NREMT STUDY

Ace your homework & exams now with Quizwiz!

The nasopharyngeal airway should be measured..... (A) from the corner of the mouth to the earlobe. (B) from the tip of the nose to the earlobe. (C) from the tip of the nose to the corner of the mouth. (D) from the tip of the nose to the chin.

(B) from the tip of the nose to the earlobe.

Aortic Dissection

damage may result in fatal bleeding, dissection is a condition where the inner layer of the wall of the aorta begins to tear; patient may complain of pain in the chest, abdomen or back will exhibit signs of shock

Three stages of labor

1) dilation 2) expulsion 3) placenta

Type 2 Diabetes

90%, patient doesn't produce enough insulin. (obesity, old age, pancreatic disease, insulin resistance)

The reason to position a pregnant woman on her left side is to A. reduce the pressure of the fetus on maternal circulation B. make labor proceed more slowly by slowing down contractions C. help turn a breech fetus in the birth canal to the vertex position D. ensure that there is sufficient blood flow to the placenta

A

To assess the motor function in the lower extremities of a responsive patient you want to: A. ask the patient to wiggle his toes B. ask the patient to bend his knees C. carefully move the patient's leg D. touch the skin of the patient's foot

A

You are treating a patient with shortness of breath. You want to deliver 6 liters per minute of oxygen. You would deliver this flow rate with a: A. nasal cannula B. simple face mask C. partial re-breather D. non-rebreather

A

MOI falls

Adults 20 feet children/infants more than 10 feet or 2or3 times their height

Common diseases from rhonchi

Aspiration Pneumonia Emphysema Chronic bronchitis

alpha1 and Alpha2

Alpha1 vessels constrict and release sweat Alpha2 try's to regulate alpha1

Compartment Syndrome

Injured cells leak watery fluid into cell spaces between them. This fluid pressure compresses tissue and increases muscle pressure. Leads to muscle & nerve damage and pain out of proportion to the injury. Potential complication of fracture.

Parietal pain

More severe pain and more localized

Urgent Move

Move used if a scne factor causes a decline in patient's condition, or if the treatment of a patient requires a move.

Medical Direction

Oversight of the patient-care aspects of an EMS system by the Medical Director.

3 components of a bomb blast

Primary: shock wave, ruptures all internal organs. Secondary: metal particles from bomb hit & injure you Tertiary: Being thrown by force to the ground or against the wall.

If patient been in cardiac arrest for more than 5 min and no CPR has been done what do you do

Provide 2 min of CPR before ard

Seizures in children most often are the result of: A. a life threatening infection B. a temperature greater than 102 F C. an abrupt rise in body temperature D. an inflammatory process in the brain

C

Snoring respirations are most rapidly managed by A. suctioning the oropharynx B. initiating assisted ventilations C. correctly positioning the head D. inserting an oropharyngeal airway

C

The focused history for patients with altered mental status should include questions about a history of trauma, diabetes, seizures and which of the following? A. heart disease B. pregnancy C. fever D. stress

C

A patient complaining of facial paralysis on one side of his face with tearing, localized pain, and sensitivity may be suffering from the most common form of facial paralysis called: A. dystonia B. muscular dystrophy C. amyotrophic lateral sclerosis D. bell's palsy

D

In a multiple casualty situation, which patient should be assigned the highest priority? A. adequate breathing, responsive, venous bleeding B. adequate breathing, responsive, suspected spine injury C. inadequate breathing, responsive, suspected broken tibia D. inadequate breathing, unresponsive, suspected internal bleeding

D

In addition to caring for injuries, emergency care for a rape victim should focus on which of the following? A. performing a pelvic or rectal exam on the patient B. collecting evidence of the rape and bagging it in plastic C. allowing the patient to shower and change clothes D. preserving evidence in a paper bag and reassuring the victim

D

distal

Far from point of reference

What are two critical fractures

Femur and pelvis

Meconium

Fetal feces. Very dangerous if the baby inhales it, suction if present.

Limb presentation

TRUE EMERGENCY. Left lateral trendelenberg position, high flow oxygen for mom. Leave limb out and transport, c section is required.

signs of severe hypoxia

Tachypnea Dyspnea Cyanosis Tachycardia may lead to dysrhythmias and eventually bradycardia Confusion Loss of coordination sleepy appearance Head bobbing Slow reaction time Altered mental status

signs of mild hypoxia

Tachypnea Dyspnea Pale cool clammy skin ( early) Elevation of blood pressure Agitation Disorientation and confusion ( from high carbon dioxide levels) Headache

Implied Consent

The consent it is presumed a patient or patient's parent or gaurdian would give if they could, such as for an unconscious patient or a parent who cannot be contacted when care is needed.

inferior

away from the head or below point of reference

Inhalation

breathing in

Glasgow scale

This is an assessment tool used to determine consciousness in clients. The Score of 15 reveals a fully oriented person. 3 is deep coma., Used for head trauma- Score 3-15, based on Eye opening, Verbal response and Motor response.

medial

Toward center of body

anterior

Toward the Front

posterior

Toward the back

dorsal

Toward the back/spine

ventral

Toward the front/ belly

Stop ambulance when using aed

True and shut off engine

Cystis

UTI, bladder infection. Caused by bacteria.

Nuchal cord

Umbilical cord around the neck; assess by as soon s the head is out of the vagina, run index finger around the neck; if unable to stretch over the babies head, clamp cord 2" apart and cut, then deliver as normal.

Umbilical cord blood vessels

Unique cause they do opposite Veins bring oxygen blood Arteries bring deoxygenated blood

McRobert's Maneuver

Pull knees to head to open up pelvis.

Example of obstructive shock

Pulmonary edema Blood can't move forward Pericardial tamponade Tension pneumothorax Both put pressure on Herat and can't fill appropriately

What is PMS

Pulses motor function Sensation

Unruptured amniotic sac

Puncture the sac and push it away from the baby.

Ribs

Vertebrosternal = 'True ribs" -ribs 1-7 attach directly to the sternum through their costal cartilage. Vertebrochondral = "False ribs" =ribs 8-10 costal cartilage articulate indirectly with the sternumb by joing the costal cartilages of ribs above. Vertebral Ribs = "Floating ribs" ribs 11 and 12 no anterior attachment.

Good indicator of internal blood loss

Weak pulse the into fast pulse Also no radial pulse body brings blood to core

Scope of Practice

What we are allowed to do or trained to do

Respiratory arrest

When breathing completely stops.

Second stage of Labor

When cervix is fully dilated and ends when infant is born.

Crush syndrome

Results from area of body trapped/crushed 4+ hours and artery flow is poor. metabolic derangement, Renal failure and sometimes death.

The Bends or Decompression Sickness

SLOW onset. Bubbles of gas obstruct blood vessels, from rapid ascent. Abdominal/joint pain. Left lateral or supine treatment for diving emergencies.

Hemothorax

Same as pneumothorax, except blood is the cause (not air). Something that can bleed has been damaged, so now blood fills the chest cavity.

ipsilateral

Same side

Secondary assessment for medical responsive

Sample Focused on injury Vitals

plantar

Sole of foot

Infant heart rate 60-80bpm?

Start chest compressions if not responding to ventilations. <100 is ventilations. <60 is compressions WITH ventilations.

Offline Direction

consists of standing orders issued by the medical director that allows EMTs to give certain medications or perform certain procedures without speaking to the medical director or another physician.

Herniation Syndrome

happens when ICP occurs. same symptoms as ICP.

Hypoperfusion

inadequate perfusion also known as shock

bariatric stretcher

Stretcher for obese patients

Neurological deficit

any deficiency in the nervous system's functioning, typically exhibited as a motor, sensory, or cognitive deficit

Atrioventricular node

located in the lower wall of the right atrium; delays impulses from the SA node to allow the atria to completely empty before the ventricles contract

Sinoartrial node

located in the upper portion of the "R" atrium. Has its own rhythm. Sets the basic pace for the cardiac rate.

Heat cramp

muscle pain or spasm due largely to the loss of salt from the body from sweating, or inadequate intake of salt

Luq

stomach spleen left lobe of liver body of pancreas L kidney and

Hemorrhagic stroke

stroke caused by the rupture of a blood vessel in the brain

Distention

swelling

edema

swelling

cricoid cartilage

the ring-shaped structure that forms the lower portion of the larynx

superior

toward the head or above point of reference

Thrombotic stroke

type of stroke caused by a blood clot blocking an artery in the brain

Where do I put epipen when done

Sharp object container

5 Stages of hypothermia

Shivering Apathy and decreased muscle function Decreased level of responsiveness Decreased vital signs Death

A loss of 15% of blood volume is

Significant and can lead to shock

Narrow pulse pressure indicates

Significant loss of blood

Nondisplaced fracture

Simple crack of the bone, no angulation/deformity.

Air Embolism

FAST onset, an air bubble lodged nto circulation. Muscle/join pain

Petit Mal Seizure

"Absence spells", usually in kids 6-12. Staring episodes, mistaken for lack of attention.

Glascow Coma Scale (eyes)

(1 point) Does not open eyes (2 points) Opens eyes in response to painful stimuli (3 points) Opens eyes in response to voice (4 points) Opens eyes spontaneously

Glascow Coma Scale (Motor)

(1 point) Makes no movement (2 points) Extension to painful stimuli ---> Decerebrate response (3 points) Abnormal flexion to painful stimuli --->Decorticate response (4 points) Flexion/withdrawal to painful stimuli (5 points) Localizes painful stimuli (6 points) Obeys commands

Glascow Coma Scale (Verbal)

(1 point) Makes no sound (2 points) Incomprehensible sounds (3 points)Utters inappropriate words (4 points) Confused and disoriented (5 points) Oriented, converses normally

Flexion posturing

(Formerly known as decorticate posturing). Client flexes one or both arms on the chest and may extend the legs stiffly. Indicates nonfunctioning cortex. Lesions of cerebral hemispheres or internal structures of brain cause this posturing.

beta 2

relaxes smooth muscle such as bronchioles and some vessels

Organs of the Right Lower Abdominal Quadrant

-APPENDIX -large & small intestines -female reproductive organs

Organs of the Left Lower Abdominal Quadrant

-Female reproductive organs -large & small intestines

Opiates & their effects

-Hydrocodone (Vicodin) -Propoxyphene (Darvon) -Heroin, morphine, codeine, meperidine (Demerol) These type of drugs suppress the CNS, depressing the respiratory system, heart rate and blood pressure.

Organs of the Right Upper Abdominal Quadrant

-LIVER -GALL BLADDER -large and small intestines

Organs of the Left Upper Abdominal Quadrant

-STOMACH -SPLEEN -PANCREAS -large & small intestine

Different Abnormal Lung Sounds

-snoring: tongue partially blocking upper airway -wheezing: constriction of bronchioles in lungs -gurgling: fluid in UPPER airway -stridor/crowing: partial UPPER airway obstruction

Hollow organs

-stomach, small intestine, colon, bladder, gallbladder

tonsil tip

... Rigid suction

oropharyngeal

..., -curved plastic device used to establish an airway in a patient by displacing the tongue from the posterior wall of the oropharynx -used in unconscious patients who do not have a gag reflex

cyanosis

..., A bluish discoloration of the skin and mucous membranes

crowing

..., A breathing sound similarto the cawing of a crow; may indicate that muscles around the larynx are in spasm.

patent airway

..., An airway that is open and clear of obstructions.

methods of artifical ventilation

...Mouth to mask Two person bag valve Fropvd One person bag valve

french tip

...Soft suction

4 Contraindications for Nitroglycerin

1) BP < 100 systolic 2) Already taken 3 doses 3) Last dose within 5 minutes 4) Sexual enhancement drugs within 48 hours

3 stages of Shock

1) Compensated 2) Decompensated 3) Irreversible (death will occur)

Cushing's Triad

1) Hypertension (widening pulse pressure) 2) Bradycardia 3) Respiration Changes

2 types of stroke

1) Ischemic (from a clot) 2) Hemorrhagic (from a bleed)

5 types of Distributive Shock

1) Neurogenic: Problem with brain/spinal cord, can't communicate with site of injury's blood vessels. 2) Drug abuse: Nitro or opiates (vasodilator) causes neurogenic shock. 3) Anaphylactic: dilated blood vessels/low BP 4) Septic: from bacteria producing vasodilating toxins 5) Psychogenic: emotional (blood vessels dilate in the brain and you faint)

Cincinnati Stroke Test (3 components)

1) Smile 2) Close eyes and raise arms, should be at equal levels 3) Repeat "the sky is blue in Cincinnati"

Patient Positions (7)

1) Supine 2) Prone 3) Fowlers: (sitting up) patient's upper body is at a 45-60 degree angle. 4) Semi-fowlers: patient's upper body is at less than a 45 degree angle. 5) Trendelenberg: patient's legs are elevated higher than the head. 6) Shock position 7) Lateral recumbent: (recovery position) patient is laying on their left or right side.

Stages of shock

1. Compensatory Shock 2. Decompensatory Shock 3. Irreversible Stage

anatomical planes

1. coronal (vertical cut into front and back halves) 2. transverse (horizontal cut into upper and lower) 3. sagittal (vertical cut into left and right halves)

Type 1 Diabetes

10%, patient does not produce ANY insulin (by beta cells of pancreas). Insulin=hormone enabling the cells to produce glucose.

Normal heart rate of children 1-6 years old

100-120 bpm

Decompensated Shock

A sudden decrease in blood pressure, tachypnea, and continued elevation of the heart rate. (Low blood pressure, high heart rate, and high # of breaths)

Subdural Hematoma

Accumulation of blood BENEATH dura mater, outside brain. More common than epidural. Fluctuating LOC or slurred speech.

Post-ical phase

After the tonic-clonic convulsions of a grand mal seizure, the patient is altered, disoriented, and recovering.

APGAR scoring

Appearance Pulse- >100 bpm Grimace- vigorous w. strong cry Activity-moving all extremities Respirations-normal w. good cry 0-2 score for each

Common diseases from wheezing

Asthma Emphysema Chronic bronchitis

lateral

Away from center of body

The golden hour in emergency medicine refers to the first 60 minutes after the A. arrival of EMS B. occurrence of multisystem trauma C. arrival at the emergency room D. start of surgery

B

Third stage of Labor

Begins with birth of infant & ends with delivery of placenta.

Convection

Body heat lost to surrounding air, which becomes warmer then rises and is then replaced by cool air and then repeat the cycle

bilateral

Both sides

What is a sign of placenta previa

Bleeding but no pain

Intracerebral Hematoma

Bleeding within brain tissue. High mortality rate

Ligaments connect: A. muscle to bone B. bones to tendon C. bone to bone D. muscle to tendons

C

Esophageal varices

Capillary networks of esophagus leak & bleed out.

Cardiac Tamponade

Collection of fluid between heart and sac, usually from blunt or penetrating trauma. Blood leaks into pericardium and accumulates in sac, compresses heart. BECK'S TRIAD

Pleural Effusion

Collection of fluid outside the lung, compresses lung and causes dypsnea. Stems from infection, CHF or cancer.

Epilepsy

Condition of reoccuring seizures.

CHF

Congestive Heart Failure failure of the heart to pump blood away from the heart causing accumulation of fluid in the tissues and lungs

Snake bite treatment

DON'T APPLY ICE OR TOURNIQUET. Splint area to minimize movement, place at or below level of heart! Circle the bite area and note the time.

Emphysema

Dead space in lungs, body retains carbon dioxide. Very prone to upper resp. diseases like pneumonia.

quadrants of the abdomen

Describing where an abdominal organ or pain is located is made easier by dividing the abdomen into four imaginary quadrants. *Right upper quadrant (RUQ) *Left upper quadrant (LUQ) *Right lower quadrant (RLQ) *Left lower quadrant (LLQ)

Type 2

Diabetes mellitus initially begins with INSULIN RESISTANCE, where the cells are not able to use the insulin properly and pancreas loses its ability to produce adequate quantities of insulin.

Peritoneum

Double-layered membrane surrounding the abdominal organs

Rhyme for glasgow

Evm 456 Eye 4 Verbal 5 Motor 6

Hypovolemic Shock

Fluid/blood loss, classic shock. Hemorrhagic=blood loss. Non-hemorrhagic=fluid loss/dehydration (from burns)

Secondary assessment for trauma minor injury

Focused on injury Vitals Sample Then detailed exam in the back of truck

Epiphyseal

Fracture at growing plate (kids)

Examples of hypovelemic shock

Hemorrhagic loos whole blood Non hemmorrhgagic Fluid loss and burns

Shock

Inadequate tissue perfusion Also know as hypoperfusion

Clonic Phase

Jerking of limbs and face, after the tonic phase.

Retroperitoneal cavity

Kidneys are in this cavity

What position is used for a patient with a protruding umbilical cord?

Knee-chest position

Grand Mal Seizure

LOC & violent muscle contractions (what most people think of when they picture a seizure).

Transport position for pregnant women not in labor

Left lateral if no trauma!

Ligaments vs Tendons

Ligaments: bone to bone Tendon: muscle to bone connection

Solid organs in abdominal

Liver Spleen Kidneys Pancreas

Penetrating truma

Made a cut through

Medium and high velocity injuries

Med/High velocity projectiles are from pellets or bullets. The damage caused, depends on the Trajectory and the dissipation of energy. Dissipation of energy is affected by Drag,Profile,Cavitation, & Fragmentation.

Treatment of a Minor

Must be given by legal guardian

proximal

Near the point of reference

Blunt force trauma

No cut but internal problem

Non urgent move

No immediate threat to life, are carried out in such a way as to prevent injury and to avoid discomfort and pain.

Obstructive Shock

Obstruction in system inhibits blood/oxygen flow. EX: clot, pulmonary embolism, tension pneumothorax, cardiac tamponade. Inhibits heart from beating properly.

unilateral

One side

Visceral pain

Pain is less severe and poorly localized because of lack of pain receptors on the organ pain is generalized

Expressed Consent

Permission that must be obtained from every conscious, mentally competent adult before emergency treatment may be provided

Pre-ictal (aura)

Phase of Grand Mal Seizure, RIGHT before the seizure occurs.

Polst

Physicians orders for life sustaining treatment. May include order for DO NOT RESUSCITATE

Placenta Previa

Placenta grows over cervix. Lots of painless vaginal bleeding (mistaken for a period). left lateral position.

PCR

Prehospital care report

less than 13

Rapid transport (what's the number on the GCS)

Secondary assessment for trauma major injury

Rapid trauma assessment Vitals Sample

Secondary assessment for medical unresponsive

Rapid trauma assessment Vitals Sample

Status Epilepticus

Repeated seizures without a break in between. Continuous seizure lasting more than 10 minutes, 3 or more seizures in one hour.

Beck's Triad

Symptoms of cardiac tamponade. Jugular Vein Distention, Muffled heart sounds, Widening systolic/diastolic BP.

Oral

Swallowed

Heat stroke

The failure of the heat-regulating ability of an individual under heat stress. Skin will be hot and red possibly not sweating

Cardiac arrest

The heart and breathing stop suddenly and without warning

Physical exam, Baseline vitals, History

Three major steps in secondary assessments

When can I apply a splint on scene

To extremity fractures only if the patient is stable and there is no life threat

Glasgow coma scale

Used for head trauma- Score 3-15, based on Eye opening, Verbal response and Motor response.

Spontaneous Pneumothorax

Usually caused by trauma. Pressure in pleural pace keeps lungs inflated, pleuritic chest pain. Collapsed lung.

Croup

VIRAL infection of airway below vocal cords, laryngotracheal bronchitis. sounds like a "seal bark"

Effects of Epinephrine

Vasoconstrictor & Bronchodilator

Effects of Nitroglycerin

Vasodilator

When would you ventilate an infant? Start CPR?

Ventilate 40-60 bpm when their heart rate is less than 100bpm. CPR when heart rate is below 60 bpm.

V-fib

Ventrical Fibrillation, chaotic electrical activity from many sites in the heart. http://en.wikipedia.org/wiki/Ventricular_fibrillation

V-tach

Ventricular tachycardia (an increased ventricular heart rate).

Expulsion stage

When the baby is expelled from its mother's body

When to ventilate patients?

When their breaths/min do not fall between 8<x<30

Tension Pneumothorax

Worsening pneumothorax. Damage to lung tissue. Air that's usually in the lungs escapes into chest cavity, applying pressure to heart and organs. Complete lung collapse on affected side. JVD

Peritoneum

a transparent membrane that lines the abdominal cavity in mammals and covers most of the viscera

Thrombus

a blood clot formed within a blood vessel and remaining attached to its place of origin

Preeclampsia

abnormal state of pregnancy characterized by hypertension and fluid retention and albuminuria

Apnea

absence of breathing

Epidural Hematoma

accumulation of blood between skull and dura mater (outermost layer), almost always because of linear fracture. Lucid intervals of LOC

Pneumonia

acute inflammation and infection of alveoli, which fill with pus or products of the inflammatory reaction

AMI

acute myocardial infarction (heart attack)

epinephrine

adrenaline; activates a sympathetic nervous system by making the heart beat faster, stopping digestion, enlarging pupils, sending sugar into the bloodstream, preparing a blood clot faster

Secondary assessment

after immediate life- or limb-threatening injuries/illnesses have been identified, this more thorough evaluation is performed to identify more subtle, yet still important, injuries

Duty to Act

an obligation to provide care to a patient

Epidermis

an outer layer of cells designed to provide protection

Sublingual

beneath the tongue

Angina pectoris

chest pain caused by a temporary loss of oxygenated blood to heart muscle often caused by narrowing of the coronary arteries

Epilepsy

chronic brain disorder characterized by recurrent seizure activity

pulse pressure

difference between systolic and diastolic pressure no more than 25%

Dyspnea

difficult or labored respiration

Red blood cell

erythrocyte

shock position

feet elevated 12 inches higher than head

Pericardial tamponade

filling of the pericardial sac with fluid, which in turn limits the filling and function of the heart.

Placenta

filters nutrients and oxygen from mom's system to the baby.

Dilation stage

first stage of labor; the uterine muscle contracts strongly to expel the fetus; the fetus presses on the cervix and causes it to dilate or expand

Grand mal

generalized tonic-clonic (it is sudden cry, fall, rigidity, followed by muscle jerking; shallow, irregular breathing; possible loss of bladder or bowel control; usually lasts seconds to minutes, followed by some confusion, a period of sleep (postical lethargy), and then return to full consciousness.

beta 1

increases HR, inotrophy (forced contraction of heart) increases electrical impulse in heart

Hepatitis B

infectious inflammation of the liver caused by the hepatitis B virus (HBV) that is transmitted sexually or by exposure to contaminated blood or body fluid

Intramuscular

into the muscle

Distributive shock

less distribution of blood to areas = venous pooling, most common in general anesthesia and spinal cord injuries and septic shock (bacterial infection where toxins in blood). cause: general anesthesia and spinal cord injuries and septic shock

mid axillary

line drawn veritcally from the middle of the armpit to the ankle

Protocols

lists of steps, such as assessments and interventions, to be taken in different situations. Protocols are developed by the Medical Director of an EMS system

Sputum

material expelled from the lungs by coughing

Define para

number of live births

Define gravida

number of pregnancies

Chronic bronchitis

obstructive pulmonary disease characterized by excessive production of mucus and chronic inflammatory changes in the bronchi, resulting in a cough with expectoration for at least 3 months of the year for more than 2 consecutive years.

Emphysema

obstructive pulmonary disease characterized by overexpansion of the alveoli with air, with destructive changes in their walls resulting in loss of lung elasticity and gas exchange

Antepartum

occurring or existing before birth

Acute pulmonary edema

occurs when an excessive amount of fluid collects in the spaces between the alveoli and capillaries, disturbs normal gas exchange

contralateral

opposite side

Placenta previa

pregnancy in which the placenta is implanted in the lower part of the uterus (instead of the upper part)

Ectopic pregnancy

pregnancy resulting from gestation elsewhere than in the uterus

Pea

pulseless electrical activity. Heart is doing something but not pumping

Dermis

second layer of skin, holding blood vessels, nerve endings, sweat glands, and hair follicles

phrenic nerve

stimulates the diaphragm

vertebrae

the 33 bones of the spinal column

Coronary arteries

the two arteries that supply blood to the heart muscle

Jugular vein distention

the visible bulging of the jugular vein when the Pt is in semi-fowlers position or full fowlers position. this is indicative of inadaquate blood movement through the heart and/or lungs

scoop stretcher

this cot splits in tow or four sections, so it can be used where larger stretchers cannot fit.

You are dispatched to a residence for a 20-year-old male with respiratory distress. When you arrive, you find that the patient has a tracheostomy tube and is ventilator dependent. His mother tells you that he was doing fine, but then suddenly began experiencing breathing difficulty. You should: • A:detach the ventilator, suction the tracheostomy tube, and reassess the patient. • B:remove the ventilator tubing and place an oxygen mask over the tracheostomy tube. • C:remove him from the mechanical ventilator and ventilate him manually. • D:check the settings on the ventilator to ensure that it is functioning properly.

you selected C; This is correct! Reason:If a ventilator-dependent patient experiences a sudden onset of respiratory distress, you should first remove him or her from the mechanical ventilator and begin manual ventilation with a bag-mask device; if the patient improves, you will know that the problem was a malfunction with the mechanical ventilator. If the patient does not improve, the tracheostomy tube is likely plugged with thick mucus secretions and requires suctioning. Unless you are familiar with the mechanical ventilator (most EMTs are not), do not attempt to troubleshoot the device by checking the settings; this only wastes time.

The MOST obvious way to reduce heat loss from radiation and convection is to: • A:move away from a cold object. • B:increase metabolism by shivering. • C:wear a thick wind-proof jacket. • D:move to a warmer environment.

you selected D; This is correct! Reason:In a cold environment, the body has two ways of staying warm: generating heat (thermogenesis) and reducing heat loss. Radiation is the transfer of heat by radiant energy. The body can lose heat by radiation, such as when a person stands in a cold room. Convection occurs when heat is transferred to circulating air, as when cool air moves across the body's surface. A person standing in windy cold weather, wearing lightweight clothing, is losing heat to the environment mostly by convection. The quickest and most obvious way to decrease heat loss from radiation and convection is to move out of the cold environment and seek shelter from wind. Shivering increases the body's metabolism and is a mechanism for generating heat, not reducing heat loss. Layers of clothing trap air and provide excellent insulation; thus, layered clothing decreases heat loss better than a single, thick jacket. Conduction is the direct transfer of heat from a part of the body to a colder object by direct contact, as when a warm hand touches cold metal or ice. The most obvious way to decrease heat loss by conduction is to remove your hand from the cold object.

snoring sounds

..., airway blocked, open patients airway promt transport

dyspnea

..., difficult or labored respiration

internal respiration

..., exchange of gases between the blood and the cells of the body

external respiration

..., exchange of gases between the lungs and the blood

An elderly man is found outside. He is unconscious, unresponsive, apneic, and pulse less. His skin is cold to touch, his muscles are rigid, and his core body temperature is 89.6°F. What is the initial treatment? (A) CPR. (B) Turn the heat up in the ambulance (C) starting passive rewarming. (D) placing in a Trendelenburg position.

(A) CPR. CPR is indicated even if signs of death are present. A hypothermic patient cannot be presumed dead until they are warm and dead. Remember hypothermia can cause Ventricular Fibrillation so rewarming needs to be done in a controlled setting.

Which statement correctly relates to pediatric seizures? (A) Febrile seizures typically occur on the 1st day of a febrile illness and have tonic-clonic activity. (B) Always suspect a fever to be the cause of a seizure if the temperature is more than 101°F. (C) Infants and children between the ages of 4 months and 8 years commonly suffer from febrile seizures. (D) Febrile seizures do not require transport if the patient can be cooled.

(A) Febrile seizures typically occur on the 1st day of a febrile illness and have tonic-clonic activity.

The documentation of a baseline neurological status should NOT include which of the following: (A) Pulse oximetry (B) Response to sensory stimulation (C) Pupillary reaction (D) Motor function

(A) Pulse oximetry

What would you suspect in a 22yo female complaining of severe, stabbing, unilateral pain in the lower abdomen. Vital signs are BP 88/64, HR 128, RR 22. Her skin is cool and she missed her menstrual cycle. (A) a ruptured ectopic pregnancy. (B) endometriosis. (C) pelvic inflammatory disease. (D) amenorrhea.

(A) a ruptured ectopic pregnancy. The classic triad of symptom includes abdominal pain, vaginal bleeding, and amenorrhea. Other symptoms include referred pain to the shoulder, nausea, vomiting, or syncope. Signs of shock may be present if the ectopic ruptures.

You are caring for a female who is 36 weeks pregnant and involved in a car accident. After you fully immobilize her on a long spine board, you would: (A) carefully tilt the board to the left side 10-15 degrees. (B) carefully tilt the board to the right side 10-15 degrees. (C) manually displace the uterus to the right. (D) allow the patient to roll onto her side.

(A) carefully tilt the board to the left side 10-15 degrees...To prevent supine hypotension syndrome. Tilting the board will allow the uterus to displace to the left and prevent compression of the inferior vena cava.

Cushing's reflex includes increased blood pressure, bradycardia, and: (A) irregular respirations. (B) irregular pulse. (C) increased respirations. (D) pupil dilation.

(A) irregular respirations. Cushing's reflex is indicative of a head injury with increased intracranial pressure (ICP).

How would you open the airway of an unresponsive child who was struck by a car traveling approximately 25 MPH? (A) jaw-thrust with c-spine stabilization. (B) head-tilt/chin-lift. (C) hyperextension with jaw-thrust. (D) hyperflexion of the head to the "sniffing" position.

(A) jaw-thrust with c-spine stabilization.

When caring for a child in respiratory distress, you hear a harsh high-pitched sound with each breath. What do you suspect? (A) stridor. (B) wheezing. (C) crackles. (D) rhonchi.

(A) stridor.

What is the rate of chest compressions for a two-year-old in cardiac arrest? (A) 80/minute. (B) 100/minute. (C) 110/minute. (D) 120/minute.

(B) 100/minute.

A patient was burned and has blisters to the chest, abdomen, entire right arm, and front of the right leg. According to the rule of nines, what is the percent of burns? (A) 40.5% (B) 36% (C) 45% (D) 31.5%

(B) 36% (chest = 9%, abdomen = 9%, entire arm 9%, front of the leg = 9%)

A nasal cannula can deliver ____% of oxygen at a flow rate of 6 LPM in optimal conditions. (A) 30% (B) 40% (C) 50% (D) 65%

(B) 40% of oxygen.

Which law protects an EMT if the EMT acts in good faith, is not negligent, acts within the scope of practice, and does not accept payment for service? (A) implied consent laws. (B) Good Samaritan laws. (C) malfeasance laws. (D) absolute immunity laws.

(B) Good Samaritan laws

Injury to which abdominal organ is most likely to cause death in the pediatric patient? (A) Pancreas (B) Liver (C) Spleen (D) Kidney

(B) Liver The liver is a solid, vascular organ in the RUQ. Rupture or laceration of the liver can cause severe hemorrhage. Injury to the liver is the most common abdominal injury that leads to death in the pediatric patient.

Which of the following is treatment for a flail chest injury? (A) Position of comfort with O2 per NRB mask (B) Positive pressure ventilation (C) Stabilization of the flail segment with a sandbag (D) Applying tape to the uninjured side

(B) Positive pressure ventilation

Which of the following is the valve located between the left atrium and left ventricle? (A) tricuspid valve. (B) mitral valve. (C) aortic valve. (D) pulmonic valve.

(B) The mitral valve is located between the left atrium and the left ventricle.

Tidal volume is defined as: (A) the amount of gas moved in and out of the respiratory tract per minute. (B) the amount of gas contained in the lungs at the end of a maximal inspiration. (C) the amount of gas inhaled and exhaled during a normal breath. (D) the amount of gas that remains in the respiratory system after forced expiration.

(B) Tidal volume is defined as the amount of gas inhaled or exhaled during a normal breath

What are the three parameters of the pediatric assessment triangle? (A) airway, breathing, circulation. (B) appearance, breathing, circulation. (C) airway, respiratory rate, pulses. (D) appearance, lung sounds, skin color.

(B) appearance, breathing, circulation. The pediatric assessment triangle is based on appearance, which includes mental status and muscle tone; breathing, which includes visible movement, effort, and audible sounds; and circulation, which includes skin color. All are obtained as part of the initial assessment to formulate a rapid determination of the child's stability.

You delivered a full-term baby in the back of your ambulance and the initial APGAR score is 5. What treatment should you do? (A) CPR. (B) effective ventilation. (C) vascular access. (D) administration of epinephrine.

(B) effective ventilation. Most newborns respond favorably to warming, drying, suctioning, and stimulation.

Shaken Baby Syndrome can cause which of the following conditions? (A) Hypoglycemia (B) Abdominal bruising (C) Subdural hematoma (D) Lower extremity fracture

(C) Subdural hematoma

Your patient is 28 weeks pregnant with complaints of dizziness and blurred vision. Her hands and feet are edematous. Vitals are BP 178/108, HR 104, RR 24. You suspect that she: (A) is emotionally upset. (B) has preeclampsia. (C) has borderline hypotension. (D) has postpartum depression.

(B) has preeclampsia. Common signs and symptoms of preeclampsia include headache, dizziness, blurred vision, nausea, vomiting, hypertension and edema.

You arrive at a two car accident and the driver of the 1st car is unconscious. Consent for this patient would be? (A) informed. (B) implied. (C) expressed. (D) involuntary.

(B) implied

An incomplete or illegible patient care report: (A) may be acceptable in some situations. (B) may cause subsequent caregivers to provide inappropriate treatment. (C) is of no use. (D) is useful in the ongoing care and treatment.

(B) may cause subsequent caregivers to provide inappropriate treatment. The accuracy and completeness of the patient care report may have a significant impact on ongoing care and treatment of the patient.

Beck's triad (associated with pericardial tamponade) includes: distant heart tones, JVD, and... (A) pulse deviation. (B) narrowing pulse pressure. (C) wheezing. (D) delayed capillary refill.

(B) narrowing pulse pressure.

A dissecting aortic aneurysm may cause the following sign or symptom: (A) a pain sensation on the neck and back of head (B) pain in the lower back and lower quadrants (C) the same blood pressure in each arm (D) respiratory distress

(B) pain in the lower back and lower quadrants The description of pain associated with a dissecting aortic aneurysm is severe back pain because the peritoneum can be rapidly stripped away from the wall of the main abdominal cavity.

Organ responsible for insulin production. (A) adrenal (B) pancreas (C) thyroid (D) thymus

(B) pancreas

The chest wall is lined by a layer of smooth, glistening tissue called: (A) visceral pleura. (B) parietal pleura. (C) visceral peritoneum. (D) parietal peritoneum.

(B) parietal pleura.

The large vessel of the body that carries oxygenated blood is the (A) coronary sinus. (B) pulmonary vein. (C) superior vena cava. (D) femoral vein.

(B) pulmonary vein Veins carry blood from the body back to the heart. The pulmonary vein carries oxygen-rich blood from the lungs back to the heart.

Supine hypotensive syndrome occurs from? (A) reduction of cardiac output due to compression of the aorta. (B) reduction of cardiac output due to compression of the inferior vena cava. (C) reduction of cardiac output due to compression of the superior vena cava. (D) the uterus pushing up on the diaphragm.

(B) reduction of cardiac output due to compression of the inferior vena cava.

You arrive on the scene of a multiple vehicle crash. Upon arrival, each of the following actions are appropriate EXCEPT to: (A) establish scene command. (B) treat and transport the most critically ill patient. (C) perform a scene size up. (D) control scene hazards.

(B) treat and transport the most critically ill patient. The first unit to arrive on the scene should begin the scene size up which includes establishing command, calling for back-up units, controlling any hazards and locating and triaging patients. It would not be appropriate for the primary unit to locate and transport the most critically injured patient.

Which of the following statements is true regarding chemical burns? (A) Always use antidotes or neutralizing agents. (B) Acid burns are typically more serious than alkali burns. (C) Both acids and alkalis cause burns by disrupting cell membranes and damaging tissues on contact. (D) Always approach a chemical spill from downwind.

(C) Both acids and alkalis cause burns by disrupting cell membranes and damaging tissues on contact.

The elements of a negligence claim include all of the following EXCEPT: (A) duty to act. (B) breach of duty. (C) code of conduct. (D) proximate cause.

(C) Code of conduct. The four elements of a negligence claim include: the duty to act, breach of duty, actual damages, and proximate cause.

Your patient injured his left arm in a motorcycle accident. When assessing the injured extremity, which of the following is most important? (A) Check for compartment syndrome. (B) Check deep tendon reflexes. (C) Do a thorough neurovascular exam. (D) Check for position sense

(C) Do a thorough neurovascular exam. It is important to check for pulses, motor, and sensory functions distal to any injury in an extremity. Assessment and documentation of the neurovascular status are required before and after splinting.

You arrive to find a 6 year old child who is in the tripod position and drooling. The child has a fever and difficulty swallowing. What do you suspect? (A) bronchiolitis. (B) croup. (C) epiglottitis. (D) meningitis.

(C) Epiglottitis Epiglottitis is a bacterial infection causing the acute swelling of the epiglottis and the soft tissues above the glottic opening. Clinical findings include fever, pain on swallowing, typically sitting in tripod position, and drooling.

Which federal agency regulates telecommunications in the United States? (A) FBI. (B) FDIC. (C) FCC. (D) FACEP.

(C) FCC.

When assessing an injured eye, you see a collection of blood in front of the pupil and iris. What is it called? (A) Raccoon's eye (B) Retinal detachment (C) Hyphema (D) Corneal laceration

(C) Hyphema

Where should you place your hands on the chest of an adult victim when you are performing chest compressions? (A) On the top of the sternum (B) On the lower one-third of the sternum (C) On the lower half of the sternum, at the nipple line (D) Over the very bottom of the sternum

(C) On the lower half of the sternum, at the nipple line

Which of the following indicate that a pediatric patient is progressing from respiratory distress to respiratory failure? (A) Nasal flaring (B) Respiratory rate over 32 (C) Poor muscle tone (D) Grunting/head bobbing

(C) Poor muscle tone Nasal flaring, elevated respiratory rate, grunting, and head bobbing all indicate respiratory distress. The development of poor muscle tone indicates the patient is tiring and is an ominous sign of respiratory failure.

A high-pitched noise associated with upper airway constriction is known as (A) wheezes. (B) rhonchi. (C) stridor. (D) rales.

(C) Stridor Stridor is defined as a high-pitched sound associated caused by obstruction in the trachea or larynx.

What are the anatomical differences in the pediatric airway compared to the adult airway. (A) The airway diameter is larger, the larynx is more anterior, the tongue is proportionally smaller. (B) The airway diameter is larger, the larynx is more posterior, the tongue is proportionally smaller. (C) The airway diameter is smaller, the larynx is more anterior, the tongue is proportionally larger. (D) The airway diameter is smaller, the larynx is more posterior, the tongue is proportionally larger.

(C) The airway diameter is smaller, the larynx is more anterior, the tongue is proportionally larger.

When using the rule of nines for calculating burns, which of the following is a correct modification for a child? (A) The front of each arm for a child is 7% compared to 9% for an adult. (B) The front of each leg for a child is 4% compared to 9% on an adult. (C) The head of a child is 18% compared to 9% on an adult. (D) The entire leg of a child is 9% compared to 18% on an adult.

(C) The head of a child is 18% compared to 9% on an adult.

Stimulation of the vagus nerve may result in: (A) vasodilation. (B) an increase in heart rate. (C) a decrease in heart rate. (D) an increase in cardiac output

(C) a decrease in heart rate.

During your assessment, you find crepitus and instability to the right rib cage. Lung sounds are equal but there is paradoxical chest movement. You suspect: (A) a tension pneumothorax. (B) a massive hemothorax. (C) a flail chest. (D) an abdominal aneurysm.

(C) a flail chest Flail chest is defined as three or more adjacent ribs that are fractured in two or more places. This segment of the chest is free to move with the pressure changes of respirations.

Manual maneuvers used to open a patient's airway (A) are contraindicated in most patients. (B) are difficult to perform. (C) include head-tilt/chin-lift and jaw thrust. (D) do not usually work without more extensive intervention.

(C) include head-tilt/chin-lift and jaw thrust.

The Sellick maneuver (A) is used to clear a foreign body airway obstruction in an infant or child. (B) is used to clear blood or mucus from the nasopharynx. (C) may be used to minimize gastric distention. (D) is the preferred method for opening the airway of an unconscious patient when cervical spine injury is suspected.

(C) may be used to minimize gastric distention. Posterior pressure exerted on the cricoid cartilage (the Sellick maneuver) will effectively compress the esophagus, minimizing the potential for gastric distention. This maneuver will also reposition the vocal cords for clearer visualization of anatomic structures

A BVM with a reservoir and oxygen source (at least 15 LPM) delivers an oxygen concentration of (A) 21%. (B) 40% to 60%. (C) 80%. (D) 100%.

(D) 100%.

Which of the following is NOT indicative of a tension pneumothorax? (A) Dyspnea (B) Hyperresonance to percussion (C) Distended jugular veins (D) Clear lung sounds

(D) Clear lung sounds.

How would you transport a 22 year old man who was hit in the head with a baseball and had a period of unconsciousness lasting 7 seconds? (A) Recovery position (B) Sitting up at a 90 degree angle (C) Trendelenburg (D) Immobilized on a backboard

(D) Immobilized on a backboard

Which of the following can help determine possible injuries that may be sustained in an MVC? (A) Length of skid marks (B) Debris at the scene (C) Patient symptoms (D) Mechanism of injury

(D) Mechanism of injury

You arrive on scene to find a 7 y/o boy who is lethargic, cyanotic and has labored breathing. His RR is 50bpm with diminished breath sounds and crackles over the right lung. His temp is 104°F. What do you suspect? (G) cardiogenic shock. (H) a drug overdose. (C) respiratory distress. (D) respiratory failure, probably d/t pneumonia.

(D) Respiratory failure, most probably due to pneumonia. This is based on tachypnea, increased work of breathing, diminished breath sounds, and elevated temperature.

All of the following are phases of the stress response EXCEPT: (A) alarm reaction. (B) resistance. (C) exhaustion. (D) acceptance.

(D) acceptance. The 3 phases of the body's response to stress: (1) alarm reaction (fight-or-flight response) (2) resistance: the level of resistance to stress raises (3) exhaustion-as stress continues, coping mechanisms begin to fail

Your assessment must be thorough when dealing with an intoxicated patient because: (A) alcohol increases pain tolerance. (B) alcohol decreases the patient's judgment. (C) alcohol is a CNS stimulant. (D) alcohol can mask signs and symptoms of injury.

(D) alcohol can mask signs and symptoms of injury. Alcohol is a central nervous system depressant and, therefore, can mask signs and symptoms of injury.

Which blood component transports oxygen? (A) plasma. (B) platelets. (C) leukocytes. (D) erythrocytes.

(D) erythrocytes. Also known as RBC's. Leukocytes are white blood cells and play a role in fighting infection. Platelets play a role in clotting. Plasma is the liquid that carries the cells throughout the body.

All of the following can affect the accuracy of a pulse oximeter EXCEPT: (A) hypoperfusion. (B) anemia. (C) carbon monoxide poisoning. (D) hyperthermia.

(D) hyperthermia.

Treatment for a patient with preeclampsia would include: (A) rapid transport with lights and siren. (B) administering nitroglycerin to lower the blood pressure. (C) transport for psychiatric evaluation. (D) keeping the patient calm; transport without lights and siren.

(D) keeping the patient calm; transport without lights and siren. If you suspect preeclampsia, you should take precautions to prevent seizures, which include keeping the patient calm and transporting without lights and sirens.

A small child fell through ice when skating. After 20 minutes, the child is pulled out of the water apneic and pulseless. What is your first action? (A) keep the child cool to maintain the mammalian dive reflex. (B) defibrillate the child immediately. (C) pronounce the child dead at the scene. (D) secure the child's airway and control the cervical spine.

(D) secure the child's airway and control the cervical spine Since it is unknown if there are any c-spine injuries from the fall through the ice, you must secure the airway and control the cervical spine during resuscitation.

What happens during inspiration? (A) the diaphragm contracts and the intercostal muscles relax. (B) the diaphragm relaxes and the intercostal muscles contract. (C) the diaphragm relaxes and the intercostal muscles relax. (D) the diaphragm contracts and the intercostal muscles contract.

(D) the diaphragm contracts and the intercostal muscles contract.

An indicator of severe abdominal trauma after an MVC may include: (A) absent bowel sounds. (B) back pain. (C) referred pain to the clavicle. (D) unexplained shock.

(D) unexplained shock. When assessment of a trauma patient does not reveal significant injury, the presence of unexplained shock should lead you to suspect serious abdominal/thorax trauma.

upper airway

(Nose, mouth, pharynx, larynx)FUNCTION:*Conducts air to lower airway *Protects lower airways *Warms, filters & humidifies air

A patient was struck in the head by a large tree branch while trimming a tree. The next day, he complains of dizziness and vomiting. You suspect? (A) an epidural hematoma. (B) a subdural hematoma. (C) a concussion. (D) a contusion.

(a) subdural hematoma. A subdural hematoma is usually due to rupture of small venous vessels. This type of bleeding is slow and the onset of symptoms may take several hours to develop after the injury.

Para

(obstetrics) the number of live-born children a woman has delivered

tachypnea

..., fast breathing, an abnormally rapid rate of respiration, usually >20 breaths per minute

nasopharyngeal

..., flexible airway inserted through the patients nose

The air sacs in the lung where oxygen-carbon dioxide exchange occurs are the A. bronchioles B. bronchi C. epiglottis D. alveoli

D

intercostal muscles

..., Muscles which move the rib cage during breathing

cricoid pressure

..., Pressure on the trachea, prevents air from entering the esophagus/stomach and vomiting

agonal respiration

..., Shallow, slow or infrequent breathing

respiratory arrest

..., When breathing completely stops.

respiratory failure

..., a condition in which the level of oxygen in the blood becomes dangerously low or the level of carbon dioxide becomes dangerously high

stridor

..., a whistling sound when breathing (usually heard on inspiration) upper airway

rales

..., abnormal crackling sound made during inspiration

n-95

..., Mask used for a patient with TB

pulmonary ventilation

..., Movement of air into and out of the lungs

retractions

..., Movements in which the skin pulls in around the ribs during inspiration.

FROPVD

..., flow-restricted, oxygen-powered ventilation device: a device that uses oxygen under pressure to deliver artifical ventilations. Its trigger is placed so that the rescuer can operate it while still using both hands to maintain a seal on the face mask. Has automatic flow restriction to prevent overdelivery of oxygen to the patient.

gurgling

..., indicates presence of fluid in the upper airway, need for suctioning

visceral pleura

..., inner layer of pleura that surrounds each lung

rhonchi

..., lower-pitched sounds like snoring or rattling, secretions in larger airways (pneumonia, bronchitis, aspiration)

pleura

..., membrane surrounding the lungs

parietal pleura

..., pleura that lines the inner chest walls and covers the diaphragm

cellular respiration

..., process that releases energy by breaking down glucose and other food molecules in the presence of oxygen

bradypnea

..., slow respiratory rate, usually below 10 respirations per minute

bronchioles

..., smallest branches of the bronchi

capillary refill

..., tested by pressing the nail tip briefly and watching for color change. A normal finding is the pink tone returns immediately when pressure is released. An abnormal finding is slow (greater than 2 seconds) return of pink tonewith respiratory or cardiovascular diseases that cause hypoxia.

respiration

..., the bodily process of inhalation and exhalation

carina

..., the fork at the lower end of the trachea where the two mainstem bronchi branch.

pleural space

..., the small potential space between the parietal and visceral layers of the pleura

What is the primary action of nitroglycerin? A. lower the blood pressure B. contract the heart muscles C. slow the heart rate down D. dilate the coronary arteries

D

Factors that may increase bleeding

Movement Low body temp clotting less effective Medication anticlotting like aspirin Intravenous fluids Removal of bandages

Placenta Abrupto

Placenta torn off endometrium, causes bleeding, abdominal pain, increased heart rate. Fetus is lost, mom can be saved. Code 3. Usually caused by trauma or chemicals. Left lateral position.

Common diseases from crackles

Pneumonia Pulmonary edema

Normal heart rate of Newborn

140-160 bpm

Glucose after meal

120-140

How long does angina pain last

2-15 min

How many shocks can I give

2-3 with CPR in-between

The patient you are treating has white, waxy skin on both hands. The patient's hands feel as if they're frozen. They are swollen and you notice that blisters are forming. Which of the following would you not want to do in caring for the patient. A. rub the hands B. remove any jewelry C. cover the hands with dry sterile dressings D. leave blisters intact

A

The production of insulin occurs in the A. pancreas B. liver C. gallbladder D. kidney

A

You arrive at an emergency room to find that the nurses are busy taking care of other patients. You are getting off shift and want to get back to the station. You and your partner move the patient into a hospital bed, lay the report on the bed with the patient, and leave. You and your partner have just committed A. abandonment B. negligence C. appropriate patient care D. vicarious liability

A

You arrive at the scene of a possible unconscious patient located in a lab setting. On the exterior of the room there is a NFPA diamond with a 4 in the blue portion of the diamond. This is an: A. extreme health hazard B. extreme fire hazard C. extreme reactivity hazard D. extreme water reactivity

A

You arrive on the scene to find an unconscious patient about 14 years of age. Where would you assess for a pulse on this patient? A. carotid artery B. brachial artery C. femoral artery D. radial artery

A

You have just intubated a patient in cardiac arrest. Your partner tells you he hears breath sounds on the right side but not on the left. You would suspect: A. intubated the right stem bronchus B. intubated the left stem bronchus C. are in the proper position D. are in the esophagus

A

To which patient should you administer oral glucose? A. 60-year-old female behaving as if she is intoxicated, and whose daughter informs you that she takes insulin by injection B. 45-year-old male with a history of diabetes behaving erratically after falling and hitting his head in the bathtub C. 70-year-old male with a long history of diabetes who is unconscious and cannot swallow D. 52-year-old female who tells you that she is feeling dizzy and has low blood sugar

A

Vitreous humor is found: A. behind the lens of the eye B. in the bone marrow of the upper arm C. in front of the lens of the eye D. in the joint lubrication of the upper arm

A

When established a landing zone for a medical helicopter, the minimum area secured should be A. 100' x 100' B. 200' x 200' C. 50' x 50' D. 60' x 60'

A

When is it most appropriate to clamp and cut the umbilical cord? A. as soon as the cord stops pulsating B. after the placenta has completely delivered C. before the newborn has taken its first breath D. immediately following delivery of the newborn

A

Hepatitis C

inflammation of the liver caused by the hepatitis C virus, transmitted by exposure to infected blood (rarely contracted sexually)

When treating an eye injury involving an impaled object, it is important to: A. cover both eyes and stabilize the object B. cover the affected eye and stabilize the object C. stabilize the object and do not cover either eye D. remove the object and cover both eyes

A

When you listen to the lungs of an asthma patient you would expect to hear: A. wheezes B. rales C. stridor D. rhonchi

A

Meninges

3 layers that surrounds the brain & spinal cord 1) Dura Mater 2) Arachnoid 3) Pia Mater

Flail Segment

3+ ribs fractured in 2+ places. Causes free floating segment of the chest wall, other internal injuries probably present. Splint using a pillow or towel.

Which artery do you typically assess on an infant? A. brachial artery B. carotid artery C. femoral artery D. radial artery

A

Subdural hematoma

pertaining to below the dura mater, tumor of blood

Which of the following actions should be carried out during the initial assessment? A. assessing the skin B. palpating the cranium C. auscultating the lungs D. obtaining a blood pressure

A

Which of the following are the signs and symptoms of shock in the early stages? A. tachycardia, anxious, restless, skin pale, cool, clammy B. bradycardia, anxious, restless, skin pale, cool, and clammy C. tachycardia, hypotension, increased breathing D, bradycardia, hypotension, skin pale, cool, clammy

A

Which of the following indicates that a patient with hyperthermia is in serious danger? A. hot skin B. moist skin C. muscles cramps D. dizziness

A

Which of the following is a late sign of shock? A. hypotension B. tachycardia C. cyanosis D. anxiety

A

Which of the following is not part of the history or secondary exam? A. form a general impression of the patient B. take the patient's history C. conduct a physical exam D. take baseline vitals

A

Which of the following is the highest priority patient? A. 57 year old male with chest pain and systolic blood pressure of 80 B. 40 year old female with moderate pain from a leg injury C. 75 year old male who appears confused but responds to commands D. 25 year old female in labor with contractions six minutes apart

A

Which of the following pressures is considered narrowed? A. 30 B. 20 C. 20 D. 35

A

Your patient has profuse bleeding from a wound on her lower leg but no signs of skeletal injury. The steps you should take to stop the bleeding in the correct order, are: A. direct pressure, elevation, pressure dressing and pressure point B. pressure point, tourniquet, and concentrated or diffuse direct pressure C. pneumatic anti-shock garments (PASG), lower extremity elevation, and diffuse direct pressure D. elevation, pressure point, pressure dressing and PASG

A

Your patient has what appears to be a dislocated shoulder. Treatment for this patient should be: A. sling and swathe the injured shoulder B. immobilize the arm to the side of the patient C. reset the shoulder and then immobilize D. wrap the upper body in a pillow and immobilize both arms

A

Your patient, a 69-year-old male, is in cardiac arrest. His wife informs you that their physician has written a DNR order for the patient, but she does not have the written order. You should: A. provide all necessary care to save the patient's life B. obey the DNR order and leave immediately

A

Which of the following situations represents your abandonment of a patient? A. you begin assessing a patient, but turn responsibility for that patient over to a first responder B. you begin CPR on a cardiac arrest patient but stop when the ALS team takes over care C. with the approval of medical direction, you do not transport a patient who feels fine after having a seizure D.you refuse to help a patient administer nitroglycerin that has been prescribed for someone else

A

Which patient should receive a rapid trauma survey to determine hidden injuries? A. alert 2-year-old child in car seat who was in a medium-speed crash B. alert 20-year-old male who fell ten feet and is complaining of leg pain C. alert 65-year-old female who fell in the bathtub and is complaining of wrist pain D. alert 11-year-old who tripped while roller-skating and fell down three steps

A

Which situation requires that an emergency patient be moved? A. your patient has undergone cardiac arrest while seated in a chair B. your patient is found on the ground, unresponsive, and alone C. your patient is found in his bed, displaying early symptoms of shock D. your patient is showing signs of inadequate breathing and shock

A

You are suctioning your patient's airway. One of the biggest side effects of suctioning is: A. hypoxia B. aspiration C. suction power D. suction catheter clogging

A

Atherosclerosis

the most common form of CVD; a disease characterized by plaques along the inner walls of the arteries.

Your unconscious patient has blood in his airway. You should: A. use a suction unit to immediately clear the airway B. apply oxygen using a nonrebreather mask at 15L/min C. use a bag-valve mask to clear the airway D. perform a finger sweep to remove the blockage

A

Ischemic stroke

the most common kind of stroke

You are the triage officer at the scene of a mass casualty incident. Which of the following patients should be treated first? A. A 37-year-old female patient who is unresponsive B. An 18-year-old male patient who is not breathing and has no pulse C. A 29-year-old male patient with a femur fracture D. An 8-year-old patient who is conscious but is having trouble breathing

A

You are treating a patient who has overdosed on a narcotic. The greatest risk for this patient is: A. respiratory depression B. seizures C. hypertension D. hyperactivity

A

Gravida

the number of the pregnancy that a woman is in

Normal heart rate of children 6+

80-100 bpm

Normal glucose

80-120

Glucose after 8-10hr of no food

80-90

What temp and humidity are bad for the body to regulate temp

90 degrees and 75 humidity

% oxygen by BVM

90%

A 2 year old male is in respiratory failure when he has A. altered mental status and breathing rate of 68 per minute B. limp muscle tone and weak or absent distal pulses C. nasal flaring and mottled skin color D. breathing rate of 6 per minute and heart rate of 50 per minute

A

A 34-year-old woman, who is 36 weeks pregnant, is having a seizure. After you protect her airway and ensure adequate ventilation, you should transport her: A. on her left side B. in the prone position C. in the supine position D. in a semisitting position

A

After arriving at the hospital you notice blood on your stretcher. The most appropriate way to disinfect the stretcher is a(n) A. 1:10 ratio of bleach to water solution B. 1:100 ratio of bleach to water solution C. 1:1000 ratio of bleach to water solution D. straight bleach solution

A

During your assessment you find a possible fractured leg and hear the bones grinding together. This is called: A. crepitus B. complete fracture C. subcutaneous fracture D. multi-linear fracture

A

Ethics is best described as: A. the principles of conduct, concerns for what is right or wrong, good or bad B. a code of conduct put forward by a society or some other group such as religion C. the principle of doing good for the patient D. the obligation to treat all patients fairly

A

In legal terms, a tort is a(n) A. civil wrong committed by one individual against another B. criminal wrongdoing C. action by an employee for a workers' compensation claim D. breach of contract

A

In pedestrian versus automobile impacts, which of the following statements are true? A. children often turn toward the impact and are often thrown down and under the vehicle B. adults tend to turn toward the vehicle before impact C. children often turn towards the impact and are often scooped and thrown over the vehicle D. there tends to be no difference in the way adults and children respond in these situations

A

In which of the following situations should you call for immediate assistance? A. you must care for two critical patients with gunshot wounds B. your patient is a 26 year old female in active labor C. your patient is a child with fever who has had a brief seizure D. your partner is needed to stabilize the cervical spine

A

The head of a newborn infant has just been delivered. You should A. suction the baby's mouth and nostrils with a bulb syringe B. push down on the baby's upper shoulder to facilitate the rest of the delivery C. push up on the baby's lower shoulder to facilitate the rest of the delivery D. ventilate the baby with a pediatric bag-valve mask and high-flow oxygen

A

You are treating a patient that has been involved in a motor vehicle accident. You can lift a flap of skin on the patient's head. This type of injury would e referred to as a(n): A. avulsion B. laceration C. evisceration D. puncture

A (A puncture is created by an object that is typically sharp and pointed. A laceration is defined as a jagged cut. An evisceration is typically referred to as organs protruding.)

The mitral or bicuspid valve, A. prevents blood from back flowing into the left atrium B. prevents blood from back flowing into the right atrium C. prevents the blood from back flowing into the lungs D. is located between the left atrium and the right ventricle

A (The mitral or bicuspid valve is located between the left atrium and the left atrium and the left ventricle. It prevents the blood from back flowing into the left atrium.)

Tidal volume is best defined as the: A. volume of air inhaled on a single breath B. volume of air that remains in the upper airway C. total volume of air that the lungs are capable of holding D. volume of air moved in and out of the lungs each minute

A (average is 500 ml)

Low velocity injuries

A knife or other impailed object in the body exerts damage to the immediate area of impact and it's underlying structures. The length of the object used is important

Stroke

A medical injury to brain that is not related to truma

noepinephrine

A neurotransmitter from nerve endings and a hormone from the adrenal gland. It is release in times of stress and is involved in hunger regulation, blood glucose regulation and other body processes.

Standing Orders

A policy or protocol issued by a Medical Director that authorizes EMT-Bs and others to perform particular skills in certain situations.

Extension posturing

A posture in which the pt arches the back and extends the arms straight out parallel to the body. A sign of serious head injury

Rule of nines

A system that assigns percentages to sections of the body, allowing calculation of the amount of skin surface involved in the burn are, a method of estimating the extent of a burn. For an adult, each of the following surfaces represents 9% of the body surface: the head and neck, each upper extremity, the chest, the abdomen, the upper back, the lower back and buttocks, the front of each lower extremity, and the back of each lower extremity. The remaining 1% is assigned to the genital region. For an infant or child the percentages are modified so that 18% is assigned to the head, 14% to each lower extremity.

Tension pneumonthorax

A type of pneumothorax in which air can enter the pleural space but cannot escape via the route of entry. This leads to increased pressure in the pleural space, resulting in lung collapse. The increase in pressure also compresses the heart and vena cavae, which impairs circulation.

During your assessment of a patient's chest you note that the left side of the chest moves opposite to the right. This is called: A. crepitus B. paradigmal motion C. subcutaneous emphysema D. paradoxical movement

D

Soild organs

Spleen liver pacreas kidneys

Sprain vs. Strain

Sprain: overstretching/damaging ligaments STrain: T for TENDON, overstretching/damaging tendon.

Under medical direction, the EMT-Basic may administer epinephrine to a patient with respiratory distress or hypoperfusion resulting from an allergic reaction if the: A. patient has no history of heart disease B. patient is in severe respiratory distress or arrest C. medication has been prescribed for this patient D. medication has been stored in the refrigerator

C

When arriving at the scene of a possible hazardous materials incident, you would identify hazards by A. thoroughly investigating the scene yourself B. interviewing victims and bystanders C. scanning binoculars from a safe distance D. assisting law enforcement officers in the search

C

There are hollow and solid organs in the abdomen. The gallbladder is considered a hollow organ. A ruptured hollow organ such as the gallbladder is most commonly associated with: A. severe bleeding B. inflammation and infection C. gall stones D. appendicitis

B

Pertussis

Acute infectious disease characterized by a cough that has a "whoop" sound; also called whooping cough

What is the best method to assess circulation in an infant? A. palpate the carotid pulse B. palpate the brachial pulse C. palpate the radial pulse D. observe capillary refill time

B

hypo perfusion

Also known as shock (decreased blood flow through an organ, as in hypovolemic shock; if prolonged, it may result in permanent cellular dysfunction and death.)

What is the first thing you should do after receiving orders from the medical direction physician? A.carry out the orders immediately B. repeat the orders exactly as you heard them C. question anything you did not understand D. document the orders in your report

B

When you grab a backboard, you realize that blood remains from an earlier call. With your hand covered in blood, your first step towards decontamination should be: A. contacting your Infection Control Officer B. washing your hands with soap and water C. completing exposure control forms D. going to the emergency department for admission

B

Tertiary phase injuries

Are due to being throw and then landing on the ground similar to being ejected out of a car

Secondary phase injuires

Are due to flying debris propelled by the blast They are usually lacerations fracture burns

Primary phase injuries

Are due to the pressure wave of the blast. Injuries primarly effect the gas-containing organs, such as the lungs, stomach, intestines, inner ears and sinuses. Death may occur from this stage w/o any sign of external injury

A 20-year-old patient was sexually assaulted. The patient states that she is hemorrhaging profusely from the vagina. You should A. do nothing and transport immediately B. apply a sterile sanitary napkin C. pack the vagina with sterile dressings D. have the patient squeeze her leg together and transport immediately

B

A 23-year-old pregnant female is bleeding profusely from her vagina. All of the following actions are appropriate EXCEPT: A. providing high-concentration oxygen B. placing a sanitary napkin in the vagina C. replacing pads as they become soaked D. rapid transport to the hospital

B

A sign or symptom of a predelivery emergency is: A. the mother's skin is dry B. profuse vaginal bleeding C. the presence of a bloody show D. a contraction every 20 minutes

B

A soft-tissue injury that results in a flap of torn skin is referred to as: A. an incision B. an avulsion C. an abrasion D. a laceration

B

After a patient has a seizure, they will have a gradual state of awakening. This phase of a seizure is called: A. the tonic phase B. the postictal phase C. the clonic phase D. an aura

B

All of the following are signs of possible child abuse except: a. the presence of multiple bruises in various stages of healing B. a single, severe traumatic event that occurred for no reason C. injuries inconsistent with the mechanism described D. conflicting histories of the injury from the guardians/parents

B

EMTs should wear high-efficiency particulate air (HEPA) respirators when they are in contact with patients who would have which of the following? A. HIV B. TB C. open wounds D. hepatitis B

B

Emergency care for a responsive 7 year old child with a foreign body airway obstruction includes A. holding the child on your knee and performing back blows B. standing behind the child and performing sub-diaphragmatic thrusts C. placing the child supine on the floor and attempting to see the obstruction D. placing the child supine on the floor and performing abdominal thrusts

B

In which age is capillary refill most useful? A. infants only B. patients less than six years of age C. patients greater than six years of age D. all ages

B

Patients commonly describe heart attack pain as which of the following characteristics? A. like pin needles B. crushing or squeezing C. intermittent (it comes and goes) D. less severe than indigestion

B

Select the correct-sized oral airway for a small child by measuring from the corner of the patient's mouth to what structure: A. pinnea of the ear B. angle of the jaw C. corner of the mouth D. mastoid process

B

The most common electrical rhythm disturbance that results in sudden cardiac arrest is called: A. pulseless electrical activity B. ventricular fibrillation C. ventricular tachycardia D. asystole

B

The patient is a 29-year-old female pregnant with her second child. She is 39-weeks pregnant and a saw a bloody show approximately 4 hours ago. Her contractions are 2 minutes apart and lasting 60 seconds. Transport time is approximately 45 minutes. You should A. protect the airway and monitor vital signs while transporting B. prepare for an imminent on-scene delivery C. position the mother on her left side and begin transport D. notify dispatch of the need for ALS assistance

B

The scene size-up includes all of the following components, except: A. determining scene safety B. applying personal protective gear C. assessing the need for assistance D. evaluating the mechanism of injury

B

Which of the following are signs of early respiratory distress in children and infants? A. breathing rate of less than ten per minute, limp muscle tone, slow or absent heart rate, weak or absent distal pulses B. increased rate of breathing, nasal flaring, intercostal or supraclavicular retractions, mottled skin color, abdominal muscle tone C. altered mental status, respiratory rate of over 60 or under 20 breaths per minute, severe retractions, severe use of accessory muscles D. inability to cough, crying with tears but no sounds, cyanosis, abdominal or chest-wall movements with absent breath sounds

B

Which of the following arryhythmias should be shocked using an AED? A. asystole B. ventricular tachycardia C. pulseless electrial activity D. atrial fibrillation

B

Which of the following is NOT an imminent sign that birthing is going to occur? A. crowning has occurred B. contractions are 5 minutes apart C. the patient feels the infant's head moving down her birth canal D. the patient's abdomen is very hard

B

Which of the following is not considered a significant mechanism of injury for an adult? A. roll-over vehicle B. falls greater than 10 feet C. penetrations of head, chest, or abdomen D. ejection from vehicle

B

Which of the following is the first line of defense in fighting against infectious diseases? A. vaccinations B. hand washing C. using BSI D. using 100% bleach

B

Which patient can safely receive only a focused physical examination rather than a rapid trauma assessment? A. 10 year old male with deformed right lower leg who is responsive after falling off his bicycle B. 20 year old female who complains of severe pain in her ankle after stepping off a curb C. 70 year old male who complains of neck pain after a medium speed car collision D. 30 year old male who is unresponsive but only has minor cuts on the extremities

B

Which rhythm often converts to ventricular fibrillation? A. asystole B. ventricular tachycardia C. atrial fibrillation D. atrial tachycardia

B

Which statement about a patient's right to refuse care is correct? A. a child who is old enough to understand danger is old enough to refuse care and transport B. an adult patient who is of sound mind and understands the consequences can refuse treatment C. no one can authorize treatment or transport for any other individual, regardless of his or her age D. EMTs should leave immediately whenever a patient says that he or she will refuse care

B

You are assessing a 24-year-old male who was involved in a bar room brawl. Witnesses say the patient was struck with a barstool. The patient is unresponsive. You note battle signs on your assessment. This is a sign of: A. intracranial bleeding B. basilar skull fracture C. epidural hematoma D. subdural hematoma

B

You are at the scene of a mass casualty incident. Who is in charge of the overall scene? A. the EMT-B B. the incident commander C. the paramedic D. the medical director

B

You are treating a 35 year old conscious choking victim. The patient suddenly goes unresponsive. Your next step is to: A. attempt to ventilate the patient B. perform CPR C. deliver 5 abdominal thrusts D. deliver 5 chest thrusts

B

You are treating a 62-year-old female patient who is complaining of shortness of breath, chest pain, and is coughing up a frothy sputum. You would suspect this patient has: A. right sided congestive heart failure B. left sided congestive heart failure C. emphysema D. pneumonia

B

You arrive on the scene of a patient who fell from a ladder. You should open the patient's airway by using: A. head-tilt/chin-lift B. jaw thrust maneuver C. neck-lift/head-tilt D. head-tilt/jaw-thrust

B

You arrive on the scene to discover a patient who refuses to allow you to touch her. You feel the patient is in need in treatment and attempt to take her blood pressure. You have just committed: A. an assault B. battery C. negligence D. an act befitting your profession

B

You arrive on the scene to find an unresponsive female patient who is in her car in the garage. The car is still running and the door is closed. Looking through an outside window, your next action would be to: A. attempt and immediate rescue B. open the large garage door and any other outside openings C. wait for the car to run out of gas D. shut the car off immediately and remove the patie

B

You arrive on the scene, finding an unconscious patient. There are no other individuals in the vicinity to give any other information or permission to treat the patient. Even though the patient cannot give you consent to treat him, you begin to do so because of" A. expressed consent B. implied consent C. advanced directives D. emergency consent

B

You can assess a pregnant woman's uterine contractions by placing your gloved hand on A. her abdomen, below the navel B. her abdomen, above the naval C. the right side of her abdomen D. the left side of her abdomen

B

You should never approach a helicopter from the: A. front B. rear C. left D. right

B

Your patient is a 25-year-old female who is severely hypothermic after having plunged into an icy river. Although she was rescued after only a few minutes in the water, she is showing a diminished level of responsiveness. Your care should include A. encouraging the patient to walk in order to improve her circulation B. covering the patient in blankets and turning up the heat in the ambulance C. giving her hot coffee or tea to drink and massaging her extremities D. beginning active rewarming measures under direct medical direction

B

Your patient is behaving abnormally but refuses treatment after falling down a flight of stairs. Before transporting the patient without consent, you should: A. document the presence of any injury B. ask bystanders to serve as witnesses C. have bystanders help talk him into care D. contact medical direction for advice

B

Your patient is vomiting large amounts of bright red blood. The patient does not complain of any pain or tenderness in the abdomen during your assessment. The patient's pulse is 128 and he is having difficulty breathing. His skin is pale, cool and clammy. You noticed he has a jaundiced appearance. You would suspect this patient has: A. pancreatitis B. esophageal varices C. abdominal aortic aneurysm D. appendicitis

B

Your pregnant patient is experiencing contractions. She feels like she needs to move her bowels. This may indicate that A. birth is still some time away B. birth is imminent C. she is going into shock D. the baby is still very high in the birth control

B

Albuterol is a medication administered by: A. injection B. inhalation C. absorption D. orally

B (Albuterol is a bronchodilator)

The appropriate care for a patient with epistaxis is to: A. have the patient lay down and remain calm B. pinch the nostrils and have the patient lean forward C. pinch the nostrils and have the patient lean back D. have the patient lay in a supine position with his head lower than the body

B (Epistaxis is a nose bleed. In these cases you should have the patient pinch their nose and lean forward.)

You arrive to find a 48 year old male complaining that his chest feels heavy. The patient is awake and talking to you. During your assessment, you note that his skin is pale, cool, and clammy. Your first step is to: A. apply your AED B. administer supplemental oxygen C. obtain a past medical history D. assist the patient in taking his neighbor's nitroglycerin

B (Your first step is to administer supplemental oxygen. When treating chest pain, it is important to get oxygen to the patient as soon as possible to help alleviate damage to the heart muscle.)

The process of losing heat through direct contact is called: A. radiation B. convection C. conduction D. evaporation

C

There are three stages of labor. In the second stage of labor. A. labor pains develop C. the cervix becomes dilated C. the baby is born D. the placenta is expelled

C

Fetal Demise

Baby dies in utero and decomposes/mummifies.

Epiglottitis

Bacterial infection of soft tissue in the area above the vocal cords, mistaken for croup. Most commonly in children. High grade fever >104

Post Partum Hemorrhage

Bleeding in excess of 500cc after delivery. FUNDAL MASSAGE and put infant to breast. Place pad over vaginal opening and treat for shock, elevate hips.

Melena

Blood appears in feces, serious sign of internal bleeding.

Pulmonary embolism

Blood clot that has travelled throughout the blood stream and is blocking an artery of the lung.

Conduction

Body heat is lost through direct physical touch of other objects

Radiation

Body heat is lost to the atmosphere or nearby objects without physically touching them

Coup contra coup

Brain hits skull on one side of the head, then the other. Front of the brin hits, then the back of the brain hits.

Syncope/Syncopal Episode

Brief LOC from transiet cerebral hypoxia. Lasts less than 30 seconds, should then be immediately alert and oriented. From vagal stimulation. (Fainting)

A 45 year old male is experiencing chest discomfort. After placing him in his position of comfort, your next action should be to A. ventilate the patient with a nonrebreather mask at 15L/min B. ventilate the patient with the bag-valve-mask at 15L/min C. administer oxygen by nonrebreather mask at 15L/min D. administer oxygen by the nasal cannula at 6L/min

C

A 45-year-old male was involved in a MVA. The patient is complaining of shortness of breath and chest pain. You recognize that the patient has JVD and the patient's pulse is weak and thready. You believe the patient has a pericardial tamponade. You distinguish that this is the problem with the patient by nothing or discovering A. decreased heart sounds B. patient has JVD C. narrowing pulse pressure D. patient is complaining of chest pain

C

Bacterial meningitis has an incubation period of A. weeks to months, depending on the type B. 11-21 days C. 2-10 days D. 2-6 weeks

C

During transport of a patient with a head injury, what assessment factor will provide you with the most information regarding the patient's condition? A. pupil size B. heart rate C. mental status D. blood pressure

C

Following an explosion, a patient is trapped in a collapsed structure and suffers crush injuries to both lower extremities. How would the injuries be classified based on the blast-injury phase? A. primary blast injury B. secondary blast injury C. tertiary blast injury D. none of the above

C

If a person was hit by an object described in the following, which would have the potential to cause the most damage? A. one-pound object traveling at 10 mph B. two-pound object traveling at 20 mph C. one-pound object traveling at 30 mph D. one-pound object traveling at 20 mph

C

Of the following, which body fluid has the most potential to transmit blood-borne diseases? A. nasal discharge B. vomitus C. amniotic fluid D. feces

C

The AED is used to treat patients in: A. asystole B. ventricular tachycardia with a pulse C. ventricular fibrillation D. pulseless electrical activity

C

The electrical impulse generated in the right atrium is called the: A. atrioventricular node B. purkinje fibers C. sinoatriual node D. bundle of his

C

The most serious side effect of alcohol consumption is: A. liver damage B. hepatitis C. death D. loss of consciousness

C

The patient is bleeding severely from the lower leg. You have applied direct pressure and elevated the leg. Your next step is to: A. apply a tourniquet B. apply direct pressure C. apply pressure at the pressure point D. elevate the arm

C

When treating children, all of the following are important considerations except: A. the fact that they are more susceptible to hypothermia B. the padding is needed when immobilizing C. that they should be treated just like adults D. that they have smaller airways

C

Which of the following is NOT a risk for a stroke? A. previous TIA B. diabetes C. hypervolemia D. hypertension

C

Which of the following is the most common cause of suicide? A. alcohol B. drugs C. depression D. the holidays

C

Which of the following is true regarding using a pocket mask to ventilate a nonbreathing patient? A. there is direct contact between the rescuer and the patient's mouth B. oxygen cannot be connected to the mask C. a one-way valve prevents exhaled air from contacting the rescuer D. oxygen levels of 100% may be achieved

C

Which of the following patient characteristics does NOT represent a potential for violence? A. quick irregular movements B. threatening posture C. large, muscular individual D. loud, thunderous voice

C

Which of the following would findings would be most significant during an assessment of a patient with a severe headache? A. pain in both legs B. chest discomfort C. unilateral weakness D. abdominal tenderness

C

You are assessing a conscious patient complaining of chest pain. Which mnemonic will assist you in assessing the patient's current chief complaint? A. DCAP-BTLS B. SAMPLE C. OPQRST D. ABC

C

You are assisting with childbirth in the field. As the infant's head is delivered, you discover that the umbilical cord is wrapped tightly around the neck. You should immediately: A. place the mother on her side and transport B. deliver the infant with the cord wrapped around its neck C. clamp the cord in two places and cut it between clamps D. suction the infant's mouth and nose to clear secretions

C

You are called to a store where a holdup has been committed. Police are already on the scene searching for the gunman. Through the store window, you see the store manager, who has been shot. You should A. enter the store immediately to care for the manager B. leave immediately and seek cover a distance away C. wait until the police tell you it is safe to enter the scene D. request medical direction to determine if you can enter

C

You are called to assist a 60-year-old female who consists of a severe headache. Upon entering the home, you smell a strong odor of natural gas. What is your first action? A. check the patient's airway, breathing and circulation B. insert a nasopharyngeal airway and assess vital signs C. remove the patient from the house to your ambulance D. open all the windows and determine the source of the gas leak

C

You are transporting a patient who has been resuscitated but is still unresponsive. You should check the patient's pulse every: A. 30 seconds B. 1 minute C. 5 minutes D. ten minutes

C

You should not suction a patient's airway for more than 15 seconds because: A. the patient's tongue may be injured B. the suction unit's batter may drain too quickly C. the patient will become hypoxic during this time D. you may cause the patient to vomit

C

Your patient has been stung by a bee, and the stinger is present in the wound. You should attempt to remove it by: A. grabbing it with sterile tweezers B. cutting around it with a knife C. scraping it away with a rigid object D. grabbing it with your fingers

C

Your patient is a 6-year-old female who fell off her bicycle. She has a suspected broken ankle, no respiratory compromise, and no suspected internal injuries. After providing necessary care at the scene, you are transporting the child and her father to the hospital. The father loudly insists that you use your siren and lights en route. You should A. request medical direction in dealing with the father B. request permission from dispatch to use lights and sirens C. refuse, because it may cause an unnecessary hazard D. comply, it will relax the father and comfort the patient

C

Your patient is an 11-month-old female. How can you determine if she was decreased mental status and is responsive to verbal stimuli? A. she will be upset when you take her from her mother's arms B. she will be unable to tell you how old she is if you ask her C. she will attempt to locate her parents' voices when they speak D. she will try to pull away from a painful stimulus on her toe

C

Your patient is an 84-year-old female having difficulty breathing. Her daughter, age 45, is with her. When communicating with this patient, you should assume that she is A. incompetent; speak directly with the daughter B. hard of hearing; speaking extremely slowly and loudly C. competent and able to understand; speak respectfully D. confused explain your treatment clearly to the daughter

C

You arrive on the scene of an incident where a 16 year old male fell approximately 12 feet. That would be considered: A. a trauma alert B. a significant mechanism of injury C. not a significant mechanism of injury D. a case with a load 'n' go patient

C (A patient over the age of 8 is considered an adult for most medical care purposes. A fall of greater than 20 feet is considered a significant injury in adults. If this had been an infant or child, it would have been a significant mechanism of injury since any fall greater than 10 feet would be considered a significant mechanism of injury.)

You are called to the home of a 20-year-old male who is depressed. The patient states he is going to kill himself. Your first step in treating this patient after the scene is safe is to: A. restrain the patient to prevent him from hurting himself B. ask the patient why he wants to do something so stupid C. ask the patient how he plans t kill himself D. turn the scene over to law enforcement

C (At this point, the scene is safe and the patient does not appear to be a threat to anyone. Restraining him would not be appropriate. No matter how you feel about what the patient is going to do, it is inappropriate for you to say that the patient is going to do something stupid. You should continue your treatment of the patient and not turn the scene over to law enforcement.)

Which of the following conditions would most likely cause flushed skin? A. shock B. hypoxia C. exposure to heat D. low blood pressure

C (Flushed or red skin commonly is seen in patients who are exposed to heat. Fever can also cause flushed skin. Shock and low blood pressure generally cause the skin to become pale, and hypoxia causes cyanosis, a bluish-gray tint to the skin.)

Which of the following is NOT true of the treatment of an impaled object? A. remove if blocking the airway B. stabilize in place C. remove to accommodate transport of the patient D. control the bleeding

C (The treatment of a patient with an impaled object includes: securing the object in place, exposing the wound area, controlling bleeding, and using a bulky dressing to help stabilize the object. The only time it is permissible to remove an impaled object is if it is impeding the breathing of a patient.)

Emergency care for an infant when meconium is present in the amniotic fluid includes A. stimulating the infant to cough to expel the meconium B. performing bag-valve-mask ventilation to improve lung compliance C. performing back blows and chest thrusts to remove the meconium D. suctioning and notifying the hospital that meconium was present

D

Prolapsed Cord

Cord comes out before the baby does. Left lateral trendelenberg and use hand to keep baby from compressing it (it should be pulsating, this means the baby is getting oxygen). Can use knee-chest position too.

For which of these procedures should you wear gloves, gown, mask, and protective eyewear? A. performing endotracheal intubation B. performing oral/nasal intubation C. cleaning contaminated instruments D. bleeding control with spurting blood

D

Immediately after delivering a shock with an AED t a patient in cardiac arrest, you should: A. check for a pulse B. check breathing and provide rescue breaths as necessary C. analyze with the AED and shock again if indicated D. do CPR

D

Immediately upon delivery of a newborn's head, you should first: A. dry the face B. cover the eyes C. suction the nose D. suction the mouth

D

In which of the following situations may an EMT-B place their hands in a patient's vagina? A. there is never an incident that allows this B. a breech birth C. to examine the vagina D. to relieve pressure on a prolapsed cord

D

A 29 year old male has taken LSD. The patient appears very anxious and in a panic state. He is showing signs of paranoia. You should: A. agree withe everything the patient says B. restrain the patient C. be aggressive and talk very straight and stern to the patient D. talk the patient down by reassuring the patient

D

A minute after a baby is born, the heart rate is less than 60 beat per minute. You should: A. blow by oxygen B. attempt to stimulate the newborn C. aggressively warm the newborn D. begin chest compressions

D

A sign of generalized cold emergency, or hypothermia, is cool skin on the A. feet or hands B. ears C. face D. abdomen

D

All of the following may be signs of an allergic reaction except: A. headache and dizziness B. rapid, labored breathing C. decreased blood pressure D. decreased heart rate

D

Pink or bloody sputum is often seen in patients with A. pulmonary edema B. anaphylaxis C. allergic reaction D. flu

D

When performing the modified jaw-thrust maneuver to open your patient's airway, which of the following steps is NOT correct? A. stabilize the patient's cervical spine with your forearms B. rest your elbows on the same surface as the patient C. tilt the head by applying gentle pressure to the forehead D. use your index fingers to push the angles of the lower jaw forward

D

When splinting an injured limb, you should assess pulse, motor function, and sensation distal to the injury A. after applying the splint B. before applying the splint C. while applying the splint D. before and after applying the splint

D

Where should you place your hands when using the head-tilt chin-lift maneuver to open an unconscious patient's airway? A. on the nose, with the fingertips pinching it closed, and under the neck B. on the nose, with the fingertips pinching it closed, and on the forehead C. on the forehead, with the other hand under the neck D. on the forehead, with the fingertips of the other hand under the lower jaw

D

Which of the following is a sign of inadequate breathing? A. warm, dry skin B. no audible sounds C. equal chest expansion D. accessory muscle use

D

Which of the following is the correct flow of blood through the heart and lungs? A. inferior/superior vena cavae, lungs, right atrium, right ventricle, left venntricle, aorta B. inferior/superior vena cavae, left atrium, left ventricle, lungs, right atrium, left ventricle, aorta C. inferior/superior vena cavae, lings, aorta, left atrium, left ventricle, right atrium, left ventricle D. inferior/superior vena cavae, right atrium, right ventricle, lungs, left ventricle, aorta

D

Which statement about patient confidentiality is correct? A. patients who are cared for in a public place lose their right to confidentiality B. the right to confidentiality does not apply to minors or to wards of the state C. the patient who signs a statement releasing confidential information relinquishes all rights to privacy D. a patient must sign a written release before an confidential information can be disclosed

D

You are called to the scene of a possible drowning at a local pool. When you arrive on the scene, a bystander is holding the patient at the surface of the water. The patient is unconscious. Your next step is to: A. being rescue breathing B. remove the patient from the pool C. start CPR D. apply cervical and spinal immobilization

D

You are the first on the scene of a mass casualty incident. Your responsibility is to A. treat the first patient with life-threatening injury B. begin removing the patients from the scene C. establish treatment D. begin triaging the patients

D

You are treating a patient suffering from anaphylactic shock. Which of the signs and symptoms would you likely see in this patient? A. urticaria, bradycardia, tachypnea, and stridor B. subcutaneous emphysema, bradycardia, dyspnea, and wheezing C. urticaria, hypertension, tachypnea, and tachycardia D. urticaria, tachycardia, tachypnea, and hypotension

D

You are treating a patient with frostbite. Which of the following actions should be taken? A. break any blisters on the wound B. apply direct heat to the affected area C. rub or massage the affected area D. remove any jewelry from the afflicted limb

D

You arrive on scene to find a patient who had a productive cough for the past two weeks. The patient is complaining of a fever and night sweats. Your next step would be to A. obtain a better medical history B. obtain vital signs C. immediately transport the patient to the hospital D. put the HEPA mask on

D

You arrive on the scene of a tanker truck carrying a hazardous material. You should position your vehicle: A. 2,000 feet from the tanker truck B. uphill C. upwind D. all of the above

D

You arrive on the scene to discover a 65-year-old female lying on the floor of her living room. Your first step in the care of this patient is to" A. begin chest compressions B. apply the AED C. maintain an open airway D. assess level of consciousness

D

You have successfully converted your cardiac arrest patient out of v-fib with your AED. The patient has a pulse and respirations are 6 per minute. You should: A. continue CPR B. monitor patient C. administer 15 LPM via NRB D. continue to assist the patient with ventilations via a BVM and oxygen

D

Your patient has experienced a spontaneous abortion or miscarriage. You should A. remove any tissues from the vagina B. discard any expelled tissues C. place a sanitary napkin in the vagina D. treat the patient for shock

D

Your patient is an 8-year-old female who had a single, brief seizure at school. Her mother arrives at the same time you do and reports that her daughter has seizures often and is under medical treatment. What should you do? A. request advanced life support (ALS) and law enforcement backup so you can transport your child B. administer a dose of the child's prescribed seizure-control medication C. maintain ABCs, monitor vital signs and transport the patient immediately D. ensure a patent airway and request medical direction regarding transport

D

You are treating a patient with a fractured femur. The most appropriate splint to use for this patient would be: A. a ladder splint B. the other leg C. two board splint D. a traction splint

D (A traction splint is used for femur fractures. Ladder splints are used for angulated fractures. Board splints are good to use for extremity fractures.)

You are called to assist s SCUBA diver who is having trouble breathing. The patient states that it started approximately 15 minutes after she surfaced. She complains of chest pain, dizziness, blurred vision and nausea and vomiting. The patient is most likely suffering from: A. pneumothorax B. decompression sickness C. barotrauma D. an air embolism

D (Any of these could be associated with dive incidents but these signs and symptoms are indicative of an air embolism. Divers who suffer from decompression sickness, or the bends, typically do not develop signs and symptoms of the bends for 12 to 24 hours after their dive. Barotrauma usually occurs as the diver is ascending or descending.)

Firefighters have rescued a man from his burning house. He is conscious and in considerably respiratory distress. He has a brassy cough and singed nasal hairs. The most immediate threat to this patient's life is: A. hypothermia B. severe burns C. severe infection D. closure of the airway

D (Because of the signs and symptoms that this patient is exhibiting, you must be immediately concerned with the potential for closure of the airway and be prepared to assist ventilations. Signs of airway burns include respiratory distress, singed nasal hairs, a brassy cough, difficulty breathing and coughing up soot sputum. Infection, the burns themselves and hypothermia should concern you; however, airway problems are the greatest threat to human life.)

Which of the following heat emergencies is considered a true emergency? A. heat cramps B. heat exhaustion C. heat infarction D. heat stroke

D (Heat strokes are considered a true life threatening emergency. A heat stroke typically occurs after a patient as experienced heat cramps and heat exhaustion.)

When assessing a 35 year old patient complaining she is short of breath, you note that her breathing is in excess of 28 times per minute. This is considered: A. neuropnea B. apnea C. bradypnea D. tachypnea

D (Respiratory rates greater than 20 breaths per minute are considered tachypnea. Less than 12 breaths per minute is bradypnea and absent breathing is apnea.)

Brain herniation

Increasing intracranial pressure related to the presence of lg pocket of blood (hematoma)

Respiratory Distress

Indicates that a patient is breathing but is having trouble doing so. May lead to respiratory arrest.

Example of cardiogenic shock

Heart attack Congenital heart failure Abnormal rhythm Beta blockers

Congestive Heart Failure

Heart can't pump blood sufficiently to meet the body's needs. Commonly caused by MI. Pulmonary edema & rales from fluid, sit patient upright (High fowlers, legs down) and administer high flow O2 via NRB mask.

Cardiogenic Shock

Heart failure, heart isn't beating correctly (don't confuse with obstructive shock, there is no outside agent here). Pulmonary edema

3 major heat emergencies, in order of increasing severity

Heat Cramps, Heat Exhaustion, Heat stroke (ALTERED LOC)

HEPA Mask

High Efficiency Particulate Air respirator; used for patients with suspected TB; worn by the EMT provider to prevent airborne transmission

You have been called to the top of a mountain resort for a patient that is complaining of a sudden onset of shortness of breath and coughing. Your assessment of the patient reveals an anxious patient with basilar crackles in his lungs. The patient's vital signs include a heart rate of 136, respirations of 28, and a blood pressure of 176/94 mmHg. What is your field diagnosis of this patient? High altitude pulmonary edema New onset congestive heart failure Pneumonia High altitude congestive heart failure

High altitude pulmonary edema, a result of increased pulmonary pressure and hypertension caused by changes in blood flow at high altitudes.

Tuberculosis

Infectious disease caused by the tubercle bacillus, Mycobacterium tuberculosis. Most commonly affects the respiratory system and causes inflammation and calcification of the system.

Meningitis

Inflammation of meninges (layer protecting spinal cord & brain). Flu like symptoms with Nuchal rigidity (stiff neck) and body rash of anterior trunk

You are assessing a 53-year-old female who neighbors discovered acting funny. The patient appears to be fatigued and confused and exhibiting slurred speech. She is breathing and has a pulse and skin that is cool and mottled. Vital signs are pulse 68 beats per minute, respirations 14 per minute, blood pressure 108/60 mmHg, and temperature 92.7 degrees Fahrenheit. There are no signs of trauma to the patient. Patient prescriptions of Verapamil, Digoxin, Synthroid, and nitroglycerin are found in the bedroom. Allergies are unknown. The temperature in her apartment is 55 degrees Fahrenheit. Based on the assessment findings, which of the following conditions is most likely a contributing factor to the patient's condition? Hypothyroidism Hypertension Atrial fibrillation Seizure

Hypothyroidism.

Types of shock

Hypovolemic Distributive Cardiogenic Obstructive

Subluxation

INCOMPLETE join location/disruption

Cavacation

Is the cavity that is formed by a pressure way

Involuntary

Mentally incompetent person

aed shock then pulse check?

No aed shock then 2min CPR then pulse check

Heat exhaustion

Occurs when the body's cooling mechanism have been expended Skin is normal to cool in temp Pale or Gregory in color And sweaty

palmar

Palm of hand

Transient Ischemic Attack

Precedes 1/3 of all strokes. Blockage or spasm of blood vessel (clots), signs & symptoms must be gone within 24 hours.

Pre-eclampsia

Pregnancy Induced Hypertension. Sudden weight gain, headache, edema. Left lateral positioning. Any pregnant woman with high BP

Breech Delivery

Presenting part=buttocks or both legs. Insert sterile gloved hand to create airway for infant. Delivery can be similar to normal birth.

Supine hypertensive syndrome

Pressure on the vena cava by baby when mother is in supine position

Pericardial Tamponade

SAME THING AS CARDIAC TAMPONADE. Peri=around & cardial=heart. Rapid accumulation of fluid around the heart, leading to pressure around the heart so it can't expand and pump adequately. BECKS TRIAD

Ruptured uterus sign

Searing or tearing feeling in abdomen

Examples if distributive shock

Septic Infection that causes the vessels to dilate Neurogenic Spinal cord injury cause vessels to dilate Anaphylactic Body wide vessel dilation

Glucagon

a hormone secreted by the pancreas

Advanced Directive

a legal document prepared by a living, competent adult to provide guidance to the health care team if the individual should become unable to make decisions regarding his or her medical care; may also be called a living will or durable power of attorney for health care

Emergency Move

a move made when there is an immediate danger to the patient.

Hypothalamus

a neural structure lying below the thalamus; directs eating, drinking, body temperature; helps govern the endocrine system via the pituitary gland, and is linked to emotion

Good Samaritan

a person who voluntarily offers help or sympathy in times of trouble

Tension pneumothorax

a pneumothorax with rapid accumulation of air in the pleural space causing severely high intrapleural pressures with resultant tension on the heart and great vessels

The wall that separates the left and right sides of the heart is called the: • A:pericardium. • B:carina. • C:mediastinum. • D:septum.

The correct answer is D; Reason: The septum is the wall that separates the left and right sides of the heart. There is a septum for both the atria and the ventricles. The carina is the bifurcation point of the trachea, and the mediastinum is the space between the lungs in which the heart, great vessels, and a portion of the esophagus lie. The pericardium is the sac that surrounds the heart and contains pericardial fluid.

Status epilepticus

a seizure that lasts longer than 10 minutes or a seizure that begins again after one seizure stops without that patient regaining consciousness. It is a dire medical emergency, as the patient may sustain bone fractures, airway occlusion, and possible death.

Syncope

a sudden, and generally temporary, loss of consciousness and postural tone, due to inadequate flow of oxygenated blood to the brain (fainting)

Eclampsia

a toxic condition characterized by convulsions and possibly coma during or immediately after pregnancy

Embolic stroke

a type of ischemic stroke that causes a clot to travel to the brain, mostly from the left side of the heart

Increasing Intercranial Pressure

Usually from intercranial hematoma or cerebral edema, can crush brain tissue and cause great damage. CUSHING'S REFLEX/TRIAD. Altered LOC, Dilation of ipsilateral or both pupils, contralateral hemiparesis (weakness on one side of the body)

Glucose and sodium bring what with them?

Water

Tonic Phase

Stiffening of the limbs during a grand mal seizure. Breathing may cease or decrease

Hollow organ in abdominal

Stomach Gallbladder Urinary bladder Ureters Internal urethra Fallopian tubes Intestines

Glycogen

Storage form of glucose

Can I aed on 1-8 year olds

Yes but prefer to use less electrical dose If u can't it is ok

A football player complains of severe neck pain and tingling in his arms and legs after being tackled. He is conscious and alert, has a patent airway, and is breathing adequately. He is in a supine position and is still wearing his helmet, which is tight-fitting. The MOST appropriate treatment for this patient includes: • A:manually stabilizing his head with his helmet still on, removing the face mask, administering high-flow oxygen, placing him onto a long backboard, and restricting spinal motion with straps and a lateral head stabilizer. • B:carefully removing his helmet, manually stabilizing his head, applying a cervical collar, administering high-flow oxygen, placing him onto a long backboard, and restricting spinal motion with straps and a lateral head stabilizer. • C:manually stabilizing his head, leaving his helmet on, applying a vest-style spinal immobilization device, placing him onto a long backboard, and restricting spinal motion with straps and a lateral head stabilizer. • D:manually stabilizing his head, carefully removing his helmet, administering high-flow oxygen, applying a cervical collar, placing him onto a long backboard, and restricting spinal motion with straps and a lateral head stabilizer.

The correct answer is A; Reason: A helmet that fits well prevents the patient's head from moving and should be left on, provided (1) there are no impending airway or breathing problems, (2) it does not interfere with your assessment and treatment of airway or ventilation problems, and (3) you can properly immobilize the spine. You should also leave the helmet on if there is any chance that removing it will further injure the patient. The mask on most sports helmets can be removed, without affecting helmet position or function, by using a trainer's tool designed for cutting retaining clips or unscrewing the retaining clips for the face mask. Your patient has severe neck pain and tingling in his extremities; these are obvious signs of a spinal injury. However, he is conscious and alert, has a patent airway, and is breathing adequately. Thus, the safest approach is to remove the face mask as previously described, apply high-flow oxygen, and restrict spinal motion by securing him to a long backboard. A vest-style device is more suitable for seated patients; it is impractical to use on supine patients.

Which of the following actions is MOST important when immobilizing a patient with a suspected spinal injury? • A:Select and apply the appropriate size of extrication collar. • B:A vest-style immobilization device should routinely be used. • C:Check range of motion by asking the patient to move the head. • D:Secure the patient's head prior to immobilizing the torso.

The correct answer is A; Reason: Although an extrication (cervical) collar is not the sole means of immobilizing the patient's spine, it must be of the appropriate size in order to minimize flexion/extension of the patient's neck. When immobilizing any patient, whether with a vest-style device or long spine board, the head is immobilized after the torso. Immobilizing the head first may cause potential cervical spine compromise as the torso is immobilized. Determining whether to use a vest-style immobilization device or a long spine board is based on the patient's condition. Obviously, you should never ask a patient with a potential spinal injury to move his or her head around.

Your actions at the scene of a critically injured patient who was shot during a robbery should include: • A:caring for the patient while manipulating the scene minimally. • B:performing a primary assessment only. • C:providing care when the police authorize you to. • D:starting immediate care as you would with any other patient.

The correct answer is A; Reason:After ensuring your own safety, your priority while functioning at a crime scene is to provide care to the patient. However, you should make a reasonable effort to avoid manipulating the scene in order to preserve potential evidence. If furniture or other objects do not need to be moved to gain access to the patient and provide adequate working space, they should be left in place. Conversely, if any obstacles impede your care of the patient, they must be moved as needed. Seeking law enforcement approval before treating a critically injured patient would clearly waste valuable time.

Which of the following is an abnormal finding when using the Cincinnati stroke scale to assess a patient who presents with signs of a stroke? • A:One arm drifts down compared with the other side. • B:Both arms drift slowly and equally down to the patient's side. • C:The patient's face is symmetrical when he or she smiles. • D:One of the pupils is dilated and does not react to light.

The correct answer is A; Reason: The Cincinnati Stroke Scale is used to assess patients suspected of experiencing a stroke. It consists of three tests: speech, facial droop, and arm drift. Abnormality in any one of these areas indicates a high probability of stroke. To test arm drift, ask the patient to hold both arms in front of his or her body, palms facing upward, with eyes closed and without moving. Over the next 10 seconds, observe the patient's arms. If one arm drifts down toward the ground, you know that side is weak; this is an abnormal finding. To test for facial droop, have the patient smile, showing his or her teeth. The face should be symmetrical (both sides of the face should move equally). If only one side of the face moves well, you know that something is wrong with the part of the brain that controls the facial muscles. You should assess the pupils of a patient with a suspected stroke; however, this is not a component of the Cincinnati Stroke Scale.

All of the following are components of the Cincinnati stroke scale, EXCEPT: • A:pupil size. • B:arm movement. • C:facial droop. • D:speech pattern.

The correct answer is A; Reason: The Cincinnati Stroke Scale, which tests speech, facial droop, and arm drift, is a reliable tool that should be used during your assessment of a patient suspected of having had a stroke. To test speech, ask the patient to repeat a simple phrase; he or she should be able to repeat the phrase without slurred speech. If the patient cannot repeat the phrase, or repeats it with slurred speech, a stroke should be assumed. To test facial movement, ask the patient to smile and show his or her teeth; both sides of the face should move symmetrically (equally). If only one side of the face is moving well (facial droop), a stroke should be assumed. To test arm movement, ask the patient to hold both arms in front of his or her body, palms up, with eyes closed and without moving. Over the next 10 seconds, watch the patient's hands. If one arm drifts down toward the ground, you know that side is weak. You should assess pupillary size, equality, and reactivity in any patient with a neurological disorder; however, this is not a part of the Cincinnati Stroke Scale.

Which of the following interventions would the EMT be the LEAST likely to perform while attempting to resuscitate a cardiac arrest patient? • A:Insertion of a supraglottic airway device. • B:Assisting a paramedic with intubation • C:Rhythm analysis with the AED • D:Ventilation with a bag-mask device

The correct answer is A; Reason: The insertion of advanced airway devices (eg, endotracheal [ET] tube, multilumen airway, supraglottic airway) is generally outside the EMT's scope of practice. However, he or she may be asked to assist a paramedic in the placement of such devices. For example, the paramedic may ask the EMT to retrieve the appropriate equipment or preoxygenate the patient before he or she inserts the device. Operation of the AED and ventilating with a bag-mask device are within the EMT's scope of practice.

You are assessing an elderly man with respiratory distress. He is coughing up bloody sputum and has an oxygen saturation of 85%. You auscultate his breath sounds and hear coarse crackles in all lung fields. This patient MOST likely has: • A:congestive heart failure. • B:severe bacterial pneumonia. • C:acute onset emphysema. • D:decompensated asthma.

The correct answer is A; Reason: This patient's signs and symptoms are classic for left-sided congestive heart failure and pulmonary edema. As the left side of the heart weakens, in which case it can no longer effectively pump blood, blood backs up into the lungs, resulting in pulmonary edema. As pulmonary edema gets worse, the patient begins coughing up pink, frothy sputum (hemoptysis). The presence of fluid in the lungs impairs the exchange of oxygen and carbon dioxide, resulting in hypoxemia and a low oxygen saturation (SpO2). Auscultation of the patient's lungs often reveals coarse crackles, which indicates the presence of fluid. Emphysema is a chronic respiratory disease, not an acute one. Furthermore, hemoptysis is not a common finding with emphysema. Likewise, patients with decompensated asthma often have markedly diminished lung sounds owing to severe bronchospasm; hemoptysis and crackles are not common. Bacterial pneumonia can cause respiratory distress; however, it usually presents with fever and diminished breath sounds to a localized area of a lung (for example, the left lower lobe).

Ventilation is defined as the: • A:movement of air into and out of the lungs. • B:volume of air inhaled into the lungs in a single breath. • C:elimination of carbon dioxide from the body. • D:exchange of oxygen and carbon dioxide at the cell level.

The correct answer is A; Reason: Ventilation is defined as the movement of air into and out of the lungs. During negative-pressure ventilation (normal breathing), the diaphragm and intercostal muscles contract, which increases the vertical and horizontal dimensions of the chest cavity. As a result, a vacuum is created in the chest and air is drawn into the lungs. Positive-pressure ventilation is the act of forcing air into the lungs (ie, bag-mask ventilation). The volume of air inhaled or exhaled in a single breath is called tidal volume. The exchange of gases between the body and its environment is called respiration; therefore, the exchange of oxygen and carbon dioxide at the cell level is called cellular (internal) respiration. During pulmonary (external) respiration, oxygen and carbon dioxide are exchanged in the lungs; oxygenated blood returns to the left side of the heart and carbon dioxide is eliminated from the body during exhalation.

Which of the following is the MOST correct technique for ventilating an apneic adult who has a pulse? • A:Deliver each breath over 1 second at a rate of 10 to 12 breaths/min. • B:Hyperventilate at a rate between 20 and 24 breaths/min. • C:Ventilate at a rate of 15 breaths/min and look for visible chest rise. • D:Deliver each breath over 1 second at a rate of 8 to 10 breaths/min.

The correct answer is A; Reason: When ventilating an apneic adult who has a pulse, deliver each breath over a period of 1 second, at a rate of 10 to 12 breaths/min (one breath every 5 to 6 seconds), while observing for visible chest rise. A ventilation rate of 8 to 10 breaths/min (one breath every 6 to 8 seconds) is appropriate for infants (except newborns), children, and adult patients in cardiac arrest after an advanced airway device (eg, ET tube, multilumen airway, supraglottic airway) has been inserted. Do NOT hyperventilate the patient; doing so may impede blood return to the heart, thus reducing cardiac output, secondary to hyperinflation of the lungs. Hyperventilation also increases the incidence of gastric distention, regurgitation, and aspiration.

The transition phase of the pediatric assessment process would be the LEAST appropriate if: • A:the child is unstable and needs rapid transport. • B:you determine that the child's condition is stable. • C:a parent is available to help keep the child calm. • D:your transport time is greater than 30 minutes.

The correct answer is A; Reason:If the child's condition does not require immediate transport, the transition phase can allow the infant or child to become familiar with you and your equipment. This will help to alleviate the child's anxiety, allowing you to perform a more thorough and accurate assessment. Remember that sick or injured children are afraid and do not understand why you are there and what you are doing. As a result, they are less likely than an adult to trust you. The transition phase will facilitate the trust-building process between you and the child.

In the patient with diabetes, insulin shock typically presents with: • A:clammy skin and a rapid onset. • B:dry skin and a slow onset. • C:dry skin and a rapid onset. • D:clammy skin and a slow onset.

The correct answer is A; Reason:In the patient with diabetes, insulin shock (hypoglycemic crisis) presents with cool, clammy skin and a rapid onset. The brain is critically dependent on glucose and responds quickly when the body is in short supply. Diabetic coma (hyperglycemic crisis) typically presents with warm, dry skin and a slow onset, sometimes occurring over a period of days.

Your assessment of a mother in labor reveals that a fetal limb is protruding from the vagina. Management of this situation should include: • A:positioning the mother with her hips elevated, administering high-flow oxygen, and providing transport. • B:applying gentle traction to the protruding limb to remove pressure of the fetus from the umbilical cord. • C:positioning the mother in a semi-Fowler's position, administering oxygen, and providing transport. • D:giving the mother 100% oxygen and attempting to manipulate the protruding limb so that delivery can occur.

The correct answer is A; Reason:Limb presentations represent a dire emergency for the newborn and do not spontaneously deliver in the field. You should position the mother in a manner so that her hips are elevated in an attempt to slide the infant slightly back into the birth canal and remove pressure from the umbilical cord. Administer high-flow oxygen to the mother, cover the protruding limb with a sterile sheet (or any clean sheet, if a sterile sheet is not available), and transport immediately. Do NOT pull on the protruding limb as this may cause injury to the newborn.

A 29-year-old woman, who is 38 weeks pregnant, presents with heavy vaginal bleeding, a blood pressure of 70/50 mm Hg, and a heart rate of 130 beats/min. She is pale and diaphoretic, and denies abdominal cramping or pain. Her signs and symptoms are MOST consistent with a/an: • A:placenta previa. • B:abruptio placenta. • C:ruptured ectopic pregnancy. • D:ruptured ovarian cyst.

The correct answer is A; Reason:Of the conditions listed, placenta previa would be the least likely to present with abdominal pain, although some patients may have pain or cramping. Placenta previa is a condition in which the placenta develops over and covers some or all of the cervix. As the cervix dilates, the vasculature that attaches the placenta to the uterine wall tears, resulting in vaginal bleeding that is often severe enough to cause shock. By contrast, abruptio placenta is a condition in which the placenta prematurely separates from the uterine wall; it is characterized by tearing abdominal pain, heavy vaginal bleeding, and shock. Placenta previa and abruptio placenta occur during the later stages of pregnancy. A ruptured ovarian cyst typically causes lower abdominal pain, often unilateral. Ectopic pregnancy, a condition in which the egg implants and grows outside the uterus (usually in a fallopian tube), is a first trimester condition; it is typically discovered between 8 and 10 weeks of pregnancy. If the ectopic pregnancy ruptures, the patient often presents with a sudden stabbing pain in the lower abdomen and shock due to intraabdominal hemorrhage.

Propoxyphene (Darvon) is categorized as what type of drug? • A:Narcotic • B:Amphetamine • C:Benzodiazepine • D:Barbiturate

The correct answer is A; Reason:Propoxyphene (Darvon) is in the narcotic (opiate) class of drugs. Other narcotics include heroin, morphine, codeine, and meperidine (Demerol). Phenobarbital is an example of a barbiturate. Drugs such as Valium, Klonopin, and Xanax are benzodiazepines. Amphetamines include drugs such as Ritalin, Adderall, and Focalin.

Upon arriving at the scene of a crash involving a large truck, you immediately note the presence of an orange placard on the side of the tank that the truck is pulling. This indicates that the vehicle is carrying a/an ______________ agent. • A:explosive • B:flammable • C:corrosive • D:radioactive

The correct answer is A; Reason:The color of a warning placard indicates the general classification of agent being carried, while the United Nations (UN) number in the center of the placard indicates the exact agent being carried. For example, a red placard bearing the UN number 1203 indicates gasoline; red indicates the classification (flammable), and 1203 indicates the exact agent (gasoline). Orange placards indicate explosive or blasting agents, placards that are half yellow and half white indicate radioactive agents, and black placards indicate corrosive agents. Use your emergency response guidebook (and binoculars, if appropriate) to identify both the classification and exact agent involved.

The appropriate technique for performing two-rescuer CPR on a 4-year-old child includes: • A:15 compressions to 2 ventilations, compressing the sternum with the heel of your hand, and ventilating until visible chest rise occurs. • B:30 compressions to 2 ventilations, compressing the chest one third the depth of the chest, and delivering each breath over 1 second. • C:15 compressions to 2 ventilations, compressing the sternum with your thumbs, and delivering at least 100 compressions per minute. • D:30 compressions to 2 ventilations, compressing the sternum with the heel of both hands, and delivering each breath over 1 to 2 seconds

The correct answer is A; Reason:When performing two-rescuer CPR on an infant (less than 1 year of age) or a child (1 year of age to the onset of puberty [12 to 14 years of age]), use a compression to ventilation ratio of 15:2. Compress the chest one-third the depth of the chest (about 1 1/2" for infants; about 2" for children), at a rate of at least 100/min, and allow the chest to fully recoil in between compressions. Deliver each breath over 1 second, just enough to produce visible chest rise, and allow complete exhalation. For a child, use the heel of one or both hands to compress the chest, depending on the size of the child. For two-rescuer infant CPR, use the tips of your thumbs to compress the chest (two-thumb, encircling hands technique); the two-finger technique may be used for one-rescuer infant CPR. A 30:2 compression to ventilation ratio is used for all adult and one-rescuer CPR.

A 60-year-old woman presents with acute respiratory distress. She is conscious and alert, but restless. Her respiratory rate is 26 breaths/min with adequate chest expansion, her breath sounds are clear to auscultation bilaterally, and her oxygen saturation is 90%. Which of the following is the MOST appropriate treatment for this patient? • A:A nasopharyngeal airway and supplemental oxygen • B:Supplemental oxygen with a nonrebreathing mask • C:A nasal cannula with the flowmeter set at 4 to 6 L/min • D:A nasopharyngeal airway and assisted ventilations

The correct answer is B; Reason: Although the patient is restless—a sign of hypoxemia—she is conscious and alert and able to maintain her own airway; therefore, an airway adjunct is not needed at this point. Furthermore, her respirations, although increased in rate, are producing adequate tidal volume as evidenced by adequate chest expansion. Therefore, she is not in need of assisted ventilation at this point. Considering her oxygen saturation of 90%, the most appropriate treatment would be to administer high-flow oxygen with a nonrebreathing mask and closely monitor her for signs of inadequate breathing (ie, shallow breaths [reduced tidal volume], decreased level of consciousness, cyanosis). An acutely hypoxemic patient requires more oxygen than a nasal cannula can provide.

When assessing distal circulation in a patient with a swollen deformed femur, you should: • A:touch his foot with a blunt object. • B:palpate for a dorsalis pedis pulse. • C:ask the patient to wiggle his toes. • D:assess the pulse behind the knee.

The correct answer is B; Reason: Care for a musculoskeletal injury includes assessing distal circulatory, sensory, and motor functions before and after applying a splint. In the case of a femur injury, the dorsalis pedis (pedal) pulse, located on top of the foot, is the most distal pulse relative to the injury. If a pedal pulse can be palpated, circulation distal to the injury is present. The popliteal pulse is located behind the knee; it is proximal to the pedal pulse. Touching the patient's foot and asking him if he can feel it and asking him to wiggle his toes are assessing sensory and motor functions, respectively, not circulatory function.

A 30-year-old woman has an open deformity to her left leg and is in severe pain. She is conscious and alert, has a patent airway, and is breathing adequately. Your primary concern should be: • A:administering high-flow oxygen. • B:controlling any external bleeding. • C:assessing pulses distal to the injury. • D:covering the wound to prevent infection.

The correct answer is B; Reason: Initial care for any open injury involves controlling external bleeding. Further care involves manually stabilizing the injury site; applying a sterile dressing to keep gross contaminants from entering the wound; assessing distal perfusion (eg, a pulse), motor, and sensory functions; and stabilizing the injury with an appropriate splint. The patient in this scenario is conscious, alert, has a patent airway, and is breathing adequately. Depending on other assessment findings, oxygen may be indicated. Your primary concern, however, should be to ensure that all external bleeding has been controlled.

Initial treatment for a patient with external blood loss depends upon: • A:whether or not hypotension is present. • B:his or her signs and symptoms. • C:his or her past medical history. • D:the amount of estimated blood loss.

The correct answer is B; Reason: It can be difficult to estimate the amount of external blood loss. This is because blood looks different on different surfaces, such as when it is absorbed by clothing or when it has been diluted with water. If possible, you should attempt to estimate the amount of external blood loss; however, the patient's presentation (signs and symptoms) will ultimately direct your treatment. Do not wait to begin treatment until the patient's blood pressure falls. Hypotension following bleeding, internal or external, indicates decompensated shock. Your goal is to recognize shock in its earliest stages (eg, restlessness, anxiety, tachycardia, tachypnea) and begin immediate treatment. Information regarding the patient's past medical history and current medications should be obtained, and may lead you to alter your treatment in some cases. However, it does not direct your initial treatment.

In which of the following circumstances would external bleeding be the LEAST difficult to control? • A:Lacerated brachial artery; BP of 140/90 mm Hg • B:Lacerated femoral vein; BP of 70/40 mm Hg • C:Lacerated carotid artery; BP of 90/50 mm Hg • D:Lacerated jugular vein; BP of 100/60 mm Hg

The correct answer is B; Reason: It is generally less difficult to control external bleeding from a lacerated vein rather than an artery. Unlike arteries, veins are under low pressure. Furthermore, the presence of a low blood pressure (hypotension), which causes less pressure against the vascular wall, would make external bleeding that much easier to control.

Following administration of nitroglycerin to a man with crushing chest pressure, he experiences a significant increase in his heart rate. This is MOST likely the result of: • A:a cardiac dysrhythmia. • B:a drop in blood pressure. • C:coronary vasoconstriction. • D:preexisting hypertension.

The correct answer is B; Reason: Nitroglycerin (NTG) is a vasodilator drug. It dilates not only the coronary arteries, but also other arteries in the body. In some patients, NTG may cause a drop in blood pressure, especially if they are taking other medications that cause vasodilation (eg, erectile dysfunction drugs [Viagra, Levitra, Cialis]). In response to a drop in blood pressure, the nervous system attempts to compensate by increasing the heart rate (tachycardia). Common side effects of NTG include a headache, a burning sensation under the tongue, or a bitter taste in the mouth. Always assess the patient's vital signs, before and after administering nitroglycerin.

A patient with a mild foreign body airway obstruction: • A:has progressive difficulty breathing. • B:is typically not cyanotic. • C:presents with a weak cough. • D:has a low oxygen saturation.

The correct answer is B; Reason: Patients with a mild (partial) airway obstruction are able to move adequate amounts of air, but will have varying degrees of respiratory distress. The patient can cough forcefully, although you may hear wheezing in between coughs. Because the patient is able to move air effectively, the level of oxygen in his or her blood remains adequate; therefore, cyanosis is typically absent. By contrast, the patient with a severe (complete) airway obstruction cannot move air effectively and cannot speak. If a cough is present, it is weak and ineffective. As the level of oxygen in the blood falls, cyanosis develops, oxygen saturation falls, and the patient's level of consciousness decreases. A foreign body airway obstruction, mild or severe, is an acute event that presents with an acute onset of difficulty breathing. Progressive (gradually worsening) difficulty breathing is more consistent with diseases such as congestive heart failure and pneumonia.

Which of the following describes pulseless electrical activity (PEA)? • A:A rapid cardiac rhythm that does not produce a pulse, but responds to defibrillation • B:Any organized cardiac rhythm, slow or fast, that does not produce a palpable pulse • C:The presence of a palpable pulse in the absence of any electrical activity in the heart • D:A disorganized, chaotic quivering of the heart muscle that does not generate a pulse

The correct answer is B; Reason: Pulseless electrical activity (PEA) is a condition in which the heart produces organized electrical activity (slow or fast), despite the absence of a palpable pulse. A disorganized, chaotic quivering of the heart muscle that does not generate a pulse is called ventricular fibrillation (V-Fib), and is treated with defibrillation. Defibrillation is not indicated for patients with PEA; it is only indicated for patients with V-Fib or pulseless ventricular tachycardia (V-Tach). If the AED gives a no shock message, and the patient is still pulseless, he or she is either in asystole or PEA, neither of which are shockable rhythms.

An elderly woman with COPD presents with a decreased level of consciousness, cyanosis to her face and neck, and labored respirations. Her pulse is rapid and weak and her oxygen saturation is 76%. You should: • A:apply oxygen via nasal cannula and reassess her respiratory status. • B:assist her ventilations with a bag-mask device and high-flow oxygen. • C:avoid high-flow oxygen because this may cause her to stop breathing. • D:insert a nasal airway and give her oxygen via a nonrebreathing mask.

The correct answer is B; Reason: The patient in this scenario is experiencing an exacerbation (worsening) of her COPD. Her decreased level of consciousness; cyanosis; weak, rapid pulse; low oxygen saturation (SpO2); and labored breathing clearly indicate that she is not breathing adequately. Therefore, you should assist her ventilations with a bag-mask device and high-flow oxygen; if you don't, she will continue to deteriorate, possibly to the point of cardiac arrest. If needed, insert a nasal airway adjunct to help keep her airway open. Regardless of the patient's history of COPD, you must NOT withhold oxygen from her. Respiratory depression in COPD patients who receive high-flow oxygen is highly uncommon. Death due to hypoxia, however, is very common.

Which of the following airway sounds indicates a lower airway obstruction? • A:Stridor • B:Wheezing • C:Gurgling • D:Crowing

The correct answer is B; Reason: Wheezing is a whistling sound that results from narrowing and/or inflammation of the bronchioles in the lungs and is an indicator of a lower airway disease (ie, asthma, bronchiolitis). Crowing and stridor are both high-pitched sounds that indicate an upper airway disease or obstruction (ie, croup, epiglottitis, foreign body obstruction), and gurgling indicates secretions in the oropharynx.

A 21-year-old man partially amputated his right arm when the chainsaw he was using to trim trees slipped. You can feel a weak radial pulse and his arm is cool to the touch. Dark red blood is flowing heavily from the wound. You should: • A:control the bleeding, manipulate the arm to improve circulation, and apply a splint. • B:apply bulky compression dressings to the wound and splint the extremity. • C:carefully pack sterile dressings into the wound and fully splint the extremity. • D:apply a tourniquet proximal to the injury and tighten it until the bleeding stops.

The correct answer is B; Reason: When caring for a partially amputated extremity, control bleeding with bulky compression (pressure) dressings and splint the extremity to prevent further injury. If direct pressure does not immediately control the bleeding, however, a proximal tourniquet should be applied without delay. Never pack dressings into a wound; this may cause further damage and increases the risk of infection. Although your patient's radial pulse is weak, it is present and indicates blood flow distal to the injury. Do not manipulate his arm; doing so may lacerate or compress an artery and compromise distal circulation.

Which of the following questions would be MOST appropriate to ask when assessing a patient with chest pain? • A:Does the pain radiate to your arm? • B:What does the pain feel like? • C:Is the pain worse when you take a deep breath? • D:Would you describe the pain as sharp?

The correct answer is B; Reason: When questioning any patient about any type of pain, you should avoid asking leading questions that can simply be answered yes or no. To obtain the most reliable assessment, open-ended questions should be asked to allow the patient to describe the quality of the pain in his or her own words.

A 56-year-old man is found to be pulseless and apneic. His wife states that he collapsed about 5 minutes ago. As your partner gets the AED from the ambulance, you should: • A:ask the wife if the patient has a living will. • B:begin CPR, starting with chest compressions. • C:open the airway and give 2 rescue breaths. • D:provide rescue breaths until the AED is ready

The correct answer is B; Reason: When you arrive on scene and determine that a patient is in cardiac arrest, you should immediately begin CPR, starting with chest compressions. Perform 30 chest compressions and then open the airway and deliver 2 rescue breaths. Chest compressions are a crucial part of cardiopulmonary resuscitation and must be started without delay. Apply the AED as soon as it is available. In the interest of this patient, whose arrest interval is short, you should begin resuscitative efforts immediately. In some cases, it is appropriate to inquire about the presence of a living will; however, this should be done after resuscitative efforts have begun.

A middle-aged female with a history of hypertension and high cholesterol complains of chest discomfort. She asks you to take her to the hospital where her personal physician practices, which is 15 miles away. Her blood pressure is 130/70 mm Hg, pulse is 84 beats/min and regular, and respirations are 18 breaths/min and unlabored. Which of the following actions is clearly NOT appropriate for this patient? • A:Contacting her physician via phone • B:Allowing her to walk to the ambulance • C:Taking her to her choice hospital • D:Giving oxygen via nasal cannula

The correct answer is B; Reason: You should NEVER allow a patient with a possible cardiac problem to walk to the ambulance. This causes exertion, which increases cardiac oxygen consumption and demand and could worsen his or her condition. Give the patient oxygen in a concentration sufficient to maintain his or her oxygen saturation equal to or greater than 94%. In general, you should transport patients to the hospital of their choice. However, transport to a closer hospital should be considered if you believe the patient is unstable or is at high risk for becoming unstable. If necessary, consult with the patient's physician via phone to determine if he or she thinks the patient should go to a closer hospital.

How should you classify a patient's nature of illness if he or she has a low blood glucose level, bizarre behavior, and shallow breathing? • A:Behavioral emergency • B:Altered mental status • C:Respiratory emergency • D:Cardiac compromise

The correct answer is B; Reason: The nature of illness (NOI) is the medical equivalent to mechanism of injury (MOI). Altered mental status should be the suspected NOI in any patient with any fluctuation in level of consciousness, which can range from bizarre behavior to complete unresponsiveness. Causes of an altered mental status include hypo- or hyperglycemia, head trauma, stroke, behavioral crises, drug overdose, and shock, among others.

A middle-aged man was found floating facedown in a small pond. When you arrive at the scene, bystanders are present, but nobody has removed him from the water because they thought he was dead. After reaching the victim, you should: • A:float a buoyant backboard under him, remove him from the pond, and begin rescue breathing. • B:move him to a supine position by rotating the entire upper half of his body as a single unit. • C:stabilize his head as you remove it from the water and open his airway by tilting his head back. • D:grab him by his clothing, remove him from the pond, and assess for breathing and a pulse.

The correct answer is B; Reason:After safely reaching a drowning victim, you should first turn him to a supine position by rotating the entire upper half of the body as a single unit; protect the cervical spine as you do this because a spinal injury cannot be ruled out. Open the patient's airway, without manipulating the neck, and begin rescue breathing while still in the water. Float a buoyant backboard under the patient, secure him to it, and remove him from the water. After removing the victim from the water, assess for a pulse. If the victim is pulseless, begin CPR, dry him off, and apply the AED as soon as possible.

During the triage process, which of the following injuries or conditions would classify a patient as a high priority? • A:Partial-thickness burns with no respiratory difficulty • B:A large avulsion to the arm and an altered mental status • C:Pulselessness and apnea • D:Unilateral femur fracture and tachycardia

The correct answer is B; Reason:During triage, patients with an altered mental status, who are in shock, or who have problems with airway, breathing, or circulation, are potentially salvageable and are given immediate priority. Patients who are pulseless and apneic have low priority in a mass-casualty situation. If you focus your efforts on cardiac arrest patients, who will most likely not survive anyway, patients who could have potentially been saved will die as well. Remember, the goal of triage is to provide the greatest good for the greatest number of patients.

You are giving a presentation to a group of laypeople on the importance of calling EMS immediately for cardiac arrest patients. What point should you emphasize the MOST? • A:Laypeople are incapable of providing adequate CPR. • B:CPR and defibrillation are key factors in patient survival. • C:Cardiac drug therapy is the most important EMS treatment. • D:Rapid transport significantly reduces patient mortality.

The correct answer is B; Reason:Early high-quality CPR and defibrillation are the most crucial initial treatments to provide to a patient in cardiac arrest. Adequately performed CPR can keep the heart and brain oxygenated, thus increasing the chance of defibrillation success. Ventricular fibrillation (V-Fib) is the most common initial dysrhythmia seen in adult patients with sudden cardiac arrest and requires prompt defibrillation. Untreated V-Fib will rapidly deteriorate to asystole, the mortality rate from which is very high. You should also advise the audience that compression-only CPR has been linked to patient survival.

In general, you should avoid rewarming a frostbitten body part in the field if: • A:arrival at the emergency department will be delayed. • B:the affected part could refreeze after rewarming. • C:you are unable to obtain water that is at least 120°F. • D:a paramedic is not present to administer analgesia.

The correct answer is B; Reason:Frostbitten body parts should not be rewarmed if there is a chance that they could refreeze after you have rewarmed them. If an extremity thaws and then refreezes, the amount of tissue and cellular damage may be worse than the damage caused by the initial freezing. A delay in getting the patient to the emergency department warrants rewarming. If rewarmed, the extremity should be immersed in water that is 105° to 112°F. Analgesia would certainly be a comfort to the patient, although its absence does not negate rewarming a frostbitten body part in the field.

Which of the following statements regarding the use of an escort vehicle when en route to an emergency call is correct? • A:An escort vehicle will allow you to arrive at the scene quicker. • B:An escort vehicle should be used only if you are unfamiliar with the patient's location. • C:To avoid getting separated from the escort vehicle, you should closely follow it. • D:With an escort vehicle, the risk of an accident at an intersection is reduced significantly.

The correct answer is B; Reason:Generally, escort vehicles should not be used when responding to an emergency scene. The biggest danger of using an escort occurs at intersections, which is where most ambulance crashes occur. Drivers may yield to the escort vehicle, but may not be prepared for a second vehicle following the escort (wake effect collision). The only time that an escort may be required is when you are unfamiliar with the location of the patient and need assistance getting there. If an escort must be used, you must follow at a safe distance of at least 500 feet.

A 4-year-old girl fell from a third-story window and landed on her head. She is semiconscious with slow, irregular breathing and is bleeding from her mouth and nose. You should: • A:suction her oropharynx, open her airway with the jaw-thrust maneuver, insert an oropharyngeal airway, and assist her ventilations. • B:open her airway with the jaw-thrust maneuver while manually stabilizing her head, suction her oropharynx, and assist her ventilations. • C:open her airway by carefully tilting her head back, suction her oropharynx, and administer high-flow oxygen via nonrebreathing mask. • D:manually stabilize her head, open her airway with the jaw-thrust maneuver, insert a nasopharyngeal airway, and suction her oropharynx.

The correct answer is B; Reason:In any semi- or unconscious patient with a head injury, you should manually stabilize the head and open the airway with the jaw-thrust maneuver. If there are any secretions in the mouth, suction the oropharynx. If possible, insert a simple airway adjunct. The patient in this scenario is semiconscious and likely has an intact gag reflex; therefore, you should not attempt to insert an oropharyngeal airway. Conversely, you should not insert a nasopharyngeal airway in patients with a head injury, especially if there is fluid or blood draining from the nose (a sign of a skull fracture). After ensuring a patent airway, you should turn your attention to the patient's breathing. Slow, irregular breathing will not provide adequate minute volume and should be treated with ventilatory assistance.

When insulin levels in the blood remain high: • A:a fruity odor can be detected on the patient's breath. • B:glucose is rapidly taken out of the blood to fuel the cells. • C:the patient urinates excessively and becomes dehydrated. • D:the cells starve for glucose and begin to metabolize fat.

The correct answer is B; Reason:Insulin is a hormone that promotes the cellular uptake of glucose from the bloodstream. If insulin levels remain high, such as when a diabetic inadvertently takes too much insulin, glucose is rapidly taken out of the blood to fuel the cells. This leads to low circulating blood glucose levels (hypoglycemia) and a condition called insulin shock. By contrast, if insulin levels are too low, such as when a diabetic forgets to take his or her insulin, glucose cannot enter the cells and pools in the bloodstream (hyperglycemia). In the absence of glucose, the cells begin to metabolize fat, which produces ketoacids (diabetic ketoacidosis [DKA]). The respiratory system attempts to eliminate ketoacids from the blood with an increased rate and depth of breathing that is accompanied by a fruity or acetone breath odor (Kussmaul respirations). Hyperglycemia causes the patient to eliminate excess water from the body through urination (diuresis), resulting in dehydration.

You arrive at the scene where a man fell approximately 40 feet and landed on his head. He is unresponsive, has agonal gasps, and a weak carotid pulse. Further assessment reveals an open head injury with exposed brain matter. Upon identifying this patient as an organ donor, you should: • A:recognize that the patient's injuries disqualify him as an organ donor. • B:manage the patient aggressively and provide rapid transport. • C:provide rapid transport only because the patient likely will not survive. • D:request authorization from medical control not to initiate care.

The correct answer is B; Reason:It is unlikely that the patient will survive his injury; however, he is still breathing, albeit very poorly, and has a weak carotid pulse. Therefore, you should begin immediate and aggressive treatment and transport him promptly, just as you would for any other critically injured patient. It would clearly be inappropriate to request authorization to provide no care at all. If it is determined by a physician that he will not survive his injury, his organs can potentially be harvested and save several lives.

A patient with a core body temperature of 94°F would MOST likely present with: • A:joint and muscle stiffness. • B:shivering and pale skin. • C:decreased muscle activity. • D:an altered mental status.

The correct answer is B; Reason:Mild hypothermia is defined as a core body temperature (CBT) between 93.2°F and 96.8°F (34°C and 36°C). Mildly hypothermic patients are usually alert and shivering in an attempt to generate heat through muscular activity. Their skin is typically pale due to the body's constriction of blood vessels at the skin to retain heat. In moderate hypothermia (CBT between 86°F and 93.2°F [30°C and 34°C]), shivering stops and muscular activity decreases. As the CBT falls further, all muscle activity stops. In severe hypothermia (CBT < 86°F [30°C]), vital functions decrease (eg, level of consciousness, BP, pulse, respirations) and the patient is at risk for life-threatening cardiac dysrhythmias. The muscles become rigid and the patient appears stiff. The patient may appear dead; although a pulse is present, it may not be palpable.

When caring for a patient with severe hypothermia who is in cardiac arrest, you should: • A:perform rescue breathing only. • B:perform BLS and transport. • C:avoid using the AED. • D:hyperventilate the patient.

The correct answer is B; Reason:Patients with severe hypothermia (core body temperature < 86°F) who are in cardiac arrest should be managed with basic life support (chest compressions and ventilations), passive external rewarming (ie, removal of wet clothing, applying warm blankets) and rapid transport to the hospital where they can be actively rewarmed. Because cold muscle is a poor conductor of electricity, defibrillation, if indicated, should be limited to 1 attempt until the patient's body temperature has been increased. Cardiac arrest patients with severe hypothermia generally do not respond to defibrillation. Hyperventilation should be avoided as this may increase intrathoracic pressure and impair blood flow back to the heart.

While caring for an emotionally disturbed 30-year-old male, he suddenly becomes violent and needs to be physically restrained. During the restraint procedure, you should: • A:approach the patient from the front and converge on him quickly. • B:maintain communication with him and closely monitor his airway. • C:ensure that at least three people are available to safely restrain him. • D:place him in a prone position and secure straps across his back.

The correct answer is B; Reason:Physical restraint may be necessary if a patient is a threat to him- or herself or others. If it becomes necessary to physically restrain a violent patient, there are certain, specific guidelines that must be followed. Use a minimum of four people to restrain the patient, one assigned to each extremity. Approach the patient from the front and the sides; this will prevent him or her from getting away from you. Somebody, preferably you or your partner, should talk to the patient throughout the restraint process, even if the patient is not listening to you. Never restrain a patient facedown (prone); it is impossible to adequately monitor the patient and this position may inhibit the breathing of an impaired or exhausted patient (positional asphyxia). Use just enough force to effectively restrain the patient; the patient's gender, body size, strength, and mental status can be used to determine the appropriate amount of force. Reassess airway and breathing continuously, regardless of the patient's position, and continue to talk to him or her during transport.

The energy setting for a biphasic AED: • A:increases by 50 joules with each shock. • B:must be manually set by the EMT. • C:is manufacturer specific. • D:is typically 360 joules.

The correct answer is C; Reason: A biphasic AED sends energy in two directions simultaneously. This is advantageous because it produces a more efficient defibrillation than monophasic defibrillation, which sends energy in only one direction, and uses a lower energy setting. The energy setting on a biphasic AED is device specific, but typically ranges from 120 to 200 joules. AEDs are preprogrammed by the manufacturer to deliver a specific amount of energy, which eliminates the need to manually set the energy level. Some biphasic AEDs deliver the same amount of energy for each shock; others may escalate the energy level for each shock. Refer to the manufacturer's documentation that accompanies the AED you use regarding the amount of biphasic energy it delivers.

How does a unified incident command system differ from a single incident command system? • A:In a unified incident command system, a single incident commander is identified and will function as such, regardless of the type of incident • B:In a unified incident command system, plans are made in advance by all agencies that assume a shared responsibility for decision making • C:In a unified incident command system, one agency with several incident commanders has the majority of responsibility for incident management • D:In a unified incident command system, a single person is in charge of the entire incident, even if multiple agencies respond to the scene

The correct answer is B; Reason:Regardless of the type of incident command system (ICS) used, a single incident commander (IC) must be in charge. In a unified incident command system, plans are drawn up in advance by all cooperating agencies that assume a shared responsibility for decision making. The response plan should designate the lead and support agencies for several types of mass-casualty incidents (MCIs). For example, the Haz-Mat team will take the lead in a chemical leak and the medical team will take the lead in a multi-vehicle car crash. Large MCIs often require a unified incident command system. A single incident command system is one in which one person is in charge, even if multiple agencies respond to the scene. It is generally used with incidents in which one agency has the majority of responsibility for incident management. Ideally, it is used for short-duration, limited incidents that require the services of a single agency.

An elderly man is found unresponsive in his kitchen. The patient's wife tells you that her husband has diabetes and that he took his insulin, but did not eat anything. You should suspect: • A:diabetic coma. • B:hypoglycemia. • C:ketoacidosis. • D:hyperglycemia.

The correct answer is B; Reason:Several factors point to a field impression of hypoglycemia. First, the patient is known to have diabetes and second, he took his insulin but did not eat. Because insulin promotes the uptake of glucose into the cells, if the patient does not replace this glucose by eating, the glucose level in the blood will fall to dangerously low levels. Hypoglycemia can also occur when a patient takes too much of his or her insulin. Hyperglycemia occurs when a patient does not take his or her insulin. In such cases, glucose accumulates in the bloodstream and is not able to enter the cell. In the absence of glucose, the cells begin to metabolize fat, which results in the production of ketoacids and leads to hyperglycemic ketoacidosis (diabetic coma, hyperglycemic crisis).

Which of the following is an example of the EMT providing care based on standing orders? • A:Following assessment of a patient with a terminal illness, the EMT notifies the patient's personal physician to determine whether he or she requires any special care. • B:The EMT defibrillates a patient with the automated external defibrillator, directs immediate resumption of CPR, and then contacts medical control for further guidance. • C:After confirming that a patient's blood pressure is adequate, the EMT contacts medical control and requests permission to assist the patient with his prescribed nitroglycerin. • D:Prior to administering oral glucose to a patient with hypoglycemia, the EMT confirms the proper dosage and requests authorization from the base station physician.

The correct answer is B; Reason:Standing orders define certain treatment interventions that the EMT is authorized to perform prior to contacting medical control. For example, the EMT knows that a cardiac arrest patient requires CPR, cardiac rhythm analysis with the AED, and defibrillation (if indicated); contacting medical control before performing these crucial interventions would only waste time and increase the chance of a negative patient outcome. Standing orders generally apply to interventions that the patient requires in order to prevent immediate death. Follow your local protocols regarding which interventions you are authorized to perform prior to contacting medical control.

A known diabetic female is found unresponsive. Her respirations are rapid and shallow; her skin is cool, clammy, and pale; and her pulse is rapid and weak. Which of the following would BEST explain the likely cause of her condition? • A:Failure to take insulin • B:Insulin overdose • C:Excessive eating • D:High blood sugar

The correct answer is B; Reason:The patient has classic signs of insulin shock, a condition caused by a low blood glucose level (hypoglycemia). Common causes of insulin shock include insulin overdose (accidental or intentional), failure to eat (or not eating enough), and excessive exertion. The patient's symptoms are not consistent with diabetic ketoacidosis (DKA), a condition that results from a high blood glucose level (hyperglycemia). Signs of DKA include deep, rapid breathing with a fruity or acetone breath odor (Kussmaul respirations), and warm, dry skin. Unlike insulin shock, which can result from an insulin overdose, DKA can occur if a patient fails to take his or her insulin or takes too little.

When you begin to assess a woman in labor, she states that her contractions are occurring every 4 to 5 minutes and lasting approximately 30 seconds each. Which of the following questions would be MOST appropriate to ask next? • A:Have you had regular prenatal care? • B:At how many weeks gestation are you? • C:Has your bag of waters broken yet? • D:How many other children do you have?

The correct answer is B; Reason:When assessing a patient in labor, the first question you should ask is how far along in the pregnancy she is. If she is at less than 37 weeks gestation (37 to 42 weeks is term), you should prepare for possible resuscitation of the newborn if delivery occurs in the field. Other questions, such as asking if her amniotic sac (bag of waters) has ruptured and whether or not she has received prenatal care, also can help you anticipate and prepare for potential complications. You should also inquire as to how many times the patient has been pregnant, regardless of whether she carried the baby to term (gravida), and the number of times she has carried a baby beyond 28 weeks, regardless of whether it was born dead or alive (para).

When requesting medical direction for a patient who was involved in a major motor-vehicle accident, you should do all of the following, EXCEPT: • A:describe the severity of damage to the patient's vehicle. • B:use radio codes to describe the situation. • C:use proper medical terminology when speaking. • D:question an order if it seems to be inappropriate. You selected A;

The correct answer is B; Reason:When giving a report to medical control or requesting medical direction, you should avoid the use of radio codes, such as "10-50." The physician may not be familiar with such codes. There is clearly less risk of confusion if you use plain English. At the same time, you should use proper medical terminology, especially when describing the patient's injuries. Information regarding the severity of damage to the patient's vehicle is critical information and should be relayed to the physician; this can help him or her appreciate the significance of the situation. Do not be afraid to question an order that is contrary to your training or protocols; the physician may have simply made an error. Repeating an order back to the physician, word for word, will minimize the risk of this occurring.

A 22-year-old female fell on her knee and is in severe pain. Her knee is flexed and severely deformed. Her leg is cold to the touch and you are unable to palpate a distal pulse. You should: • A:carefully straighten her leg until you restore a distal pulse and then apply padded board splints. • B:apply gentle longitudinal traction as you straighten her leg and then apply a traction splint. • C:manually stabilize her injury and contact medical control for further stabilization instructions. • D:place a pillow behind her knee and stabilize the injury by applying padded board splints.

The correct answer is C; Reason: A dislocated knee occurs when the proximal end of the tibia completely displaces from its juncture with the distal femur. In some cases, the popliteal artery behind the knee may be compressed, resulting in compromised distal blood flow. Signs of this include absent distal pulses and a pale extremity that is cool or cold. Manually stabilize the knee and assess for distal pulses. If distal pulses are absent, contact medical control immediately for further stabilization instructions. Medical control may instruct you to make ONE attempt to realign the knee to reduce compression of the popliteal artery and restore distal circulation. If you are unable to restore distal circulation or medical control advises you not to manipulate the injury, splint the knee in the position it was found and transport promptly. Traction splints are contraindicated in any injury to or near the knee

A 30-year-old man sustained partial-thickness burns to the anterior chest and both anterior arms. Based on the Rule of Nines, what percentage of his body surface area has been burned? • A:36% • B:9% • C:18% • D:27%

The correct answer is C; Reason: According to the adult Rule of Nines, the anterior trunk (chest and abdomen) accounts for 18% of the total body surface area (TBSA) and each entire arm accounts for 9%. Therefore, the anterior chest, which is one half of the trunk, would account for 9% of the TBSA, and both anterior arms (4.5% each) would account for 9% TBSA, for a total of 18% TBSA burned.

A 21-year-old male was bitten on the left forearm by a dog. He is conscious and alert and denies any other injuries. An animal control officer is at the scene and has contained the dog. Your assessment of the patient's arm reveals a large avulsion with a peeled back flap of skin. Distal circulation is adequate and the patient is able to feel and move his fingers. In addition to bleeding control, you should: • A:perform a rapid head-to-toe assessment. • B:irrigate the wound for at least 15 minutes. • C:replace the avulsed flap to its original position. • D:apply oxygen via a nonrebreathing mask.

The correct answer is C; Reason: An avulsion is an injury that separates various layers of soft tissue, usually between the subcutaneous layer and fascia, so that they become completely detached or hang as a flap. The patient's injury is isolated and not life-threatening; therefore, a rapid head-to-toe assessment is not indicated. Unless your protocols specify otherwise, oxygen is usually not necessary for patients with isolated, non-life-threatening soft tissue injuries. If the avulsed tissue is hanging from a small piece of skin, circulation through the flap may be at risk. If you can, replace the avulsed flap to its original position, as long as it is not visibly contaminated with dirt and/or other foreign materials, and then cover the wound with a dry sterile dressing. Unless the wound is grossly contaminated with dirt or debris, irrigation is usually deferred until the patient is evaluated by a physician. Furthermore, flushing an open wound may force dirt or other debris into the wound, increasing the risk of infection.

Which of the following patients would be the LEAST likely to present with classic signs and symptoms of acute myocardial infarction? • A:55-year-old female with COPD and frequent infections. • B:59 year-old male with alcoholism and angina pectoris. • C:72-year-old female with diabetes and hypertension. • D:64-year-old male with renal disease and depression.

The correct answer is C; Reason: Chest pain, pressure, or discomfort (usually lasting greater than 15 minutes) is present in the majority of patients experiencing acute myocardial infarction (AMI). Other common signs and symptoms include shortness of breath, nausea, and diaphoresis. However, elderly female patients—especially those with diabetes—are more likely to present with atypical or unusual signs and symptoms than any other patient population. Diabetic neuropathy—a degenerative nerve condition associated with diabetes—results in decreased sensitivity to pain; therefore, the patient may present without any pain or discomfort. Sometimes, the only presenting signs and symptoms of AMI are generalized weakness, fatigue, or fainting.

Which of the following processes occurs during inhalation? • A:The diaphragm contracts and the intercostal muscles relax. • B:The intercostal muscles relax and the diaphragm descends. • C:The intercostal muscles and diaphragm both contract. • D:The diaphragm descends and the intercostal muscles relax.

The correct answer is C; Reason: During the active process of inhalation, the diaphragm contracts, causing it to descend. This increases the vertical dimensions of the chest. At the same time, the intercostal muscles (muscles between the ribs) contract, increasing the horizontal dimensions of the chest. These two processes cause intrathoracic pressure to fall, and air rushes in to fill the lungs. The drawing of air into the lungs by the actions of these muscles is called negative-pressure ventilation.

You are administering oxygen at 15 L/min to a patient with respiratory distress. If you are using a D cylinder (cylinder constant, 0.16), which reads 1,500 psi, how long will it take before you need to replace the oxygen cylinder? • A:18 minutes • B:9 minutes • C:14 minutes • D:11 minutes

The correct answer is C; Reason: The length of time you can use an oxygen cylinder depends on the type of cylinder you are using, the pressure in the cylinder, and the oxygen flow rate. A D cylinder is a small oxygen cylinder that is usually carried in the jump kit to the patient; it has a cylinder constant of 0.16. The following method can be used to calculate cylinder duration: gauge pressure (in psi) - the safe residual pressure (200 psi) × the cylinder constant ÷ flow rate in L/min. Using this formula, your D cylinder will become depleted in about 14 minutes, as follows: 1,500 (psi) - 200 (safe residual pressure) × 0.16 (cylinder constant) ÷ 15 (flow rate in L/min) = 13.86 (14 minutes). A full oxygen cylinder should contain 2,000 psi. The safe residual pressure is the lowest acceptable cylinder pressure before it should be replaced; it is usually 200 psi, although some EMS systems use 500 psi as a safe residual pressure. Although you will switch to your on-board oxygen (M cylinder) source when you load the patient into the ambulance, you should always have at least one back-up portable cylinder (preferably two) when administering oxygen to a patient at the scene, especially if you are giving high-flow (12 to 15 L/min) oxygen and/or your on-scene time will be delayed (eg, lengthy extrication, moving a patient from the second floor, etc).

A 22-year-old man was stabbed in the chest with a large knife. The patient is pulseless and apneic, and the knife is impaled in the center of his chest. Treatment should include: • A:stabilizing the knife, starting CPR, and providing rapid transport. • B:removing the knife, applying an occlusive dressing, and providing rapid transport. • C:removing the knife, starting CPR, and providing rapid transport. • D:stabilizing the knife, applying an occlusive dressing, and providing rapid transport.

The correct answer is C; Reason: There are two indications for removing an impaled object: when the object is causing airway compromise and when the object interferes with your ability to perform CPR. A knife impaled in the center of the chest, which is where chest compressions are performed, in a patient who is in cardiac arrest must be carefully removed. Quickly cover the wound to control any bleeding (an occlusive dressing covered by a sterile dressing is preferred) and begin CPR immediately.

Rapid extrication of a patient from an automobile should be performed by: • A:applying a vest-style extrication device and sliding the patient out of the car onto a long spine board for full immobilization. • B:maintaining support of the head, grasping the patient by the clothing, and rapidly removing the patient from the car. • C:applying a cervical collar, sliding a long spine board under the patient's buttocks, and removing the patient from the car. • D:applying a cervical collar and removing the patient from the car using the direct carry method

The correct answer is C; Reason: Unless there is an immediate threat to your or the patient's life, you should perform a rapid extrication by applying a cervical collar, sliding a long spine board under the patient's buttocks, turning and placing the patient on the board, and removing the patient from the automobile. You must be careful to control the cervical spine at all times during extrication. A vest-style extrication device would not be appropriate for a rapid extrication because it takes too long to correctly apply; it is better suited for stable patients with neck and/or back pain. An emergency move involves removing the patient from the automobile without any immobilization equipment.

Which of the following statements regarding ventricular fibrillation (V-Fib) is correct? • A:Any patient in V-Fib must receive CPR for 2 minutes prior to defibrillation. • B:Patients in V-Fib should be defibrillated after every 60 seconds of CPR. • C:In V-Fib, the heart is not pumping any blood and the patient is pulseless. • D:Loss of consciousness occurs within minutes after the onset of V-Fib.

The correct answer is C; Reason: Ventricular fibrillation (V-Fib) is a disorganized, ineffective quivering of the heart muscle. No blood is pumped through the body and the patient is pulseless. Loss of consciousness occurs within seconds following the onset of V-Fib. Patients in V-Fib are treated with high-quality CPR and defibrillation every 2 minutes if needed. When treating a patient in cardiac arrest, whether the patient's arrest was witnessed or unwitnessed, begin immediate CPR and apply the AED as soon as it is available.

A young female is unresponsive after overdosing on an unknown type of drug. Her respirations are slow and shallow and her pulse is slow and weak. Which of the following drugs is the LEAST likely cause of her condition? • A:Seconal • B:Heroin • C:Cocaine • D:Valium

The correct answer is C; Reason: Of the drugs listed, cocaine would be the least likely cause of the patient's condition. Cocaine is a central nervous system (CNS) stimulant; you would expect her to be hypertensive, tachycardic, tachypneic, and perhaps even violent. Heroin, Valium, and Seconal are all CNS depressants and could explain her condition. Heroin is an illegal narcotic (opiate), Valium is a benzodiazepine sedative-hypnotic drug, and Seconal is a barbiturate. Narcotics, benzodiazepines, and barbiturates are all CNS depressants. When taken in excess, they cause a decreased level of consciousness, respiratory depression, bradycardia, and hypotension.

Your assessment of a 5-year-old child reveals that he is unresponsive with a respiratory rate of 8 breaths/min and a heart rate of 50 beats/min. Treatment for this child should include: • A:high-flow oxygen via nonrebreathing mask and rapid transport. • B:back slaps and chest thrusts while attempting artificial ventilations. • C:positive-pressure ventilation, chest compressions, and rapid transport. • D:assisted ventilation with a bag-mask device and rapid transport.

The correct answer is C; Reason:A heart rate less than 60 beats/min in an infant or child—especially when accompanied by signs of poor perfusion and inadequate breathing—should be treated with positive-pressure ventilation, chest compressions, and rapid transport. Respirations of 8 breaths/min and a heart rate of 50 beats/min will not maintain adequate oxygenation and perfusion in a child. Back slaps and chest thrusts are indicated for a responsive infant with a severe foreign body upper airway obstruction.

You are called to treat a 55-year-old man who is experiencing difficulty breathing. After making contact with your patient, he extends his arm out to allow you to take his blood pressure. This is an example of: • A:implied consent. • B:formal consent. • C:actual consent. • D:informed consent.

The correct answer is C; Reason:Actual consent, also referred to as expressed consent, is when the patient asks for your help outright. This may also include nonverbal gestures, such as extending the arm to you to allow you to take the blood pressure. Informed consent involves explaining your proposed treatment to the patient, including the potential benefits and risks of the treatment. Implied consent involves treating an unresponsive patient or minor child (when the parents are not present) based on the assumption that the patient (or the parents of a minor) would consent to emergency treatment.

After an advanced airway device has been inserted in a 6-month-old infant in cardiopulmonary arrest, you should deliver ventilations at a rate of: • A:6 to 8 breaths/min. • B:10 to 12 breaths/min. • C:8 to 10 breaths/min. • D:12 to 20 breaths/min.

The correct answer is C; Reason:After an advanced airway device (eg, ET tube, multilumen airway, supraglottic airway) has been inserted during cardiac arrest, ventilate the patient at a rate of 8 to 10 breaths/min (one breath every 6 to 8 seconds). This ventilation rate applies to all age groups, except the newborn. Healthcare providers often deliver excessive ventilation, particularly when an advanced airway device is in place. Excessive ventilation (eg, hyperventilation) is detrimental because it causes an increase in intrathoracic pressure, which impedes blood flow back to the heart and decreases coronary perfusion. Hyperventilation also increases the risks of regurgitation and aspiration in the patient who does not have an advanced airway in place.

In which of the following situations is an emergency move of a patient from his or her wrecked vehicle clearly indicated? • A:The patient appears unresponsive and a high-power line is lying across the hood. • B:The patient has an altered mental status; diaphoresis; and rapid, shallow breathing. • C:Gas is leaking from the vehicle and there is a small fire in the engine compartment. • D:Your primary assessment reveals that the patient has signs and symptoms of shock.

The correct answer is C; Reason:An emergency move is indicated if you or the patient's life is in immediate danger. Gas leaking from the vehicle and a fire in the engine compartment are clear indicators that you and the patient's lives are in imminent danger. An emergency move involves grabbing the patient by the clothing, protecting his or her spine as much as possible, and dragging him or her from the vehicle to a safe place. The rapid extrication technique, which involves manually stabilizing the patient's head, applying a cervical collar, and removing the patient from the vehicle onto a long backboard, is indicated if the patient's condition is unstable and/or he or she is in need of treatment that requires a supine position. Never approach or touch a vehicle that is in contact with a high-power line; have the power company cut the power to the line first and then remove the patient from the vehicle.

After the baby's head delivers, it is usually tilted: • A:anteriorly, with the chin up. • B:with the face up. • C:posteriorly, to one side. • D:posteriorly, face down.

The correct answer is C; Reason:As the baby's head begins to deliver, it is usually in a posterior, face down position. After the head delivers completely, however, it usually tilts to the side in preparation for delivery of the shoulders. Remember to check for the presence of a nuchal cord (umbilical cord wrapped around the neck), and to suction the baby's mouth and nose as soon as its head delivers.

A 46-year-old man presents with generalized weakness and shortness of breath after he was bitten on the leg by a rattlesnake. His blood pressure is 106/58 mm Hg and his pulse rate is 112 beats/min. In addition to supplemental oxygen, further treatment for this patient should include: • A:ice packs to the wound and splinting. • B:proximal arterial constricting band and splinting. • C:splinting and lowering of the affected part. • D:elevation of the affected part and ice packs.

The correct answer is C; Reason:Care for a patient with a bite from a pit viper (rattlesnake, copperhead, water moccasin) includes keeping the patient calm, administering supplemental oxygen, splinting the affected part, and keeping it below the level of the heart. Do NOT apply ice to a snakebite; this will cause local vasoconstriction and may force the venom deeper into the patient's circulation. If a constricting band is applied, it should be proximal to the bite and should be tight enough to slow venous return only, not cut off arterial supply.

You are called to a residence for a woman in cardiac arrest. Shortly after starting CPR, the patient's husband presents you with an unsigned document that states "do not attempt resuscitation." You should: • A:contact medical control prior to continuing any resuscitative efforts. • B:stop all resuscitative efforts in accordance with the document. • C:continue CPR until you have contacted medical control for guidance. • D:stop CPR until the document can be validated by a physician.

The correct answer is C; Reason:Do not attempt resuscitate (DNAR) orders are particularly challenging for EMS providers. When presented with documentation, especially if it does not appear to be valid (in this case, an unsigned document), you should err on the side of patient care and continue resuscitative efforts until medical control orders you to stop.

A patient who overdosed on heroin would be expected to present with: • A:tachycardia. • B:hyperpnea. • C:hypotension. • D:dilated pupils.

The correct answer is C; Reason:Heroin is a Schedule I (illegal) narcotic that is typically injected. As with all narcotics, legal or illegal, overdose causes depression of the central nervous system (CNS), resulting in a decreased level of consciousness; bradycardia; hypotension; and slow, shallow (reduced tidal volume) breathing. Hyperpnea (deep breathing) would not be present in a patient who overdosed on a narcotic. In a narcotic overdose, the pupils are typically constricted (miosis). Barbiturates, such as phenobarbital, are also CNS depressants and cause the same symptoms seen with narcotic overdose. The patient's pupils, however, are typically dilated (mydriasis), not constricted.

A patient presents with severe bradycardia, hypersalivation, vomiting, and excessive tearing. Which of the following agents would MOST likely cause his signs and symptoms? • A:Phosgene • B:Chlorine • C:Soman • D:Anthrax

The correct answer is C; Reason:Nerve agents (eg, V agent [VX], sarin [GB], soman [GD], tabun [GA]) are among the most deadly chemicals developed. Designed to kill large numbers of people with small quantities, nerve agents can cause cardiac arrest within seconds to minutes of exposure. Nerve agents, discovered while in search of a superior pesticide, are in a class of chemical called organophosphates, which are found in household bug sprays, agricultural pesticides, and some industrial chemicals. Organophosphates block an essential enzyme in the nervous system, which cause the body's organs to become overstimulated. The mnemonic "DUMBELS" can help you recall the signs and symptoms of nerve agent exposure; it stands for Diarrhea; Urination; Miosis (constricted pupils); Bradycardia; Emesis (vomiting), Lacrimation (excessive tearing); and Seizures, Salivation, and Sweating. You can also use the mnemonic "SLUDGEM," which stands for Salivation, Lacrimation, Urination, Defecation, GI distress, Emesis, and Miosis.

You are dispatched to a residence for a child having a seizure. When you arrive at the scene, the 4-year-old child's grandfather tells you that he has had several full body seizures over the past 20 minutes, but never woke up in between the seizures. The child's skin is hot and flushed. This is MOST indicative of: • A:a febrile seizure. • B:an absence seizure. • C:status epilepticus. • D:a focal motor seizure.

The correct answer is C; Reason:Status epilepticus is defined as a prolonged (greater than 20 minutes) seizure or multiple seizures without a return of consciousness in between seizures. A febrile seizure is caused by an abrupt rise in body temperature, usually due to a non-life-threatening infection (eg, middle ear infection). Most febrile seizures last less than 5 minutes, have resolved by the time EMS arrives at the scene, and are not followed by a postictal period. The child in this scenario, although febrile (hot, flushed skin), did not experience a seizure caused by fever alone; you should suspect other causes of fever and seizures, such as meningitis. An absence seizure, also called a petit mal seizure, is characterized by a blank stare and an absence of tonic-clonic motor activity. Like febrile seizures, absence seizures are usually of short duration and are not followed by a postictal period. A focal motor seizure is isolated to one part of the body, such as an extremity, but can progress to a generalized (full body) seizure.

You are called to a local state park where a hiker fell from a cliff into a thick, wooded area with rough terrain. It is cold and foggy and a thunderstorm is approaching. You will MOST likely move the patient to the ambulance by: • A:bringing the ambulance stretcher to the patient, loading him onto it, and removing him from the wooded area. • B:requesting a helicopter to hoist the patient out of the wooded area and carry him to a site near the ambulance. • C:placing the patient in a basket stretcher and using at least four people to carry him to the ambulance. • D:assisting the patient in walking from the wooded area to the ambulance if his injuries are not life-threatening

The correct answer is C; Reason:The basket stretcher, also called a Stokes basket, is ideal for moving patients across rough terrain. Because the patient fell from a cliff, he will require spinal motion restriction precautions. Secure him to a long backboard and place the backboard in the basket stretcher. When carrying a patient across rough terrain, you should utilize at least four people to ensure a safe patient move. Clearly, the patient should not walk; he should be suspected of having a spinal injury due to the fall. A wheeled ambulance stretcher is impractical in situations were the patient must be moved over rough terrain. In many cases, a helicopter can be used to move patients from remote areas to a landing site near the ambulance or directly to the hospital. However, in this case, the weather (eg, fog, approaching thunderstorm) will likely prohibit a helicopter from being able to safely fly.

You are assessing a 5-year-old boy with major trauma. His blood pressure is 70/40 mm Hg and his pulse rate is 140 beats/min. and weak. The child's blood pressure: • A:is appropriate based on his age. • B:suggests increased intracranial pressure. • C:indicates decompensated shock. • D:reflects adequate compensation.

The correct answer is C; Reason:The low normal systolic blood pressure (SBP) for a child between 1 and 10 years of age is calculated by multiplying his or her age (in years) by 2 and adding 70. Using this formula, the low normal SBP for a 5-year-old child is 80 mm Hg; anything less indicates hypotension. Therefore, a SBP of 70 mm Hg in a 5-year-old child, especially in the context of major trauma, indicates decompensated shock. Patients with increased intracranial pressure are typically hypertensive, not hypotensive.

The role of triage officer at a mass-casualty incident should be assumed by the: • A:first EMS provider who is willing to perform the task. • B:EMS medical director via telephone communication. • C:most knowledgeable EMS provider. • D:EMS provider with the most years in EMS.

The correct answer is C; Reason:The role of triage officer should be assumed by the most knowledgeable EMS provider at the scene. Knowledge and experience will enable this person to most effectively manage the triage process. Just because a person has been in the EMS profession for a long period of time does not mean that he or she has been active or has maintained clinical competence.

An elderly woman, who was removed from her burning house by firefighters, has full-thickness burns to approximately 50% of her body. Appropriate treatment for this patient should include: • A:peeling burned clothing from the skin and removing all rings, necklaces, and bracelets. • B:cooling the burns with sterile saline and covering them with dry, sterile burn pads. • C:applying moist, sterile dressings to the burned areas and preventing hypothermia. • D:covering the burns with dry, sterile dressings and preventing further loss of body heat.

The correct answer is D; Reason: After moving the patient to safety, stopping the burning process, and supporting the ABCs, full-thickness burns should be cared for by applying dry, sterile dressings or burn pads and preventing hypothermia. Cooling full-thickness burns (ie, applying moist dressings, pouring saline or water on the burn) should be avoided as this increases the risks of hypothermia and infection. Rings, necklaces, and other potentially constrictive devices should be removed in the event that severe swelling occurs. If portions of clothing are adhered to the skin, they should be cut around, not peeled from the skin, in order to prevent further soft-tissue damage.

When treating a patient with chest pain, pressure, or discomfort, you should first: • A:request an ALS ambulance response to the scene. • B:administer supplemental oxygen. • C:assess the blood pressure and give nitroglycerin. • D:place the patient in a position of comfort.

The correct answer is D; Reason: An important aspect of treating a patient with chest pain, pressure, or discomfort is to ensure that the patient is in a comfortable position. Most of the time, the patient will already be in this position upon your arrival. A position of comfort will aid in minimizing anxiety, which in turn decreases cardiac oxygen consumption and demand. After ensuring that the patient is in a comfortable position, administer supplemental oxygen in a concentration sufficient to maintain an SpO2 of greater than 94%. Following your assessment, if you feel that ALS support is needed, you should request it. If the patient has prescribed, unexpired nitroglycerin; the systolic blood pressure is greater than 100 mm Hg; and the patient has not taken the maximum of three doses, you should contact medical control to obtain permission to assist the patient in taking the nitroglycerin.

A 19-year-old female has a closed, swollen deformity to her left forearm. You are unable to palpate a radial pulse and the skin distal to the injury is cold and pale. Several attempts to contact medical control have failed and you are approximately 45 miles away from the closest hospital. You should: • A:apply an air splint to her forearm, keep her arm below the level of her heart, place an icepack over the injury, and transport. • B:begin transport at once, gently manipulate her arm en route until distal circulation is restored, and apply an air splint. • C:splint her entire arm with rigid board splints, elevate the limb above the level of her heart, and transport immediately. • D:make one attempt to restore distal circulation by applying gentle manual traction in line with the long axis of the limb.

The correct answer is D; Reason: Cold, pale skin and an absent distal pulse indicates that blood flow distal to the injury is compromised. You should notify medical control, who will likely direct you to attempt to restore distal circulation. However, if you are unable to contact medical control and your transport time will be lengthy, you should make ONE attempt to restore distal circulation by applying gentle manual traction in line with the long axis of the limb. Be careful, as excessive manipulation can worsen the vascular problem. If you are unsuccessful after one attempt, splint the limb in the most comfortable position for the patient and transport at once. If distal circulation is restored, splint the limb in whatever position allows the strongest distal pulse. You should elevate the limb above the level of the heart to help minimize swelling. An icepack may also help reduce pain and swelling.

As you begin ventilating an unresponsive apneic man, you hear gurgling in his upper airway. Your MOST immediate action should be to: • A:suction the patient's airway for no longer than 15 seconds. • B:squeeze the bag-mask device with less force and reassess. • C:reposition the patient's airway and continue ventilations. • D:quickly turn the patient onto his side so secretions can drain.

The correct answer is D; Reason: Gurgling in the airway indicates the presence of vomitus or other secretions. If this is noted, you should immediately turn the patient onto his side to allow the secretions to drain. After placing the patient on his side, suction his airway for no longer than 15 seconds. To continue ventilating a patient whose airway is full of vomitus or secretions will force the secretions into the trachea, resulting in aspiration. Aspiration significantly increases mortality!

While transporting an elderly woman who was complaining of nausea, vomiting, and weakness, she suddenly becomes unresponsive. You should: • A:feel for a carotid pulse for at least 5 seconds. • B:analyze her heart rhythm with the AED. • C:open her airway and ensure that it is clear. • D:quickly look at her chest for obvious movement.

The correct answer is D; Reason: If a patient is found unresponsive or becomes unresponsive in your presence, your first action should be to assess for breathing; this should be done by quickly (no more than 10 seconds) looking at the chest for obvious movement. If the patient is not breathing or only has agonal gasps, you should check for a pulse for at least 5 seconds but no more than 10 seconds. If the patient has a pulse but is not breathing, open the airway and provide rescue breathing. If the patient does not have a pulse, begin CPR (starting with chest compressions), and apply the AED as soon as possible. If you are transporting a patient who becomes unresponsive, pulseless, and apneic, you should begin CPR and instruct your partner to stop the ambulance and prepare the AED.

Which of the following injury mechanisms is associated with hangings? • A:Subluxation • B:Axial loading • C:Hyperextension • D:Distraction

The correct answer is D; Reason: Injury to the cervical spine following a hanging occurs via distraction, or stretching, of the vertebrae and spinal cord. A subluxation is a partial or incomplete dislocation; it is an injury, not an injury mechanism. Injuries related to hyperextension mechanisms are common in patients who strike their head on the windshield during a motor-vehicle crash. Axial loading is a mechanism of injury in which the spinal column is compressed vertically. Injuries caused by axial loading include cervical spine injuries after diving head first into shallow water and lumbar spine injuries after a fall from a significant height in which the patient lands feet first.

A 44-year-old male experienced burns to his anterior trunk and both arms. He is conscious and alert, but is in extreme pain. Assessment of the burns reveals reddening and blisters. This patient has ________________ burns that cover _____ of his total body surface area. • A:first-degree, 27% • B:full-thickness, 18% • C:second-degree, 45% • D:partial-thickness, 36%

The correct answer is D; Reason: Partial-thickness (second-degree) burns damage the epidermis and part of the dermis, and are characterized by blistering and severe pain. Areas of superficial (first-degree) burns, which cause reddening of the skin, commonly surround a partial-thickness burn. The anterior trunk (chest and abdomen) accounts for 18% of the total body surface area (TBSA) and each entire arm accounts for 9%. Therefore, this patient has partial-thickness burns that cover 36% of his TBSA. Full-thickness (third-degree) burns are characterized by charred or white, leathery skin. Because the entire dermis, including the nerves, is destroyed, full-thickness burns are usually painless. The surrounding areas of partial-thickness burns, however, are very painful.

The automated external defibrillator (AED) should NOT be used in patients who: • A:have a nitroglycerin patch applied to the skin. • B:are between 1 and 8 years of age. • C:experienced a witnessed cardiac arrest. • D:are apneic and have a weak carotid pulse.

The correct answer is D; Reason: The AED is only applied to patients in cardiac arrest (eg, pulseless and apneic), whether the arrest was witnessed or unwitnessed. According to the 2010 guidelines for CPR and Emergency Cardiac Care (ECC), AEDs can safely be used in infants and children less than 8 years of age in conjunction with a dose-attenuating system (energy reducer) and pediatric pads. However, if pediatric pads and an energy reducer are unavailable, adult AED pads should be used. A nitroglycerin patch is not a contraindication to the use of an AED; simply remove the patch (with gloved hands) and apply the AED as usual.

A patient with a closed head injury opens his eyes in response to pain, is mumbling words that you cannot understand, and pushes your hand away when you apply a painful stimulus. His Glasgow Coma Scale (GCS) score is: • A:6 • B:8 • C:7 • D:9

The correct answer is D; Reason: The Glasgow Coma Scale (GCS) is a valuable tool used when assessing patients with a neurological injury. It assesses three parameters: eye opening, verbal response, and motor response. A minimum score on the GCS is 3 and a maximum score is 15. A patient who opens his or her eyes in response to pain would receive a score of 2. Mumbling speech, moaning, or incomprehensible words equate to a score of 2 for verbal response. Localization of a painful stimulus, such as pushing your hand away from the source of pain, equates to a score of 5. Therefore, the patient has a GCS score of 9. It is important to note that a patient's GCS score should be reassessed frequently. Review the entire GCS in your EMT text and commit it to memory.

A patient reports pain in the upper midabdominal area. This region of the abdomen is called the: • A:retroperitoneum. • B:peritoneum. • C:mediastinum. • D:epigastrium.

The correct answer is D; Reason: The mid-upper region of the abdomen is referred to as the epigastrium because of its location over the stomach (epi = upon, gastric = stomach). This is a common site of pain or discomfort in patients experiencing a cardiac problem, which frequently causes them to attribute their pain or discomfort to indigestion.

A patient overdosed on several drugs and is unresponsive with shallow breathing and facial cyanosis. As you continue your assessment, the patient suddenly vomits. You should: • A:insert an oropharyngeal airway. • B:begin assisting his ventilations. • C:suction his oropharynx at once. • D:turn the patient onto his side.

The correct answer is D; Reason: The patient's airway must be clear of foreign bodies or secretions before it can be assessed or managed. If the patient begins to vomit, he must first be rolled onto his side to allow for drainage of the vomitus. Use suction to remove secretions after you have positioned him on his side. After the airway is clear, you should insert an appropriate airway adjunct (oral or nasal airway) and ensure adequate ventilation and oxygenation. In this patient, this involves assisting his ventilations with a bag-mask device.

After an adult cardiac arrest patient has been intubated by a paramedic, you are providing ventilations as your partner performs chest compressions. When ventilating the patient, you should: • A:deliver each breath over 2 seconds at a rate of 12 to 15 breaths/min. • B:deliver 2 breaths during a brief pause in chest compressions. • C:hyperventilate the patient to maximize carbon dioxide elimination. • D:deliver each breath over 1 second at a rate of 8 to 10 breaths/min.

The correct answer is D; Reason: When ventilating an adult cardiac arrest patient with an advanced airway in place (ie, ET tube, multilumen airway, supraglottic airway), you should deliver each breath over a period of 1 second—just enough to produce visible chest rise—at a rate of 8 to 10 breaths/min (one breath every 6 to 8 seconds). Do not attempt to synchronize ventilations with chest compressions once the airway has been secured with an advanced device. Hyperventilation should be avoided as it may result in increased intrathoracic pressure, decreased blood return to the heart, and as a result, less effective chest compressions.

The 5-minute Apgar assessment of a newborn reveals a heart rate of 130 beats/min, cyanosis to the hands and feet, and rapid respirations. The infant cries when you flick the soles of its feet and resists attempts to straighten its legs. These findings equate to an Apgar score of: • A:8 • B:10 • C:7 • D:9

The correct answer is D; Reason: The Apgar score, which is obtained at 1 and 5 minutes after birth (and every 5 minutes thereafter), assigns numbers (0, 1, or 2) to the following five areas: Appearance, Pulse, Grimace, Activity, and Respirations. A score of 1 is assigned for appearance if the newborn's body is pink, but its hands and feet remain blue. If its heart rate is greater than 100 beats/min, it receives a score of 2 for the pulse. If it cries and tries to move its foot away when soles of its feet are flicked, it is assigned a score of 2 for grimace/irritability. If it resists attempts to straighten its hips and knees, a score of 2 is assigned for activity/muscle tone. If its respirations are rapid, a score of 2 is assigned. Based on these parameters, the newborn in this scenario would receive an Apgar score of 9. Refer to your EMT textbook for a complete review of the Apgar score.

Which of the following creates a secure loop at the working end of a rope, which can be used to attach the end of the rope to a fixed object or a piece of equipment? • A:Half hitch • B:Figure eight knot • C:Clove hitch • D:Figure eight on a bite

The correct answer is D; Reason:Although EMTs infrequently perform special rescue operations (ie, urban search and rescue [USAR]), they should have a basic working knowledge of the different ropes and knots used for rescue purposes. The figure eight on a bite knot creates a secure loop at the working end of the rope, the part of the rope used for forming the knot. This loop can be used to attach the end of the rope to a fixed object or a piece of equipment, or to tie a life safety rope around a person. The loop may be of any size, from an inch to several feet in diameter. The figure eight is a basic knot used to produce a family of other knots, including the figure eight on a bite and the figure eight with a follow-through. A simple figure eight knot is seldom used. The half hitch is not a secure knot by itself, which is why it is used in conjunction with other knots. The clove hitch is used to attach a rope firmly to a round object, such as a tree or fencepost.

Which of the following is an abnormal finding? • A:Respiratory rate of 26 breaths/min in a 2-year-old child • B:Systolic BP of 100 mm Hg in a 10-year-old child • C:Rapid, irregular breathing in a newly born infant • D:Heart rate of 80 beats/min in a 3-month-old infant

The correct answer is D; Reason:An infant between 1 month and 1 year of age typically has a heart rate between 100 and 160 beats/min; a heart rate less than 100 beats/min in this age group is considered to be bradycardic. Newborn infants normally have irregular breathing that ranges between 40 and 60 breaths/min. The systolic BP for a child between 6 and 12 years of age typically ranges between 80 and 110 mm Hg. The respiratory rate in a child between 1 and 3 years of age typically ranges between 20 and 30 breaths/min.

A 5-year-old boy was struck by a car when he ran out into the street. When you arrive at the scene and approach the child, you see him lying supine approximately 15 feet from the car. Based on the child's age and mechanism of injury, which of the following should you suspect to be his PRIMARY injury? • A:Head injury • B:Upper thorax injury • C:Lower leg injury • D:Pelvic injury

The correct answer is D; Reason:Children are smaller than adults; therefore, when they are injured by the same mechanism of injury as an adult, the location of their injuries may differ from those of an adult. For example, when an adult is struck by a vehicle, the primary injury typically occurs at or below the knees, depending on the height of the bumper at the time of impact. Because the child is shorter, initial impact typically occurs at or near the pelvis. Secondary injury occurs when child's chest collides with the vehicle's grille. Tertiary injury occurs when the child strikes the side of his or her head on the pavement after being propelled away from the vehicle. In some cases, the child is pulled underneath the vehicle and is dragged.

You arrive at a residence shortly after a 4-year-old boy experienced an apparent febrile seizure. The child is alert and crying. His skin is flushed, hot, and moist. His mother tells you that the seizure lasted about 2 minutes. You should: • A:begin rapid cooling measures at once. • B:allow the mother to take her child to the doctor. • C:give him acetaminophen or ibuprofen. • D:provide supportive care and transport.

The correct answer is D; Reason:Febrile seizures are common in children between the ages of 6 months and 6 years; they occur when the child's body temperature suddenly rises or when an already febrile child experiences an acute fever spike. Treatment for a child who has experienced a febrile seizure involves providing supportive care (eg, monitoring ABCs, administering oxygen as tolerated) and transporting to the hospital. Any infant or child who experienced a seizure should be transported by EMS. Although febrile seizures are typically self-limiting and are rarely life-threatening, there are other causes of fever and seizures, such as meningitis. The EMT is generally not authorized to administer medications unless directed by local protocol or direct medical control. Active cooling (eg, cool or cold water baths) should be avoided; a sudden lowering of the child's temperature may cause him to shiver, which may cause a sudden fever spike and induce another seizure.

A 60-year-old male complains of a tearing sensation in his abdomen. He tells you the pain began suddenly and feels like someone is sticking a knife into his abdomen. He is conscious and alert with a blood pressure of 148/88 mm Hg, a pulse of 120 beats/min, and respirations of 22 breaths/min. In addition to administering high-flow oxygen, you should: • A:perform a rapid head to toe assessment and prepare for transport. • B:auscultate over his epigastrium to assess for bowel sounds. • C:vigorously palpate his abdomen to assess for a pulsating mass. • D:transport at once and be prepared to treat him for severe shock.

The correct answer is D; Reason:Given the onset and nature of the patient's pain (eg, sudden onset, tearing sensation), you should suspect that he has an acute dissection of the abdominal aorta. Administer high-flow oxygen and transport him at once. Acute aortic dissection can quickly cause an aneurysm, which could rupture and cause profound shock. Therefore, you must carefully monitor him and be prepared to treat him accordingly. Unnecessary or vigorous palpation of his abdomen could cause his aorta to rupture and should be avoided. A rapid head-to-toe assessment is indicated for trauma patients with a significant mechanism of injury and unresponsive medical patients; this patient falls into neither of these categories. Auscultating bowel sounds in the field is time-consuming, will yield little information, and only delays transport.

A 7-year-old child has an altered mental status, high fever, and a generalized rash. You perform your assessment and administer supplemental oxygen. En route to the hospital, you should be MOST alert for: • A:respiratory distress. • B:combativeness. • C:hypotension. • D:convulsions.

The correct answer is D; Reason:High fever and an alerted mental status indicate sepsis (severe infection). A generalized rash should alert you to the possibility of meningitis—a condition caused by infection and inflammation of the meninges that protect the brain and spinal cord. Children with meningitis are at risk for seizures (convulsions), usually due to increased intracranial pressure (ICP) and/or high fever; therefore, you must continually monitor the child's condition en route to the hospital and be prepared to treat seizures if they occur. Remember that seizure deaths are caused by cerebral hypoxia. You should also be alert for vomiting, which can jeopardize the airway. Hypotension can occur in patients with sepsis and should also be of concern; however, seizures directly compromise adequate ventilation and oxygenation.

Treatment for a responsive 4-year-old child with a mild airway obstruction, who has respiratory distress, a strong cough, and normal skin color, includes: • A:oxygen, back slaps, and transport. • B:assisted ventilations, back slaps, and transport. • C:subdiaphragmatic thrusts until the object is expelled. • D:supplemental oxygen and transport.

The correct answer is D; Reason:If a child (1 year of age to the onset of puberty [12 to 14 years of age]) with a mild airway obstruction is alert and has adequate air movement (ie, a strong cough, normal skin color), you should offer oxygen, avoid agitating the child, and provide transport to the hospital. Attempts to relieve a mild airway obstruction may result in a severe airway obstruction. If signs of a severe airway obstruction develop, you must take immediate action to remove the object (eg, back slaps and chest thrusts in a responsive infant; subdiaphragmatic [abdominal] thrusts in a responsive adult or child). Finger sweeps are ONLY indicated if the patient is unresponsive and you can see the object in his or her mouth.

Which of the following vaccinations is NOT currently recommended by the Centers for Disease Control and Prevention (CDC)? • A:Tetanus, diphtheria, and pertussis • B:Measles, mumps, rubella • C:Hepatitis B • D:Smallpox

The correct answer is D; Reason:Immunization requirements are usually set by the state department of health, but generally follow recommendations made by the Centers for Disease Control and Prevention (CDC). Recommended immunizations for health care providers include tetanus, diphtheria, and pertussis (Tdap, DTaP) boosters (every 10 years); measles, mumps, rubella (MMR) booster; influenza vaccine (yearly); and hepatitis B vaccine (single or three-shot series). Smallpox vaccinations ceased in 1972 after the World Health Organization (WHO) officially declared the disease eradicated.

Upon assessing a newborn immediately after delivery, you note that the infant is breathing spontaneously and has a heart rate of 80 beats/min. What is the MOST appropriate initial management for this newborn? • A:Start chest compressions and contact medical control. • B:Assess the newborn's skin condition and color. • C:Provide blow-by oxygen with oxygen tubing. • D:Initiate positive-pressure ventilations.

The correct answer is D; Reason:Positive-pressure ventilations are indicated in the newborn if he or she is apneic or has gasping respirations, if the heart rate is less than 100 beats/min, or if central cyanosis persists despite the delivery of blow-by oxygen. Chest compressions are indicated if the heart rate is less than 60 beats/min, despite 30 seconds of adequate positive-pressure ventilation. In many cases, the newborn's heart rate will increase to greater than 100 beats/min with adequately performed positive-pressure ventilation.

Following the initial steps of resuscitation, a newborn remains apneic and cyanotic. You should: • A:start CPR if the heart rate is less than 80 beats/min. • B:immediately resuction its mouth and nose. • C:gently flick the soles of its feet for up to 60 seconds. • D:begin ventilations with a bag-mask device.

The correct answer is D; Reason:The initial steps of newborn resuscitation, which are performed on all newborns following delivery, include drying, warming, positioning, suctioning, and tactile stimulation. If the newborn remains apneic after the initial steps of resuscitation, or has a heart rate less than 100 beats/min, you should begin positive-pressure ventilations (PPV) with a bag-mask device at a rate of 40 to 60 breaths/min. Continued tactile stimulation (eg, flicking the soles of the feet, rubbing the lateral thorax) of an apneic newborn wastes time; you must ventilate at once. If the newborn's heart rate is less than 60 beats/min despite effective PPV, you should begin chest compressions.

A young male experienced a syncopal episode after working in the heat for several hours. He is conscious and alert; has cool, clammy skin; and complains of nausea and lightheadedness. You should: • A:provide rapid cooling. • B:give him cold water to drink. • C:advise him to go home and rest. • D:transport him on his side.

The correct answer is D; Reason:The patient is experiencing heat exhaustion and should be transported to the hospital for evaluation, especially since he experienced a syncopal episode (fainting). Because he is nauseated, he should not be given anything to drink and should be placed on his side during transport to prevent aspiration if he vomits. Loosen any restrictive clothing that may trap heat. Rapid cooling is indicated for patients with heatstroke, the signs of which include an altered mental status and hot, flushed skin (dry or moist).

Following an apparent terrorist attack, numerous patients present with shortness of breath and persistent coughing. A green haze is noted in the area in which the patients are located. Which of the following agents should you suspect they were exposed to? • A:Tabun (GA) • B:Phosgene oxime • C:V agent (VX) • D:Chlorine (CL)

The correct answer is D; Reason:The patient's signs and symptoms are indicative of a pulmonary (choking) agent, specifically chlorine (CL). Chlorine (CL) was the first chemical agent ever used in warfare. It has a distinct odor of bleach and creates a green haze when released as a gas. Initially, it produces upper airway irritation and a choking sensation. Later signs and symptoms include shortness of breath, chest tightness, hoarseness and stridor as the result of upper airway swelling, and gasping or persistent coughing. Phosgene, not to be confused with phosgene oxime (a blistering [vesicant] agent), is also a pulmonary (choking) agent. Tabun (GA) and V agent (VX) are examples of chemical nerve agents. Nerve agents are among the most deadly chemicals developed. Designed to kill large numbers of people with small quantities, nerve agents can cause cardiac arrest within seconds to minutes of exposure.

Which of the following statements regarding the length-based resuscitation tape measure is correct? • A:The red end of the tape measure is placed at the heel of the child's foot. • B:The resuscitation tape estimates a child's age based on his or her height. • C:It is not a reliable tool in children who are less than 5 years of age. • D:The tape measure can be used in children who weigh up to 75 pounds.

The correct answer is D; Reason:There are a number of ways to identify the appropriate size equipment for a pediatric patient; however, the length-based resuscitation tape measure is perhaps the most accurate. Pediatricians generally agree that length (height), not age, is the most reliable estimator of weight. The resuscitation tape estimates a child's weight based on his or her height; it is a reliable tool to use in children who weigh up to 75 lb (34 kg). To use the resuscitation tape, lay it next to the child with the multicolored side up. Place the red end of the tape at the top (crown) of the child's head, and stretch the tape out the full length of the child, stopping at the heel of the foot. Place your free hand, side down, at the bottom of the child's heel. Note the color or letter and weight range on the edge of the tape where your hand is.

Which of the following statements regarding lightning-related injuries is correct? • A:Multiple fractures, including those of the cervical spine, are the most common cause of lightning-related deaths. • B:The majority of patients who are struck by lightning die, even if CPR is provided immediately. • C:Because of the high electrical energy associated with lightning, full-thickness burns are a common finding. • D:The cardiovascular and nervous systems are most commonly injured during a lightning strike.

The correct answer is D; Reason:The cardiovascular and nervous systems are most commonly injured during a lightning strike; therefore, respiratory or cardiac arrest is the most common cause of lightning-related deaths, especially if CPR is delayed. A lightning strike typically induces asystole (cardiac standstill); however, this spontaneously resolves in some people or can be reversed with early, high-quality CPR. The energy associated with lightning is composed of direct current (DC) of up to 200,000 amperes and a potential of 100 million volts or more; however, the duration of a lightning strike is short. Therefore, skin burns are usually superficial; full-thickness (third-degree) burns are rare. Lightning causes massive contraction of all the body's muscles, potentially resulting in long bone and spinal fractures. Although this can clearly increase morbidity, it is not the most common cause of lightning-related deaths.

Respiratory Failure

The reduction of breathing to the point where oxygen intake is not sufficient to support life

perfusion

The supply of oxygen to and removal of wastes from the cells and tissues of the body as a result of the flow of blood through the capillaries.

Hypercapnia

Too much carbon dioxide

What should I do with someone had a spontaneous abortio.

Transport because there is lots of blood that may be lost

An inaccurate pulse oximetry reading may be caused by: • A:excessive red blood cell production. • B:severe peripheral vasoconstriction. • C:a heart rate greater than 100 beats/min. • D:heat illnesses, such as heat stroke.

You selected A; The correct answer is B; Reason: A pulse oximeter measures the percentage of hemoglobin that is saturated with oxygen. Under normal conditions, a patient's oxygen saturation (SpO2) ranges between 95% and 100% while breathing room air. Although no definitive threshold for normal SpO2 values exists, an SpO2 that is less than 95% in a nonsmoker may indicate hypoxemia. Of the factors listed, several peripheral vasoconstriction (ie, hypothermia, cigarette smoking, chronic hypoxia) would be the most likely to produce an inaccurate SpO2 reading. When the peripheral vasculature constricts, blood is shunted to the core of the body; in such cases, the pulse oximeter would likely yield a falsely low reading (or no reading at all). Other factors that can cause inaccurate readings include dark or metallic nail polish, dirty fingers, and abnormal hemoglobin binding (ie, carbon monoxide [CO] poisoning). It is important to note that the pulse oximeter is designed to detect gross abnormalities, not subtle changes, and should be used in conjunction with a thorough clinical assessment of the patient.

Sudden cardiac arrest in the adult population is MOST often the result of: • A:myocardial infarction. • B:a cardiac dysrhythmia. • C:an acute stroke. • D:respiratory failure.

You selected A; The correct answer is B; Reason: Most cases of sudden cardiac arrest (SCA) in the adult (70% to 75%) are the result of a cardiac dysrhythmia, most commonly ventricular fibrillation (V-Fib). This fact underscores the importance of early defibrillation. Stroke, respiratory failure, and myocardial infarction can all cause cardiac arrest in the adult, but a cardiac dysrhythmia is more common.

The presence of subcutaneous emphysema following blunt trauma to the anterior neck should make you MOST suspicious for a: • A:pneumothorax. • B:carotid artery injury. • C:fractured larynx. • D:ruptured esophagus.

You selected A; The correct answer is C; Reason: Crushing or blunt trauma to the anterior neck can injure the trachea or larynx. Once the cartilages of the upper airway and larynx are fractured, they do not spring back to their normal position. Such a fracture can lead to loss of voice, airway obstruction, and leakage of air into the soft tissues of the neck. Air leakage into the soft tissues is called subcutaneous emphysema. Subcutaneous emphysema may also be observed in patients with a tension pneumothorax, although it is typically located in the chest. Esophageal rupture would likely present with difficulty swallowing (dysphagia) and vomiting blood (hematemesis). You should suspect injury to a carotid artery or jugular vein if you observe a rapidly expanding hematoma to the neck following blunt trauma.

The scene size-up includes all of the following components, EXCEPT: • A:evaluating the mechanism of injury. • B:determining if the scene is safe. • C:assessing the need for assistance. • D:donning personal protective gea

You selected A; The correct answer is D; Reason: The components of the scene size-up include determining scene safety, assessing the mechanism of injury (MOI) or nature of illness (NOI), determining the number of patients, and requesting additional help if needed. Personal protective equipment (PPE) should be donned prior to beginning the scene size-up.

Which of the following types of stroke would MOST likely present with a sudden, severe headache? • A:Hemorrhagic • B:Ischemic • C:Thrombotic • D:Embolic

You selected A; This is correct! Reason: A stroke occurs when blood flow to a portion of the brain is interrupted. There are two types of stroke: ischemic and hemorrhagic, both of which are acute events. Hemorrhagic stroke is caused by a ruptured artery in the brain, resulting in intracerebral bleeding, cerebral ischemia, and increased intracranial pressure. A classic feature of a hemorrhagic stroke is a sudden, severe headache (indicates cerebral artery rupture) that is followed by rapid deterioration of the patient's condition. By contrast, ischemic stroke (accounts for approximately 80% of all strokes) occurs when a clot occludes a cerebral artery. If the clot forms locally, it is referred to as a thrombotic stroke; if the clot travels to the brain from another part of the body, it is referred to as an embolic stroke. Acute ischemic stroke typically presents with a sudden onset of confusion, facial droop, slurred speech, and weakness to one side of the body. A headache is uncommon in patients with acute ischemic stroke.

Which of the following statements regarding the automated external defibrillator (AED) is correct? • A:AEDs can safely be used in infants and children less than 8 years of age • B:AEDs will analyze the patient's rhythm while CPR is in progress • C:The AED should be applied to patients at risk for cardiac arrest • D:The AED should not be used in patients with an implanted defibrillator

You selected A; This is correct! Reason: According to the 2010 guidelines for CPR and Emergency Cardiac Care (ECC), the AED can safely be used in infants and children less than 8 years of age. Although a manual defibrillator is preferred in infants, an AED can be used. When using the AED in infants and children, you should use pediatric pads and a dose-attenuating system (energy reducer); however, if these features are not available, adult AED pads should be used. The AED should only be applied to patients in cardiac arrest; if a patient is at risk for cardiac arrest, have the AED ready but not applied. The AED will not analyze the cardiac rhythm if the patient is moving (ie, CPR is in progress). AEDs can be used in patients with an automated implanted cardioverter/defibrillator (AICD) or implanted pacemaker; ensure that the pads are at least 1" away from the implanted device.

How should you treat an unresponsive, uninjured patient with respirations of 16 breaths/min and good chest expansion? • A:Airway adjunct and oxygen via nonrebreathing mask • B:Suctioning as needed and artificial ventilations • C:Jaw-thrust maneuver and frequent suctioning • D:Oropharyngeal suctioning and assisted ventilations

You selected A; This is correct! Reason: After opening the airway of an unresponsive patient, an airway adjunct (oral or nasal airway) should be inserted to keep the tongue from occluding the posterior pharynx. Oral and nasal airways are used in conjunction with manual head positioning to help maintain a patent airway. Unresponsive patients who are breathing adequately (good rate, adequate depth [tidal volume]) should receive high-flow oxygen via nonrebreathing mask. The patient must be monitored closely for signs of inadequate breathing, which will require ventilatory assistance with a bag-mask device. Suction the oropharynx only if blood or other secretions are in the patient's mouth.

Which of the following patients has signs of inadequate breathing? • A:A 41-year-old woman with shallow respirations of 14 breaths/min • B:A 30-year-old man with respirations of 12 breaths/min with adequate depth • C:A 50-year-old woman with respirations of 12 breaths/min and pink, dry skin • D:A 60-year-old man with clear and equal breath sounds bilaterally

You selected A; This is correct! Reason: Although the 41-year-old woman has a respiratory rate that falls within the normal range for an adult, the depth of her breathing is shallow (reduced tidal volume). Signs of inadequate breathing in the adult include a slow (less than 12 breaths/min) or fast (greater than 20 breaths/min) respiratory rate, shallow depth (reduced tidal volume), altered level of consciousness, tachycardia, an irregular pattern of inhalation and exhalation, diminished breath sounds during auscultation, and cyanosis. It is important to assess ALL components of a patient's breathing: rate, regularity, depth, and quality. Do not rely solely on one parameter.

A soft-tissue injury that results in a flap of torn skin is called a/an: • A:avulsion. • B:incision. • C:abrasion. • D:laceration.

You selected A; This is correct! Reason: An avulsion is a soft-tissue injury in which a portion of the skin is torn away, leaving a flap of skin. A laceration is a jagged soft-tissue injury that can be caused by glass or other sharp objects. An abrasion is the scraping away of the epidermis, causing oozing of serous fluid from the capillary bed. Road rash is a classic example of an abrasion. An incision is similar to a laceration, but has smooth edges. Scalpels or knives are examples of instruments that would make an incision.

Which of the following patients obviously needs positive-pressure ventilation assistance? • A:Responsive to pain only; respiratory rate of 8 breaths/min and shallow • B:Combative; respiratory rate of 24 breaths/min and deep • C:Restless; respiratory rate of 12 breaths/min with adequate tidal volume • D:Semiconscious; respiratory rate of 14 breaths/min and good chest rise

You selected A; This is correct! Reason: Any patient with a decreased level of consciousness should be assessed for inadequate breathing (eg, fast or slow respiratory rate, reduced tidal volume [shallow breathing]). Of the patients listed, the patient who is responsive to pain only and has shallow respirations of 8 breaths/min clearly needs positive-pressure ventilation assistance. Slow, shallow respirations will not produce the minute volume needed to support adequate oxygenation of the blood.

You are performing a secondary assessment on a severely injured patient while en route to a trauma center. During the assessment, you note that the patient's respiratory rate has increased. You should: • A:repeat the primary assessment and treat as needed. • B:immediately notify the receiving facility. • C:assess his oxygen saturation with a pulse oximeter. • D:count the number of respirations per minute

You selected A; This is correct! Reason: Any time a patient's condition deteriorates, such as your patient whose respirations have increased, you should immediately repeat the primary assessment and adjust your treatment accordingly. For example, a patient who initially had adequate breathing may now require assisted ventilation. After stabilizing the patient's condition, reassess his or her vital signs, including oxygen saturation, and notify the receiving facility.

The pain associated with acute aortic dissection: • A:is typically described as a stabbing or tearing sensation. • B:typically comes on gradually and progressively worsens. • C:is usually preceded by nausea, sweating, and weakness. • D:originates in the epigastrium and radiates down both legs.

You selected A; This is correct! Reason: Aortic dissection occurs when the inner layers of the aorta become separated, allowing blood to flow between the layers at high pressure. This separation of layers significantly weakens the aortic wall, making it prone to rupture. Signs and symptoms of acute aortic dissection include a sudden onset of a ripping, tearing, or stabbing pain in the anterior part of the chest or in the back between the scapulae. It may be difficult to differentiate the pain of acute aortic dissection from that of an acute myocardial infarction (AMI), but a number of distinctive features may help. The pain from an AMI is often preceded by other signs and symptoms (ie, nausea, indigestion, weakness, sweating [diaphoresis]). It tends to come on gradually and becomes more severe as time progresses, and is usually described as a crushing pain or as a feeling of heaviness or pressure. By contrast, the pain associated with aortic dissection is acute and is often of maximum intensity from the onset; it is typically described as a ripping, tearing, or stabbing sensation.

When ventilating an apneic patient, you note decreased ventilatory compliance. This means that: • A:the lungs are difficult to ventilate. • B:the upper airway is blocked. • C:you meet no resistance when ventilating. • D:fluid is occupying the alveoli.

You selected A; This is correct! Reason: As it applies to artificial ventilation, compliance is the ability of the lungs to expand during ventilation. Increased ventilatory compliance means that no resistance is met when you ventilate the patient; you can ventilate the lungs with ease. Decreased ventilatory compliance means that significant resistance is met when you ventilate the patient; the lungs are difficult to ventilate. Conditions such as upper airway obstruction, widespread bronchospasm, fluid in the alveoli (eg, pulmonary edema), and COPD can all cause decreased ventilatory compliance.

Basic shock treatment includes: • A:applying oxygen, elevating the lower extremities per protocol, and providing warmth. • B:elevating the lower extremities, applying and inflating the PASG, and applying oxygen. • C:applying and inflating the PASG, applying oxygen, and thermal management. • D:applying oxygen, elevating the upper body, and taking measures to prevent hypothermia.

You selected A; This is correct! Reason: Basic shock treatment, which should be initiated as soon as possible, includes applying high-flow oxygen, elevating the lower extremities 6 to 12 inches (if dictated by your local protocols), and providing warmth. The pneumatic antishock garment (PASG) has been largely abandoned for shock treatment. The theory behind the PASG is that it shunts blood from the lower extremities to the vital organs of the body; however, further research and evidence has shown that this is not the case. The PASG is still used by some EMS systems, but for the purpose of stabilizing pelvic fractures and bilateral femur fractures. Elevation of the upper body in a patient with shock will decrease, not increase, blood flow to the brain.

A patient has severe facial injuries, inadequate breathing, and copious secretions coming from the mouth. How should this situation be managed? • A:Alternate suctioning for 15 seconds and ventilations for 2 minutes. • B:Insert an oropharyngeal airway and suction until the secretions clear. • C:Turn the patient to the side and provide continuous oral suctioning. • D:Provide artificial ventilations and suction for 30 seconds as needed.

You selected A; This is correct! Reason: Both inadequate breathing and secretions in the mouth (ie, blood, vomitus, etc) must be addressed simultaneously. This is best accomplished by suctioning in 15-second increments, then providing assisted ventilations for 2 minutes. This pattern must be continued until the airway is clear of secretions or the airway has been definitively secured (ie, endotracheal intubation). Oral suctioning should not exceed 15 seconds in the adult. The insertion of an airway adjunct should not occur until the airway is clear of secretions or potential obstructions.

Which of the following signs or symptoms is indicative of cerebral hypoxia? • A:Decreased level of consciousness • B:Heart rate greater than 120 beats/min • C:Diffuse wheezing on exhalation • D:Chief complaint of dyspnea

You selected A; This is correct! Reason: Dyspnea, a feeling of shortness of breath, is a symptom of a condition that can cause cerebral hypoxia (eg, CHF, COPD); however, dyspnea itself does not indicate cerebral hypoxia. Wheezing, a whistling sound that indicates bronchospasm, is a sign; like dyspnea, it indicates the presence of a condition that can cause cerebral hypoxia (eg, asthma). Tachycardia can occur for many reasons; cerebral hypoxia is but one. Of the choices listed, a decreased level of consciousness is most indicative of cerebral hypoxia. As oxygen levels in the brain decrease and carbon dioxide levels increase, the patient's mental status deteriorates.

Which of the following describes the MOST appropriate method of performing chest compressions on an adult patient in cardiac arrest? • A:Compress the chest at least 2", allow full recoil of the chest after each compression, minimize interruptions in chest compressions • B:Do not interrupt chest compressions for any reason, compress the chest no more than 1 1/2", allow partial recoil of the chest after each compression • C:Allow full recoil of the chest after each compression, compress the chest to a depth of 2", deliver compressions at a rate of at least 80/min • D:Minimize interruptions in chest compressions, provide 70% compression time and 30% relaxation time, deliver compressions at a rate of 100/min

You selected A; This is correct! Reason: Effective chest compressions are essential for providing blood flow during CPR. To perform adequate chest compressions, the EMT should "push hard and push fast." Compress the chest of an adult at a rate of at least 100 compressions/min to a depth of at least 2". When performing chest compressions on an infant or child, compress the chest at least one third the depth of the chest (about 1 1/2" for infants, about 2" for children). Allow the chest to fully recoil after each compression, and allow equal time for compression and relaxation. Minimize interruptions in CPR to 10 seconds or less. Obviously, chest compressions must be paused when using the AED to analyze the patient's cardiac rhythm or defibrillate and when assessing for a pulse.

Internal or external bleeding would be especially severe in a patient: • A:with hemophilia. • B:with heart disease. • C:who is hypotensive. • D:who takes aspirin.

You selected A; This is correct! Reason: Hemophilia is a condition in which the patient lacks one or more of the blood's clotting factors. There are several forms of hemophilia, most of which are hereditary and some of which are severe. Sometimes bleeding occurs spontaneously in patients with hemophilia. Because the patient's blood does not clot, all injuries, no matter how minor they appear, are potentially serious. Aspirin does not destroy the blood's clotting factors; it decreases the ability of platelets to stick together. Although this may cause prolonged bleeding time, the patient with hemophilia, who lacks key clotting factors, will bleed more severely. Many patients with heart disease take aspirin daily to prevent clot formation in a coronary artery. When blood pressure is low (hypotension), the driving force of the blood through the blood vessels is reduced; as a result, bleeding tends to be less severe relative to patients with high blood pressure. Unfortunately, however, hypotension indicates decompensated shock.

Hypoxia is defined as: • A:inadequate oxygen to the body's cells and tissues. • B:an excess amount of carbon dioxide in arterial blood. • C:decreased oxygen content in arterial blood. • D:an absence of oxygen to the vital body organs.

You selected A; This is correct! Reason: Hypoxia is a dangerous condition in which the body's cells and tissues do not have enough oxygen. Hypoxemia is a decreased amount of oxygen in arterial blood. Untreated hypoxemia will lead to hypoxia. An absence of oxygen to any part of the body is called anoxia. An excess amount of carbon dioxide in arterial blood is called hypercarbia. If the body cannot bring in enough oxygen, it is also unable to eliminate carbon dioxide from the blood; therefore, hypoxemia and hypercarbia occur together.

The AED analyzes your pulseless and apneic patient's cardiac rhythm and advises that a shock is NOT indicated. You should: • A:resume CPR, starting with chest compressions. • B:open the patient's airway and check for breathing. • C:reanalyze the cardiac rhythm for positive confirmation. • D:assess for a pulse for no more than 10 seconds.

You selected A; This is correct! Reason: If the AED advises "no shock," you should immediately resume CPR, starting with chest compressions. Unless the patient starts to move or has other signs of life (ie, coughing), stopping CPR to assess for a pulse should be avoided because it only causes an unnecessary delay in performing chest compressions. The AED is a highly reliable device; if it advises that a shock is or is not indicated, take its word for it! There is no need to reanalyze the patient's cardiac rhythm; this only causes an unnecessary delay in defibrillation (if indicated) and chest compressions

Which of the following occurs during positive-pressure ventilation? • A:Intrathoracic pressure increases • B:The esophagus remains closed • C:Oxygen is pulled into the lungs • D:Blood is drawn back to the heart

You selected A; This is correct! Reason: Negative-pressure ventilation, the act of normal breathing, occurs when the diaphragm and intercostal muscles contract. The actions of these muscles create a vacuum (negative pressure), which pulls oxygen-rich air into the lungs. Because of the negative pressure created in the chest, blood is naturally drawn back to the heart. The esophagus remains closed during normal breathing. In contrast, positive-pressure ventilation involves the forcing of air into the lungs, such as what is provided during rescue breathing. Positive-pressure ventilation causes an increase in intrathoracic pressure, which can impair blood flow back to the heart and cause a decrease in cardiac output. During positive-pressure ventilation, the esophagus is forced open and air enters the stomach (gastric distention); this could result in vomiting and aspiration.

Following blunt trauma to the chest, a 33-year-old male has shallow, painful breathing. On assessment, you note that an area to the left side of his chest collapses during inhalation and bulges during exhalation. These are signs of a/an: • A:flail chest. • B:isolated rib fracture. • C:pneumothorax. • D:pulmonary contusion.

You selected A; This is correct! Reason: If two or more ribs are fractured in two or more places or if the sternum is fractured along with several ribs, a segment of chest wall may be detached from the rest of the thoracic cage. This injury is called a flail chest. In a flail chest, the detached portion of the chest wall moves opposite of normal. It moves in during inhalation and out during exhalation (paradoxical motion). Isolated (single) rib fractures are not associated with paradoxical motion because they are usually only fractured in one place. In a pneumothorax, the patient's respirations are often labored; in severe cases, an entire side of the chest may not move at all (asymmetrical chest movement). A pulmonary contusion (bruising of the lung tissue) does not cause paradoxical chest motion unless associated with a flail chest

During your initial attempt to ventilate an unresponsive apneic patient, you meet resistance and do not see the patient's chest rise. You should: • A:reposition the head and reattempt to ventilate. • B:begin CPR, starting with chest compressions. • C:suction the airway for no longer than 15 seconds. • D:assume that a foreign body is blocking the airway.

You selected A; This is correct! Reason: If your initial attempt to ventilate a patient is met with resistance and/or does not make the chest visibly rise, you should reposition the patient's head and reattempt to ventilate. In many cases, this simple action will open the airway and enable you to ventilate the patient. However, If both of your breaths are met with resistance and/or do not make the chest visibly rise, you should assume that a foreign body is obstructing the airway and begin chest compressions. The airway should be suctioned if secretions are present in the mouth; if oral secretions are not present, do not suction.

You arrive at the scene of a 56-year-old man who collapsed. The patient's wife tells you that he suddenly grabbed his chest and then passed out. Your assessment reveals that he is apneic and pulseless. As your partner begins one-rescuer CPR, you should: • A:prepare the AED for use. • B:notify medical control. • C:insert an airway adjunct. • D:obtain a SAMPLE history.

You selected A; This is correct! Reason: Immediate treatment for a patient in cardiac arrest involves performing CPR and applying the AED as soon as possible. After applying the AED pads to the patient's chest (around your partner's compressing hands), analyze his cardiac rhythm, deliver a shock if indicated, and immediately resume CPR (starting with chest compressions). Management of the airway, including insertion of an airway adjunct, should occur during the 2-minute period of CPR in between cardiac rhythm analysis and defibrillation. While CPR is in progress, obtain as much of the patient's medical history from his wife as possible, and notify medical control when it is practical (ie, you have more help at the scene).

During the inhalation phase of breathing: • A:pressure within the thorax decreases and air is drawn into the lungs. • B:the diaphragm and intercostal muscles contract and ascend. • C:air passively enters the lungs as pressure within the thorax increases. • D:the muscles in between the ribs relax, which lifts the ribs up and out.

You selected A; This is correct! Reason: Inhalation is the active, muscular part of breathing. During inhalation, the diaphragm and intercostal muscles contract. When the diaphragm contracts, it moves down (descends) slightly and enlarges the thoracic cage from top to bottom. Contraction of the intercostal muscles, the muscles in between the ribs, causes the ribs to move up and out. As we inhale, the combined actions of these structures enlarge the thorax in all directions. The air pressure outside the body, called the atmospheric pressure, is normally higher than the air pressure within the thorax. As we inhale and the thoracic cage expands, the air pressure within the thorax decreases, creating a slight vacuum. This draws air in through the trachea and into the lungs, a process called negative-pressure ventilation.

Tachycardia can be detrimental to a patient who is experiencing a cardiac problem because it causes: • A:increased cardiac oxygen usage and demand. • B:increased cardiac filling in between beats. • C:a profound decrease in oxygen consumption. • D:an associated increase in breathing difficulty.

You selected A; This is correct! Reason: Many patients experiencing a cardiac problem are tachycardic (heart rate greater than 100 beats/min); others are bradycardic (heart rate less than 60 beats/min). As the heart beats faster, it consumes, and therefore requires, more oxygen. This can be detrimental to the patient because the heart is already deprived of oxygen (ischemia) and may not be able to accommodate the increased need for oxygen; this may worsen the ischemia, potentially causing further cardiac damage. A very fast heart rate (>150 beats/min) may cause hypotension due to a decrease in cardiac output; it occurs because the ventricles are not adequately filling with blood in between beats. Some patients with a cardiac problem may have difficulty breathing, which may be the result of congestive heart failure and pulmonary edema; it is not caused by the tachycardia itself.

A gang member was cut on the left side of the neck during a fight and is bleeding heavily from the wound. His airway is patent and his breathing is adequate. You should immediately: • A:cover the wound with an occlusive dressing and apply direct pressure. • B:perform a head-to-toe assessment to find and treat other injuries. • C:apply high-flow oxygen via a nonrebreathing mask at 15 L/min. • D:apply a tight pressure dressing and secure it in place with tape.

You selected A; This is correct! Reason: Neck lacerations are extremely dangerous and can result in severe bleeding and shock, air embolism, or both. If a jugular vein is lacerated, air can be sucked into the wound, enter the circulatory system, and cause a pulmonary embolism. You should immediately apply an occlusive dressing to the wound (prevents entrainment of air), place a bulky dressing over the occlusive dressing, and apply direct pressure. Your patient has a patent airway and is breathing adequately; although high-flow oxygen is important and should be given as soon as possible, it does not take priority over bleeding control for this particular patient. After treating all airway, breathing, and circulation problems, perform a head-to-toe assessment (if indicated) and prepare for rapid transport.

Shortly after assisting a 60-year-old woman with her second nitroglycerin treatment, she tells you that she is lightheaded and feels like she is going to faint. Her symptoms are MOST likely due to: • A:low blood pressure. • B:an irregular heartbeat. • C:a drop in her blood sugar. • D:nervousness and anxiety.

You selected A; This is correct! Reason: Nitroglycerin (NTG) is a vasodilator; as such, it may cause a drop in blood pressure (hypotension) in some patients. Signs and symptoms of hypotension include dizziness, lightheadedness, and fainting (syncope), among others. For this reason, you should always assess a patient's blood pressure before and after administering nitroglycerin. If the patient's systolic BP is less than 100 mm Hg, NTG should not be given. The patient is probably nervous and anxious, and may even have an irregular heartbeat; however, hypotension is a more likely cause of her symptoms. NTG does not affect a patient's blood sugar level.

By which of the following mechanisms does nitroglycerin relieve cardiac-related chest pain or discomfort? • A:Vasodilation and increased myocardial oxygen supply • B:Vasoconstriction and increased cardiac oxygen demand • C:Vasoconstriction and increased myocardial workload • D:Vasodilation and decreased myocardial oxygen supply

You selected A; This is correct! Reason: Nitroglycerin is a smooth muscle relaxant. Smooth muscle is found within the walls of the blood vessels. Nitroglycerin causes vasodilation, including dilation of the coronary arteries, which in turn increases the flow of oxygenated blood to the heart and reduces myocardial workload. However, care must be taken when administering nitroglycerin to a patient. Because of its vasodilatory effects, nitroglycerin can cause hypotension; therefore, it should not be given to patients with a systolic blood pressure of less than 100 mm Hg.

Which of the following statements regarding artificial ventilation of an apneic patient who has dentures is correct? • A:Tight-fitting dentures should be left in place because they facilitate the delivery of adequate tidal volume. • B:Because of the risk of airway obstruction, the EMT should routinely remove a patient's dentures. • C:If a patient's dentures are loose, the EMT should use the jaw-thrust maneuver to keep the airway open. • D:The EMT should not attempt to remove a patient's dentures because this may cause an airway obstruction.

You selected A; This is correct! Reason: Providing artificial ventilation with a bag-mask or pocket face mask device is usually much easier when dentures can be left in place. Leaving the dentures in place provides "structure" to the face and will assist you in maintaining a good mask-to-face seal, thus facilitating the delivery of adequate tidal volume. However, loose dentures make it much more difficult to perform artificial ventilation by any method and can easily obstruct the airway. Therefore, dentures and dental appliances that do not stay firmly in place should be removed. When ventilating a patient who has dentures or a dental appliance, periodically reassess his or her airway to ensure they remain firmly in place.

The MOST important initial treatment for a patient whose cardiac arrest was witnessed is: • A:high-quality CPR. • B:rapid transport. • C:defibrillation. • D:cardiac drug therapy.

You selected A; This is correct! Reason: Regardless of whether a patient's cardiac arrest is witnessed or unwitnessed, the single most important initial treatment is high-quality CPR. Delays in performing CPR have been clearly linked to poor patient outcomes. After CPR has been initiated, apply the AED as soon as it is available. Cardiac drug therapy and rapid transport enhance the patient's chance of survival, but are useless without minimally-interrupted, high-quality CPR.

You assess an unresponsive 65-year-old man and find that he is apneic and pulseless. The patient's wife tells you that he has an automatic implanted cardioverter/defibrillator. After initiating CPR, you should: • A:apply the AED as soon as possible and analyze his cardiac rhythm. • B:deactivate the implanted defibrillator by running a magnet over it. • C:avoid using the AED because the implanted defibrillator is more effective. • D:ask the wife why and when he had the automatic defibrillator implanted.

You selected A; This is correct! Reason: Some patients who are at high risk for sudden cardiac arrest due to ventricular fibrillation (V-Fib) have a small automatic implanted cardioverter/defibrillator (AICD). The AICD attaches directly to the heart and continuously monitors the cardiac rhythm, delivering a shock if V-Fib or another lethal dysrhythmia is detected. Regardless of whether the patient has an AICD, he or she should be treated like all other cardiac arrest patients. Perform CPR and use the AED as usual; however, you should ensure that the AED pads are at least 1" away from the implanted device. Generally, the electricity from the AICD is so low that it will have no effect on rescuers and therefore should not be of concern to you. Do not deactivate an implanted AICD, especially if it is working and delivering shocks as it is supposed to. When treating a cardiac arrest patient who has an AICD, your priority is to provide CPR and defibrillate with the AED if indicated, not to determine when and why the AICD was implanted

The MOST effective way to determine if you are providing adequate volume during artificial ventilation is: • A:assessing the chest for adequate rise. • B:checking the skin for improvement of cyanosis. • C:assessing the pulse for an improving heart rate. • D:checking the pupils for increased reactivity

You selected A; This is correct! Reason: The goal of providing artificial ventilation is to provide adequate tidal volume to the patient so that enough oxygen is delivered to the lungs, and ultimately, the cells of the body. The most effective way to determine if adequate tidal volume is being delivered is to watch for the chest to rise during each ventilation. Other signs of adequate artificial ventilation include improvement in skin color, the return of the heart rate to a normal range, and ensuring that you are ventilating the patient at the appropriate rate. If the adult is apneic but has a pulse, provide 10 to 12 breaths/min. If the adult is apneic and pulseless, provide 8 to 10 breaths/min after an advanced airway device (ie, ET tube, multilumen airway, supraglottic airway) has been inserted.

To obtain the MOST reliable assessment of a patient's tidal volume, you should: • A:look at the rise of the chest. • B:assess for retractions. • C:count the respiratory rate. • D:listen for airway noises.

You selected A; This is correct! Reason: Tidal volume is the amount of air, in milliliters, breathed into or out of the lungs in a single breath. The most effective (and practical) way to assess tidal volume is to evaluate the rise of the patient's chest. If the patient's chest rises minimally during inhalation, his or her respirations are shallow; shallow respirations reflect a reduced tidal volume.

An unresponsive patient with multi-systems trauma has slow, shallow breathing; weak radial pulses; and severe bleeding from a lower extremity wound. You should direct your partner to: • A:assist the patient's ventilations while you control the bleeding. • B:apply oxygen via nonrebreathing mask while you control the bleeding. • C:prepare the long spine board and straps for rapid spinal immobilization. • D:radio for a paramedic ambulance to respond to the scene.

You selected A; This is correct! Reason: The goal of the primary assessment is to rapidly identify and correct all life-threatening injuries or conditions. In the case of this patient, as your partner maintains in-line cervical spine control, he or she should assist the patient's ventilations. An unresponsive patient with slow, shallow breathing is not breathing adequately and should be treated with ventilatory assistance, not a nonrebreathing mask. As your partner is managing the patient's airway and providing ventilatory assistance, you should apply direct pressure (or a tourniquet, if needed) to the extremity wound to control the bleeding. It is important for you and your partner to work together so that all life-threats can be corrected as soon as possible. Most EMS systems work with two-person crews and do not have the luxury of a third EMT. If the police or fire department is on the scene, you can ask them to gather equipment for you. The request for an ALS ambulance is based on factors such as the patient's condition and transport time to the closest appropriate hospital.

In which of the following situations should the jaw-thrust maneuver be used? • A:When the mechanism of injury is unclear • B:In any patient who is in cardiac arrest • C:In a patient who is in need of frequent suctioning • D:In a patient with apnea with no signs of trauma

You selected A; This is correct! Reason: The jaw-thrust maneuver should be used to open the airway any time the mechanism of injury suggests trauma or when the mechanism of injury is unclear (ie, in a patient who became unresponsive without witnesses). When performed correctly, the jaw-thrust maneuver maintains a patent airway without manipulating the spine. It should be noted, however, that if the jaw-thrust maneuver does not adequately open the patient's airway, the head tilt-chin lift maneuver should be used.

If an adult patient presents with a respiratory rate of 26 breaths/min, your initial action should be to: • A:evaluate his mental status and the depth of his respirations. • B:begin assisting his ventilations with a bag-mask device. • C:apply oxygen via a nonrebreathing mask and take his vital signs. • D:apply the pulse oximeter and assess his oxygen saturation.

You selected A; This is correct! Reason: The normal respiratory rate for an adult at rest is 12 to 20 breaths/min. If a patient presents with a respiratory rate outside of the normal range, you should immediately assess him or her for other signs of inadequate breathing, such as a decreased level of consciousness, shallow breathing (reduced tidal volume), brief inhalations followed by prolonged exhalations, and cyanosis. If the patient is conscious, alert, and has adequate tidal volume (eg, his or her chest rises adequately with each breath), supplemental oxygen via nonrebreathing mask or nasal cannula would be appropriate, depending on his or her chief complaint and oxygen saturation. However, if the patient's mental status is decreased and his or her tidal volume is reduced (eg, shallow breathing), some form of positive-pressure ventilation should be initiated (eg, bag-mask or pocket face mask ventilations). It is important to note that breathing adequacy is not determined solely by the patient's respiratory rate; you must assess all aspects of breathing (rate, regularity, depth) as well as the patient's mental status. A patient can be breathing at a "normal" rate; however, if his or her tidal volume is reduced, minute volume will decrease and some form of positive-pressure ventilation may be required

Which of the following patients is the BEST candidate for an oropharyngeal airway? • A:An unresponsive trauma patient with blood draining from the nose • B:Any patient that you suspect of being acutely hypoxemic • C:An unresponsive patient with uncontrolled oropharyngeal bleeding • D:A semiconscious patient who ingested a large quantity of aspirin

You selected A; This is correct! Reason: The oropharyngeal airway is used to keep the tongue off of the posterior pharynx and is indicated for unresponsive patients without a gag reflex. If an unresponsive patient has severe, uncontrolled oropharyngeal bleeding, your priority is to suction his or her airway in order to prevent aspiration and transport rapidly. Semiconscious patients typically have a gag reflex, although it may be somewhat depressed. Oxygen should be administered to any patient with suspected hypoxemia.

A patient who is experiencing an acute myocardial infarction: • A:has chest pain or discomfort that does not change with each breath. • B:often complains of a different type of pain than a patient with angina. • C:most often describes his or her chest pain as being sharp or tearing. • D:often experiences relief of his or her chest pain after taking nitroglycerin. You selected A; This is correct!

You selected A; This is correct! Reason: The type of chest pain or discomfort associated with acute myocardial infarction (AMI) is the same that is experienced by patients with angina pectoris (eg, dull, crushing, pressure, heaviness); thus, you cannot distinguish AMI from angina pectoris based solely on the type or quality of pain. Furthermore, the pain associated with AMI, like that of angina, often radiates to the arm, jaw, back, or epigastrium. Relative to other causes of chest pain or discomfort (eg, pleurisy, pneumothorax), the pain associated with AMI and angina does not worsen or improve when the patient takes a breath. Rest and nitroglycerin often relieve the pain associated with stable angina, but are less likely to relieve the pain associated with AMI.

A 56-year-old man has labored, shallow breathing at a rate of 28 breaths/min. He is responsive to pain only. You should: • A:insert a nasopharyngeal airway and begin assisting his ventilations. • B:place him on his side and administer oxygen via nonrebreathing mask. • C:suction his mouth for 15 seconds and insert an oropharyngeal airway. • D:ventilate him with a bag-mask device at a rate of 30 breaths/min.

You selected A; This is correct! Reason: This patient in this scenario is not breathing adequately. He is responsive to pain only, and his respirations are rapid, labored, and shallow. You should insert a nasopharyngeal airway, which is usually well-tolerated in patients who are semiconscious and have a gag reflex, and assist his ventilations with a bag-mask device. When assisting a patient's breathing, you should squeeze the bag-mask device to ensure that he or she receives 10 to 12 adequate breaths per minute. Do not hyperventilate the patient as this increases the risks of vomiting and aspiration. Hyperventilation also increases intrathoracic pressure, which may impair venous return to the heart (preload) and cause a decrease in cardiac output. Oxygen via nonrebreathing mask is appropriate for patients who are breathing adequately, but are suspected of being hypoxic. The recovery position (patient is placed on his or her side) is appropriate for unresponsive, uninjured patients with adequate breathing.

You arrive at the scene shortly after a 55-year-old man collapsed. Two bystanders are performing CPR. Your FIRST action should be to: • A:stop CPR so you can assess breathing and pulse. • B:attach the AED and analyze his cardiac rhythm. • C:check the effectiveness of the CPR in progress. • D:insert an oropharyngeal airway and continue CPR

You selected A; This is correct! Reason: Upon arriving at a scene where bystander CPR is in progress, you must first confirm that the patient is indeed apneic and pulseless and needs CPR. Bystanders who are not properly trained often perform CPR on patients who do not need it. After confirming cardiac arrest, you should resume CPR and attach the AED as soon as possible.

A 42-year-old man was ejected from his car after it struck a bridge pillar at a high rate of speed. You find him in a prone position approximately 50 feet from his car. He is not moving and does not appear to be breathing. You should: • A:manually stabilize his head. • B:administer high-flow oxygen. • C:use the jaw-thrust maneuver. • D:assess his breathing effort.

You selected A; This is correct! Reason: When a trauma patient is found in a prone (face-down) position, especially if he or she is unresponsive, your first action should be to manually stabilize his or her head; this action is based on the assumption that he or she has a spinal injury. Next, log roll the patient to a supine position (while continuing to manually stabilize the head), open the airway with the jaw-thrust maneuver, clear the airway with suction if needed, and assess for breathing. It would be extremely difficult to adequately open the patient's airway while he or she is in a prone position. Depending on the patient's breathing effort, administer high-flow oxygen or ventilate using a bag-mask device.

When assessing and treating a patient with a gunshot wound, you should routinely: • A:look for the presence of an exit wound. • B:determine why the patient was shot. • C:apply ice directly to the wound. • D:evaluate the pulses proximal to the wound.

You selected A; This is correct! Reason: When assessing a patient who sustained a gunshot wound, you should routinely look for an exit wound, which may be difficult to find. Exit wounds can be a source of continued bleeding, both externally and internally. They may or may not follow the same path as the entrance wound. This is why it is important to conduct a thorough examination of the patient. Ice can be applied to the wound, but only after the wound has been covered by a sterile dressing. Determining why the patient was shot is the responsibility of law enforcement, not the EMT. If the wound is close to an extremity, pulse, motor, and sensory function should be assessed distal to the wound.

When suctioning copious secretions from a semiconscious adult's airway, you should: • A:avoid touching the back of the airway with the suction catheter. • B:suction for up to 20 seconds while withdrawing the catheter. • C:use a flexible catheter because it will remove the secretions faster. • D:apply suction as you carefully insert the catheter into the mouth.

You selected A; This is correct! Reason: When suctioning a patient's airway, especially if he or she is semiconscious, you should avoid touching the back of the airway with the suction catheter. Inserting the catheter too far may stimulate the gag reflex, cause vomiting, and increase the risk of aspiration. Rigid (tonsil-tip) catheters are best for removing large amounts of fluid from the airway. Flexible (whistle-tip) catheters are used in situations in which rigid catheters cannot be used, such as with a patient who has a stoma, patients whose teeth are clenched, or if suctioning the nose is necessary. Apply suction while you are withdrawing the catheter. In the adult, suction for no longer than 15 seconds (10 seconds in children, 5 seconds in infants); suction not only removes secretions from the airway, it also removes oxygen.

During your assessment of a patient with a gunshot wound to the chest, you note that his skin is pale. This finding is the result of: • A:decreased blood flow to the skin. • B:a significantly elevated heart rate. • C:a critically low blood pressure. • D:peripheral dilation of the vasculature.

You selected A; This is correct! Reason: When the body attempts to compensate for shock, peripheral vasoconstriction shunts blood away from the skin to the more vital organs in the body such as the brain, heart, lungs, and kidneys. When there is minimal or no peripheral blood flow, the skin assumes a pale appearance. By contrast, when peripheral circulation increases (ie, vasodilation), the skin assumes a red (flushed) appearance. Pallor does not necessarily indicate hypotension. Tachycardia is a compensatory response of the nervous system in an attempt to increase cardiac output and maintain blood pressure

You are ventilating an apneic 50-year-old woman with a bag-mask device. After squeezing the bag and noting visible chest rise, you should: • A:allow the patient to completely exhale. • B:squeeze the bag again in 3 seconds. • C:reopen the airway and ventilate again. • D:suction the airway for up to 15 seconds.

You selected A; This is correct! Reason: When ventilating an apneic patient, it is important to allow for complete exhalation. To do this, deliver each breath over 1 second, just enough to make the chest visibly rise, and then deliver the next breath 5 to 6 seconds later (3 to 5 seconds later for infants and children). Failure to allow for complete exhalation may cause the patient to retain carbon dioxide (carbon dioxide elimination occurs during exhalation) and may also impair venous return to the heart secondary to hyperinflation of the lungs. If you observe visible chest rise when you ventilate the patient, there is no need to reopen the airway. Suction the airway only if secretions are present.

A newborn is considered to be premature if it: • A:is born before 37 weeks gestation. • B:weighs less than 6.5 pounds. • C:is born to a heroin-addicted mother. • D:has meconium in or around its mouth.

You selected A; This is correct! Reason: A term gestation is between 37 and 42 weeks. A premature newborn is one that is born before 37 weeks gestation or weighs less than 5.5 lb (2.5 kg). Compared to women who do not abuse drugs, smoke, or drink alcohol during pregnancy, women who do are more likely to deliver prematurely or deliver a low-birth-weight baby. The risk of the fetus voiding its first bowel movement (meconium) in utero increases any time the fetus is distressed, regardless of its gestational age or weight; the presence of meconium does not define a premature newborn.

As an EMT, your primary responsibility is to: • A:keep yourself as safe as possible. • B:transport all patients to the hospital. • C:ensure the safety of your partner. • D:provide competent patient care.

You selected A; This is correct! Reason:As an EMT, your primary responsibility is to yourself. An injured or dead EMT is of no use to a patient. After ensuring the safety of yourself, your crew, and any bystanders, patient care should be initiated.

When is it MOST appropriate to clamp and cut the umbilical cord? • A:As soon as the cord has stopped pulsating • B:Immediately following delivery of the newborn • C:Before the newborn has taken its first breath • D:After the placenta has completely delivered

You selected A; This is correct! Reason: Generally, it is safe to clamp and cut the umbilical cord once it has stopped pulsating and the baby is breathing adequately. When blood flow through the umbilical cord ceases, it will stop pulsating; this indicates that the baby is oxygenating its own blood. If the cord does not stop pulsating and/or the baby is not breathing adequately, the cord should not be clamped and cut and the baby should be kept at the level of the mother's perineum and managed appropriately while en route to the hospital.

During your assessment of a woman in labor, you see the baby's arm protruding from the vagina. The mother tells you that she needs to push. You should: • A:cover the arm with a sterile towel and transport immediately. • B:gently push the protruding arm back into the vagina. • C:encourage the mother to push and give her high-flow oxygen. • D:insert your gloved fingers into the vagina and try to turn the baby.

You selected A; This is correct! Reason: On rare occasions, the presenting part of the fetus is neither the head nor buttocks, but a single arm or leg. This is called a limb presentation. You cannot successfully deliver such a presentation in the field. These infants usually must be delivered surgically. If you encounter a limb presentation, transport immediately. Cover the protruding limb with a sterile towel. If the mother feels the urge to push, you should discourage this; pushing may put pressure on the fetus, potentially causing injury. Never try to push the limb back in, and never pull on it. Place the mother on her back, with head down and pelvis elevated. Because both mother and fetus are likely to by physically stressed in this situation, give the mother high-flow oxygen.

Which of the following statements BEST describes a mass-casualty incident? • A:The number of patients overwhelms your resources. • B:More than three vehicles are involved in the incident. • C:More than five patients are involved. • D:At least half of the patients are critically injured.

You selected A; This is correct! Reason:A mass-casualty incident (MCI) occurs any time the number of injured patients overwhelms your available resources. It is not necessarily defined by the number of patients, but rather your ability to effectively manage them. For example, if you and your partner arrive at a scene and find two critically injured patients, you have an MCI (albeit a small-scale MCI) because two EMTs and one ambulance can only care for one critically ill or injured patient effectively. While caring for a trauma patient, blood splashes into an EMT's eyes. This is an example of: • A:infection. • B:indirect contact. • C:transmission. • D:exposure.

You are transporting a 35-year-old male who has a history of alcoholism. He stopped drinking 4 days ago and is now disoriented, diaphoretic, and tachycardic. You should be MOST concerned that he: • A:may have a seizure. • B:is severely dehydrated. • C:will develop a high fever. • D:will begin hallucinating

You selected A; This is correct! Reason:A patient in alcohol withdrawal may experience frightening hallucinations, or delirium tremens (DTs), a syndrome characterized by restlessness, fever, disorientation, sweating, agitation, and even seizures. About 1 to 7 days after a person stops drinking or when alcohol consumption levels are suddenly decreased, DTs may develop. Patients with DTs are often dehydrated due to sweating, fluid loss, insufficient fluid intake, or vomiting, and their hallucinations can be extremely frightening. However, the possibility of a seizure, clearly the most life-threatening complication associated with DTs, should concern you the most.

After assisting a patient with her epinephrine auto-injector, you should: • A:place the device in a puncture-proof container. • B:place the device in a red biohazard bag. • C:give it to the patient to have it refilled. • D:replace the cover and place it in the trash can.

You selected A; This is correct! Reason:After any device is used that has the potential for causing an accidental needle stick or is otherwise contaminated, it should be placed in a puncture-proof container, which usually is red and has a biohazard logo on it. The cover of the auto-injector should never be replaced, nor should a needle be recapped. Epinephrine auto-injectors are not refillable.

After moving a hypothermic patient to a warmer area, your primary focus should be to: • A:prevent further body heat loss. • B:provide rapid rewarming. • C:assess his or her body temperature. • D:give warm, humidified oxygen.

You selected A; This is correct! Reason:After moving a hypothermic patient to a warmer area, your primary focus should be to prevent further body heat loss. Remove wet or damp clothing and cover the patient with blankets (passive rewarming). It is especially important to cover the patient's head; most heat is lost around the head and neck. You should not, however, try to actively rewarm the patient (ie, placing heat on or into the body). Rewarming too quickly may induce a lethal cardiac dysrhythmia and cardiac arrest. For this reason, active rewarming should be done in the controlled setting of a hospital. In some areas, medical control may call for the administration of warm, humidified oxygen; follow your local protocols. Assessing a hypothermic patient's core body temperature requires a special hypothermia thermometer, which many EMS systems do not carry.

When caring for any patient with a decreased level of consciousness, your primary concern should be the: • A:potential for airway compromise. • B:patient's blood glucose level. • C:possibility of a spinal injury. • D:possibility of a drug overdose.

You selected A; This is correct! Reason:Altered mental status could be caused by a high or low blood glucose level, drug overdose, or head injury, among other causes. Furthermore, the possibility of a spinal injury should be considered if the patient was injured. However, your primary concern should be the status of the patient's airway. Patients with a decreased level of consciousness are at risk for aspiration if vomiting occurs. Unless spinal trauma is present or the patient is breathing inadequately (eg, fast or slow rate, shallow breathing [reduced tidal volume]), place him or her in the recovery position to facilitate drainage if vomiting occurs. Remember this: no airway, no patient!

In most states, the EMT is required to report which of the following occurrences? • A:Animal bite • B:Motor vehicle crash • C:Injury to a minor • D:Drug overdose

You selected A; This is correct! Reason:Although each state may have slightly differing reporting laws, most require the EMT to report cases such as child or elderly abuse, sexual assault, animal bites, and injury that occurs during the commission of a crime. Injury to a minor is typically not a reportable case unless abuse is suspected. Motor vehicle crashes and drug overdoses are not reportable cases either, unless they occur during the commission of a crime.

When providing care to multiple patients at the scene of a mass-casualty incident, your goal should remain focused on: • A:transporting patients to the hospital. • B:keeping all bystanders at a safe distance. • C:initiating CPR for those in cardiac arrest. • D:immobilizing all patients at the scene.

You selected A; This is correct! Reason:At the scene a mass-casualty incident, you will be faced with many challenges, including ensuring your safety, extrication, triage, and patient care. In the midst of all of these activities, however, you must never lose sight of your ultimate goal, which is to transport all patients to the hospital as soon as possible.

You are responding to a call for a 2-year-old child who fell from a second-story window. With the mechanism of injury and the age of the patient in mind, you should suspect that the primary injury occurred to the child's: • A:head. • B:lower extremities. • C:abdomen. • D:chest.

You selected A; This is correct! Reason:Because a child's head is proportionately larger than the rest of the body when compared to an adult, the head commonly is the primary site of injury. This is especially true in fall-related injuries, in which gravity causes the head to precede the rest of the body. Head injury is a leading cause of traumatic death in infants and small children.

A sudden onset of respiratory distress in a 5-year-old child with no fever is MOST likely the result of: • A:a foreign body airway obstruction. • B:inflammation of the upper airway. • C:infection of the lower airways. • D:a progressive upper airway infection.

You selected A; This is correct! Reason:Children with no fever who have a sudden onset of respiratory distress should be treated for a foreign body airway obstruction. If the child is able to cough, cry, or speak, he or she is moving adequate air and has a mild airway obstruction. If the child is unable to cough, cry, or speak, and if he or she is cyanotic and has a decreased level of consciousness, he or she is not moving adequate air and has a severe airway obstruction. Epiglottitis, a bacterial infection of the upper airway, also causes a sudden onset of respiratory distress, but is accompanied by a high fever. Croup, a viral infection of the upper airway, typically does not present acutely; however, it is often accompanied by a low-grade fever. Lower airway infections (ie, bronchitis, bronchiolitis) typically present with a progessive onset of respiratory distress and abnormal lung sounds, such as wheezing and rhonchi.

Which of the following actions should be carried out during the primary assessment of an unresponsive patient? • A:Assessing the skin • B:Obtaining a blood pressure • C:Palpating the cranium • D:Auscultating the lungs

You selected A; This is correct! Reason:Components of the primary assessment for both responsive and unresponsive patients include assessing and managing the airway and assessing and managing circulation, which includes controlling any major bleeding; assessing the rate, regularity, and quality of the pulse; and assessing the color, condition, and temperature of the skin.

While functioning at a large-scale terrorist incident, it is important for the EMT to: • A:use triage and base patient care on available resources. • B:avoid placing any casualty in a "delayed" treatment status. • C:identify the person or persons responsible for the event. • D:begin immediate treatment of the most critically injured.

You selected A; This is correct! Reason:During a terrorist incident, the basic foundations of triage and patient care remain the same; however, the treatment can and will vary. Terrorist incidents can produce a single casualty, hundreds of casualties, or thousands of casualties. When presented with wide-spread mass casualties, you must remember situational awareness. What you do in one situation may not be appropriate for another situation. In large-scale terrorist incidents, it is important to use triage and base patient care on available resources. When triaging casualties, use the same triage process that you would for any other mass-casualty incident. Remain focused on providing the greatest good for the greatest number of people, not the person or persons responsible for the incident.

You are caring for a conscious patient who you believe is having an acute ischemic stroke. After administering oxygen, your next priority should include: • A:providing prompt transport for possible fibrinolytic therapy. • B:administering 1 tube of oral glucose to rule out hypoglycemia. • C:closely monitoring the blood pressure every 15 to 20 minutes. • D:determining whether the patient has prescribed nitroglycerin.

You selected A; This is correct! Reason:Fibrinolytic (clot-buster) therapy is critical to a patient who is having a stroke if it is initiated within 3 hours after the onset of symptoms. It is critical to ask a family member or bystander when the patient was last seen normal. In addition to providing supplemental oxygen, transport the patient without delay so that she may receive fibrinolytic therapy (if she is a candidate) at the hospital. Because fibrinolytic therapy decreases the blood's ability to clot, its use is limited to patients with ischemic strokes. It would increase intracerebral bleeding in patients with hemorrhagic stroke; therefore, fibrinolytic therapy is contraindicated for these patients. Hypoglycemia can easily be ruled out by assessing the patient's blood glucose level with a glucometer. Nitroglycerin is of no relevance in this scenario because the patient is not suspected of having a cardiac event. Monitor the patient's vital signs every 5 to 15 minutes, or as dictated by her clinical condition, while en route to the hospital.

Proper body mechanics when lifting and moving a patient include: • A:keeping the weight as close to you as possible. • B:twisting at the waist when moving around a corner. • C:using the muscles of your lower back to lift. • D:maintaining a slight curvature of your back.

You selected A; This is correct! Reason:General guidelines for safe lifting and moving include keeping the weight as close to your body as possible; keeping your back in a straight, locked-in position; using the muscles of your thighs to lift; and avoiding twisting when moving a patient around a corner. Back injuries are the most common injury sustained by the EMT and can be easily avoided if proper lifting and moving techniques are observed.

Which of the following signs and symptoms are MOST characteristic of hyperglycemic ketoacidosis? • A:Warm, dry skin and a slow onset • B:Warm, dry skin and a rapid onset • C:Cool, clammy skin and a rapid onset • D:Cool, clammy skin and a slow onset

You selected A; This is correct! Reason:Hyperglycemic ketoacidosis (diabetic coma) is characterized by a dangerously high blood glucose level (hyperglycemia); slow onset; warm, dry skin (from dehydration); and Kussmaul's respirations, which are deep and rapid and have a fruity or acetone odor. Insulin shock results from a low blood glucose level (hypoglycemia) and is characterized by a rapid onset; altered mental status; and cool, clammy skin.

Supplemental oxygen via the blow-by technique is MOST appropriate for a child who presents with respiratory difficulty and: • A:is agitated, tachycardic, and clinging to his or her parent. • B:has a heart rate of 70 beats/min and signs of physical exhaustion. • C:is breathing with a significant reduction in tidal volume. • D:has facial cyanosis and a decreased level of consciousness.

You selected A; This is correct! Reason:If a child presents with respiratory difficulty, the method of oxygen delivery depends on his or her mental status, respiratory effort, and heart rate. A child with respiratory distress has an increased work of breathing, is agitated and tachycardic, and is clinging to his or her parent. Oxygen for a child with respiratory distress should be given by the least threatening method. You should avoid further agitation of the child, which may cause deterioration of his or her condition. Give the child oxygen via the blow-by technique; allow the parent to hold the mask or oxygen tubing near the child's face. By contrast, respiratory failure in the child is characterized by a decreased level of consciousness, signs of physical exhaustion, reduced tidal volume (shallow breathing), cyanosis, and bradycardia. Children with respiratory failure need assisted ventilation with a bag-mask device and high-flow oxygen. Remember, respiratory failure is the most common cause of cardiac arrest in infants and children

When using the power lift to lift a stretcher, you should: • A:ensure that you lift with your palms facing up. • B:place your hands palms down on the stretcher. • C:maintain a slight inward curve to your back. • D:bend at the waist and keep your back straight.

You selected A; This is correct! Reason:In order to achieve the best grip and to avoid injury to your wrists, you should lift a stretcher, backboard, or other carrying device with your palms facing up. Do not bend at the waist; bend at the knees and keep your back in a straight, locked-in position.

In order to maintain neutral alignment of an 18-month-old child's airway, you should: • A:pad in between the shoulder blades. • B:hyperextend the head. • C:ensure that the head is slightly flexed. • D:place a rolled towel behind the head.

You selected A; This is correct! Reason:Infants and small children have proportionately large heads, specifically the occiput (back of the head). Therefore, it is often necessary to place padding in between the scapulae (shoulder blades) in order to ensure neutral alignment of the head. Padding behind the head places the child's head in the sniffing position, which is used to facilitate intubation. If the infant or child's head is hyperextended, the large occiput may push the head forward, resulting in hyperflexion. Flexing the child's head will collapse the trachea, resulting in obstruction of the airway.

Most crashes involving ambulances occur: • A:at intersections. • B:at stop lights. • C:at stop signs. • D:on the highway.

You selected A; This is correct! Reason:Intersection crashes are the most common and usually the most serious type of collision in which ambulances are involved. When approaching an intersection, you should come to a complete stop, look in both directions for pedestrians and other motorists, and then proceed with caution. Remember, your lights and siren do NOT give you the right of way; they ask other motorists for the courtesy of the right of way. If you proceed through an intersection without stopping and strike another vehicle that had the right of way, you will be held liable. Whether or not your lights and siren were in use at the time of the incident is irrelevant.

As you step out of the ambulance at the scene of a nighttime motor vehicle crash on the highway, your MOST immediate concern should be: • A:the presence of oncoming traffic. • B:whether the car will catch on fire. • C:rapid assessment of all injured patients. • D:placing safety flares by the ambulance.

You selected A; This is correct! Reason:Nighttime traffic crashes, especially those that occur on a highway, pose a significant risk to the safety of the EMT. Therefore, immediately upon exiting the ambulance, the concern for oncoming traffic should be at the front of the EMT's mind. Drivers can be blinded by all of the emergency lighting and ¬inadvertently veer off of the road and strike the rescuer. Safety flares are used less commonly than in the past; they have been known to blow off the road and start grass fires or ignite gasoline leaking from a vehicle. Reflective cones or triangles are safer and are used more often than flares. After ensuring your own safety, which includes notifying the fire department if the vehicle is leaking gas, you should then proceed to the patients.

When you arrive at a residence for a man who is "not acting right," you enter the house and find him sitting on his couch. Which of the following findings would be MOST indicative of an altered mental status? • A:Slurred speech. • B:Tired appearance. • C:Odor of alcohol. • D:Eyes are closed.

You selected A; This is correct! Reason:Often, an altered mental status can be difficult to assess, especially if you do not know how the patient normally acts. However, there are key findings that should increase your index of suspicion. An abnormal speech pattern, such as slurring or incoherent words, can be the result of a diabetic problem, alcohol intoxication, or drug ingestion. All of these can cause an altered mental status. The odor of alcohol suggests intoxication as a potential cause of his problem, but cannot be quantified. Just because the patient's eyes or closed or he has a tired appearance does not necessarily indicate that he has an altered mental status.

Which of the following patients with diabetes is the BEST candidate for oral glucose? • A:A confused patient who has cool, clammy skin • B:A semiconscious patient with pale, clammy skin • C:An unresponsive patient who took too much insulin • D:A confused patient with suspected hyperglycemia

You selected A; This is correct! Reason:Oral glucose is indicated for patients with diabetes who are in insulin shock (hypoglycemic crisis), or for any patient with symptomatic hypoglycemia (as documented by glucometer). The patient must be conscious and alert enough to be able to swallow the glucose, which comes in a tube of gel. If the patient is unresponsive or otherwise unable to swallow the glucose, you should provide rapid transport, providing the appropriate airway management en route, and consider an ALS intercept.

You should be MOST suspicious that a patient with chest pressure has an underlying cardiac problem if his or her pulse is: • A:slow. • B:irregular. • C:rapid. • D:bounding.

You selected B; This is correct! Reason: Of the choices listed, an irregular pulse should make you the most suspicious that a patient with chest pain, pressure, or discomfort has an underlying cardiac problem. An irregular pulse indicates a cardiac dysrhythmia (abnormal cardiac rhythm), which may be a precursor to cardiac arrest. A fast pulse (tachycardia), slow pulse (bradycardia), or bounding pulse can be caused by numerous conditions, not all of which are cardiac related.

A 50-year-old woman with a history of epilepsy is actively seizing. Care for this patient should focus primarily on: • A:protecting her from injury and ensuring adequate ventilation. • B:administering high-flow oxygen and requesting an ALS ambulance. • C:frequently suctioning her airway and carefully restraining her. • D:placing a bite block in between her molars and giving her oxygen.

You selected A; This is correct! Reason:Seizure deaths are most frequently the result of hypoxia. When a person is actively seizing, he or she is not breathing adequately. Your primary focus when treating a seizure patient is to protect him or her from injury and to ensure adequate ventilation and oxygenation. Many seizing patients require assisted ventilation. Suction the oropharynx only if the patient has secretions in his or her mouth. Do NOT insert anything into the mouth of a seizing patient; doing so may cause an airway obstruction or damage the soft tissues of the mouth, resulting in bleeding. Do not attempt to restrain an actively seizing patient; doing so may result in musculoskeletal injuries. Request an ALS ambulance per your local protocols.

A 34-year-old female complains of persistent fever, fatigue, and night sweats. During your assessment, you note that she has purple blotches on her arms and legs. She MOST likely has: • A:HIV/AIDS. • B:chickenpox. • C:tuberculosis. • D:viral hepatitis.

You selected A; This is correct! Reason:Signs of human immunodeficiency virus (HIV) infection include persistent fever, weight loss, fatigue, a cough, and night sweats. The presence of purple skin blotches (malignant lesions called Kaposi's sarcoma) is consistent with acquired immune deficiency syndrome (AIDS), also known as late stage HIV infection. Patients with tuberculosis (TB) also present with fever, fatigue, weight loss, a cough, and night sweats; however, skin lesions are not common. Hepatitis is characterized by right upper quadrant abdominal pain, fever, nausea and vomiting, and a yellow tint to the skin and sclera (jaundice). Chickenpox is unlikely; the associated rash is characterized by pustules that crust over, not purple blotches.

A behavioral crisis is MOST accurately defined as: • A:any reaction to an event that interferes with the activities of daily living or has become unacceptable to the patient, family, or community. • B:an acute psychiatric emergency characterized by violent behavior, mood swings, and a loss of connection to reality. • C:a persistent feeling of sadness, despair, or hopelessness that incapacitates the patient and prevents him or her from interacting socially. • D:a chronic mental health problem in which the patient experiences frequent thoughts of suicide or other self-destructive behavior.

You selected A; This is correct! Reason:The definition of a behavioral crisis or emergency is any reaction to an event that interferes with the activities of daily living (eg, bathing, dressing, eating) or has become unacceptable to the patient, his or her family, or the general community. Some patients react to an event with violent behavior or suicidal thoughts; others react with depression. Regardless of the reaction, it significantly interferes with the patient's life or is unacceptable to his or her family and/or the community

Which of the following structures is NOT part of the endocrine system? • A:Gallbladder • B:Pancreas • C:Thyroid • D:Pituitary

You selected A; This is correct! Reason:The gallbladder, which concentrates and stores bile, is not an endocrine organ; it is a digestive organ. Endocrine organs produce hormones, which regulate other body organs and systems. The thyroid regulates metabolism; the pancreas produces insulin and glucagon; and the pituitary gland, which is located within the brain, is the "master" endocrine gland and regulates the function of all endocrine glands in the body.

The information that would be of LEAST pertinence when educating the public on injury prevention is: • A:how to provide rescue breathing. • B:teaching children to wear bicycle helmets. • C:the proper usage of child safety seats. • D:building a childproof fence around a pool.

You selected A; This is correct! Reason:The goal of an injury prevention program is just that, prevention. If rescue breathing is needed in a situation, the injury has already occurred. As EMS providers, we are consequence managers. Additionally, we have a responsibility to educate the public on how to avoid injuries in the first place.

When the incident command system is activated at the scene, you should expect to: • A:report back to your section officer in between assignments. • B:receive instructions and then function independently. • C:be assigned one responsibility for the duration of the incident. • D:be immediately directed to the established treatment area.

You selected A; This is correct! Reason:The incident command system (ICS) was established in order to maximize effective operations at the scene and maximize the number of lives saved. Deviation from the ICS jeopardizes lives and increases the risk of losing control over the situation. When you arrive at the scene in which the ICS has been activated, you should report to the staging area, where you will be directed to the area in which you are needed. Report to the section officer of that area, receive your instructions, and carry them out. When you have completed your assignment, you must return to the section officer for further instructions. Depending on the situation, you may be sent to another section. The sections that you work in and the responsibilities that you are given may change during the incident. At no time should you attempt to function independently (freelancing); this defeats the purpose of the ICS and puts lives in jeopardy.

Which of the following organs are contained within the right upper quadrant of the abdomen? • A:Liver and gallbladder • B:Stomach and gallbladder • C:Liver and spleen • D:Liver and stomach

You selected A; This is correct! Reason:The liver and gallbladder lie within the right upper quadrant of the abdomen. Most of the stomach is within the left upper quadrant, as is the entire spleen.

When applying a vest-style spinal immobilization device to a patient with traumatic neck pain, you should: • A:gently flex the head forward as you position the device. • B:secure the torso section prior to immobilizing the head. • C:immobilize the head prior to securing the torso straps. • D:ask the patient to fully exhale as you secure the torso.

You selected B; This is correct! Reason: When you apply a vest-style immobilization device such as a KED, you must immobilize the patient's head after the torso is adequately secured. If you immobilize the head first, the cervical spine may be unnecessarily manipulated as you secure the torso. Prior to securing the torso straps, you should ask the patient to inhale as much as possible so that when the straps are secured, enough space is allowed for the patient to breathe adequately. After full immobilization, the patient's spine should be completely in-line, from the head to the pelvis. During the entire immobilization procedure, the patient's head must be maintained in a neutral in-line position.

While treating a patient in cardiac arrest, you turn the AED on and attach the pads to the patient. However, when the AED begins to analyze the patient's cardiac rhythm, it signals "low battery" and then shuts off. The patient subsequently died. Which of the following statements regarding this scenario is MOST correct? • A:You and your partner may be held liable for negligence. • B:The crew that preceded you may be held liable for negligence. • C:Most errors associated with the AED involve equipment failure. • D:The manufacturer of the AED will be held liable for negligence.

You selected A; This is correct! Reason:The most common errors that occur with the AED are the result of operator error (not equipment failure), usually because no one made sure that the batteries were fully charged when checking the ambulance at the start of the shift. Because the patient died, you and your partner could be held liable for negligence. Remember, the entire ambulance must be checked by the oncoming shift to ensure that all equipment is functional and that all supplies are present. Even though the preceding crew is morally responsible for not replacing the batteries, the legal ramifications will rest on you and your partner's shoulders.

The technique of rapid extrication from a vehicle involves: • A:manually stabilizing the head, applying a cervical collar, and removing the patient from the vehicle onto a long backboard. • B:grabbing the patient by his or her clothing, protecting his or her spine as much as possible, and dragging him or her from the vehicle. • C:applying a cervical collar, grasping the patient by the clothing, and quickly removing him or her onto the stretcher. • D:applying a vest-style extrication device, sliding a long backboard under the patient's buttocks, and removing him or her from the vehicle.

You selected A; This is correct! Reason:The rapid extrication technique is indicated if the patient has life-threatening injuries and/or is in need of treatment that requires a supine position. It is performed by manually stabilizing the patient's head (an EMT in the backseat typically does this), applying a cervical collar, sliding a backboard under the patient's buttocks, and removing him or her from the vehicle and onto the backboard. The vest-style extrication device is not appropriate to use when performing the rapid extrication technique; it takes too long to correctly apply. Do not confuse the rapid extrication technique with an emergency move. An emergency move, which is indicated if you or the patient's safety is in imminent danger, involves grabbing the patient by the clothing, protecting his or her spine as much as possible, and dragging him or her from the vehicle to a safe place.

The function of the uterus is to: • A:house the fetus as it grows for 40 weeks. • B:provide oxygen and other nutrients to the fetus. • C:dilate and expel the baby from the cervix. • D:provide a cushion and protect the fetus from infection.

You selected A; This is correct! Reason:The uterus is a muscular organ where the fetus grows for 37 to 42 weeks (average of 40 weeks). It is responsible for contracting during labor, which in conjunction with dilation of the cervix (the opening of the uterus), expels the baby from the uterus into the birth canal. The placenta is the organ of exchange that delivers oxygen and other nutrients from the mother to the fetus and returns metabolic waste products from the fetus to the mother. The amniotic sac, also called the bag of waters, provides a cushion for the developing fetus and helps protect it from infection.

Your patient is a 75-year-old female who, according to her son, tripped on a throw rug and fell. The patient is conscious and alert, is sitting on the couch, and has a hematoma to her forehead. When you ask her what happened, her son interjects by saying, "I already told you, she tripped and fell." As you further question her, you find that she is hesitant to answer your questions and keeps looking at her son. You should: • A:interview the patient and her son separately if possible. • B:tell the son that you suspect his mother has been abused. • C:notify law enforcement and have the son removed. • D:ask the patient if someone hurt her intentionally.

You selected A; This is correct! Reason:There are several indicators that suggest this patient's injury was intentionally inflicted: the son's interjection, her hesitance to answer your questions, and the fact that she keeps looking at her son. If possible, interview the patient and her son separately; doing say may reveal inconsistencies regarding the event, which would further increase your index of suspicion for abuse. You must not, however, accuse her son of abuse or insinuate such. If you are wrong, you could be liable for slander. At present, the son is not verbally or physically abusive; however, if he becomes abusive, you should notify law enforcement at once. Remember, you are legally obligated to report any suspicions of abuse or neglect to the emergency department physician. You should make every effort to convince the patient to consent to treatment and transport.

If a woman is having her first child, the first stage of labor: • A:is usually the longest and lasts an average of 16 hours. • B:generally does not allow time for you to transport. • C:is shorter than in women who have had other children. • D:is typically very short and only lasts about 2 hours.

You selected A; This is correct! Reason:There are three stages of labor: dilation of the cervix, delivery of the baby, and delivery of the placenta. The first stage begins with the onset of contractions and ends when the cervix is fully dilated. Since assessing for cervical dilation is not performed in the prehospital setting, the first stage of labor is said to have ended when crowning occurs. Because the cervix has to be stretched thin by uterine contractions until the opening is large enough for the fetus to pass through into the vagina, the first stage is usually the longest, lasting an average of 16 hours for a first delivery. With subsequent pregnancies, the first stage of labor typically progresses more quickly. You will usually have enough time to transport the mother during the first stage of labor, especially if this is her first pregnancy. It should be noted, however, than some primigravida (pregnant for the first time) women progress through the first stage of labor very quickly

You respond to a shooting at a local bar. Law enforcement is present and the scene has been secured. Your patient is a young male, who is sitting against the wall screaming in pain. Bright red blood is spurting from a wound near his groin. You should: • A:transport the patient at once. • B:apply pressure to the wound. • C:ensure an open airway. • D:administer high-flow oxygen

You selected B; This is correct! Reason: You must first address problems that pose the most immediate threat to life. The injury to the groin area most likely is an arterial bleed from the femoral artery; this bleeding must be controlled immediately or the patient will bleed to death. Because the patient is screaming in pain, it is clear that his airway is patent. After the bleeding has been controlled, administer high-flow oxygen, treat the patient for shock, and transport without delay.

Upon arriving at the scene of an overturned tanker truck, you see a clear liquid leaking from the rear of the tank. The driver is still in the vehicle and you can see that his face is covered with blood. You should: • A:request fire department assistance if they are not already en route. • B:ensure that the ambulance is downwind and downhill from the tanker. • C:put on gloves, a gown, and a mask and quickly remove the driver. • D:approach the rear of the tanker to identify the type of fluid leaking.

You selected A; This is correct! Reason:Upon determining that you are dealing with a potential hazardous materials incident, you should immediately request specially trained personnel (eg, fire department, Haz-Mat team) if they are not already en route to the scene. Do not approach a vehicle that may be leaking a hazardous material, even if the patient is still in the vehicle, or you may become a casualty as well. Most ambulances are not equipped with the personal protective equipment required for hazardous materials. Park your ambulance in a location that is both upwind and uphill from the incident. Until additional personnel arrive, try to identify the material being carried by reading the United Nations (UN) number on the safety placard affixed to the tanker; however, this should be done from a safe distance and with the use of binoculars.

A 50-year-old female complains of severe pain to the right lower quadrant of her abdomen. You should: • A:palpate the left upper quadrant of her abdomen first. • B:keep her supine with her legs fully extended. • C:quickly palpate that area first to assess for rigidity. • D:suspect that she has an acute problem with her liver.

You selected A; This is correct! Reason:When assessing a patient with abdominal pain, determine where the pain is most severe (in this case, the right lower quadrant) and then palpate the quadrant furthest from that area first (in this case, the left upper quadrant). If you palpate the painful area first, the patient is less likely to allow you to assess the remainder of his or her abdomen; it also causes the patient unnecessary pain. Pain to the right lower quadrant suggests a problem with the appendix (eg, appendicitis), not the liver. Patients with abdominal pain commonly prefer to lie on their side with their legs drawn into their abdomen; this position often provides pain relief by taking pressure off of the abdominal muscles. If the patient prefers this position, do not discourage it.

The immobilization device MOST appropriate to use for a patient with multiple injuries and unstable vital signs is the: • A:long spine board immobilization device. • B:short spine board immobilization device. • C:vest-style immobilization device. • D:scoop immobilization device.

You selected A; This is correct! Reason:When caring for a critically injured patient with multiple injuries, the patient's entire body should be immobilized. This is most quickly and effectively accomplished using a long spine board. Vest-style devices or short spine boards take too long to apply and will not provide full body immobilization. The scoop (orthopaedic) stretcher is effective for maneuvering patients in narrow spaces but will not allow for full spinal immobilization because of the vertical opening down the center of the device.

You should not attempt to actively rewarm a patient with moderate or severe hypothermia in the field because: • A:rewarming too quickly can cause a fatal cardiac dysrhythmia. • B:the risk of inadvertently inducing hyperthermia is too high. • C:it is painful for the patient and you cannot give analgesic drugs. • D:active rewarming has been shown to cause severe hypertension.

You selected A; This is correct! Reason:When caring for a patient with hypothermia, your goal is to prevent further heat loss; this involves removing wet clothing, applying warm blankets, and allowing the patient's body temperature to rise gradually and naturally (passive rewarming). If the patient is moderately or severely hypothermic, you should not try to rewarm him or her actively (placing heat on or into the body). Rewarming too quickly may cause a fatal cardiac dysrhythmia, such as ventricular fibrillation (V-Fib). Active rewarming may also cause rewarming shock, a condition in which the blood vessels dilate when heat is applied to the body, resulting in significant hypotension. For these reasons, active rewarming should only be performed in the controlled setting of a hospital.

You are caring for a 6-year-old child with a swollen, painful deformity to the left forearm. As you communicate with the parents of this child, you should: • A:make sure that they remain aware of what you are doing. • B:tell them that the child will be transported to the hospital. • C:ask them repeatedly how the child was injured. • D:use appropriate medical terminology at all times.

You selected A; This is correct! Reason:When caring for any patient, it is important to keep both the patient and family aware of what you are doing. You should avoid medical terminology whenever possible because most laypeople will not understand what you are saying. The plain English approach is much more effective. When caring for children specifically, you should inform the parents of the need for ambulance transportation and why; doing so will provide them with the information necessary to make an informed decision. Asking the parents repeatedly how the child was injured may be construed by some as implying that the child was abused.

You are transporting a 30-year-old man who is experiencing an emotional crisis. The patient does not speak when you ask him questions. How should you respond to his unwillingness to speak? • A:Remain silent until the patient speaks to you. • B:Do not speak to the patient, even if he begins to speak to you. • C:Tell the patient that you cannot help if he won't talk. • D:Continually encourage the patient to talk to you.

You selected A; This is correct! Reason:When dealing with a psychiatric patient who is silent and unwilling to speak to you, do not fear the silence. The patient simply does not wish to speak. You should not press the issue, for doing so may upset the patient. You should remain calm until the patient speaks to you, and then respond accordingly.

A prolapsed umbilical cord is dangerous because the: • A:cord might pull the placenta from the uterine wall during delivery. • B:baby's head may compress the cord, cutting off its supply of oxygen. • C:mother may die of hypoxia due to compromised placental blood flow. • D:cord may be wrapped around the baby's neck, causing strangulation

You selected B; This is correct! Reason:A prolapsed umbilical cord, a condition in which a portion of the umbilical cord delivers before the baby, is a dangerous condition; the baby's head may compress the cord, cutting off its own supply of oxygen. Therefore, when a prolapsed umbilical cord is discovered, it is important to take immediate action. Place the mother in a position in which her hips are elevated. It may be necessary to insert your gloved fingers into the vagina and lift the baby's head off of the cord. A nuchal cord occurs when the umbilical cord is wrapped around the baby's neck; it is relatively common and is usually easily treated by simply sliding the cord from around the baby's neck. A nuchal cord and a prolapsed umbilical cord usually do not occur at the same time.

Your FIRST action in managing a patient with an altered mental status should be to: • A:determine if the patient is breathing adequately. • B:administer one tube of oral glucose. • C:administer high-flow supplemental oxygen. • D:ask a family member how the patient normally acts.

You selected A; This is correct! Reason:When treating a patient with an altered mental status, you must first ensure a patent airway and determine if the patient is breathing adequately. If the patient is breathing adequately, administer supplemental oxygen and continue your assessment. If the patient is not breathing adequately (ie, fast or slow rate, shallow breathing [reduced tidal volume]), assist his or her ventilations. It is important to ask a family member, if available, how the patient normally acts; this will help establish a baseline. Before administering oral glucose, you should assess the patient's blood glucose level to determine if hypoglycemia is the cause of his or her problem and then decide if the patient is able to swallow the glucose, if indicated. If the patient is unable to swallow, do not administer oral glucose, even if he or she is hypoglycemic. Request a paramedic ambulance if possible so the patient can receive intravenous glucose.

You arrive at the scene of an 80-year-old woman who is weak and lightheaded. Her son, who called 911, is present and asks you to transport his mother to the hospital. You should: • A:assess the woman and determine if she wishes to be treated and transported. • B:comply with the son's request and transport the woman to the hospital. • C:advise the son that he can probably drive his mother to the hospital. • D:take the woman's vital signs and apply supplemental oxygen if necessary.

You selected A; This is correct! Reason:You must obtain consent from any mentally competent adult patient prior to initiating treatment. Just because the patient is 80 years old does not mean that she does not have decision-making capacity. And just because her son wants her to be transported does not mean that she does. Ask her if she wishes to be treated and transported to the hospital. If she does, then you have obtained consent and should proceed accordingly. If she does not, you should determine if she has decision-making capacity; that is, whether or not she is mentally competent. If she is determined to have decision-making capacity, then you cannot legally treat or transport her. If she does not have decision-making capacity (eg, she is confused, under the influence of drugs or alcohol), then you may treat and transport under the law of implied consent. It is not the EMT's decision to determine, let alone recommend, that a patient be taken to the hospital via privately owned vehicle (POV). If the patient requests EMS treatment and transport, you are legally obligated to do so.

You are dispatched to a call for an unresponsive patient. Which of the following is the MOST important information that you should initially obtain from the dispatcher? • A:The exact location of the patient • B:Whether or not the patient is breathing • C:The patient's sex and approximate age • D:The call back number of the caller

You selected A; This is correct! Reason:You should attempt to obtain as much patient information from the dispatcher as you can. However, in order to help the patient, you must know his or her location; therefore, this is the most important initial question to ask. Once you determine where the patient is, you should then try to ascertain the sex and approximate age of the patient, whether or not he or she is breathing, and any other information that can help you prepare to provide care. The caller's phone number is generally not disclosed to the responding EMTs, although this is important information for the dispatcher to obtain. In many dispatch systems, the caller's phone number (and location in some cases) automatically appears on the computer screen when the 911 call is answered.

You receive a call at 3:00 a.m. for a patient who is slumped over the steering wheel of his car, which is parked on the shoulder of the road. Your unit and a police officer arrive at the scene at the same time. You should: • A:stay in your unit until the police officer checks the patient. • B:shine a spotlight in the side view mirror of the patient's vehicle. • C:approach the vehicle from the front to ensure maximum visibility. • D:park the ambulance 25 feet in front of the patient's vehicle.

You selected A; This is correct! Reason:Unfortunately, it is not uncommon for people to fake illness or injury with the intent of harming responding personnel. In this situation, you should utilize the safety resource at the scene: the police officer. You and your partner should stay in the unit until the police officer checks the patient to ensure it is safe for you to approach. Remember, the safety of you and your partner comes first!

You are dispatched to the scene of a motorcycle crash. Upon arrival, you find the patient lying facedown approximately 25 feet from his bike. He is not wearing a helmet and is moaning. You should: • A:log roll him to a supine position. • B:apply a cervical collar. • C:stabilize his head manually. • D:evaluate the status of his airway.

You selected B; The correct answer is C; Reason: The mechanism of injury for this patient was significant. In his present position (prone), you cannot effectively assess his airway. Therefore, your first action should be to manually stabilize his head. Then, you must log roll him into a supine position, keeping his head in an in-line position. If possible, log roll him directly onto a long backboard. After the patient is supine, assess the status of his airway, assess his breathing adequacy, administer high-flow oxygen or begin assisted ventilations if needed, and continue with your primary assessment. Apply a cervical collar as soon as possible, but assess his posterior neck first.

Which of the following structures is responsible for regulating body temperature? • A:Cerebrum • B:Medulla oblongata • C:Hypothalamus • D:Cerebellum

You selected B; The correct answer is C; Reason:The hypothalamus, which is located within the brainstem, regulates body temperature by acting as the body's thermostat. During a heat-related emergency, the hypothalamus can "reset" the body's normal temperature to a much higher temperature in response to the environment and the body's inability to eliminate heat.

A 3-year-old boy is found to be in cardiopulmonary arrest. As you begin one-rescuer CPR, your partner prepares the AED. The appropriate compression to ventilation ratio for this child is: • A:3:1 • B:30:2 • C:5:1 • D:15:2

You selected B; This is correct! Reason:A universal compression to ventilation ratio of 30:2 is used for all one-rescuer CPR (adult, child, and infant), with the exception of the newborn. A compression to ventilation ratio of 3:1 is used for newborns (one- and two-rescuer). Two-rescuer infant and child CPR is performed at a compression to ventilation ratio of 15:2. In this scenario, you are performing one-rescuer CPR as your partner prepares the AED; therefore, you should give 30 compressions and 2 breaths. However, when you and your partner resume CPR, give 15 compressions and 2 breaths.

Ischemic heart disease is a condition in which: • A:an acute event leads to a significant decrease in the pumping force of the heart. • B:a portion of the heart muscle dies because of a prolonged lack of oxygen. • C:the coronary arteries dilate, thus preventing effective blood flow to the heart. • D:there is a decrease in blood flow to one or more portions of the heart muscle.

You selected B; The correct answer is D; Reason: Chest pain or discomfort that is related to the heart usually stems from a condition called ischemia (insufficient oxygen). Because of a partial or complete blockage of blood flow through one or more coronary arteries, the tissue of the heart muscle (myocardium) fails to get enough oxygen and nutrients relative to its needs. Therefore, ischemic heart disease is a condition involving a decrease in blood flow, and therefore oxygen, to one or more portions of the myocardium. If blood flow to the ischemic portion of the myocardium is not restored, it eventually dies (myocardial infarction). Dilation of the coronary arteries increases, not decreases, blood flow to the heart. If an event such as a myocardial infarction damages the heart and significantly decreases its ability to contract forcefully, heart failure may occur.

Freshly oxygenated blood returns to the heart via the: • A:aorta. • B:vena cavae. • C:pulmonary artery. • D:pulmonary vein.

You selected B; The correct answer is D; Reason: The pulmonary vein is the only vein that carries oxygen-rich blood. It carries blood from the lungs back to the left atrium. All other veins in the human body, including the vena cavae, carry deoxygenated blood back to the heart. The aorta is the largest artery in the body and branches immediately from the left ventricle, carrying freshly oxygenated blood to the rest of the body. The pulmonary artery carries deoxygenated blood from the right ventricle to the lungs for reoxygenation.

Oxygen that is administered through a nasal cannula would be of LEAST benefit to a patient who: • A:is breathing greater than 12 times per minute. • B:breathes through his or her mouth. • C:has COPD and an oxygen saturation of 94%. • D:is in need of long-term oxygen therapy.

You selected B; This is correct! Reason: A patient who breathes through the mouth or has a nasal obstruction will get little or no benefit from a nasal cannula. Many patients with COPD (eg, emphysema, chronic bronchitis) require long-term, low-flow oxygen therapy; the nasal cannula is ideal in this situation. Considering their chronic respiratory problem, an oxygen saturation of 94% in a COPD patient is good; in fact, many COPD patients maintain an oxygen saturation lower than 94%, even with supplemental oxygen. A nasal cannula is appropriate to use in patients breathing greater than 12 times per minute, provided they have adequate tidal volume and are not significantly hypoxemic. Regardless of the oxygen delivery device used, you should maintain a patient's oxygen saturation at greater than 94%.

During the rapid head-to-toe assessment of a patient with multiple injuries, you expose the chest and find an open wound with blood bubbling from it. You should: • A:apply high-flow supplemental oxygen. • B:prevent air from entering the wound. • C:place a porous dressing over the wound. • D:stop your assessment and transport.

You selected B; This is correct! Reason: A sucking chest wound (open pneumothorax) is a life-threatening condition that must be corrected immediately upon discovery. You must take immediate action to prevent air from entering the wound or the patient's condition may continue to deteriorate. Cover a sucking chest wound with an occlusive (non-porous) dressing and secure it on three sides. This will prevent air from entering the pleural space during inhalation. It is important to note, however, that when you cover the wound, you have converted it from an open pneumothorax to a closed pneumothorax. Therefore, you must closely monitor the patient; if signs of a tension pneumothorax develop (ie, worsened respiratory distress, cyanosis, signs of shock), lift the unsecured corner of the dressing to allow air to escape from the pleural space.

Which of the following assessment parameters is the MOST reliable when determining if a patient with a head injury is improving or deteriorating? • A:pupillary reaction. • B:level of consciousness. • C:systolic blood pressure. • D:rate and depth of breathing.

You selected B; This is correct! Reason: All of the options in this question are important parameters to monitor in a patient with a head injury. However, the single most reliable parameter is the patient's level of consciousness (LOC); a person's LOC indicates how the brain is functioning from a global perspective. It should be monitored frequently in order to determine whether the patient's condition is improving (ie, concussion), or deteriorating (ie, intracerebral hemorrhage). In general, level of consciousness serves as the most reliable indicator of perfusion.

You have analyzed a cardiac arrest patient's rhythm three times with the AED, separated by 2-minute cycles of CPR, and have received no shock messages each time. You should: • A:consider terminating resuscitation. • B:continue CPR and transport at once. • C:remove the AED and continue CPR. • D:request a paramedic unit at the scene.

You selected B; This is correct! Reason: Although protocols vary from system to system, it is generally agreed that if you receive three consecutive no shock messages, separated by 2-minute cycles of CPR, you should continue CPR and transport at once; it is unlikely that the patient will convert to a shockable rhythm (eg, V-Fib, pulseless V-Tach). En route, coordinate a rendezvous with a paramedic unit if possible; waiting at the scene would only delay further treatment. The decision to terminate resuscitative efforts is made by a physician, and in some cases, a paramedic, after adequately performed BLS and ALS have proven unsuccessful.

A young woman who overdosed on heroin is unresponsive with slow, shallow breathing. As you attempt to insert an oropharyngeal airway, she begins to gag. You should: • A:place her on her side until she stops gagging and then suction her mouth. • B:remove the oropharyngeal airway and be prepared to suction her mouth. • C:suction the patient's oropharynx as you insert a nasopharyngeal airway. • D:make sure you are using the most appropriate size of oropharyngeal airway.

You selected B; This is correct! Reason: Although uncommon, an unresponsive patient may have an active gag reflex. If an unresponsive patient begins to gag as you are attempting to insert an oropharyngeal airway, you must remove the airway immediately and be prepared to suction if vomiting should occur. Turn the patient on his or her side to facilitate drainage of secretions. Once the airway has been cleared, a nasopharyngeal airway, which is better tolerated in patients with a gag reflex, should be inserted.

A young male has an open abdominal wound through which a small loop of bowel is protruding. There is minimal bleeding. The BEST way to treat his injury is to: • A:cover the wound with a dry sterile trauma dressing and tightly secure it in place by circumferentially wrapping roller gauze around the abdomen. • B:apply a sterile trauma dressing moistened with sterile saline directly to the wound and secure the moist dressing in place with a dry sterile dressing. • C:gently clean the exposed loop of bowel with warm sterile saline, carefully replace it back into the wound, and cover it with a dry sterile dressing. • D:apply dry sterile gauze pads to the wound and then keep them continuously moist by pouring sterile saline or water on them throughout transport.

You selected B; This is correct! Reason: An abdominal evisceration occurs when a loop of bowel, an organ, or fat protrudes through an open abdominal injury. Never try to replace an organ that is protruding from an open abdominal wound, whether it is a small fold of peritoneum or nearly all of the intestines; this significantly increases the risk of infection. Instead, cover it with sterile gauze pads or a sterile trauma dressing moistened with sterile saline and secure the moist dressing in place with a dry sterile dressing. Some EMS protocols call for an occlusive dressing over the organs, secured by trauma dressings. Do not apply excessive pressure when dressing and bandaging the wound; this may force the protruding organ or loop of bowel back into the abdominal cavity.

The lower airway begins at the: • A:trachea. • B:larynx. • C:epiglottis. • D:cricoid cartilage.

You selected B; This is correct! Reason: Anatomically, the lower airway begins at the larynx (voice box). The cricoid cartilage is a firm cartilage ring that forms the inferior (lower) part of the larynx. The trachea is connected to the larynx and extends downward to form the left and right mainstem bronchi. The epiglottis is an upper airway structure; it is a leaf-shaped structure above the larynx that prevents food and liquid from entering the trachea during swallowing.

During your assessment of a trauma patient, you note massive facial injuries, weak radial pulses, and clammy skin. What should be your MOST immediate concern? • A:Internal bleeding and severe shock • B:Potential obstruction of the airway • C:Providing rapid transport to a trauma center • D:Applying 100% supplemental oxygen

You selected B; This is correct! Reason: Any trauma patient with severe maxillofacial trauma is at an extremely high risk of airway compromise. The airway can be compromised by either mandibular fractures, in which the tongue may occlude the airway, or severe oral bleeding, in which blood clots can obstruct the airway. Correct ANY airway problems immediately upon discovery, ensure adequate ventilation and oxygenation, assess for and treat other life-threatening injuries, and prepare for rapid transport.

Appropriate care for an amputated body part includes all of the following, EXCEPT: • A:keeping the part cool, but not allowing it to freeze. • B:placing it directly on ice to prevent tissue damage. • C:wrapping it in a sterile dressing and placing it in a plastic bag. • D:laying the wrapped body part on a bed of ice.

You selected B; This is correct! Reason: Appropriate care for an amputated body part includes wrapping the part in a sterile dressing and placing it in a plastic bag. Follow your local protocols regarding how to preserve amputated parts. In some areas, dry sterile dressings are recommended for wrapping amputated parts; in other areas, dressings moistened with sterile saline are recommended. Put the bag in a container filled with ice. Lay the wrapped part on a bed of ice; do not pack it in ice or place it in direct contact with ice. The goal is to keep the part cool without letting it freeze or develop frostbite. Freezing may cause cellular and tissue damage, which decreases the chance of successful reattachment.

Which of the following is a common side effect of nitroglycerin? • A:Nausea • B:Headache • C:Hypertension • D:Anxiety

You selected B; This is correct! Reason: Because nitroglycerin (NTG) causes vasodilation, including the vessels within the brain, cerebral blood flow increases following the administration of NTG. This often causes a pounding headache for the patient. As uncomfortable as it is for the patient, headaches are a common and expected side effect of the drug. The vasodilatory effects of nitroglycerin could result in hypotension; therefore, the patient's blood pressure should be carefully monitored. Nausea and anxiety are common symptoms of acute coronary syndrome; they are not common side effects of nitroglycerin.

A 50-year-old man, who fell approximately 20 feet and landed on a hard surface, is semiconscious. You should: • A:gently tilt the patient's head back to assess for breathing. • B:stabilize his head while performing the jaw-thrust maneuver. • C:check for a carotid pulse if the patient is breathing rapidly. • D:begin positive-pressure ventilations with a bag-mask device.

You selected B; This is correct! Reason: Because of the significant mechanism of injury (fall of greater than 15 feet), spinal injury should be assumed. The first step in managing this patient is to manually stabilize his head in a neutral position and open his airway with the jaw-thrust maneuver, both of which can be performed simultaneously. After the patient's airway is open, assess the rate and quality of his breathing and treat accordingly. The head tilt-chin lift maneuver should not be used on a patient with a possible spinal injury unless the jaw-thrust maneuver does not adequately open his or her airway. The patient in this scenario is semiconscious; therefore, he has a pulse (pulseless patients are unresponsive). If an uninjured patient is found to be unresponsive, you should quickly assess for breathing by visualizing the chest. If the patient is not breathing or only has agonal gasps, you should check for a carotid pulse.

A patient's skin will MOST likely become cyanotic if he or she has: • A:a decrease in the amount of carbon dioxide. • B:a decrease in the amount of arterial oxygen. • C:an increase in the amount of arterial oxygen. • D:an overall increase in circulating red blood cells.

You selected B; This is correct! Reason: Cyanosis, a blue or purple tint to the skin, reflects an inadequate amount of oxygen in the arterial blood. More specifically, cyanosis indicates that a significant amount of hemoglobin has separated from the red blood cells (desaturation), which makes the arterial blood less able to carry oxygen. An overall increase in the number of circulating red blood cells (polycythemia), would likely cause a patient's skin to remain pink, not become cyanotic. Patients with cyanosis must be given high-flow oxygen and, if needed, positive-pressure ventilations if they are apneic or breathing inadequately (eg, fast or slow rate, shallow breathing [reduced tidal volume]).

A 33-year-old female presents with acute respiratory distress. She is conscious but anxious, and tells you that she has a history of asthma. She took two puffs of her albuterol inhaler prior to your arrival, but states that it did not help. Her oxygen saturation reads 89% and you hear diffuse wheezing while auscultating her lungs. You should: • A:give her 100% humidified oxygen to dilate her bronchioles, monitor her oxygen saturation, and transport her to an appropriate medical facility. • B:administer high-flow oxygen, contact medical control to request permission to assist her with another albuterol treatment, and prepare for transport. • C:ventilate her with a bag-mask device until her oxygen saturation is at least 94% and rapidly transport her to the closest appropriate medical facility. • D:assist her with a third albuterol treatment, contact medical control for further advice, give her high-flow oxygen, and transport her to the hospital.

You selected B; This is correct! Reason: Despite two albuterol treatments, the patient is still experiencing respiratory distress. Furthermore, the presence of wheezing indicates continued bronchospasm. After administering high-flow oxygen via a nonrebreathing mask, you should contact medical control and request permission to assist the patient with a third albuterol treatment. Drugs such as albuterol (Proventil, Ventolin) and metaproterenol (Alupent) stimulate beta-2 receptors in the lungs, resulting in bronchodilation. Up to three bronchodilator treatments are typically given in the prehospital setting. In most EMS systems, EMTs are not allowed to assist patients with their medication without medical control authorization. After assisting the patient with a third albuterol treatment, reassess her breath sounds and oxygen saturation and transport her promptly.

After restoring a pulse in a cardiac arrest patient, you begin immediate transport. While en route to the hospital, the patient goes back into cardiac arrest. You should: • A:begin CPR and proceed to the hospital. • B:tell your partner to stop the ambulance. • C:analyze the patient's rhythm with the AED. • D:contact medical control for further advice.

You selected B; This is correct! Reason: If you restore a pulse in a cardiac arrest patient, the AED pads should remain attached to the patient's chest during transport in case cardiac arrest recurs en route to the hospital. You should, however, turn the AED off or disconnect the pads from the AED. If cardiac arrest recurs, you should immediately tell your partner to stop the ambulance and assist you as you begin CPR. Remember that the AED will not analyze the cardiac rhythm if the patient is moving. Once your partner is available to assist, you should analyze the cardiac rhythm, defibrillate if indicated, and immediately resume CPR. Contact medical control as soon as possible, but not before performing CPR and defibrillation.

Which of the following is MOST indicative of decompensated shock in a trauma patient with internal bleeding? • A:Restlessness • B:Hypotension • C:Clammy skin • D:Tachycardia

You selected B; This is correct! Reason: In compensated shock, the body is able to maintain blood pressure, usually above 90 to 100 mm Hg, through the physiologic responses of tachycardia and shunting of blood from the skin to the vital organs of the body. Signs of compensated shock include restlessness; pallor; tachycardia; tachypnea; and cool, clammy (diaphoretic) skin. If the signs of compensated shock are not recognized and treatment is delayed, the body's compensatory mechanisms will fail and blood pressure will fall (hypotension). At this point, the patient is said to be in decompensated shock. Do not wait for a trauma patient's blood pressure to fall before initiating treatment; it may be too late.

You arrive at a residence where you find a man lying unresponsive in his front yard. There were no witnesses to the event. In assessing this man, you should assume that he: • A:has a heat-related emergency. • B:has sustained an injury. • C:is having a diabetic reaction. • D:is having a heart attack.

You selected B; This is correct! Reason: In the absence of any witnesses, you should assume that any patient who is found unresponsive has an injury until ruled out at the hospital. Apply spinal motion restriction precautions as needed. Do not be so hasty to label your patient as a "medical" or "trauma" patient. Many patients have injuries and medical conditions at the same time. For example, a patient can be driving his or her vehicle, experience a heart attack, and run off the road and strike a tree.

Your partner has applied the AED to a cardiac arrest patient and has received a shock advised message. While the AED is charging, you should: • A:retrieve the airway equipment and prepare to ventilate the patient. • B:continue chest compressions until your partner tells you to stand clear. • C:cease all contact with the patient until the AED has delivered the shock. • D:perform rescue breathing only until the AED is charged and ready to shock.

You selected B; This is correct! Reason: It is important to minimize interruptions in CPR, especially chest compressions, when at all possible. All contact with the patient must cease while the AED is analyzing. However, if the AED gives a shock advised message and begins charging, you should resume chest compressions until the AED is charged and ready to deliver the shock; at this point, you should cease contact with the patient. As soon as the AED delivers the shock, resume CPR starting with chest compressions.

A young male has a large laceration to his lateral neck, directly over his jugular vein. His airway is patent and his breathing is adequate. Your MOST immediate priority should be to: • A:obtain vital signs to determine if he is hypotensive. • B:keep air out of the wound and control the bleeding. • C:apply high-flow oxygen via a nonrebreathing mask. • D:perform a rapid assessment to detect other injuries.

You selected B; This is correct! Reason: Jugular vein lacerations pose two immediate life threats: entrainment of air into the wound (which may cause a fatal air embolism) and severe external bleeding. The patient's airway is patent and his breathing is adequate; therefore, your most immediate priority is to apply an occlusive dressing directly over the wound, which will keep air from entering the venous circulation, and then cover the occlusive dressing with bulky dressings to control the external bleeding. Apply high-flow oxygen via a nonrebreathing mask (your partner can do this as you are treating the neck wound). The need to perform a rapid head-to-toe assessment is based on the presence of a significant mechanism of injury (MOI). If a significant MOI is present, the rapid assessment is performed only after problems with airway, breathing, and circulation have been addressed. Vital signs are typically obtained after the rapid assessment, although they can be obtained by another EMT as you perform the rapid assessment.

The process of loading oxygen molecules onto hemoglobin molecules in the bloodstream is called: • A:respiration. • B:oxygenation. • C:ventilation. • D:diffusion.

You selected B; This is correct! Reason: Oxygenation is the process of loading oxygen molecules onto hemoglobin molecules in the blood. Adequate oxygenation is required for internal (cellular) respiration to take place. Diffusion is the process in which gases (oxygen and carbon dioxide) move from an area of higher concentration to an area of lower concentration. Ventilation is the act of moving air into and out of the lungs. Negative-pressure ventilation is the act of normal, unassisted breathing, and occurs when the diaphragm and intercostal muscles contract, which creates a vacuum and draws air into the lungs. Positive-pressure ventilation is the act of forcing air into the lungs, such as when you are providing rescue breathing to an apneic patient or assisting the ventilations of a patient who is breathing inadequately. Respiration is the exchange of gases between the body and its environment. Pulmonary (external) respiration occurs when gases are exchanged in the lungs and cellular (internal) respiration occurs when gases are exchanged at the cellular level.

Prior to administering nitroglycerin to a patient with chest pain, you should: • A:elevate the patient's lower extremities. • B:obtain vital signs to detect hypotension. • C:auscultate the patient's breath sounds. • D:inquire about an allergy to salicylates.

You selected B; This is correct! Reason: Prior to assisting a patient with his or her prescribed nitroglycerin, there are two things that you must do: take the patient's vital signs and obtain authorization from medical control. Nitroglycerin is contraindicated for patients with a systolic blood pressure that is less than 100 mm Hg. If the patient develops hypotension after being given nitroglycerin, elevating his or her lower extremities would be indicated. Salicylates are a class of drugs that include aspirin, not nitroglycerin (nitroglycerin is a nitrate). Although you should inquire about medication allergies in general, it is not necessary to inquire specifically about an allergy to salicylates unless you are going to administer aspirin. Assessment of a patient with a possible cardiac or respiratory problem should include auscultation of breath sounds; however, this does not necessarily have to be done before assisting the patient with his or her nitroglycerin.

The MOST appropriate treatment for a semiconscious patient with slow, shallow respirations includes: • A:an oropharyngeal airway and assisted ventilation with a bag-mask device. • B:a nasopharyngeal airway and assisted ventilation with a bag-mask device. • C:a nasopharyngeal airway and high-flow oxygen via a nonrebreathing mask. • D:an oropharyngeal airway and high-flow oxygen via a nonrebreathing mask.

You selected B; This is correct! Reason: Semiconscious patients are not fully able to protect their own airway and require an airway adjunct. The nasopharyngeal airway is indicated for semiconscious patients because they often have an intact gag reflex; the oropharyngeal airway is contraindicated in any patient with an intact gag reflex. Slow, shallow respirations will not provide the minute volume needed to support adequate oxygenation and should be treated with positive-pressure ventilation assistance (eg, bag-mask device, pocket face mask).

When assessing a patient who complains of chest pain, which of the following questions would you ask to assess the "R" in OPQRST? • A:Is there anything that makes the pain worse? • B:Is the pain in one place or does it move around? • C:Did the pain begin suddenly or gradually? • D:What were you doing when the pain began?

You selected B; This is correct! Reason: The "R" in OPQRST stands for radiation or referred pain. An appropriate way to determine whether the pain radiates or not is to ask the patient if the pain remains in one place or if it moves around. When determining if the patient has referred pain, ask him or her if he or she hurts somewhere other than his or her chest. If you use the term "radiating pain," chances are the patient will not understand what you are asking.

A patient with a spinal injury may still be able to use his or her diaphragm to breathe, but would lose control of the intercostal muscles, if the spinal cord is injured: • A:above the C5 level. • B:below the C5 level. • C:above the C3 level. • D:between C1 and C2.

You selected B; This is correct! Reason: The nerves that supply the diaphragm (the phrenic nerves) exit the spinal cord at C3, C4, and C5. A patient whose spinal cord is injured below the C5 level will lose the ability to move his or her intercostal muscles (the muscles in between the ribs), but the diaphragm will still contract. The patient may still be able to breathe because the phrenic nerves remain intact. Patients with spinal cord injuries at C3 or above often lose their ability to breathe entirely. Remember this: C3, 4, and 5 keep the diaphragm alive.

A 58-year-old man is found unresponsive by his wife. According to his wife, he was complaining of a "dull ache" in his chest the day before, but refused to allow her to call 9-1-1. His blood pressure is 70/50 mm Hg, his pulse is 120 beats/min and weak, and his respirations are 28 breaths/min and labored. Further assessment reveals that his skin is cool, pale, and clammy. You should suspect: • A:pulmonary embolism. • B:cardiogenic shock. • C:acute septic shock. • D:hypovolemic shock.

You selected B; This is correct! Reason: The patient in this scenario likely experienced an acute myocardial infarction the day before. However, because he refused medical care, the infarction has likely damaged a significant portion of his heart, resulting in cardiogenic shock. Cardiogenic shock occurs when the heart fails as a pump and can no longer meet the metabolic needs of the body; it has a very high mortality rate. The patient's labored breathing is likely caused by pulmonary edema, which occurs when blood backs up into the lungs because the heart cannot effectively pump. There is no evidence of hypovolemia (ie, poor skin turgor) or sepsis (ie, fever). A pulmonary embolism typically presents with an acute onset of pleuritic (sharp) chest pain and difficulty breathing, and is also commonly associated with cyanosis.

Which of the following signs is LEAST suggestive of a diabetic emergency? • A:Fruity breath odor • B:Bradycardia • C:Combativeness • D:Tachycardia

You selected B; This is correct! Reason:Bradycardia is not commonly associated with either hyperglycemia or hypoglycemia. Tachycardia and combativeness can occur in patients with hyperglycemia or hypoglycemia. A fruity breath odor is noted exclusively in patients with diabetic ketoacidosis (diabetic coma, hyperglycemic crisis).

A 33-year-old male struck a parked car with his motorcycle and was ejected from the motorcycle. He was not wearing a helmet. He is unresponsive, has a depressed area to his forehead, bilaterally deformed femurs, and widespread abrasions with capillary bleeding. Which of the following statements regarding this patient is false? • A:You should suspect that the patient has a skull fracture and increased intracranial pressure. • B:You must stop the bleeding from his abrasions immediately or he will die from hypovolemic shock. • C:Internal hemorrhage cannot be controlled in the field and requires prompt surgical intervention. • D:Femur fractures are a common injury when a motorcyclist is ejected from his or her motorcycle.

You selected B; This is correct! Reason: The patient's abrasions (road rash) and capillary bleeding are the least of his problems. Capillary bleeding, blood that oozes from the capillary beds, is the least severe type of external bleeding and will not kill your patient. Wasting time at the scene to cover his abrasions, however, will delay definitive care at a trauma center; this may kill him! The patient likely has a depressed skull fracture, and the fact that he is unresponsive indicates a traumatic brain injury with increased intracranial pressure. When a motorcyclist is ejected from his or her motorcycle, the femurs typically strike the handlebars, resulting in unilateral or bilateral fractures. You cannot control internal hemorrhage in the field, regardless of your level of training. Internal bleeding requires surgical intervention; therefore, you must transport the patient without delay.

A reduction in tidal volume would MOST likely result from: • A:increased minute volume. • B:unequal chest expansion. • C:flaring of the nostrils. • D:accessory muscle use.

You selected B; This is correct! Reason: Unequal (asymmetrical) or minimal expansion of the chest results in a decrease in the amount of air inhaled per breath (tidal volume). Accessory muscle use and nasal flaring are signs of increased work of breathing, which represents an attempt to maintain adequate tidal volume (and therefore, minute volume). An increase in tidal volume, respiratory rate, or both, would result in an increase in minute volume. It should be noted, however, that a markedly fast respiratory rate would cause a natural decrease in tidal volume. For example, a patient breathing at a rate of 40 breaths/min would likely only inhale air into the anatomic dead space before promptly exhaling it.

Which of the following statements regarding sudden cardiac arrest and ventricular fibrillation is correct? • A:Patients with ventricular fibrillation are typically unconscious, apneic, and have a weak and irregular pulse. • B:For each minute that defibrillation is delayed, the chance of survival decreases by as much as 10%. • C:Most patients develop ventricular fibrillation within 10 minutes after the onset of sudden cardiac arrest. • D:High-quality CPR often reverses ventricular fibrillation if it is initiated within 2 minutes of the onset.

You selected B; This is correct! Reason: Ventricular fibrillation (V-Fib), a chaotic quivering of the heart muscle, is the most common dysrhythmia that results in sudden cardiac arrest (SCA). In V-Fib, the heart is not beating effectively and is not pumping blood; therefore, the patient will not have a pulse. The single most important treatment for V-Fib is early defibrillation. Even if CPR is begun right at the time of the patient's collapse, the likelihood of survival decreases by as much as 10% for each minute that defibrillation is delayed. CPR alone rarely, if ever, converts V-Fib to a cardiac rhythm with a pulse. V-Fib is often a transient dysrhythmia, and the window of opportunity for successful defibrillation is very narrow. After 10 minutes of cardiac arrest, most patients deteriorate to asystole (absence of electrical and mechanical activity in the heart).

Which of the following statements regarding one-rescuer CPR is correct? • A:Ventilations should be delivered over a period of 2 to 3 seconds. • B:The chest should be allowed to fully recoil after each compression. • C:You should assess the patient for a pulse after 3 cycles of CPR. • D:A compression to ventilation ratio of 15:2 should be delivered.

You selected B; This is correct! Reason: When performing CPR on any patient, you should allow the chest to fully recoil after each compression. Incomplete chest recoil causes increased intrathoracic pressure, which may impair blood return to the heart. Assess the patient's pulse after every 5 cycles (about 2 minutes) of CPR; take no longer than 5 to 10 seconds to do this. A compression to ventilation ratio of 30:2 should be performed during all adult and one-rescuer CPR (adult, child, and infant), except for newborns. A compression to ventilation ratio of 15:2 is used during two-rescuer infant and child CPR. Ventilations should be delivered over a period of 1 second each, just enough to produce visible chest rise.

Damaged small blood vessels beneath the skin following blunt trauma causes: • A:mottling. • B:ecchymosis. • C:hematoma. • D:cyanosis.

You selected B; This is correct! Reason: When small blood vessels beneath the skin are damaged, blood seeps into the soft tissues. This manifests as a bruise, also referred to as ecchymosis. A hematoma develops when larger blood vessels are ruptured and the internal bleeding forms a "lump." Cyanosis is a blue or purple discoloration of the skin and signifies a low content of oxygen in the blood. Mottling occurs when the skin takes on a blotched, purple appearance and is a sign of shock (hypoperfusion).

When ventilating an unresponsive apneic adult with a bag-mask device, you should ensure that: • A:the pop-off relief valve is manually occluded. • B:an airway adjunct has been inserted. • C:ventilations occur at a rate of 20 breaths/min. • D:you are positioned alongside the patient.

You selected B; This is correct! Reason: When ventilating an unresponsive apneic patient with a bag-mask device, you should ensure that an oral or nasal airway adjunct is inserted, which will keep the tongue off of the posterior pharynx. When ventilating a patient with a bag-mask device, it is best for you to be positioned at the patient's head to allow for better control of the head. Ventilations in the apneic adult with a pulse (ie, not in cardiac arrest) should be provided at a rate of 10 to 12 breaths/min (one breath every 5 to 6 seconds). Generally, only pediatric sized bag-mask devices have pop-off relief valves, which should NOT be occluded, because they help prevent overinflation of the patient's lungs.

At the peak of the inspiratory phase, the alveoli in the lungs contain: • A:high quantities of carbon dioxide. • B:equal levels of oxygen and carbon dioxide. • C:minimal levels of oxygen and carbon dioxide. • D:more oxygen than carbon dioxide.

You selected D; This is correct! Reason: At the peak of the inspiratory (inhalation) phase, the alveoli are filled with fresh oxygen that the patient just breathed in. During the expiratory (exhalation) phase, the oxygen moves from the alveoli to the left side of the heart and the carbon dioxide is exhaled into the atmosphere. The process of oxygen and carbon dioxide exchange in the lungs is called pulmonary (external) respiration.

You are assessing a young male who was stabbed in the right lower chest. He is semiconscious and has labored breathing, collapsed jugular veins, and absent breath sounds on the right side of his chest. This patient MOST likely has a: • A:liver laceration. • B:hemothorax. • C:pneumothorax. • D:ruptured spleen.

You selected B; This is correct! Reason: You should suspect a hemothorax if a patient with chest trauma presents with shock, especially if the injury was caused by penetrating trauma. Hemothorax occurs when blood collects in the pleural space and compresses the lung, resulting in shock and respiratory compromise. Other signs include collapsed jugular veins (due to low blood volume), labored breathing, and decreased or absent breath sounds on the side of the injury. A pneumothorax (air in the pleural space) is also associated with difficulty breathing and unilaterally decreased or absent breath sounds; however, the jugular veins are usually not collapsed. If excessive air accumulates within the pleural space, however, pressure will shift across the mediastinum and affect the uninjured lung (tension pneumothorax); if this occurs, the jugular veins may become engorged (distended). Splenic injury is unlikely; the patient's injury is on the right side and the spleen is on the left. A liver laceration can cause severe shock; however, it is not associated with unilaterally decreased breath sounds or labored breathing.

After attaching the AED to a 7-year-old child in cardiac arrest, you push the analyze button and receive a shock advised message. After delivering the shock, you should: • A:reanalyze the cardiac rhythm. • B:immediately perform CPR. • C:assess for a carotid pulse. • D:open the airway and ventilate.

You selected B; This is correct! Reason: After the AED delivers a shock, you should immediately begin or resume CPR, starting with chest compressions. Perform CPR for 2 minutes and then reanalyze the child's cardiac rhythm. If the AED states that a shock is advised, defibrillate without delay and then continue CPR. If the AED states no shock advised, resume CPR and reassess in 2 minutes. Do not check for a pulse after defibrillation or if the AED gives a no shock message; this only causes an unecessary delay in performing chest compressions. Continue CPR and cardiac rhythm analysis and defibrillation (if indicated) every 2 minutes until ALS personnel arrive or the patient starts to move spontaneously.

Which of the following is a later sign of hepatitis? • A:Fever and vomiting • B:Jaundice • C:Loss of appetite • D:Fatigue

You selected B; This is correct! Reason: Early signs and symptoms of viral hepatitis include loss of appetite (anorexia), vomiting, fever, fatigue, and muscle and joint pain. Jaundice (yellow sclera and skin) and right upper quadrant abdominal pain are not common early manifestations of hepatitis; they usually develop within 1 to 2 weeks into the disease process.

You are at the scene where a man panicked while swimming in a small lake. Your initial attempt to rescue him should include: • A:rowing a small raft to the victim. • B:reaching for the victim with a long stick. • C:throwing a rope to the victim. • D:swimming to the victim to rescue him.

You selected B; This is correct! Reason: General rules to follow when attempting to rescue a patient from the water include "reach, throw, row, and then go." In this case, you should attempt to reach the victim by having him grab hold of a large stick or similar object. If this is unsuccessful, throw the victim a rope or flotation device (if available). If these are not available, row to the patient in a small raft (if available). Going into the water to retrieve the victim is a last resort. The rescuer must be a strong swimmer because patients who are in danger of drowning are in a state of blind panic and will make every attempt to keep themselves afloat, even if it means forcing the rescuer underwater.

Which of the following statements regarding heatstroke is correct? • A:Heatstroke is caused by a hyperactive sweating mechanism. • B:Not all patients experiencing heatstroke have dry skin. • C:As core body temperature rises, the patient becomes more agitated. • D:Heatstroke is more likely to occur when the humidity is low.

You selected B; This is correct! Reason: Heatstroke occurs when the body is exposed to more heat than it can eliminate and normal mechanisms for eliminating heat, such as sweating, are overwhelmed. The core body temperature then rises rapidly to the point where tissues are destroyed. Heatstroke can develop in patients during prolonged vigorous physical activity or when they are in a closed, poorly ventilated, hot and humid space. High humidity impairs a person's ability to eliminate heat via the sweating mechanism. Many patients with heatstroke have hot, dry, flushed skin; however, early in the course of heatstroke, the skin may be moist due to residual perspiration, as with exertional heatstroke. As the core body temperature rises, the patient's level of consciousness decreases. Untreated heatstroke will result in death.

After an initial attempt to ventilate an unresponsive apneic patient fails, you reposition the patient's head and reattempt ventilation without success. You should next: A: turn the patient onto his side and deliver 5 to 10 back slaps. B: perform chest compressions, open the airway, and look in the mouth. C: administer 5 to 10 abdominal thrusts and reattempt to ventilate. D: perform continuous chest compressions until ALS personnel arrive.

You selected B; This is correct! Reason: If you are unable to ventilate an unresponsive, apneic patient after two attempts, you should assume that he or she has a severe (complete) foreign body airway obstruction. Immediately perform 30 chest compressions (15 compressions if two EMTs are present and the patient is an infant or child). Next, open the patient's airway and look inside the mouth. If you can see the object, attempt to remove it with your finger (never perform blind finger sweeps of the mouth). If you cannot see the object, continue chest compressions. If you are able to remove the object, reattempt to ventilate. Unless paramedics are nearby, begin transport while continuing chest compressions, opening the airway and looking in the mouth, and attempting to ventilate (if you can remove the object). Abdominal thrusts are indicated for responsive children and adults with a severe airway obstruction. Back slaps are indicated for a responsive infant with a severe airway obstruction.

General care for an amputated body part includes: • A:thoroughly cleaning the amputated part and wrapping it in a sterile dressing. • B:immersing the amputated part in ice cold water to prevent further damage. • C:wrapping the amputated part in a moist, sterile dressing and keeping it warm. • D:wrapping the amputated part in a moist, sterile dressing and placing it on ice.

You selected D; This is correct! Reason: General care for an amputated body part includes wrapping the part in a moist, sterile dressing and keeping it cool. Placing the wrapped part in a plastic bag and putting it on ice can accomplish this. The amputated part must never be placed directly on ice because this may cause cell and tissue damage. Attempting to clean the amputated part or immersing it directly in water can also cause further cell and tissue damage.

You are called to transport a patient with terminal lung cancer from a skilled nursing facility to the emergency department for evaluation of possible pneumonia. As you are reviewing the transfer paperwork, you see that the patient has a valid "do not attempt resuscitation" order. During transport, you should: • A:monitor the patient because a DNAR order prohibits you from providing care. • B:provide supportive care, such as oxygen, and keep the patient comfortable. • C:perform CPR for only 2 minutes if the patient develops cardiac arrest. • D:disregard the DNAR order because it is only valid in the hospital setting.

You selected B; This is correct! Reason:A valid do not attempt resuscitation (DNAR) order gives you the legal authority not to initiate resuscitative efforts if the patient develops cardiac arrest. Because laws vary from state to state, you must be familiar with the DNAR laws inherent to the state in which you work as an EMT. Many states have adopted out-of-hospital DNAR orders; these are legal documents and should be adhered to if they are valid. Generally speaking, a valid DNAR order must clearly state the patient's medical problem(s), be signed by the patient or legal guardian, and be signed by one or more physicians. In some states, DNAR orders have an expiration date, while in others, no expiration date is included. Contact medical control for guidance if the validity of a DNAR order is questionable. Even in the presence of a valid DNAR order, you are still obligated to provide supportive measures, such as oxygen, pain relief, and comfort. DNAR does not mean do not treat.

Which of the following situations is an example of abandonment? • A:An EMT gives a verbal report to an emergency nurse. • B:A paramedic transfers care to an advanced EMT. • C:An EMT departs the scene after a paramedic arrives. • D:An EMT transfers care of a patient to a paramedic.

You selected B; This is correct! Reason:Abandonment occurs any time you disengage from a patient while he or she still requires care or you relinquish your responsibility of patient care to a provider of lesser training. If a paramedic transfers patient care to an advanced EMT (AEMT)—clearly a provider with a lower level of training—then the paramedic has abandoned his or her patient. When delivering a patient to the emergency department, you must give your verbal report to a registered nurse or a physician.

The MOST effective means of preventing the spread of disease is: • A:wearing a mask with all patients. • B:effective handwashing. • C:wearing gloves with all patients. • D:up-to-date immunizations.

You selected B; This is correct! Reason:According to the Centers for Disease Control and Prevention (CDC), the most effective way of preventing the spread of disease is to frequently and effectively wash your hands, especially in between patients. The regular use of gloves with all patients and wearing a mask when managing a patient with a communicable disease (ie, tuberculosis) will decrease your chance of disease exposure. Remaining up-to-date with your immunizations will reduce your risk of contracting certain diseases if you are exposed.

Immediately following a generalized motor seizure, most patients are: • A:hyperactive. • B:confused. • C:apneic. • D:awake and alert.

You selected B; This is correct! Reason:After a generalized (grand mal) motor seizure, the patient typically will be confused, sleepy, or in some cases, combative. This is referred to as the postictal phase. The patient's level of consciousness typically improves within 30 minutes. In many cases, the patient's respirations will be fast (tachypnea) following a seizure; this is the body's attempt to eliminate excess carbon dioxide that accumulated in the blood during the seizure.

When sizing up a motor-vehicle crash in which a small passenger car struck a bridge pillar and sustained severe damage, you should: • A:disconnect the battery cables and then request heavy extrication tools. • B:ensure that there are no hazards and then try to open one of the doors. • C:immediately approach the vehicle and determine if it is stable. • D:break the driver's side window and gain rapid access to the patient.

You selected B; This is correct! Reason:After ensuring your own safety, you should attempt simple access to the patient, trying to get to him or her as simply and quickly as possible without using any tools or breaking any glass. It may be necessary to use tools or other forcible entry methods, some of which may require specialized extrication equipment (eg, Jaws of Life). Many times, however, the patient can be accessed simply by opening a door, even if the door is badly damaged. If the door is locked and the patient is responsive, ask him or her to unlock the door. Stabilizing the vehicle (eg, placing shoring blocks under the car) and disconnecting the car's battery cables are typical functions of the fire department.

Which of the following situations presents the greatest risk for suicide? • A:A woman who quit her job for one that pays a lot more • B:A man who was recently diagnosed with stage 4 lung cancer • C:An EMT who saved a drowning child and receives no media attention • D:A woman who is planning a family trip, but gets called away to work

You selected B; This is correct! Reason:Any patient with a significant, most often negative, life change is at risk for suicide. Common catalysts to suicide include chronic depression, the loss of a loved one or a job, relationship problems, financial difficulties, and the diagnosis of a serious or terminal illness. There have been a number of cases in which EMS personnel who performed a heroic act have committed suicide after receiving excessive media attention.

Immediately upon delivery of a newborn's head, you should: • A:suction the nose. • B:suction the mouth. • C:dry the face. • D:cover the eyes.

You selected B; This is correct! Reason:As soon as the newborn's head has delivered, you should first suction the mouth, then the nose. As the infant is forced through the birth canal, the thoracic cavity is squeezed, which causes the infant to expel amniotic fluid from the lungs. If this fluid is not thoroughly suctioned, it can be aspirated, resulting in inadequate ventilation and hypoxia. Immediately before or after suctioning the infant's airway, you should check for the presence of a nuchal cord (umbilical cord wrapped around the neck).

You arrive at the scene of a traffic accident in which multiple vehicles are involved. You see at least two patients who are lying on the road and are not moving. You should: • A:begin triaging the patients. • B:request additional ambulances. • C:begin immediate patient care. • D:notify medical control for advice.

You selected B; This is correct! Reason:As soon as you determine that there are more patients than you and your partner can effectively manage, you should immediately request additional help. Waiting until you are overwhelmed with critically injured patients is not the time to call for help. When in doubt, it is best to call for help. You can always cancel any incoming ambulances if you later determine that they are not needed. After you have called for assistance, you should begin triaging and caring for the patients to the best of your ability.

The MAIN reason why small children should ride in the backseat of a vehicle is because: • A:their legs are highly prone to injury from striking the dashboard. • B:they can experience severe injury or death if the airbag deploys. • C:they are much less likely to be ejected from the vehicle. • D:the back of the front seat will provide a cushion during a crash.

You selected B; This is correct! Reason:Children under 12 years of age should ride in the backseat of a vehicle, preferably in the middle, and restrained in a device that is appropriate for their size. Merely placing the child in the backseat does not reduce the risk of ejection; the child must be properly restrained. Young children, especially those restrained in a child safety seat, may be critically injured or killed by airbags if they are riding in the front passenger seat of a car. This occurs because the child safety seat positions the child too close to the airbag; the force of the deploying airbag may cause severe head and spinal trauma.

You are cleaning the back of the ambulance after transporting a patient with major trauma. Which of the following contaminated items should NOT be placed in a plastic biohazard bag? • A:Suction canister • B:Plastic IV catheter • C:Rigid suction catheter • D:Blood-soaked gauze pads

You selected B; This is correct! Reason:Contrary to popular belief, the end of a Teflon IV catheter (the plastic catheter without the needle) can cut you; it is sharp! Therefore, it should be placed in a puncture-proof sharps container, not a plastic biohazard bag. Items that are blood-soaked or are otherwise contaminated, but cannot puncture, can safely be placed in a plastic biohazard bag.

Which of the following statements regarding crowning is correct? • A:It is safe to transport the patient during crowning if the hospital is close. • B:Gentle pressure should be applied to the baby's head during crowning. • C:Crowning always occurs immediately after the amniotic sac has ruptured. • D:Crowning represents the end of the second stage of labor.

You selected B; This is correct! Reason:Crowning occurs when the baby's head is visible at the vaginal opening; it is an obvious sign of delivery in progress. When crowning is observed, you should apply gentle pressure to the infant's head to prevent an explosive delivery. Care must be taken to avoid putting pressure on the fontanelles (the soft spots on the infant's head). Crowning represents the end of the first stage of labor and the beginning of the second stage; it does not always occur immediately after the amniotic sac has ruptured. If the infant's head is born and the amniotic sac is still intact, you need to pinch the thin membrane with your fingers, which will usually cause the sac to easily rupture, and then suction the infant's mouth and nose.

A 44-year-old woman was bitten on the ankle by an unidentified snake while working in her garden. She is conscious and alert, has stable vital signs, and denies shortness of breath. Her only complaint is a burning sensation at the wound site. Your assessment reveals two small puncture wounds, redness, and swelling. You should: • A:conclude that envenomation likely did not occur, provide reassurance, and allow a friend to take her to the hospital. • B:give supplemental oxygen, splint her leg to decrease movement, and keep her leg below the level of her heart. • C:elevate her leg, cover the wound with a dry sterile dressing, and apply an ice pack to reduce pain and swelling. • D:administer high-flow oxygen, apply a constricting band proximal to the bite, and use ice to prevent venom spread.

You selected B; This is correct! Reason:Given the fact that the snake was not identified, you should assume that it was poisonous. Furthermore, the presence of puncture wounds, burning, redness, and swelling are suggestive of envenomation. Therefore, you should provide emergency care and transport the patient to the hospital. Treatment for a snake bite involves keeping the patient calm, administering oxygen, splinting the affected extremity to decrease movement (helps slow the spread of venom), keeping the extremity below the level of the heart, and transporting the patient to the hospital. Do NOT apply ice to a snake bite; it may constrict the blood vessels and force venom further into the bloodstream. The use of a proximal constricting band is controversial; if one is used, it should be loose, not tight. En route to the hospital, monitor the patient's vital signs and mental status, and be alert for vomiting.

A man is experiencing a severe allergic reaction after being stung by a scorpion. He does not have his own epinephrine; however, his wife is allergic to bees and has a prescribed epinephrine auto-injector. You should: • A:request an ALS unit to respond to the scene to administer epinephrine. • B:provide rapid transport and consider an ALS rendezvous. • C:assist the patient with the wife's prescribed epinephrine. • D:assist the patient with one half the usual dose of the wife's epinephrine.

You selected B; This is correct! Reason:If a patient does not have a prescribed epinephrine auto-injector and is experiencing a severe allergic reaction, you should administer supplemental oxygen, assist the patient's ventilations if needed, and transport without delay. Closely monitor the patient's airway and breathing status en route and coordinate an ALS rendezvous if possible. If you carry an epinephrine auto-injector on your ambulance and your protocols allow you to administer it, do so without delay. Otherwise, the ALS unit will be able to administer epinephrine via the intramuscular or intravenous route. Never assist a patient with a medication that is not prescribed to him or her specifically.

Which of the following statements regarding the high-efficiency particulate air (HEPA) respirator is correct? • A:A HEPA respirator is necessary only if the patient with suspected tuberculosis is coughing. • B:Long sideburns or a beard will prevent the proper fit of a HEPA respirator. • C:A HEPA respirator should be placed on any patient with tuberculosis. • D:A surgical mask provides better protection against tuberculosis than a HEPA respirator.

You selected B; This is correct! Reason:If you are caring for a patient with known or suspected tuberculosis (TB), regardless of whether the patient is coughing, you should place a surgical mask (or high-flow oxygen, if indicated) on the patient and a high-efficiency particulate air (HEPA) respirator (N-95 or higher) on yourself. Unlike a surgical mask, the HEPA respirator is specifically designed to prevent exposure to the bacterium that causes TB. A surgical mask, however, will reduce the transmission of germs from the patient into the air. Do not place a HEPA respirator on the patient; it is unnecessary and uncomfortable. Use of a HEPA respirator should comply with OSHA standards, which state that facial hair, such as long sideburns or beards, will prevent a proper fit.

You have completed your patient care report and left a copy at the hospital when you realize that you forgot to document a pertinent finding on the front of the report. You should: • A:take no action and report the event to your supervisor. • B:attach an addendum to the original run report. • C:complete a new run report and add the information. • D:write the information on the original run report.

You selected B; This is correct! Reason:If you discover that you forgot to include pertinent information on your patient care report (PCR) after leaving a copy at the hospital, you should write the information on a separate addendum and attach it to the original PCR. A copy of the addendum also should be sent to the receiving facility. Once you leave a copy of your PCR at the hospital, you should not add anything to the original. Legally, this would result in two different records for the same patient. Many EMS systems use electronic PCR (E-PCR) software, in which case the PCR is electronically submitted to the receiving facility upon completion. As with the hand-written PCR, you should add an addendum to the E-PCR if you forget to include pertinent information on the original.

Medical control has ordered you to administer one tube of oral glucose to a hypoglycemic patient. Immediately after receiving this order, you should: • A:ask medical control to repeat the order word for word. • B:repeat the order back to medical control word for word. • C:document the order on the prehospital care report. • D:administer the medication and reassess the patient.

You selected B; This is correct! Reason:Immediately after receiving an order from medical control, you should repeat the order back to medical control word for word. This will ensure that you heard correctly and understand the order to be carried out. If you receive an order that seems inappropriate, you should ask the physician to repeat the order back to you for clarification. Reassess the patient after administering the medication and document the time and patient's response (good or bad) on your patient care report.

Which of the following is a more reliable indicator of perfusion in children than it is in adults? • A:Heart rate • B:Capillary refill • C:Blood pressure • D:Respiratory rate

You selected B; This is correct! Reason:In children younger than 6 years of age, capillary refill time (CRT) serves as an excellent indicator of perfusion; it assesses oxygen delivery to the capillaries. As a person gets older, however, CRT becomes less reliable. It is important to remember that factors such as cold temperature can affect CRT. Early in shock, the heart and respiratory rates increase in an attempt to compensate for decreases in oxygen; this occurs in both children and adults. When these compensatory mechanisms fail, the blood pressure falls, and the patient enters a state of decompensated shock. For this reason, you should not rely upon a patient's blood pressure to determine overall perfusion; the blood pressure may be maintained, despite inadequate perfusion.

You receive a call to a restaurant where a 34-year-old man is experiencing shortness of breath. When you arrive, you immediately note that the man has urticaria on his face and arms. He is conscious, but restless, and is in obvious respiratory distress. You should: • A:obtain a set of baseline vital signs and a SAMPLE history. • B:place a nonrebreathing mask set at 15 L/min on the patient. • C:ask the patient if he has an epinephrine auto-injector. • D:remove the patient's shirt to inspect his chest for urticaria.

You selected B; This is correct! Reason:Initial management of a patient with a suspected allergic reaction is to ensure a patent airway and provide high-flow oxygen. Positive-pressure ventilation may be required if the patient is breathing inadequately. After ensuring a patent airway and adequate ventilation and oxygenation, you should inquire whether the patient has a prescribed epinephrine auto-injector. If so, you should contact medical control and obtain permission (if required by local protocol) to assist the patient with the auto-injector.

Which of the following statements regarding the function of insulin is correct? • A:It stimulates the liver to release glucose into the bloodstream. • B:It facilitates the uptake of glucose from the bloodstream into the cell. • C:It causes the pancreas to produce glucose based on the body's demand. • D:It promotes the entry of glucose from the cell into the bloodstream.

You selected B; This is correct! Reason:Insulin is a hormone produced by the beta cells in the Islets of Langerhans of the pancreas. It promotes the uptake of glucose from the bloodstream into the cells where it is used in the production of energy. Glucagon, a hormone produced by the alpha cells in the pancreas, facilitates the conversion of glycogen to glucose (glycogenolysis) in the liver. The liver does not produce glucose; it produces glycogen, a complex sugar that the body cannot utilize until it has been converted to glucose, a simple sugar.

Immediately following a generalized seizure involving tonic-clonic activity, most patients: • A:have a slow heart rate because of cardiac depression during the seizure. • B:are unresponsive but gradually regain consciousness. • C:experience prolonged apnea and require positive-pressure ventilations. • D:return to a normal level of consciousness but cannot recall the event.

You selected B; This is correct! Reason:Most generalized (grand mal) seizures last 3 to 5 minutes and are followed by a lengthy (5 to 30 minutes) period called the postictal phase, in which the patient is unresponsive at first but gradually regains consciousness. During the seizure itself, the nervous system releases excess adrenaline (epinephrine); therefore, tachycardia, even after the seizure has stopped, is common. Apnea is common during a seizure; however, breathing usually resumes after the seizure has stopped. Many patients are tachypneic following a seizure; this is the body's attempt to eliminate excess carbon dioxide that accumulated in the blood during the seizure. Unlike grand mal seizures, petit mal seizures (also called absence seizures) can last for just a fraction of a minute, after which the patient fully recovers immediately with only a brief lapse of memory of the event.

At the scene of a mass-casualty incident, you notice a bystander who is emotionally upset. An appropriate action to take would be to: • A:tell the bystander to leave the scene at once. • B:assign the bystander a simple, non-patient-care task. • C:have the bystander assist you with patient care. • D:notify the police and have the bystander removed.

You selected B; This is correct! Reason:One of the most effective ways to reduce stress in a bystander at the scene of a mass-casualty incident is to assign the bystander a task that is not related to patient care. This may involve assisting other bystanders who are having difficulties as well or providing water to the rescuers. An obviously distressed bystander should not simply be sent away from the scene, but should be looked at as a patient as well. Clearly, if the bystander becomes aggressive or violent, law enforcement personnel should get involved.

An 80-year-old woman has pain in the right upper quadrant of her abdomen and a yellow tinge to her skin. You should suspect dysfunction of the: • A:gallbladder. • B:liver. • C:pancreas. • D:spleen.

You selected B; This is correct! Reason:Pain in the right upper quadrant and skin with a yellow tinge to it (jaundice) indicates a problem with the liver. Jaundice is the result of an excess level of bilirubin in the blood. Bilirubin, a yellow-colored substance, is the waste product that remains in the bloodstream after the iron is removed from the hemoglobin. One of the functions of the liver is to filter out waste, such as bilirubin, from the blood. If the liver is not functioning properly, bilirubin accumulates in the bloodstream. Dysfunction of the pancreas would result in possible fluctuations in the levels of blood glucose. Gallbladder inflammation (cholecystitis) typically produces right upper quadrant pain and/or referred pain to the right shoulder that occurs shortly after eating. Dysfunction of the spleen would cause left upper quadrant pain and/or referred pain to the left shoulder.

When is it MOST appropriate to complete your patient care report for a critically ill or injured patient? • A:Promptly after the primary assessment • B:As soon as all patient care activities are completed • C:After the ambulance has been restocked at the station • D:Any time before you arrive at the hospital

You selected B; This is correct! Reason:Patient care activities, especially when the patient's condition is critical, take priority over the completion of your patient care report (PCR). Once all patient care activities have been completed, you can complete the PCR. This is usually accomplished at the hospital or immediately upon returning to quarters.

You and your partner have secured a trauma patient to a long backboard and are preparing to lift the backboard onto the stretcher. When doing so, you should: • A:recall that most of the patient's weight is at the foot end of the backboard. • B:ensure that the strongest EMT is positioned at the head of the backboard. • C:lift the backboard from the sides instead of from the ends. • D:be sure to lift the backboard with the powerful muscles of your back.

You selected B; This is correct! Reason:Since more than half of the patient's weight is distributed to the head end of a backboard, you should always ensure that the strongest EMT is at that position. This will reduce the risk of injury to less strong personnel as well as the risk of dropping the patient. The backboard should be lifted from the ends, not the sides; you have less control over the board if it is lifted from the sides. When lifting any patient, you should use the powerful muscles of your thighs, not your back, to lift. Keep your back straight and in a locked-in position.

The daughter of an elderly patient states that her mother is acting confused and talking incoherently. This nature of illness is MOST consistent with: • A:diabetic complications. • B:altered mental status. • C:cardiac compromise. • D:behavioral problems

You selected B; This is correct! Reason:The nature of illness is a category in which you place the patient based on his or her chief complaint. A chief complaint of confusion and incoherent speech suggests that altered mental status is the nature of illness. An altered mental status can be caused by a variety of problems, including diabetic and behavioral problems, and in some cases, cardiac problems.

A 73-year-old male presents with confusion; cool, pale, clammy skin; absent radial pulses; and a blood pressure of 70/40 mm Hg. The patient's wife tells you that he has had abdominal pain for a week and began vomiting a coffee-ground substance yesterday. His past medical history includes hypertension and gastric ulcer disease. Your MOST immediate concern should be that: • A:he is bleeding from his gastrointestinal tract. • B:he is in shock and requires prompt transport. • C:his blood glucose level is probably too high. • D:his condition requires surgery within 2 hours

You selected B; This is correct! Reason:The patient is likely bleeding from his gastrointestinal (GI) tract. Although this is a serious condition, it is not a condition you can treat; internal bleeding cannot be controlled in the field. You can, however, treat his signs and symptoms of shock by administering high-flow oxygen and keeping him warm by covering him with a blanket. Therefore, this should be your most immediate concern. Furthermore, the patient requires prompt transport to the hospital where he can receive definitive care, which may or may not involve surgery. It is highly unlikely that his blood glucose level is too high; he does not have a history of diabetes.

You and your partner arrive at the home of a 60-year-old man with shortness of breath. As you enter the residence, you find the patient sitting in his recliner; he is in obvious respiratory distress. As you approach him, he becomes verbally abusive, stating that it took you too long to get to his home. A small handgun is sitting on a table next to his recliner. You should: • A:verbally distract the patient as your partner attempts to retrieve the gun. • B:slowly back your way out of the residence and call law enforcement. • C:turn around, rapidly exit the residence, and notify law enforcement. • D:explain the reason for your delay as your partner administers oxygen.

You selected B; This is correct! Reason:There is an immediate threat to you and your partner's safety! The patient is verbally abusive and has a firearm within arm's reach. In this situation, you and your partner should slowly back away from the patient, exit the residence, and notify law enforcement. Do NOT take your eyes off of the patient; if you turn your back, you may take a bullet in the back! Backing away gives you the opportunity to take cover if you see the patient reach for his gun. Remember, your safety comes before all else.

A 32-year-old man who was stung by a bee has diffuse hives, facial swelling, and difficulty breathing. When he breathes, you hear audible stridor. What does this indicate? • A:Narrowing of the two mainstem bronchi • B:Swelling of the upper airway structures • C:Narrowing of the bronchioles in the lungs • D:Swelling of the lower airway structures

You selected B; This is correct! Reason:This patient is experiencing a severe allergic reaction (anaphylaxis). Stridor, which is a high-pitched sound heard on inhalation, indicates swelling of the structures and tissues of the upper airway. If not promptly treated, the patient's airway may close completely, resulting in respiratory arrest. Narrowing of the bronchioles in the lungs causes wheezing, a whistling sound that may be heard during inhalation, exhalation, or both.

Which of the following is a sign of an altered mental status in a small child? • A:Consistent eye contact with the EMT. • B:Inattention to the EMT's presence. • C:Recognition of the parents. • D:Fear of the EMT's presence.

You selected B; This is correct! Reason:Typically, a small child will fear the presence of a stranger in his or her environment and will maintain constant eye contact with the stranger; therefore, inattentiveness to your presence should alert you to the presence of an altered mental status.

A 52-year-old woman crashed her minivan into a tree. She is pinned at the legs by the steering wheel and is semiconscious. After gaining access to the patient, you should: • A:have the fire department disentangle the patient and quickly remove her from the car. • B:perform a primary assessment and provide any life-saving care before extrication. • C:rapidly assess her from head to toe, obtain vital signs, and apply a cervical collar. • D:immediately apply high-flow oxygen to the patient and allow extrication to begin.

You selected B; This is correct! Reason:Unless there is an immediate threat of fire, explosion, or other danger, you should perform a primary assessment and begin any live-saving care as soon as you have gained access to the patient. If you wait to do this until after the patient has been disentangled, it may be too late; the patient may already be dead. After you have assessed the patient and treated any immediate threats to life, allow extrication to commence. Once the patient has been freed from the vehicle, continue any life-saving care and perform a rapid head-to-toe assessment to identify and treat other life-threatening injuries. Another EMT can obtain vital signs as you rapidly assess the patient. Prepare for immediate transport after the rapid head-to-toe assessment has been performed and the appropriate spinal precautions have been taken.

A man armed with a shotgun has taken two people hostage and has shot one of them. Upon arriving at the scene, you should: • A:be sure and turn up your portable radio loud enough so that you can remain aware of the entire situation. • B:have the incident commander guide you to a shielded staging area and wait for the tactical team to bring the patient to you. • C:leave your lights and siren on because this will let the injured person know that you have arrived and are there to help. • D:inform the incident commander that you will assume responsibility for the entire incident since there is a confirmed patient

You selected B; This is correct! Reason:Upon arriving at the scene of a tactical situation, such as a hostage situation, the presence of a sniper, or any exchange of gunfire, you should report to the incident commander, who will take you to a shielded, safe staging area that has been selected for the ambulance and for treatment of casualties. Remain there until the tactical team brings the patient or patients to you. As an EMT, you are responsible for patient care once the patient has been brought to you; you are not in charge of the entire incident. When you arrive at the scene, turn your lights and siren off to avoid agitating the gunman. You should also keep your portable radio turned down and minimize radio traffic. If possible, use an ear speaker.

A young woman reports significant weight loss over the last month, persistent fever, and night sweats. When you assess her, you note the presence of dark purple lesions covering her trunk and upper extremities. You should suspect: • A:end-stage cancer. • B:HIV/AIDS. • C:tuberculosis. • D:rheumatic fever.

You selected B; This is correct! Reason:Weight loss, fever, and night sweats could indicate tuberculosis or HIV/AIDS; however, the dark purple lesions on the skin, which are called Kaposi's sarcoma, are malignant skin tumors and are a classic finding in patients in the later stages of AIDS.

The two MOST important steps in treating a patient with a contact poisoning are: • A:determining when the exposure occurred and irrigating the patient's entire body with copious amounts of water. • B:avoiding self-contamination and removing the irritating or corrosive substance from the patient as rapidly as possible. • C:donning a pair of gloves and performing a physical examination to determine if a significant exposure occurred. • D:immediately washing any dry chemicals off of the patient's skin and removing his or her clothing as soon as possible.

You selected B; This is correct! Reason:When caring for a patient with a contact (skin surface) exposure to a poison, the two most important steps in management are avoiding contaminating yourself and removing the substance from the patient's skin as rapidly as possible. The level of personal protective equipment (PPE) you use depends on the type of chemical the patient was exposed to. After ensuring your own safety, remove all of the patient's clothing that has been contaminated, thoroughly brush off any dry chemicals, and then flush the skin with water. Always brush dry chemicals off the skin before irrigating with water; failing to do so may increase the amount of damage caused by the chemical. A physical exam of the patient should be performed only after he or she has been properly decontaminated.

A 3-year-old child has a sudden onset of respiratory distress. The mother denies any recent illnesses or fever. You should suspect: • A:lower respiratory infection. • B:foreign body airway obstruction. • C:croup. • D:epiglottitis.

You selected B; This is correct! Reason:You should suspect a foreign body airway obstruction in any child who presents with an acute onset of respiratory distress in the absence of fever. Croup, epiglottitis, and lower airway infections (ie, bronchiolitis, bronchitis) commonly present with a fever. If the child is experiencing a mild airway obstruction, in which he or she is moving adequate air, has a normal level of consciousness, and pink skin, do not attempt to relieve the airway obstruction; doing so may result in a severe airway obstruction. Offer oxygen and transport the child to the hospital without delay. If signs of a severe airway obstruction are present (ie, ineffective cough, decreased level of consciousness, cyanosis), you should perform abdominal thrusts until the object is expelled or the child becomes unresponsive. If the child becomes unresponsive, perform chest compressions.

Your partner, a veteran EMT of 20 years, has been showing up late to work with increasing frequency over the last several shifts. When he arrives, he is in a bad mood and is clearly not interested in being at work. His behavior is MOST consistent with: • A:acute stress. • B:burnout. • C:drug use. • D:delirium.

You selected B; This is correct! Reason:Your partner's behavior is consistent with burnout. Burnout is a condition of chronic fatigue, irritability, and frustration that results from mounting stress over time. Although burnout typically manifests after years of service in EMS, some EMTs begin to experience it in a very short period of time, especially if they work in EMS systems with a high call volume and low morale. Some people with burnout abuse drugs or alcohol; if you suspect this, you should report it to your supervisor immediately. The best way to prevent burnout is to recognize the signs of stress and take action to reduce it. An acute stress reaction occurs in response to a sudden, unexpected event; it is clear that your partner has had many stressful events over his career. Delirium is an acute change in cognitive ability; it commonly results from conditions such as hypoxia, hypoglycemia, and drug toxicity.

Which of the following situations would necessitate treatment using implied consent? • A:A 17-year-old pregnant woman with an isolated extremity injury • B:A 65-year-old man who is confused and suspected of having a severe stroke • C:A 25-year-old man who is restless and has severe chest pain and diaphoresis • D:An 18-year-old man who is now fully alert after receiving oral glucose

You selected B; This is correct! Reason:A patient may be treated under the law of implied consent, also called the emergency doctrine, any time he or she is unresponsive or otherwise lacks decision-making capacity (ie, confused, under the influence of drugs or alcohol). In cases such as these, the EMT should assume that the patient would consent to treatment and transport if he or she were able to make an informed decision. Examples of such patients include those who are intoxicated or who otherwise have an altered mental status (ie, stroke, hypoglycemia). Patients younger than 18 years of age may also be treated under the law of implied consent, unless the patient is female and is emancipated or pregnant

A 50-year-old woman who is conscious and alert complains of a severe migraine headache. When caring for her, you should generally avoid: • A:transporting her in a supine position. • B:shining a light into her pupils. • C:dimming the lights in the ambulance. • D:applying ice packs to her forehead.

You selected B; This is correct! Reason:Patients with migraine or cluster headaches typically have photophobia (light sensitivity). Any type of bright light, especially if shone directly into the eyes, will cause the patient with a headache unnecessary severe pain. Dimming the lights in the ambulance and making the patient as comfortable as possible are the treatments of choice for a patient with a headache. Some patients benefit from ice packs applied to the forehead; just be sure to wrap the ice pack with roller gauze. Oxygen also should be administered as needed. Typically, the patient will prefer to lie supine or on the side

Which artery should you palpate when assessing for a pulse in an unresponsive 6-month-old patient? • A:Radial • B:Brachial • C:Femoral • D:Carotid

You selected B; This is correct! Reason:You should assess the brachial pulse in infants younger than 1 year of age. The carotid or femoral pulse can be assessed in children older than 1 year of age. A carotid pulse is difficult to locate in infants because they have minimal space between their head and shoulders.

Displaced fractures of the proximal femur are characterized by: • A:lengthening and internal rotation of the leg. • B:shortening and external rotation of the leg. • C:hip joint extension and external leg rotation. • D:a flexed hip joint and inward thigh rotation.

You selected C; The correct answer is B; Reason: Fractures of the proximal (upper) part of the femur are especially common in older people, particularly those with osteoporosis, but may also occur as a result of high-energy trauma in younger patients. Although they are usually called hip fractures, they rarely involve the hip joint. Instead, the break goes through the neck of the femur, the middle region, or across the proximal shaft. Patients with displaced fractures of the proximal femur display a very characteristic deformity. They lie with the leg externally rotated, and the injured leg is usually shorter than the uninjured leg. If the fracture is not displaced, this deformity is not present. A flexed hip joint and internal rotation of the thigh are characteristic of a posterior hip dislocation. With the less common anterior hip dislocation, the limb is in the opposite position, extended straight out, externally rotated, and pointing away from the midline of the body.

A man was struck in the side of the head with a steel pipe. Blood-tinged fluid is draining from the ear and bruising appears behind the ear. The MOST appropriate treatment for this patient includes: • A:controlling the drainage from the ear and immobilizing the entire spine. • B:elevating the lower extremities and providing immediate transport. • C:applying high-flow oxygen and packing the ear with sterile gauze pads. • D:immobilizing the spine, administering oxygen, and monitoring for vomiting.

You selected C; The correct answer is D; Reason: Patients with significant head injury should be treated by applying high-flow oxygen, assisting ventilations as needed, immobilzing the entire spine, and transporting promptly. Closely monitor the patient for vomiting and be prepared to suction the airway. Elevation of the foot of the spine board may cause more blood to engorge the brain and may increase intracranial pressure (ICP). You should never attempt to control bleeding or fluid drainage from the ears of a patient with a head injury because this too may result in increased ICP. If a patient with an isolated head injury begins showing signs of shock (ie, tachycardia, diaphoresis, tachypnea, hypotension), you should assume that he or she has internal bleeding from another injury and treat accordingly (ie, elevation of the foot-end of the backboard, preventing body heat loss).

A patient who overdosed on methamphetamine would be expected to have all of the following clinical signs, EXCEPT: • A:agitation. • B:hypertension. • C:dilated pupils. • D:bradycardia.

You selected C; The correct answer is D; Reason:Methamphetamine, an upper, stimulates the central nervous system, causing it to release excessive amounts of adrenaline (epinephrine). Epinephrine increases heart rate and blood pressure; therefore, the patient would experience tachycardia and hypertension. Other signs of methamphetamine overdose, which also indicate an adrenaline surge, include pupillary dilation, agitation, and hyperthermia. Bradycardia would be expected in patients who have overdosed on drugs that suppress, not stimulate, the central nervous system (eg, narcotics, benzodiazepines, barbiturates).

When performing two-rescuer CPR on an adult patient whose airway has not been secured with an advanced device, you should: • A:continue ventilations as the AED analyzes the patient's cardiac rhythm. • B:deliver ventilations at a rate of 8 to 10 breaths/min. • C:have your partner pause after 30 compressions as you give 2 breaths. • D:avoid synchronizing compressions with ventilations

You selected C; This is correct! Reason: When performing two-rescuer adult CPR, you should perform cycles of CPR, with a compression to ventilation ratio of 30:2. If the airway is not secured with an advanced device (eg, ET tube, multilumen airway, supraglottic airway) ventilations and chest compressions should be coordinated (synchronous). After your partner delivers 30 compressions, he or she should pause as you deliver two breaths. After the airway has been secured with an advanced device, do not attempt to synchronize compressions and ventilations. Compressions should be performed continuously at a rate of at least 100/min and ventilations should be given at a rate of 8 to 10 breaths/min (one breath every 6 to 8 seconds). All contact with the patient must cease as the AED is analyzing the cardiac rhythm.

A middle-aged woman took three of her prescribed nitroglycerin tablets after she began experiencing chest pain. She complains of a bad headache and is still experiencing chest pain. You should assume that: • A:her chest pain is not cardiac-related. • B:her blood pressure is elevated. • C:she has ongoing cardiac ischemia. • D:her nitroglycerin is no longer potent.

You selected C; This is correct! Reason: A headache and/or a bitter taste under the tongue are common side effects of nitroglycerin (NTG) that many patients experience. If the patient does not experience these side effects, the NTG may have lost its potency. However, if a patient with chest pain takes NTG and experiences these side effects, but still has chest pain, you should assume that his or her pain is the result of cardiac ischemia, a relative deprivation of oxygen to the heart. NTG is a vasodilator drug; if anything, three doses would lower her blood pressure, not raise it. Any patient with nontraumatic chest pain or pressure should be assumed to be experiencing cardiac ischemia, especially if the pain or pressure is not relieved with NTG.

Which of the following would MOST likely occur if an adult patient is breathing at a rate of 45 breaths/min with shallow depth? • A:The lungs would become hyperinflated, potentially causing a pneumothorax. • B:The volume of air that reaches the alveoli would increase significantly. • C:Most of his or her inhaled air will not go beyond the anatomic dead space. • D:Alveolar minute volume would increase due to the rapid respiratory rate.

You selected C; This is correct! Reason: Alveolar minute volume, the amount of air that reaches the alveoli per minute and participates in pulmonary respiration, is affected by tidal volume, respiratory rate, or both. If the respiratory rate decreases, tidal volume must increase in order to maintain adequate alveolar minute volume. Conversely, if tidal volume decreases, the respiratory rate must increase accordingly. However, if the respiratory rate is extremely fast, especially if the depth of breathing is shallow (reduced tidal volume), most of the inhaled air will only make it to the anatomic dead space (ie, trachea, larger bronchi) before it is promptly exhaled. As a result, alveolar minute volume would decrease, resulting in inadequate pulmonary respiration and hypoxia. For this reason, patients with rapid, shallow breathing often require ventilation assistance. Pulmonary hyperinflation would not be an issue in a patient with exceedingly fast breathing and reduced tidal volume because very little air is actually reaching the lungs.

A woman stabbed her boyfriend in the cheek with a dinner fork during an argument. Police have the woman in custody. The patient still has the fork impaled in his cheek. He is conscious and alert, breathing adequately, and has blood in his oropharynx. You should: • A:suction his oropharynx, carefully cut the fork to make it shorter, control any external bleeding, and secure the fork in place. • B:apply high-flow oxygen via a nonrebreathing mask, carefully remove the fork, and control any external bleeding. • C:suction his oropharynx, control any external bleeding, stabilize the fork in place, and protect it with bulky dressings. • D:carefully remove the fork, suction his oropharynx as needed, and pack the inside of his cheek with sterile gauze pads.

You selected C; This is correct! Reason: An impaled object in the cheek should be removed if it interferes with your ability to manage the patient's airway. In this case, however, the patient is breathing adequately and does not require aggressive airway care (eg, ventilatory assistance). The most practical approach is to suction the blood from his oropharynx, which will prevent him from swallowing it, vomiting it, and aspirating it. Stabilize the fork in place and protect it with bulky dressings; removing an impaled object from the cheek in the opposite direction it entered may cause further soft tissue damage. Transport the patient in a sitting position and suction his oropharynx en route as needed. There is no reason to cut the fork to make it shorter; this will only unnecessarily manipulate it, potentially causing further soft tissue damage and increased bleeding.

Which of the following is MOST indicative of a primary cardiac problem? • A:Tachypnea • B:Sudden fainting • C:Irregular pulse • D:Tachycardia

You selected C; This is correct! Reason: An irregular pulse signifies an abnormality within the electrical conduction system of the heart. Tachycardia, sudden fainting (syncope), and tachypnea (rapid breathing) can indicate many things other than cardiac problems, such as shock, heat-related problems, and diabetic complications. You should always consider the possibility of a cardiac problem in a patient with an irregular pulse.

The position of comfort for a patient with nontraumatic chest pain MOST commonly is: • A:supine with the legs elevated slightly. • B:on the side with the head elevated. • C:semisitting. • D:lateral recumbent.

You selected C; This is correct! Reason: As with most patients, the position of comfort for cardiac patients typically is the semisitting (semi-Fowler) position. You should allow the patient to remain in the position of comfort both during the assessment phase as well as throughout transport.

A patient experienced blunt chest trauma and has asymmetrical chest wall movement. This MOST likely indicates: • A:accumulation of blood in both of the lungs. • B:shallow breathing secondary to severe pain. • C:decreased air movement into one lung. • D:several ribs broken in numerous places.

You selected C; This is correct! Reason: Asymmetrical chest wall movement, when one side of the chest moves less than the other, indicates decreased air movement into one lung (eg, pneumothorax, hemothorax). Bleeding into both lungs and shallow breathing due to severe pain would likely cause decreased movement to both sides of the chest. If more than two ribs are fractured in several places, a free-floating (flail) segment of fractured ribs is created. This flail segment (not necessarily an entire half of the chest) collapses during inhalation and bulges during exhalation; this is called paradoxical chest movement.

During a soccer game, a 20-year-old man collided shoulder-to-shoulder with another player. He has pain and a noticeable anterior bulge to the left shoulder. Which of the following is the MOST effective method of immobilizing this injury? • A:An air-inflatable splint with the left arm immobilized in the flexed position • B:A sling to support the left arm and swathes to maintain downward traction • C:A sling to support the left arm and swathes to secure the arm to the body • D:A long board splint with the left arm immobilized in the extended position

You selected C; This is correct! Reason: Injuries to the shoulder are most effectively immobilized with the use of a sling and swathe. The sling will provide support and relieve pain to the shoulder, and the swathe will secure the arm to the body. The purpose of the swathe is not to facilitate traction. Patients with dislocated or fractured shoulders will not allow you to extend their arm, so any attempt to immobilize the injury in such a fashion will not be possible and could worsen the injury.

Epistaxis

bleeding from the nose

A 33-year-old factory worker was pinned between two pieces of machinery. When you arrive at the scene, you find him lying supine on the ground complaining of severe pain to his pelvis. He is restless, diaphoretic, and tachycardic. After performing a rapid head-to-toe assessment, you should: • A:perform a detailed secondary exam. • B:palpate his pelvis to assess for crepitus. • C:prepare for immediate transport. • D:carefully log roll him to check his back.

You selected C; This is correct! Reason: Based on the mechanism of injury and the presence of signs of shock (eg, restlessness, tachycardia, diaphoresis), you should suspect that the patient has a fractured pelvis and is bleeding internally. Therefore, after completing your primary assessment and initiating shock treatment (eg, high-flow oxygen, applying blankets), you should perform a rapid head-to-toe assessment to assess for other injuries and then prepare for immediate transport. Spinal precautions should be considered. Do not log roll the patient; doing so compresses the pelvis and may cause further injury. You should also avoid palpating his pelvis; this will only cause further pain and may cause further injury. Palpation of the pelvis is performed to assess its stability, not to elicit crepitus. A detailed secondary exam of a critically injured patient at the scene is not appropriate; it takes too long to perform and should be done en route to the hospital if time permits.

Which of the following ventilation techniques will enable you to provide the greatest tidal volume AND allow you to effectively assess lung compliance? • A:One-rescuer bag-mask ventilation • B:Flow-restricted, oxygen-powered ventilation • C:One-rescuer mouth-to-mask ventilation • D:One-rescuer demand valve ventilation

You selected C; This is correct! Reason: Because the EMT uses both of his or her hands to obtain a mask seal, the one-rescuer mouth-to-mask ventilation technique will provide the greatest tidal volume compared to the other methods listed. Furthermore, lung compliance, the ability of the lungs to expand when ventilated, can be effectively assessed because air is directly blown into the patient's lungs from the EMT's mouth. The one-rescuer bag-mask ventilation technique may allow the EMT to get a sense of lung compliance; however, because maintaining an adequate mask-to-face seal is often difficult, lesser tidal volume can be given relative to the mouth-to-mask technique. The flow-restricted, oxygen-powered ventilation device (eg, manually triggered ventilator, demand valve) provides excellent tidal volume; however, because it is a mechanical device, it does not allow the EMT to assess lung compliance.

A 40-year-old man was hit in the nose during a fight. He has bruising under his left eye and a nosebleed. After taking standard precautions, you should: • A:place a chemical icepack over his nose. • B:apply direct pressure by pinching his nostrils together. • C:ensure that he is sitting up and leaning forward. • D:determine if he has any visual disturbances.

You selected C; This is correct! Reason: During a nosebleed (epistaxis), much of the blood may pass down the throat into the stomach as the patient swallows; this is especially true if the patient is lying supine. Blood is a gastric irritant; a person who swallows a large amount of blood may become nauseated and vomit, which increases the risk of aspiration. Therefore, your first action should be to ensure that the patient is sitting up and leaning forward. This will prevent blood from draining down the back of the throat. Next, apply direct pressure by pinching the fleshy part of the nostrils together; you or the patient may do this. Placing a chemical icepack over the nose may further help control the bleeding by constricting the nasal vasculature. After controlling the nosebleed, continue your assessment, which includes assessing for facial deformities and visual disturbances.

After delivering one shock with the AED and performing 2 minutes of CPR on a woman in cardiac arrest, you reanalyze her cardiac rhythm and receive a no shock advised message. This means that: • A:the first shock restored a rhythm and pulse. • B:she has electrical activity but no pulse. • C:she is not in a shockable rhythm. • D:her rhythm has deteriorated to asystole.

You selected C; This is correct! Reason: If the AED gives a no shock advised message, it has determined that the patient is not in a shockable rhythm (eg, V-Fib, pulseless V-Tach). It does not indicate that the patient has a pulse, nor does it indicate that a normal cardiac rhythm has been restored. The AED does not distinguish pulseless electrical activity (PEA) from asystole; it only recognizes them as nonshockable. PEA is a condition in which organized cardiac electrical activity is present despite the absence of a pulse. Asystole is the absence of all cardiac electrical and mechanical activity. If the AED gives a no shock advised message, immediately resume CPR, starting with chest compressions, until ALS arrives or the patient starts to move.

Which of the following is MOST indicative of compensated shock in an adult? • A:Weak carotid pulse, cool skin, increased respiratory rate • B:Unresponsive, pallor, absent radial pulses, tachypnea • C:Restless, diaphoresis, tachypnea, BP of 104/64 mm Hg • D:Confusion, mottling, tachycardia, BP of 88/60 mm Hg

You selected C; This is correct! Reason: In compensated shock, the nervous system is mounting a physiologic response to an underlying illness or injury in order to maintain perfusion to vital organs such as the brain, heart, and kidneys. The patient with compensated shock is restless or anxious, has poor peripheral perfusion (eg, pallor, diaphoresis), tachycardia, and increased respirations (tachypnea). However, his or her blood pressure is maintained, usually above 90 to 100 mm Hg. In decompensated shock, the body's compensatory mechanisms fail, blood pressure begins to fall, and perfusion to vital organs decreases. Other signs of decompensated shock include a decreased level of consciousness, absent peripheral pulses (radial), and weak central pulses (carotid, femoral).

In patients with heart disease, acute coronary syndrome is MOST often the result of: • A:coronary artery spasm. • B:coronary artery rupture. • C:atherosclerosis. • D:atrial damage.

You selected C; This is correct! Reason: In most patients with acute coronary syndrome, or ACS (eg, unstable angina, acute myocardial infarction), atherosclerosis is the underlying problem that causes heart disease. Atherosclerosis is a disorder in which calcium and a fatty material called cholesterol build up and form a plaque inside the walls of blood vessels, obstructing blood flow. ACS due to atherosclerosis usually occurs when a fragment of plaque ruptures and occludes a coronary artery; further occlusion occurs when platelets aggregate in the area and clump together. Less commonly, acute coronary artery spasm may result in ACS. The cause of acute coronary vasospasm is largely unknown. Rupture of a coronary artery is a rare cause of ACS. Atrial or ventricular damage is usually caused by, rather than the cause of, ACS.

Which of the following would clearly be detrimental to a patient in cardiac arrest? • A:Ventilating just until the chest rises • B:Using a pocket face mask without high-flow oxygen • C:Interrupting CPR for more than 10 seconds • D:Performing CPR before defibrillation

You selected C; This is correct! Reason: Major emphasis is placed on minimizing interruptions in CPR. Even brief interruptions cause a significant decrease in blood flow to the heart and brain. If you must interrupt CPR, do not exceed 10 seconds. It is preferable to ventilate a patient with a pocket face mask attached to high-flow oxygen, but failing to do so will not be nearly as detrimental as interrupting CPR for extended periods of time. You should ventilate the patient just until the chest visibly rises; ventilations that are too forceful or too fast can cause hyperinflation of the lungs, which may reduce blood return to the heart. When caring for any patient in cardiac arrest, you should immediately begin CPR, and then apply the AED as soon as possible.

If a vehicle strikes a tree at 60 MPH, the unrestrained driver would likely experience the MOST severe injuries during the: • A:first collision. • B:fourth collision. • C:third collision. • D:second collision.

You selected C; This is correct! Reason: Motor-vehicle crashes typically consist of three separate collisions. Understanding the events that occur during each collision will help you remain alert for certain types of injury patterns. During the first collision, the vehicle strikes another object. Damage to the car is perhaps the most dramatic part of the collision, but it does not directly affect patient care. It does, however, provide information about the severity of the collision; thus, it has an indirect effect on patient care. During the second collision, the passenger collides with the interior of the vehicle. Just like the obvious damage to the exterior of the car, the injuries that result are often dramatic and usually apparent during your primary assessment. During the third collision, the occupant's internal organs collide with the solid structures of the body. Although the injuries that occur during the third collision may not be as obvious as those that occur during the second collision, they are often the most life-threatening.

Nitroglycerin is contraindicated in all of the following situations, EXCEPT: • A:systolic BP of 80 mm Hg. • B:recent use of Cialis. • C:history of cardiac bypass surgery. • D:the presence of a head injury.

You selected C; This is correct! Reason: Nitroglycerin (NTG) is a vasodilator drug used to relieve chest pain in patients with cardiac compromise by dilating the coronary arteries and improving blood flow to the heart. Because of its vasodilator effects, it should not be given to patients who have a systolic BP less than 100 mm Hg or to patients who have recently (within the past 24 to 48 hours) taken erectile dysfunction (ED) drugs (eg, sildenafil [Viagra], vardenafil [Levitra], tadalafil [Cialis]). ED drugs also cause vasodilation and may cause significant hypotension if given together with NTG. You should also avoid NTG in patients with a head injury; dilation of the cerebral blood vessels may worsen intracranial pressure caused by the head injury. By itself, a history of cardiac bypass surgery does not contraindicate the use of NTG.

After administering nitroglycerin to a patient with chest discomfort, it is MOST important for you to: • A:place the patient supine and elevate his or her legs. • B:ask the patient if the discomfort has improved. • C:reassess his or her blood pressure within 5 minutes. • D:find out how long the discomfort has been present.

You selected C; This is correct! Reason: Nitroglycerin (NTG) relaxes the muscle of blood vessel walls, dilates the coronary arteries, increases blood flow and the supply of oxygen to the heart muscle (myocardium), and decreases the workload of the heart. NTG also dilates blood vessels in other parts of the body, potentially resulting in hypotension. For this reason, you should reassess the patient's blood pressure within 5 minutes after each dose of NTG. If the systolic blood pressure is less than 100 mm Hg, do not give any more NTG. Position him or her supine and elevate his or her legs if needed. Asking the patient if his or her chest pain or discomfort has improved following NTG helps you determine if the drug is working and whether additional dosing is needed; however, detecting hypotension is clearly more important. You should determine when the chest pain or discomfort began during the focused history, which is typically performed before assisting a patient with his or her prescribed NTG.

In which of the following patients is nitroglycerin contraindicated? • A:66-year-old female with chest pressure of 6 hours' duration, lightheadedness, and a blood pressure of 110/58 mm Hg • B:41-year-old male with crushing substernal chest pressure, a blood pressure of 160/90 mm Hg, and severe nausea • C:53-year-old male with chest discomfort, diaphoresis, a blood pressure of 146/66 mm Hg, and regular use of Levitra • D:58-year-old male with chest pain radiating to the left arm, a blood pressure of 130/64 mm Hg, and prescribed Tegretol

You selected C; This is correct! Reason: Nitroglycerin is contraindicated in patients who do not have a prescription for nitroglycerin, in those with a systolic BP less than 100 mm Hg, and in patients who have taken medications for erectile dysfunction (ED) within the previous 24 to 48 hours. Such medications include sildenafil (Viagra), vardenafil (Levitra) and tadalafil (Cialis). Because ED drugs and nitroglycerin both cause vasodilation, concomitant use of these drugs may result in significant hypotension. Carbamazepine (Tegretol) is an anticonvulsant medication; there are no known interactions between Tegretol and nitroglycerin.

Which of the following patients is the BEST candidate for the administration of nitroglycerin? • A:A man with chest pain, expired nitroglycerin spray, and a blood pressure of 110/80 mm Hg • B:A woman who has taken three doses of prescribed nitroglycerin without relief of chest pain • C:A woman with chest pain, prescribed nitroglycerin, and a blood pressure of 104/76 mm Hg • D:An elderly man with crushing substernal chest pain and a blood pressure of 80/60 mm Hg

You selected C; This is correct! Reason: Nitroglycerin should be administered to patients who have the prescribed, unexpired drug with them and a systolic blood pressure of greater than 100 mm Hg. No more than three (3) nitroglycerin tablets or sprays should be administered to a patient in the prehospital setting. An expired medication should never be administered to any patient, even if the medication is otherwise indicated for his or her condition

When attaching an oxygen regulator to a D cylinder and preparing it for use, you should recall that: • A:the cylinder must remain in a standing position at all times or it will not deliver any oxygen. • B:a pressure-compensated flowmeter should be used when lying the oxygen cylinder down. • C:oxygen supports combustion and should not be used where sparks are easily generated. • D:the cylinder should be taken out of service and refilled when it contains less than 750 psi. You selected C; This is correct!

You selected C; This is correct! Reason: Oxygen does not burn or explode; however, it does support combustion. A small spark, even a lit cigarette, can become a flame in an oxygen-rich atmosphere. Therefore, you must ensure that the environment in which you will use oxygen is adequately ventilated, especially in industrial settings where hazardous materials may be present and where sparks are easily generated. Never leave an oxygen cylinder standing unattended; the cylinder can be knocked over, injuring the patient or damaging the equipment. The D cylinder (small cylinder carried to the patient) should be taken out of service and refilled when the pressure inside it falls below 500 psi. The pressure-compensated flowmeter, which contains a ball and float that rises or falls according to the gas flow, can only be used when an oxygen cylinder is upright, which is why it is used with on-board oxygen (M cylinder). The Bourdon-gauge flowmeter does not require the oxygen cylinder to be upright, which is why it is used with D cylinders.

During your assessment of a 70-year-old man with crushing chest pain, you note that his blood pressure is 80/50 mm Hg. Your MOST important action should be to: • A:assess his oxygen saturation. • B:give high-flow oxygen. • C:transport without delay. • D:keep the patient warm.

You selected C; This is correct! Reason: Patients with chest pain, pressure, or discomfort with a systolic BP less than 100 mm Hg should be transported to the hospital without delay. Hypotension in a patient with chest pain indicates cardiogenic shock due to severe cardiac damage and requires treatment that can only be given at the hospital. Any delay in transport delays definitive care and increases the patient's chance of death. High-flow oxygen, thermal management, and assessment of oxygen saturation are clearly important; however, your primary focus should be to get the patient to the hospital as soon as possible.

An unresponsive patient's respirations are 26 breaths/min and shallow. The MOST appropriate treatment includes: • A:a simple face mask set at 10 to 12 L/min. • B:a nonrebreathing mask set at 15 L/min. • C:assisted ventilations with 100% oxygen. • D:a nasal cannula set at 2 to 6 L/min.

You selected C; This is correct! Reason: Shallow respirations (reduced tidal volume) at a rate of 26 breaths/min will not provide adequate minute volume. Therefore, you should assist the patient's ventilations with a bag-mask device and high-flow oxygen. Passive oxygenation devices (eg, nonrebreathing mask, simple face mask, nasal cannula) will be of little benefit to a patient with inadequate breathing. The patient must have adequate tidal volume in order to effectively breath in oxygen from these devices.

Sonorous respirations are MOST rapidly corrected by: • A:suctioning the oropharynx. • B:inserting an oropharyngeal airway. • C:correctly positioning the head. • D:initiating assisted ventilations.

You selected C; This is correct! Reason: Sonorous (snoring) respirations, which most commonly result from partial airway obstruction by the tongue, are most rapidly corrected by simply positioning the head. This involves using either the head tilt-chin lift or the jaw-thrust maneuver if trauma is suspected. To further ensure airway patency, a simple adjunct (oral or nasal airway) may need to be inserted. The patient's airway should be suctioned if a gurgling sound is heard during breathing.

What is the function of pulmonary surfactant? • A:It facilitates the production of mucous, which is expelled during coughing. • B:It carries fresh oxygen from the lungs to the left side of the heart. • C:It lubricates the alveolar walls and allows them to expand and recoil. • D:It dilates the bronchioles in the lungs and enhances the flow of air.

You selected C; This is correct! Reason: Surfactant is a lubricant that lines the alveolar walls. It allows them to expand and recoil freely, thereby allowing for an easy exchange of oxygen and carbon dioxide. Diseases such as emphysema cause destruction of the alveolar walls and a decrease in pulmonary surfactant. This makes the normal process of breathing very difficult for these patients. Mucous-producing cells, called Goblet cells, line the trachea and larger bronchi. Provided the patient has an effective cough reflex, bacteria and other pathogens can be expelled from the body via the mucous produced by the Goblet cells.

An adult patient opens his eyes in response to a painful stimulus, moans when you ask him questions, and pulls his arm away when you palpate it. What is his Glasgow Coma Scale (GCS) score? • A:7 • B:9 • C:8 • D:6

You selected C; This is correct! Reason: The Glasgow Coma Score (GCS) assesses three neurologic parameters: eye opening, verbal response, and motor response. Your patient's GCS score is 8. For eye opening, he receives 2 points for opening his eyes in response to pain. For verbal response, he receives 2 points for moaning or making unintelligible sounds. For motor response, he receives 4 points for withdrawing to pain. The GCS is a valuable neurologic assessment tool; it should be reassessed frequently in seriously injured patients—especially patients with a head injury.

The myocardium receives its blood supply from the coronary arteries that branch directly from the: • A:left atrium. • B:right ventricle. • C:aorta. • D:vena cava.

You selected C; This is correct! Reason: The aorta, which is the largest artery in the human body, originates immediately from the left ventricle where it branches into the coronary arteries. This allows the myocardium to receive blood that has the highest concentration of oxygen. The superior and inferior venae cavae return oxygen-poor blood from the systemic circulation back to the right atrium, where it is pumped into the right ventricle. The left atrium receives freshly oxygenated blood from the lungs.

Which of the following devices is contraindicated in patients with blunt chest trauma? • A:Oral airway • B:Nasal airway • C:Oxygen-powered ventilator • D:Bag-mask device

You selected C; This is correct! Reason: The flow-restricted, oxygen-powered ventilation device (FROPVD), also referred to as an oxygen-powered ventilator or manually-triggered ventilator, should not be used in patients with chest trauma; it delivers oxygen under high pressure (40 L/min) and may worsen the patient's injury. The FROPVD is also associated with a high incidence of gastric distention. The FROPVD is also contraindicated in pediatric patients and in patients with COPD. Infants and children have small lungs; the high ventilatory pressure delivered by the FROPVD can easily cause a pneumothorax. Patients with COPD often have air trapped in their lungs; excessive ventilatory pressure may cause alveolar rupture or a pneumothorax.

You are dispatched to a residence for an elderly female who has possibly suffered a stroke. You find her lying supine in her bed. She is semiconscious; has vomited; and has slow, irregular breathing. You should: • A:perform a head tilt-chin lift and insert an oral airway. • B:administer high-flow oxygen and place her on her side. • C:manually open her airway and suction her oropharynx. • D:insert a nasal airway and begin assisting her breathing.

You selected C; This is correct! Reason: This patient's airway is in immediate jeopardy! The first step in caring for any semi- or unconscious patient is to manually open the airway (eg, head tilt-chin lift, jaw-thrust) and ensure it is clear of obstructions or secretions. Because the patient has vomited, she likely has vomitus in her mouth, which must be removed with suction before she aspirates it into her lungs. Mortality increases significantly if aspiration occurs. After opening her airway and removing any vomitus or secretions from her oropharynx with suction, you should insert an airway adjunct (a nasal airway in this case; the patient is semiconscious and likely has an intact gag reflex) and begin assisting her breathing with a bag-mask device. Her respiratory effort is inadequate and should be treated with some form of positive-pressure ventilation, not a nonrebreathing mask. Placing a semi- or unconscious patient on his or her side (recovery position) is only appropriate if he or she is breathing adequately; this patient is not.

Tidal volume is defined as the: • A:volume of air moved in and out of the lungs each minute. • B:total volume of air that the lungs are capable of holding. • C:volume of air inhaled or exhaled per breath. • D:volume of air that remains in the upper airway.

You selected C; This is correct! Reason: Tidal volume (VT) is the amount of air that is inhaled or exhaled per breath; it is normally 500 mL in a healthy adult male. Tidal volume is assessed by noting the depth of a patient's breathing. Shallow breathing, for example, indicates a reduced tidal volume. The volume of air that remains in the upper respiratory tract (eg, larger bronchi, trachea) is called dead space volume (VD); it is approximately 30% of the adult male's tidal volume and does not participate in pulmonary gas exchange. The volume of air that moves in and out of the lungs each minute, and does participate in pulmonary gas exchange, is called alveolar minute volume (VA). It is calculated by multiplying the tidal volume (minus the dead space volume) and the respiratory rate. Therefore, if an adult male has a tidal volume of 500 mL and a respiratory rate of 18 breaths/min, his alveolar minute volume is 6,300 mL (500 mL [VT] - 150 mL [VD] × 18 [breaths/min] = 6,300 mL [VA]). The maximum volume of air that the lungs are capable of holding is called the total lung capacity (TLC); it is approximately 6 L in the healthy adult male.

The tidal volume of an unresponsive patient is rapidly assessed by: • A:auscultating his or her lung sounds. • B:counting the patient's respiratory rate. • C:observing for chest rise during inhalation. • D:evaluating for the presence of cyanosis.

You selected C; This is correct! Reason: Tidal volume, a measure of the depth of breathing, is the amount of air in milliliters (mL) that is moved into or out of the lungs during a single breath. The average tidal volume for an adult male is approximately 500 mL. The quickest and most effective way to assess a patient's tidal volume is to observe his or her chest during breathing. If the chest rises adequately during inhalation, tidal volume is probably adequate. If the chest rises very little, as with shallow breathing, tidal volume is likely reduced. Auscultating breath sounds can give you an idea as to the patient's tidal volume; bilaterally diminished breath sounds may indicate a reduced tidal volume. It is quicker, however, to simply observe the chest for adequate rise. The presence of cyanosis indicates hypoxemia and is not a direct reflection of tidal volume.

After stopping the burning process, emergency care for a 68-year-old male with partial- and full-thickness burns to his chest and upper extremities includes all of the following, EXCEPT: • A:preparing to assist the patient's ventilations. • B:covering the burns with dry, sterile dressings. • C:flushing the burns with cool water for 10 minutes. • D:avoiding the use of burn ointments or antiseptics.

You selected C; This is correct! Reason: Unless the patient is on fire, do not apply water to a full-thickness (third-degree) burn, especially if the patient is already prone to hypothermia and infection (ie, older adults, small children). Cover the burns with dry, sterile dressings or a sterile burn sheet. The use of burn creams, ointments, or antiseptics should be avoided; these increase the risk of infection and will only need to be removed at the hospital. Apply high-flow oxygen, treat any associated injuries, and rapidly transport the patient. If the patient is breathing inadequately (eg, fast or slow rate, shallow breathing [reduced tidal volume]), assist ventilations with a bag-mask device.

Which of the following questions would be the MOST effective in determining if a patient's chest pain radiates away from his or her chest? • A:Do you also have pain in your arm, jaw, or back? • B:Is there any other part of your body where you have pain? • C:Does the pain stay in your chest or move anywhere else? • D:Is there anything that makes the pain better or worse?

You selected C; This is correct! Reason: When assessing a patient with any type of pain, you should avoid asking leading questions; instead, ask open-ended questions whenever possible. For example, instead of asking the patient if his or her pain is dull, crushing, or sharp, ask him or her to describe the pain using his or her own words. Patients with radiating pain often state that the pain moves or travels away from its point of origin, with pain in between point A and point B. Patients with referred pain complain of pain in more than one location, without a trail of pain in between. You should also ask the patient if anything makes the pain worse (provokes) or better (palliates).

When ventilating an apneic patient with a pocket mask device, each breath should be delivered over: • A:3 seconds. • B:4 seconds. • C:1 second. • D:2 seconds.

You selected C; This is correct! Reason: When ventilating any apneic patient, each breath should be delivered over a period of 1 second—just enough to produce visible chest rise. Excessive ventilation duration and/or volume increases the likelihood of gastric distention—especially if the patient's airway is not secured with an advanced device (ie, ET tube, multilumen airway, supraglottic airway)—and may result in increased intrathoracic pressure, decreased venous return to the heart, and decreased cardiac output.

Despite direct pressure, a large laceration continues to spurt large amounts of bright red blood. You should: • A:elevate the extremity and apply a tight pressure dressing. • B:place additional dressings on the wound until the bleeding stops. • C:apply a tourniquet proximal to the injury until the bleeding stops. • D:apply pressure to the pulse point that is proximal to the injury.

You selected C; This is correct! Reason: You must control any and all external bleeding as soon as possible. In the case of arterial bleeding (ie, bright red blood is spurting from the wound), the patient will bleed to death if immediate action is not taken. In most cases, direct pressure will effectively control external bleeding. However, if the wound continues to bleed profusely despite direct pressure, you should apply a tourniquet proximal to the injury and tighten it until the bleeding stops. Packing additional dressings on a severe external hemorrhage will only cause the patient to continue to bleed externally into the dressings. Locating and applying adequate pressure to a proximal arterial pressure point is often difficult and time-consuming.

You are assessing a middle-aged male who is experiencing respiratory distress. The patient has a history of emphysema and hypertension. He appears fatigued; has weak retractions; and labored, shallow breathing. Your MOST immediate action should be to: • A:auscultate his breath sounds to detect wheezing. • B:administer oxygen with a nonrebreathing mask. • C:assist his ventilations with a bag-mask device. • D:assess his oxygen saturation with a pulse oximeter.

You selected C; This is correct! Reason: Your patient is NOT breathing adequately. He is fatigued; has weak retractions; and labored, shallow breathing. If you do not treat him immediately, he may stop breathing altogether. You should begin assisting his ventilations with a bag-mask device and high-flow oxygen. After initiating ventilatory assistance, attach the pulse oximeter to assess his oxygen saturation and auscultate his breath sounds. A nonrebreathing mask is appropriate for patients with difficulty breathing who are moving air adequately; this patient is not!

While triaging patients at a mass-casualty incident, you encounter a responsive middle-aged female with a respiratory rate of 26 breaths/min. What should you do next? • A:Triage her as immediate (red tag) • B:Administer high-flow oxygen at once • C:Assess for bilateral radial pulses • D:Assess her ability to follow commands

You selected C; This is correct! Reason: According to the Simple Triage And Rapid Treatment (START) method, if you encounter an adult patient with a respiratory rate that is less than 10 breaths/min or greater than 29 breaths/min, you should triage him or her as immediate (red tag) and move to the next patient. However, if the patient's respiratory rate is between 10 and 29 breaths/min, you should assess his or her hemodynamic status by checking for bilateral radial pulses. If the patient's radial pulses are absent, triage him or her as immediate and move to the next patient. If the patient's radial pulses are present, assess his or her ability to follow simple commands. If the patient is able to follow simple commands, triage him or her as delayed (yellow tag) and move to the next patient. If the patient is unable to follow simple commands, triage him or her as immediate and move to the next patient. Treatment does not occur during the initial triage process.

Which of the following conditions would be the LEAST likely to be present in a patient who was submerged in water? • A:Spinal injury • B:Gastric distention • C:Hyperglycemia • D:Laryngospasm

You selected C; This is correct! Reason: Many factors can contribute to or result from a submersion injury (eg, drowning, near-drowning). It is not uncommon for a person to experience a spinal injury after diving head first into shallow water, especially if he or she is under the influence of alcohol. When a swimmer panics, he or she initially swallows large amounts of water, resulting in gastric distention. Gastric distention can cause aspiration if the patient regurgitates water during rescue breathing; protect the airway! During the panic phase, the victim expends a tremendous amount of energy (and glucose) from flailing around in the water, possibly resulting in hypoglycemia. Inhaling even a small amount of fresh or salt water can severely irritate the larynx, which sends the muscles of the larynx and vocal cords into spasm (laryngospasm), resulting in airway blockage and hypoxia.

Following delivery of a newborn and placenta, you note that the mother has moderate vaginal bleeding. The mother is conscious and alert and her vital signs are stable. Treatment for her should include: • A:treating her for shock and providing rapid transport. • B:carefully packing the vagina with sterile dressings. • C:administering oxygen and massaging the uterus. • D:massaging the uterus if signs of shock develop.

You selected C; This is correct! Reason: Postpartum bleeding is most effectively controlled by massaging the fundus (top) of the uterus. Uterine massage stimulates the pituitary gland to secrete a hormone called oxytocin, which constricts the blood vessels in the uterus and helps stops the bleeding. Do not wait for signs of shock to develop before performing uterine massage. The goal is to control the postpartum bleeding in order to prevent shock. Administer supplemental oxygen as needed, begin transport, and monitor her for signs of shock (ie, tachycardia, pallor, diaphoresis, tachypnea) en route. Vaginal bleeding is never treated by placing anything inside the vagina; this action increases the risk of maternal infection.

The ultimate goal of any EMS quality improvement program is to: • A:recognize all EMTs who demonstrate consistency in providing competent patient care. • B:provide protocols to all EMTs and hold them accountable if protocols are not followed. • C:deliver a consistently high standard of care to all patients who are encountered. • D:ensure that all personnel receive an adequate number of continuing education hours.

You selected C; This is correct! Reason: Providing continuing education to all personnel, recognizing those who consistently provide competent patient care, and holding all personnel accountable for adhering to the EMS protocols are all components of any EMS quality improvement program. The ultimate goal, however, is to provide, as a system, a consistently high standard of care to all patients who are encountered.

Unresponsiveness, shallow breathing, and constricted pupils are indicative of what type of drug overdose? • A:Marijuana • B:Amphetamine • C:Narcotic • D:Barbiturate

You selected C; This is correct! Reason:Signs of a narcotic (opiate) overdose from drugs such as heroin, morphine (Astromorph, Duramorph), meperidine (Demerol), or codeine include altered mental status; slow, shallow breathing; pupillary constriction (miosis), hypotension; and bradycardia. Narcotics are central nervous system depressants that, when taken in excess, suppress the vital functions necessary for life, such as breathing, heart rate, and blood pressure. Barbiturates produce the same effects; however, the pupils are typically dilated (mydriasis), not constricted. Marijuana and amphetamine drugs are central nervous system stimulants and would thus cause the patient to become restless or even combative.

Approximately 5 minutes after being stung by a bee, a 21-year-old male develops hives and begins experiencing difficulty breathing. When you arrive at the scene, you note that his level of consciousness is decreased, his breathing is labored, and wheezing can be heard without a stethoscope. The patient has a bee sting kit, but has not used it. You should: • A:give him high-flow oxygen via a nonrebreathing mask, cover him with a blanket and elevate his legs, and assist him in swallowing the antihistamine tablets that are in his bee sting kit. • B:provide positive-pressure ventilations, initiate rapid transport, and coordinate a rendezvous with a paramedic unit so they can administer the epinephrine from his bee sting kit. • C:assist his ventilations with a bag-mask device, administer epinephrine from his bee sting kit after receiving approval from medical control, and prepare for immediate transport. • D:administer high-flow oxygen via a nonrebreathing mask, apply a chemical cold pack to the sting, transport at once, and be prepared to administer the epinephrine from his bee sting kit.

You selected C; This is correct! Reason: The patient is in anaphylactic shock and needs epinephrine immediately. Epinephrine is given to patients with anaphylactic shock because it constricts the blood vessels and dilates the bronchioles, thus improving perfusion and breathing. Begin assisting his ventilations with a bag-mask device; his breathing is inadequate (eg, decreased LOC, wheezing, labored breathing). After receiving permission from medical control, give the epinephrine from his bee sting kit (AnaKit) via intramuscular injection and transport at once. Some EMS protocols may not require you to contact medical control before giving epinephrine to a patient whose condition is critical. Ordinarily, you would instruct the patient to chew and swallow the antihistamine tablets after giving epinephrine. However, the patient's level of consciousness contraindicates placing anything in his mouth.

A 72-year-old woman is found unresponsive in her poorly ventilated home. Her skin is flushed, hot, and dry, and her respirations are rapid and shallow. She is wearing a medical alert bracelet that states she is a diabetic and is allergic to sulfa drugs. You should be MOST suspicious for: • A:acute ischemic stroke. • B:a diabetic complication. • C:heat stroke. • D:anaphylactic shock.

You selected C; This is correct! Reason: The patient's signs and symptoms are consistent with classic heat stroke. Unlike exertional heat stroke, which usually affects otherwise healthy people who exert themselves in the heat for long periods of time, classic heat stroke commonly affects children and older adults, and typically occurs when the patient is in a hot, poorly ventilated space for a prolonged period of time. Significant underlying medical problems (eg, hypertension, diabetes, heart disease) increase the patient's risk for classic heat stroke. Both exertional and classic heat stroke present with hot, flushed skin; however, patients with exertional heat stroke may have moist skin, usually from residual perspiration, whereas patients with classic heat stroke typically have dry skin.

Appropriate treatment for an 18-year-old woman with severe vaginal bleeding may include all of the following, EXCEPT: • A:covering the vagina with a trauma dressing. • B:high concentrations of oxygen. • C:placing sterile dressings into the vagina. • D:keeping her warm with blankets.

You selected C; This is correct! Reason: The source of bleeding from the vagina cannot be directly controlled in the field. You should never pack or place any dressings directly into the vagina as this increases the risk of infection; furthermore, these dressings will only have to be removed at the hospital. Instead, you should place a trauma dressing or similar material over the vagina. If the patient has signs of shock, administer high-flow oxygen, keep her warm, and transport without delay.

Which of the following is the MOST significant finding in a patient with a severe headache? • A:Pain in both legs • B:Chest discomfort • C:Unilateral weakness • D:Abdominal tenderness

You selected C; This is correct! Reason: Unilateral weakness (weakness on one side of the body) is a clinically significant finding in a patient with a headache because it could indicate a stroke (ischemic or hemorrhagic). Abdominal, chest, and leg pain are not common complaints associated with a headache, although they should be noted and investigated if they are present.

You are called to a local park for a 7-year-old boy with respiratory distress. During your assessment, you find that the patient is wheezing and has widespread hives and facial edema. What should you suspect has occurred? • A:Poison oak exposure • B:Acute asthma attack • C:Allergic reaction • D:Heat illness

You selected C; This is correct! Reason: Wheezing, hives, and edema are hallmark findings of an allergic reaction. In this case, the patient is having a severe reaction. Although wheezing occurs in patients with asthma, hives and facial edema do not. Wheezing is not associated with head-related illnesses. Exposure to poison oak or poison ivy causes a local reaction, such as redness and itching or burning; it is not commonly associated with systemic symptoms.

Hypoxia-induced unresponsiveness during a submersion injury is usually the result of: • A:a cardiac dysrhythmia. • B:associated hypothermia. • C:laryngospasm. • D:water in the lungs.

You selected C; This is correct! Reason: When a patient falls into the water or becomes panicked when in the water, he or she begins to swallow large amounts of water. Even a small amount of water near the larynx can cause a spasm, which closes off the airway. This results in hypoxia, loss of consciousness, and cardiac dysrhythmias. If the patient is not removed from the water at once and treated aggressively, hypoxia and acidosis will eventually result in cardiac arrest.

When assessing a conscious patient who overdosed on a drug, you should FIRST determine: • A:if there is a history of prior overdose. • B:the patient's weight in kilograms. • C:the type of medication ingested. • D:when the medication was ingested.

You selected C; This is correct! Reason: When assessing a patient who has overdosed on a drug, you should first determine what was ingested, which will provide you with immediate information about whether or not the substance is toxic. You should then find out when the medication was ingested. This information will provide medical control (or poison control) with the information needed to direct the most appropriate treatment. The patient's weight also should be estimated in kilograms in the event that an antidote is required. Information regarding prior drug overdoses would normally be obtained during the SAMPLE history.

When dealing with an emotionally disturbed patient, you should be MOST concerned with: • A:gathering all of the patient's medications. • B:safely transporting to the hospital. • C:whether the patient could harm you. • D:obtaining a complete medical history.

You selected C; This is correct! Reason: When managing any patient with an emotional or psychiatric crisis, your primary concern is your own safety. Safely transporting the patient to the hospital is your ultimate goal. If possible, you should attempt to obtain a medical history and should take any of the patient's prescribed medications to the hospital. However, this should not supercede your own safety or interfere with safely transporting the patient.

Rapid transport of a patient who ingested a large dose of Tylenol is important because: • A:it only takes a small dose of Tylenol to cause cardiopulmonary arrest. • B:liver failure usually occurs within 6 hours following a Tylenol overdose. • C:an antidote may prevent liver failure if administered early enough. • D:activated charcoal cannot be given to patients who ingested Tylenol.

You selected C; This is correct! Reason:Acetaminophen (APAP), the active ingredient in Tylenol, is a safe drug if taken as directed. However, ingestion of more than 140 mg/kg in an adult can cause liver failure and death. Symptoms of APAP overdose do not present acutely; it can take up to a week before signs of liver failure are apparent. The antidote for acetaminophen poisoning is acetylcysteine (Acetadote, Mucomyst), which is only given at the hospital. However, it must be given promptly if liver failure is to be avoided. Activated charcoal can be given to patients with APAP overdose; however, it is only effective if the drug is still in the stomach. After 1 to 2 hours following ingestion, activated charcoal would likely be ineffective.

After clearing the airway of a newborn who is not in distress, it is MOST important for you to: • A:obtain an APGAR score. • B:apply blow-by oxygen. • C:keep the newborn warm. • D:clamp and cut the cord.

You selected C; This is correct! Reason:After ensuring a patent airway (ie, suctioning and positioning), it is extremely important to keep the newborn warm. Newborns cannot maintain body temperature very well and hypothermia can develop very quickly. Blow-by oxygen should be given if the newborn is breathing adequately, but has cyanosis to the face, neck, or trunk (central cyanosis). The umbilical cord should not be clamped and cut until the cord has stopped pulsating and the newborn is breathing adequately. The Apgar score, which is performed at 1 and 5 minutes after birth (and every 5 minutes thereafter), is not used to determine the need for or extent of resuscitation; respiratory effort, heart rate, skin color, and oxygen saturation (SpO2) are used to determine this.

Which of the following actions demonstrates an EMT's knowledge of crime scene preservation? • A:Carefully cutting through the hole in a patient's clothing that was made by a large caliber firearm • B:Requesting approval from law enforcement before controlling severe bleeding from a patient's arm • C:Advising a law enforcement officer after moving a coffee table to access a critically injured patient • D:Placing a knife in a plastic zip-lock bag and giving it to a law enforcement officer for safe-keeping

You selected C; This is correct! Reason:After ensuring your own safety, your priority when caring for a patient at a crime scene is to do just that, care for the patient. If you need to move a piece of furniture to gain access to a critically injured patient, move the furniture, treat the patient, and then advise a law enforcement officer of what you moved and where you moved it to. In this way, you are providing immediate care to the patient, but are remaining aware that the location of any obstacles between you and the patient may serve as evidence. Clearly, you are not going to request approval from a law enforcement officer before treating a critically injured patient. Items that may have fingerprints on them, such as knives or guns, should be placed in a paper bag; condensation can accumulate in plastic bags, potentially destroying any evidence. When removing clothing from a gunshot victim, you should make an effort to cut around (not through) the hole in the clothing that was made by the bullet. The hole in a patient's clothing may contain gunshot residue and can provide valuable information regarding the type of weapon used and the distance between the assailant and the victim.

Upon arriving at a scene in which a tanker truck overturned and is spilling an unknown liquid on the ground, you should: • A:quickly identify the material. • B:turn off your warning lights. • C:park upwind from the scene. • D:stay downhill from the scene.

You selected C; This is correct! Reason:At the scene of a potential or actual hazardous materials incident, you should park the ambulance in an area that is both upwind and uphill from the incident. However, you must be prepared to quickly relocate if the wind direction changes. Staying uphill is important because many hazardous materials collect in low-lying areas, such as valleys. After ensuring that you are in a safe place, attempt to identify the chemical involved by reading the placard on the tanker (with binoculars) and referencing the placard number in the emergency response guidebook (ERG).

Following delivery of a newborn, the 21-year-old mother is experiencing mild vaginal bleeding. You note that her heart rate has increased from 90 to 120 beats/min and she is diaphoretic. In addition to administering high-flow oxygen, treatment should include: • A:uterine massage and transport. • B:internal vaginal pads and treating for shock during transport. • C:treating for shock and uterine massage during transport. • D:placing her on her left side and transport.

You selected C; This is correct! Reason:Blood loss of up to 500 mL within the first 24 hours after delivery is considered normal and usually is well tolerated by the mother. However, any bleeding, regardless of the severity, with accompanying signs of shock, must be treated accordingly. In this case, you should apply high-flow oxygen, treat the patient for shock by elevating her legs (if allowed by local protocol) and providing warmth, and provide rapid transport to the hospital while massaging the uterine fundus en route. Placing the mother on her left side is appropriate before she delivers and prevents supine hypotensive syndrome. Dressings should never be packed into the vagina; placing pads into the vagina increases the risk for maternal infection.

A near-drowning is MOST accurately defined as: • A:complications within 24 hours following submersion in water. • B:immediate death due to prolonged submersion in water. • C:survival for at least 24 hours following submersion in water. • D:death greater than 24 hours following submersion in water.

You selected C; This is correct! Reason:Collectively, drowning and near-drowning are referred to as submersion injuries. Drowning is defined as death after submersion in a liquid medium, usually water. In a drowning, death is either immediate or occurs within 24 hours following submersion. Near-drowning is defined as survival, at least temporarily (24 hours), after submersion. It should be noted, however, that complications such as pneumonia and pulmonary edema can cause death greater than 24 hours following submersion. For this reason, all patients with a submersion injury should be transported to the hospital, even if they appear fine at the scene.

Pulmonary embolism

blockage of the pulmonary artery by foreign matter or by a blood clot

Which of the following would MOST likely cause a rapid drop in a patient's blood glucose level? • A:Mild exertion after eating a meal • B:Forgetting to take prescribed insulin • C:Taking too much prescribed insulin • D:Eating a meal after taking insulin

You selected C; This is correct! Reason:Compared to diabetic coma (hyperglycemic ketoacidosis, hyperglycemic crisis), insulin shock (hypoglycemia, hypoglycemic crisis) has a rapid onset. It is commonly caused when a patient accidentally takes too much prescribed insulin. Insulin is a fast-acting drug that rapidly causes glucose to exit the bloodstream and enter the cell. Other common causes of hypoglycemia include taking a regular dose of insulin but not eating or taking insulin and exercising heavily. Eating a meal after taking insulin typically does not cause a significant change in the patient's blood glucose level. If a person fails to take his or her insulin, glucose will not be able to enter the cell and will accumulate in the bloodstream (hyperglycemia).

Which of the following practices will provide you with the highest degree of safety when responding to an emergency call? • A:Always requesting fire department assistance • B:Routinely using the lights and siren • C:Wearing your seatbelt and shoulder harness • D:Asking the police to escort you to the scene

You selected C; This is correct! Reason:Consistent safety practices, such as wearing your seatbelt and shoulder harness, driving with due regard for others, and avoiding excessive speed, will afford you the highest degree of safety when responding to an emergency call. Unless used to guide you to an area you are unfamiliar with, escort vehicles should be avoided. Escort vehicles are especially dangerous at intersections; as the escort vehicle proceeds through the intersection, other motorists often do not expect a second emergency vehicle to follow. As a result, they may pull out into the intersection, thinking it is clear, and collide with your ambulance (wake effect collision). The use of lights and siren increases the danger factor, especially if used in conjunction with excessive speed. Lights and siren ask for, not demand, the courtesy of the right of way.

A 3-year-old female presents with respiratory distress. She is conscious, crying, and clinging to her mother. She has mild intercostal retractions and an oxygen saturation of 93%. The MOST effective way of delivering oxygen to her involves: • A:ventilations with a flow-restricted, oxygen-powered device. • B:gently restraining her and assisting her ventilations. • C:asking the mom to hold an oxygen mask near her face. • D:a nonrebreathing mask with the flow rate at 6 to 8 L/min.

You selected C; This is correct! Reason:Do not assume that a child will simply allow you to administer oxygen to him or her as you would to an adult. The child in this scenario, who is in respiratory distress and is mildly hypoxemic (SpO2 of 93%), should receive supplemental oxygen; however, it should be given in a nonthreatening manner. Agitating a sick or injured child causes an increase in oxygen consumption and demand, which may cause the child's condition to deteriorate. In this scenario, ask the child's mother to hold an oxygen mask near the child's face (blow-by oxygen). Closely monitor her condition and be prepared to assist her ventilations with a bag-mask device if she deteriorates. An oxygen flow rate of 6 to 8 L/min is too low for a nonrebreathing mask; a flow-rate of 12 to 15 L/min should be used. Do NOT use a flow-restricted, oxygen-powered ventilation device (FROPVD) on any child; doing so may cause severe gastric distention and lung injury. Allow the child to assume a position of comfort and transport.

Which of the following mechanisms cause respiratory and circulatory collapse during anaphylactic shock? • A:Bronchodilation and vasodilation • B:Bronchoconstriction and vasoconstriction • C:Bronchoconstriction and vasodilation • D:Bronchodilation and vasoconstriction

You selected C; This is correct! Reason:During anaphylaxis, histamines released from the immune system cause two negative effects that result in shock (hypoperfusion): vasodilation, which causes the blood pressure to fall and bronchoconstriction, which impairs breathing.

In contrast to the contractions associated with true labor, Braxton-Hicks contractions: • A:consistently become stronger and are not alleviated by changing position. • B:may be intensified by activity and are accompanied by a pink discharge. • C:do not increase in intensity and are alleviated by a change in position. • D:generally follow rupture of the amniotic sac and occur with regularity.

You selected C; This is correct! Reason:During pregnancy, the mother may experience false labor, or Braxton-Hicks contractions, in which there are contractions but they do not represent true labor. Unlike true labor contractions, Braxton-Hicks contractions do not increase in intensity, are not regular, and are typically alleviated by activity or a change in position. The contractions associated with true labor, once they begin, consistently get stronger and closer together and are regular; a change in position does not relieve the contractions. True labor is also commonly followed by, or in some cases preceded by, a rupture of the amniotic sac (bag of waters) and a pink or red vaginal discharge that is generally accompanied by mucus (bloody show).

Which of the following signs is MOST indicative of inadequate breathing in an infant? • A:Heart rate of 130 beats/min • B:Sunken fontanelles • C:Expiratory grunting • D:Abdominal breathing

You selected C; This is correct! Reason:Expiratory grunting in an infant or a child with a respiratory problem is an ominous sign; it indicates impending respiratory arrest. Grunting represents the child's attempt to maintain oxygen reserve in the lungs. Sunken fontanelles, the soft spots on the infant's skull, indicate dehydration. Because infants have a protuberant abdomen and rely heavily on their diaphragm to breathe, their abdomen appears to move more than their chest during breathing; this is a normal finding and is why infants are often referred to as "belly breathers." An infant or a child with inadequate breathing may be tachycardic at first; however, as hypoxia becomes more severe, bradycardia often occurs. Bradycardia in an infant or a child with a respiratory problem indicates impending cardiopulmonary arrest.

Switching compressors during two-rescuer CPR: • A:should take no more than 15 seconds to accomplish. • B:is performed after every 10 to 20 cycles of adult CPR. • C:is only necessary if the compressor becomes fatigued. • D:should occur every 2 minutes throughout the arrest.

You selected D; This is correct! Reason: Rescuer fatigue may lead to inadequate chest compression rate and/or depth. Fatigue is common after 1 minute of CPR, although the rescuer may not recognize it for 5 minutes or longer. Therefore, compressors should be changed every 2 minutes (after 5 cycles of CPR at a 30:2 ratio) throughout the resuscitation attempt. If the compressor is not switched until he or she recognizes the fatigue, the patient has likely been without effective chest compressions for at least 4 or 5 minutes. In general, interruptions in CPR should be infrequent and should not exceed 10 seconds. However, every effort should be made to switch compressors in less than 5 seconds.

Upon delivery of a baby's head, you see that the umbilical cord is wrapped around its neck. Initial treatment for this condition should include: • A:clamping and cutting the umbilical cord. • B:keeping the cord moist and providing rapid transport. • C:trying to remove the cord from around the neck. • D:gently pulling on the cord to facilitate removal.

You selected C; This is correct! Reason:If you can see the umbilical cord wrapped around the newborn's neck (nuchal cord) when the head delivers, you should gently attempt to slide the cord from around the neck. If this is unsuccessful, you should clamp and cut the cord and continue the delivery. You must never pull on the umbilical cord. In cases where the umbilical cord is prolapsed (the cord presents before the baby), you should make an attempt to move the fetus off of the umbilical cord; this often involves inserting your gloved fingers into the vagina and gently lifting the baby's head off of the cord. Cover the exposed cord with sterile moist dressings, administer oxygen to the mother, and transport without delay.

During transport of a woman in labor, the patient tells you that she feels the urge to push. You assess her and see the top of the baby's head bulging from the vagina. You should: • A:ask the mother to take short, quick breaths until you arrive at the hospital. • B:apply gentle pressure to the baby's head and notify the hospital immediately. • C:advise your partner to stop the ambulance and assist with the delivery. • D:allow the head to deliver and check for the location of the cord.

You selected C; This is correct! Reason:If, during transport, the mother begins to deliver the infant, your first action should be to advise your partner to stop the ambulance and assist you with the delivery. Delivery of a baby should never be attempted in the back of a moving ambulance. During delivery, you should apply gentle pressure to the top of the baby's head (be careful of the fontanelles) in order to prevent an explosive delivery. After the head is delivered, you should quickly run your fingers around its neck to determine if the cord is wrapped around its neck (nuchal cord). If a nuchal cord is not present, suction the baby's mouth and nose and continue with the delivery.

After removing a patient from the water, your assessment reveals that he is not breathing and is continuously regurgitating large amounts of water. You should: • A:perform abdominal thrusts to remove the water. • B:begin rescue breathing after he stops regurgitating. • C:alternate suctioning with artificial ventilations. • D:place him on his side and press on his abdomen.

You selected C; This is correct! Reason:In cases where a patient is not breathing and is regurgitating (passively vomiting) secretions at the same time, you must address both issues. This is accomplished most effectively by suctioning for 15 seconds and then ventilating for 2 minutes. This alternating sequence should be repeated until all secretions are cleared from the airway. You should turn the patient onto his side to facilitate drainage of liquid, but do not apply pressure to his abdomen. Manual gastric decompression, which involves applying pressure to the patient's abdomen, is a dangerous maneuver because it will force more water from the stomach, which the patient could potentially aspirate. Manual gastric decompression should ONLY be performed if gastric distention is so severe that it is impossible to ventilate a patient AND a paramedic is not present to insert a gastric tube into the stomach. Abdominal thrusts are used to remove a solid foreign body from the airway, not liquid.

The MOST important initial steps of assessing and managing a newborn include: • A:drying and warming the infant and obtaining an APGAR score. • B:suctioning the airway and obtaining a heart rate. • C:clearing the airway and keeping the infant warm. • D:keeping the infant warm and counting respirations.

You selected C; This is correct! Reason:In the initial steps of assessing and managing the newborn, the most important aspects include clearing the airway of amniotic fluid and making sure that the baby stays warm. The APGAR score should not be relied on as the initial indicator for resuscitation because it is not performed until the child is 1 minute old. Clearly, this is too long to wait before assessment and intervention. After the airway has been cleared and the newborn has been warmed, the respirations, heart rate, color, and oxygen saturation (SpO2) should be assessed and managed accordingly.

A 34-year-old woman, who is 36 weeks pregnant, is having a seizure. After you protect her airway and ensure adequate ventilation, you should transport her: • A:in a semisitting position. • B:in the prone position • C:on her left side. • D:in the supine position.

You selected C; This is correct! Reason:Initial care for any patient who is seizing—pregnant or otherwise—involves ensuring a patent airway, adequate ventilation, and administering high-flow oxygen. If the patient is breathing inadequately, ventilation assistance is indicated. Suction any secretions from the patient's mouth. The pregnant patient should be placed on her left side (lateral recumbent position); this will prevent supine hypotensive syndrome—a condition in which the pregnant uterus compresses the inferior vena cava and reduces cardiac output. A lateral recumbent position will also facilitate the draininge of oral secretions, thus minimizing the risk of aspiration.

The preferred method for inserting an oropharyngeal airway in a small child is to: • A:insert the airway as you would in an adult, but use an airway that is one size smaller than you would normally use. • B:open the airway with the tongue-jaw lift maneuver and insert the airway until you meet slight resistance. • C:depress the tongue with a tongue blade and insert the airway with the downward curve facing the tongue. • D:insert the airway with the curvature towards the roof of the mouth and then rotate it 180 degrees.

You selected C; This is correct! Reason:Keeping in mind that a child's tongue is proportionately large, the preferred method for inserting an oropharyngeal (oral) airway is to use a tongue blade to depress the tongue and slide the airway straight in, with the downward curve of the airway facing the tongue, until it rests just beyond the curvature of the tongue. If you use an oral airway that is too small, it will not reach the curvature of the tongue and propel it forward. If you use an oral airway that is too large, it may obstruct the airway. If you meet resistance when inserting an oral airway, you are likely using an airway that is too large.

Which of the following is a physiologic effect of epinephrine when used to treat anaphylactic shock? • A:As a vasodilator, it increases the blood pressure. • B:As an antihistamine, it blocks chemicals that cause the reaction. • C:As a vasoconstrictor, it lowers the blood pressure. • D:As a bronchodilator, it improves the patient's breathing.

You selected D; This is correct! Reason:Epinephrine possesses dual effects. As a bronchodilator, it relaxes the smooth muscle of the bronchioles and improves the patient's breathing. As a vasoconstrictor, it constricts the blood vessels and increases the patient's blood pressure. Diphenhydramine (Benadryl) is an antihistamine; it blocks H1 histamine receptor sites, which blocks the release of the chemicals (histamines) that are causing the allergic reaction.

A 42-year-old male presents with fever, a severe headache, and a stiff neck. He is conscious, but confused. His wife tells you that he does not have any medical problems and does not take any medications. You should be MOST suspicious for: • A:acute stroke. • B:influenza. • C:meningitis. • D:tuberculosis. (

You selected C; This is correct! Reason:Meningitis is an inflammation of the protective coverings of the brain and spinal cord (meninges). Common signs and symptoms of meningitis include fever, headache, neck stiffness (nuchal rigidity), and vomiting. An altered mental status is common in severe cases. Meningococcal meningitis, caused by a bacterium, is the most contagious and potentially fatal type of meningitis. The patient's signs and symptoms are not consistent with acute stroke, tuberculosis (TB), or influenza (the flu). Although fever is common with both TB and the flu, neither causes neck stiffness. Acute stroke may be associated with a headache, especially a hemorrhagic stroke; however, stroke patients typically do not have a fever.

All of the following are considered key components at the scene of a mass-casualty incident, EXCEPT: • A:an adequately staffed treatment area. • B:a supply area near the treatment area. • C:an on-scene emergency physician. • D:an on-site communication system.

You selected C; This is correct! Reason:On-site communications, an adequately staffed treatment area, and a supply area near the treatment area are some of the key components at a mass-casualty incident. Although the physician medical director is ultimately responsible for all patient care activities at the scene, he or she typically communicates with the incident commander via mobile phone or two-way radio from the base hospital. Physical presence of the physician at the scene, although optimal, is often not possible.

Which of the following signs or symptoms is MOST suggestive of a systemic reaction following ingestion of a poison? • A:Painful or difficult swallowing • B:Burns around the mouth • C:Tachycardia and hypotension • D:Nausea and vomiting

You selected C; This is correct! Reason:Once an ingested poison gets into the system, it can affect multiple organ systems. Signs that this is occurring include tachycardia or bradycardia, hypotension or hypertension, weakness, restlessness, and an altered level of consciousness, among others. Local effects of an ingested poison include nausea and vomiting as the poison irritates the gastric lining, burns in and around the mouth, and painful or difficult swallowing (dysphagia).

A 4-year-old boy ingested an unknown quantity of drain cleaner. He is alert, has a patent airway, and has adequate breathing. You should: • A:administer 1 g/kg of activated charcoal. • B:give 15 mL of ipecac and contact medical control. • C:contact poison control and give him oxygen. • D:give oxygen and perform a head-to-toe exam.

You selected C; This is correct! Reason:Once you determine that a poisoning has occurred, and have identified the poison, you should contact the poison control center at once: (800) 222-1222. Give the patient high-flow oxygen or assist his or her ventilations if necessary. Induction of vomiting with syrup of ipecac is no longer recommended because of the risk of aspiration. Activated charcoal is contraindicated in patients who have ingested a corrosive substance (eg, drain cleaner) or a petroleum product (eg, gasoline, motor oil). A head-to-toe exam is not practical in this situation, at least initially. Follow the directions given to you by the poison control center, transport the child without delay, and monitor his condition en route.

While en route back to your station from the emergency department, you discover that you forgot to include vital patient information on the front of your patient care report (PCR). Having left a copy of your PCR with the emergency department staff, you should: • A:notify the emergency department staff and advise them of your error. • B:document the information on the front of your original PCR. • C:include the information on an addendum and furnish it to the hospital. • D:do nothing, since you already left a copy of the PCR at the hospital.

You selected C; This is correct! Reason:Once you leave a copy of your patient care report (PCR) with the hospital staff, you should not add information to the front of the report. If you discover that you forgot to include vital patient information, you should document the information on a separate addendum and furnish it to the emergency department, which can be sent by fax. The copy of your PCR you leave at the hospital and the original PCR should contain the same information; adding information to the front of the PCR after leaving a copy at the hospital makes the copy and original two separate documents.

You are the first ambulance to arrive at the scene of a motor-vehicle crash. As you approach the scene, you see three patients, two who have been ejected from their vehicles and the other who is still in his vehicle. You should: • A:notify the local trauma center so they can prepare for the patients. • B:call medical control and apprise him or her of the situation. • C:request additional ambulances to respond to the scene. • D:begin triaging and treating the most critically injured.

You selected C; This is correct! Reason:One of the most important aspects of the scene size-up is to determine the need for additional resources. A single EMS unit and two EMTs cannot effectively manage three patients, especially if they are critically injured. Therefore, you must first request additional ambulances at the scene. After requesting additional resources, begin the processes of triage and treatment. Notify the local trauma center early, if possible, so they can prepare to take care of the patients; however, your first priority is to maximize patient care at the scene by requesting additional ambulances.

Which of the following techniques represents the MOST appropriate method of opening the airway of an infant with no suspected neck injury? • A:Perform the technique as you would for an older child or adult. • B:Lift up the chin and hyperextend the neck. • C:Tilt the head back without hyperextending the neck. • D:Gently lift the chin while maintaining slight flexion of the neck.

You selected C; This is correct! Reason:Opening the airway in infants and small children involves keeping the head in a neutral or slightly extended position. Because the occipital region (back of the head) of the skull is proportionately larger in infants and small children when compared to an adult, hyperextension of the neck can result in a reverse flexion of the neck and subsequent airway blockage.

Proper guidelines for safe reaching include all of the following, EXCEPT: • A:keeping your back in a locked-in position. • B:avoiding twisting of your back. • C:reaching no more than 30" in front of your body. • D:avoiding hyperextension of your back.

You selected C; This is correct! Reason:Safe reaching practices are critical to the prevention of a back injury. When reaching, you should keep your back in a locked-in position. You should avoid twisting or hyperextending your back, and should reach no more than 15 to 20 inches in front of your body.

You are dispatched to a residence for a 4-year-old female who is sick. Your assessment reveals that she has increased work of breathing and is making a high-pitched sound during inhalation. Her mother tells you that she has been running a high fever for the past 24 hours. Your MOST immediate concern should be: • A:preparing to treat her for a febrile seizure. • B:determining if the child has a history of croup. • C:assessing the need for ventilation assistance. • D:taking her temperature to see how high it is

You selected C; This is correct! Reason:The child is clearly experiencing respiratory distress and probably has croup (laryngotracheobronchitis), a viral upper airway infection. The presence of stridor (high-pitched sound heard during inhalation) indicates swelling of the upper airway. Your most immediate concern should be assessing the adequacy of her breathing and determining if ventilation assistance is necessary. If signs of respiratory failure are present (eg, signs of physical exhaustion, bradycardia, bradypnea [slow respirations]), you must begin assisting her ventilations with a bag-mask device; otherwise, she will likely deteriorate and develop cardiac arrest. She may experience a febrile seizure if her fever acutely spikes; although this is a concern, it is not the most immediate concern in a child with an airway or breathing problem.

A 30-year-old woman crashed her car into a tree at a high rate of speed. She is conscious and alert and has stable vital signs. She has some small lacerations and abrasions to her arms and face, but no obviously life-threatening injuries. As you are loading her into the ambulance, she tells you that she does not want to go to the hospital. You should: • A:advise her that she is probably too emotionally upset to be able to refuse EMS treatment and transport. • B:obtain a signed refusal from the patient and ask a law enforcement officer to transport her to the hospital. • C:advise the patient that she should be transported to the hospital because of the seriousness of the crash. • D:ask a law enforcement officer to administer a breathalyzer test to determine if she has been drinking alcohol.

You selected C; This is correct! Reason:The consequences of refusal should be explained to any patient who refuses EMS treatment and/or transport. After establishing that the patient can legally refuse treatment and transport (eg, he or she is of legal age AND has decision-making capacity), you must advise her that because of the significant mechanism of injury, the potential for critical injury or death exists, even though she may feel fine now. Once this is explained, and the patient understands and is willing to accept the possible consequences, obtain a signed refusal and ask an impartial person (eg, police officer) to witness the signed refusal.

Which of the following patients has the highest priority at the scene of a mass-casualty incident? • A:44-year-old unresponsive man with an open head injury and agonal gasps • B:29-year-old man who is pulseless and apneic with an abdominal evisceration • C:35-year-old unresponsive woman with snoring respirations and severe burns • D:38-year-old woman who remains apneic after you manually open her airway

You selected C; This is correct! Reason:The goal of triage is to provide the greatest good for the greatest number of people. Relative to the other patients, who are either in respiratory or cardiac arrest or have injuries incompatible with life, the woman with snoring respirations and severe burns has the best chance for survival. Therefore, she has the highest treatment priority.

Who has ultimate authority for all issues regarding patient care at the scene of a mass-casualty incident? • A:Incident commander • B:Most experienced EMT • C:EMS medical director • D:Treatment officer

You selected C; This is correct! Reason:The incident commander (IC) is responsible for all logistical and operational aspects of a mass-casualty incident (MCI), such as designating section officers and working in conjunction with other agencies (eg, police, fire, EMS). For all issues regarding patient care, the EMS medical director has ultimate authority. Although the treatment officer is responsible for overseeing all emergency care provided at the scene, and EMTs working in the treatment area are providing direct patient care, these personnel are still functioning under the physician's license. During an MCI, the IC (or his or her designee) is typically in contact with the medical director, who is located at the base station hospital, via mobile phone or two-way radio. In some cases, the medical director may be physically present at the incident.

Prevention of cardiac arrest in infants and small children should focus primarily on: • A:providing immediate transport. • B:avoiding upsetting the child. • C:ensuring adequate ventilation. • D:keeping the child warm.

You selected C; This is correct! Reason:The most common cause of cardiac arrest in infants and children is failure of the respiratory system. Their hearts generally are healthy, and they rarely go into ventricular fibrillation (V-Fib). The key to preventing cardiac arrest in the majority of infants and children is to ensure adequate ventilation and oxygenation.

A 40-year-old woman presents with widespread hives that she noticed about 45 minutes after taking penicillin. She is conscious and alert and denies difficulty breathing. Her breath sounds are clear to auscultation bilaterally, her vital signs are stable, and her oxygen saturation is 94%. She tells you she is allergic to wasps and has an epinephrine auto-injector. You should: • A:assist her in administering epinephrine via her auto-injector. • B:give high-flow oxygen and administer 100 mg of Benadryl. • C:give supplemental oxygen and transport her to the hospital. • D:advise her that she can probably drive herself to the hospital.

You selected C; This is correct! Reason:The patient is experiencing an allergic reaction, but she is not in anaphylactic shock. Urticaria (hives) is common to all allergic reactions, regardless of severity; however, wheezing and hypotension, which she does not have, are specific to anaphylaxis. Although she carries an epinephrine auto-injector for her allergy to wasps, she was not stung by a wasp. Furthermore, the absence of wheezing and hypotension negates epinephrine administration. Give supplemental oxygen (in a concentration sufficient to maintain an SpO2 of greater than 94%) and transport her to the hospital. Because it can take up to an hour for signs of a severe allergic reaction to manifest, the patient should not drive herself. Diphenhydramine (Benadryl), an antihistamine, is an appropriate drug based on her presentation; however, it is not typically carried on a BLS unit. Furthermore, the correct dose is 25 to 50 mg.

All of the following are signs of gastrointestinal bleeding, EXCEPT: • A:melena. • B:hematemesis. • C:tachycardia. • D:hemoptysis.

You selected D; This is correct! Reason:Signs and symptoms of gastrointestinal (GI) bleeding include abdominal pain; vomiting blood (hematemesis); the passage of dark, tarry stools (melena); and bright red rectal bleeding (hematochezia). If blood loss is significant, the patient may have signs of shock (eg, tachycardia, diaphoresis, tachypnea, hypotension). Hemoptysis (coughing up blood) is a sign of a pulmonary injury, not GI bleeding.

You are caring for a 6-year-old child with a possible fractured arm and have reason to believe that the child was abused. How should you manage this situation? • A:Call the police and have the parents arrested. • B:Transport the child to the hospital regardless of the parents' wishes. • C:Advise the parents that the child needs to be transported. • D:Inform the parents of your suspicions.

You selected C; This is correct! Reason:The responding EMT must handle cases of suspected child abuse with great care. You must never accuse the parents or caregiver of abuse. If you are wrong, you could be held liable for slander. Actions that would suggest such accusation includes summoning the police to have the parents arrested. Instead, you should advise the parents or caregiver that the child needs to be transported by ambulance, even if the injury is not life-threatening. The goal is to get the child to safety; however, this must be done legally (with parental consent). In most cases, you only need the consent of one parent to transport the child. Once at the hospital, you must apprise the physician of your suspicions.

You are called to an assisted living center where an attendant found a 72-year-old man unresponsive. The patient had recent hip surgery and has been taking Vicodin for pain. His respirations are slow and shallow and his pulse is slow and weak. You should: • A:apply the AED in case he develops cardiac arrest. • B:request an ALS ambulance to respond to the scene. • C:begin ventilation assistance with a bag-mask device. • D:apply high-flow oxygen via a nonrebreathing mask.

You selected C; This is correct! Reason:Vicodin is a combination of hydrocodone and acetaminophen (APAP), the active ingredient in Tylenol. Hydrocodone is a potent narcotic analgesic. When taken in excess, it can suppress the central nervous system and cause respiratory depression, bradycardia, and hypotension. Initial management of any patient who has overdosed on a medication of this type is to ensure a patent airway and support breathing. Because the patient is breathing inadequately (slow and shallow), immediate ventilation assistance is needed. Consider requesting an ALS ambulance if transport time to the nearest hospital will be lengthy. Paramedics can administer a drug called naloxone (Narcan) to reverse the effects of narcotic drugs. The AED is not indicated for this patient; it is only applied to patients in cardiac arrest.

When caring for a patient with an acute behavioral crisis, your primary concern should be: • A:gathering all of the patient's medications. • B:obtaining a complete past medical history. • C:ensuring you and your partner's safety. • D:providing safe transport to the hospital.

You selected C; This is correct! Reason:When caring for a patient with a behavioral or emotional crisis, your primary concern should be for your own personal safety as well as your partner's safety. Your ultimate goal is to transport the patient to the hospital safely. Gather as much medical history information as possible, but keep in mind that many patients experiencing an emotional or behavioral crisis will not readily provide this information. It is important to remember that patients with emotional or behavioral crises may appear calm initially; however, there is always the potential for them to turn violent.

You are assessing a 40-year-old male with an apparent psychiatric crisis. According to the mans's wife, he has a history of depression and schizophrenia and takes Zoloft and Zyprexa. A police officer is present at the scene. The patient, who is obviously scared, tells you that he sees snakes everywhere. You should: • A:let him know that going to the hospital will keep him safe from the snakes. • B:let him know that it is important for him to be transported as soon as possible. • C:tell him that you do not see any snakes, but they are obviously scaring him. • D:advise him that you suspect he has not been compliant with his medications.

You selected C; This is correct! Reason:When caring for any patient with a behavioral emergency, your primary responsibility is to yourself, to stay safe. The patient in this scenario, although scared, is not violent. In cases such as this, you should be prepared to spend extra time with the patient; it may take longer to assess, listen to, and prepare the patient for transport. Just because he is experiencing an acute crisis does not mean that he has not been taking his medications; to make that assumption is merely speculation on your part. You must be honest, reassuring, and nonjudgmental. Let the patient tell you how he or she is feeling in his or her own words, and acknowledge any auditory or visual hallucinations. Do not, however, play along with his or her hallucinations; this is cruel and only reinforces that what the patient is hearing or seeing is real.

You and your partner are performing CPR on an infant with suspected sudden infant death syndrome (SIDS). An important aspect in dealing with such cases is: • A:remembering that most infants with SIDS can be successfully resuscitated. • B:focusing all of your attention on the infant, with little parental interaction. • C:carefully inspecting the environment in which the infant was found. • D:discouraging the presence of the parents during your resuscitation attempt.

You selected C; This is correct! Reason:When managing an infant with suspected sudden infant death syndrome (SIDS), you will be faced with three tasks: assessment and management of the infant, communicating with and providing emotional support to the family, and assessing the scene. When assessing the scene, you should note the position in which the infant was found, any signs that suggest the infant was recently ill (eg, medications, humidifiers), and the general condition of the house (ie, clean or dirty). Because most infants die of SIDS during the night and are not discovered until the next morning, resuscitation is futile and is generally not indicated. In some cases, you may begin CPR as another member of your team discusses the situation with the parents. CPR is typically stopped after it is evident to the parents that the infant is dead and that resuscitation would be futile. If the parents want you to attempt resuscitation, encourage them to be present. Some parents wish to be present; others do not.

You are performing abdominal thrusts on a 19-year-old male with a severe airway obstruction when he becomes unresponsive. After lowering him to the ground and placing him in a supine position, you should: • A:assess for a carotid pulse for up to 10 seconds. • B:continue abdominal thrusts until ALS arrives. • C:open his airway and look inside his mouth. • D:begin CPR, starting with chest compressions.

You selected D; This is correct! Reason: A patient with a severe airway obstruction may initially be responsive and then become unresponsive during treatment. In this case, you know that an airway obstruction is the cause of his or her problem. Therefore, after placing the patient in a supine position, you should begin CPR, starting with chest compressions. Do not check for a pulse before starting chest compressions. After performing 30 chest compressions (15 compressions in infants and children when two EMTs are present), open the airway and look in the mouth. Only remove an object that you can see; do not perform a blind finger sweep in any patient. If you cannot see the object, resume chest compressions. Attempt to ventilate only if you retrieve an object from the mouth.

Which of the following statements regarding two-rescuer child CPR is correct? • A:A compression to ventilation ratio of 15:2 should be delivered without pauses in compressions to deliver ventilations • B:The chest should be compressed with one hand and a compression to ventilation ratio of 30:2 should be delivered • C:Compress the chest with one or two hands to a depth that is equal to one third the diameter of the chest • D:The chest should not be allowed to fully recoil in between compressions as this may impair venous return

You selected C; This is correct! Reason:When performing two-rescuer CPR on a child (1 year of age to the onset of puberty [12 to 14 years of age]), the chest should be compressed with one or two hands (depending on the size of the child), and a compression to ventilation ratio of 15:2 should be delivered. It is important to compress the chest to an adequate depth—one third the anterior-posterior diameter of the chest (about 1 1/2" in the child). The chest should be allowed to fully recoil in between compressions in order to maximize venous return to the heart. If an advanced airway device (ie, ET tube, multilumen airway, supraglottic airway) is not in place, two rescuers should deliver "cycles" of CPR; the compressor should pause briefly so the ventilator can deliver two breaths. A compression to ventilation ratio of 30:2 is used for one-rescuer child CPR. After an advanced airway device has been inserted, "cycles" of CPR should not be performed; compressions should be continuous at a rate of at least 100/min and ventilations should be delivered at a rate of 8 to 10 breaths/min (one breath every 6 to 8 seconds).

Which position is MOST appropriate for a mother in labor with a prolapsed umbilical cord? • A:Supine with legs elevated • B:Left lateral recumbent • C:Supine with hips elevated • D:Left side with legs elevated

You selected C; This is correct! Reason:When the umbilical cord is prolapsed, the infant typically slides down the birth canal and rests on top of the cord, shutting off its own oxygen supply. Placing the mother supine with her hips elevated will cause the baby to slide back into the birth canal slightly, thereby relieving pressure off of the cord. It may be necessary to insert your gloved fingers into the mother's vagina and lift the baby's head off of the cord. Give the mother high-flow oxygen and transport without delay. A lateral recumbent (on the side) position is appropriate for pregnant women without a prolapsed cord and will help prevent the occurrence of supine hypotensive syndrome, a condition in which the pregnant uterus compresses the inferior vena cava and compromises cardiac output.

When driving in emergency mode on a multilane highway, the emergency vehicle operator should keep to the: • A:right shoulder so that traffic flow is not disrupted. • B:extreme right lane so motorists can yield to the left. • C:extreme left lane so motorists can yield to the right. • D:center lane so the traffic can flow around the ambulance.

You selected C; This is correct! Reason:When traveling on a highway with more than one lane, the emergency vehicle operator should remain in the extreme left-hand (fast) lane. This allows other motorists to yield to the right as they see you approach. A motorist's typical initial reaction is to slam on their brakes and then look for the best direction to yield; this may be to the left or right, depending on the traffic. Do not attempt to pass a motorist until he or she is clearly aware of your presence and has yielded accordingly. Passing other motorists on the right is unsafe and should be avoided.

A 48-year-old male became acutely hypoxic, experienced a seizure, and is now postictal. The MOST effective way to prevent another seizure is to: • A:dim the lights in the back of the ambulance. • B:place him in the recovery position. • C:administer high-flow supplemental oxygen. • D:give him oral glucose if he can swallow.

You selected C; This is correct! Reason:You should administer high-flow oxygen to all patients who are actively seizing and to patients who experienced a seizure and are postictal. This is especially true if the seizure was caused by hypoxia. Increasing the oxygen content of the blood, which minimizes hypoxia, may prevent another seizure. The recovery position is appropriate for uninjured patients with a decreased level of consciousness and adequate breathing; it will help maintain the airway and facilitate drainage of secretions from the mouth, but will not prevent another seizure. Oral glucose may prevent another seizure if hypoglycemia was the cause of the seizure. You should dim the lights in the back of the ambulance to help prevent any seizure, not just those that are caused by hypoxia.

Which of the following parameters is the LEAST reliable when assessing the perfusion status of a 2-year-old child? • A:Skin color and temperature • B:Presence of peripheral pulses • C:Systolic blood pressure • D:Capillary refill time

You selected C; This is correct! Reason:You should never rely on the systolic blood pressure when assessing the perfusion status of anyone. More reliable parameters include assessing peripheral pulses, capillary refill time (most reliable in children younger than 6 years of age), and the condition and temperature of the skin. Remember that the body's compensatory mechanisms work to maintain the blood pressure, so when it falls, this corresponds to decompensated shock.

Assessment and treatment of a patient with severe abdominal pain should include: • A:giving the patient small sips of water. • B:placing the patient in a supine position. • C:administering supplemental oxygen. • D:vigorously palpating the abdomen.

You selected C; This is correct! Reason:Patients with severe abdominal pain should be given supplemental oxygen (in a concentration sufficient to maintain an SpO2 of greater than 94%) and immediate transport. Most patients with abdominal pain prefer to lie on their side with their knees drawn up into their chest (fetal position), which takes pressure off of the abdominal musculature and often affords them some relief. Vigorous palpation of the abdomen should be avoided as this will only cause the patient more pain; instead, gently palpate each of the four abdominal quadrants, palpating the most painful area last. Because the patient may become nauseated and vomit and the possibility that he or she may require surgery, do not give him or her anything to eat or drink.

During your assessment of an unresponsive adult female, you determine that she is apneic. You should: • A:place an oropharyngeal airway. • B:deliver two rescue breaths. • C:begin chest compressions. • D:assess for a carotid pulse.

You selected D; This is correct! Reason: As soon as you determine that an adult patient is apneic or only has agonal gasps, you should assess for a carotid pulse for at least 5 seconds but no more than 10 seconds. If the patient has a pulse, provide rescue breathing at a rate of 10 to 12 breaths/min (one breath every 5 to 6 seconds). If the patient does not have a pulse, perform 30 chest compressions and then open the airway and deliver 2 rescue breaths. When managing a patient who is in cardiac arrest, it is critical to minimize interruptions in chest compressions and to avoid delays in starting chest compressions. After starting CPR, apply the AED as soon as one is available. An airway adjunct should also be inserted as soon as possible.

A 24-year-old female presents with a rash to her left leg and swollen, painful knee joints. She tells you that she and her friends returned from a hiking trip in the mountains a week ago. She is conscious and alert with a blood pressure of 112/62 mm Hg, a pulse of 84 beats/min, and respirations of 14 breaths/min. Her symptoms are MOST likely the result of: • A:Rocky Mountain spotted fever. • B:tetanus. • C:Lyme disease. • D:a localized allergic reaction.

You selected C; This is correct! Reason:The patient's symptoms and her history of a recent hiking trip are consistent with Lyme disease, which was the result of a tick bite. Ticks can carry two infectious diseases: Lyme disease and Rocky Mountain spotted fever. Both are spread through the tick's saliva, which is injected into the skin when the tick attaches itself. The first symptom of Lyme disease, a rash that may spread to several parts of the body, begins about 3 days after the bite of an infected tick. The rash may eventually resemble a target bull's-eye pattern in one third of patients. After a few more days or weeks, painful swelling of the joints, particularly the knees, occurs. If recognized and treated promptly with antibiotics, many patients recover completely. Rocky Mountain spotted fever, which is not limited to the Rocky Mountains, occurs within 7 to 10 days after being bitten by an infected tick. Its symptoms include nausea, vomiting, headache, weakness, paralysis, and possibly cardiopulmonary failure.

You respond to a baseball field for a person who was struck by lightning. When you arrive, you see one patient who appears confused and is ambulatory; a second patient who is conscious, sitting on the ground, and holding his arm; and a third patient who is supine and motionless. After requesting additional responders, you should: • A:assess the motionless victim and perform CPR and defibrillation if necessary. • B:assess and begin treating the sitting patient first because he is obviously injured. • C:treat the conscious patients because the motionless patient is likely deceased. • D:proceed to the ambulatory patient, sit him down, and assess his mental status.

You selected D; The correct answer is A; Reason:The process of triaging multiple victims of a lightning strike is different than the conventional triage methods used during a mass-casualty incident. When a person is struck by lightning, respiratory or cardiac arrest, if it occurs, usually occurs immediately. Those who are conscious following a lightning strike are much less likely to develop delayed respiratory or cardiac arrest; they usually survive. Therefore, you should focus your efforts on those who are in respiratory or cardiac arrest. This process, called reverse triage, differs from conventional triage, where such patients would ordinarily be classified as deceased.

Which of the following is the preferred initial method for providing artificial ventilations to an apneic adult? • A:Flow-restricted, oxygen-powered ventilation device • B:Mouth-to-mask technique with supplemental oxygen • C:One-person bag-valve-mask technique with 100% oxygen • D:Two-person bag-valve-mask technique with 100% oxygen

You selected D; The correct answer is B; Reason: The preferred initial method for providing artificial ventilations is the mouth-to-mask technique with one-way valve and supplemental oxygen attached. Evidence has show that rescuers who ventilate patients infrequently have difficulty maintaining an adequate seal with a bag-mask device. Because both of the rescuer's hands are freed up when using a pocket face mask, it is easier to maintain an adequate seal, thus providing more effective ventilations. Of course, if two rescuers are available to manage the airway, the two-person bag-mask device technique should be used. The flow-restricted, oxygen-powered ventilation device, also referred to as the manually-triggered ventilator or demand valve, requires an oxygen source to function and would thus not be practical as an initial device for providing artificial ventilations.

A 19-year-old male was assaulted and has trauma to multiple body systems. After performing your primary assessment and treating any immediate life-threatening injuries, you should: • A:obtain a full set of baseline vital signs. • B:transport at once and intercept with ALS. • C:perform a rapid head-to-toe assessment. • D:fully immobilize his spine and transport.

You selected D; The correct answer is C; Reason: After treating all life-threatening conditions found in the primary assessment, you should perform a rapid head-to-toe assessment (rapid body scan) to look for and treat other life threats. In many cases, patients with trauma to multiple body systems have other life-threatening injuries that are not readily apparent during the primary assessment. You should obtain baseline vital signs as soon as possible; however, this should not delay or interrupt your primary or rapid head-to-toe assessments. After performing the primary and rapid head-to-toe assessments, fully immobilize the patient's spine and transport to an appropriate hospital. Consider an advanced life support (ALS) intercept, as long as it does not cause a significant delay in transport.

An unresponsive man has shallow, gurgling respirations at a rate of 8 breaths/min. Initial treatment should include: • A:oxygen via nonrebreathing mask. • B:positive-pressure ventilations. • C:suctioning of the oropharynx. • D:oropharyngeal airway insertion.

You selected D; The correct answer is C; Reason: Before breathing can be assessed, let alone managed, the airway must be cleared of any and all secretions. When you hear gurgling respirations, you should provide immediate suctioning of the oropharynx for up to 15 seconds. After the airway is clear, insert an oral or nasal airway and begin assisting his ventilations. Shallow respirations at a rate of 8 breaths/min will not produce adequate minute volume and will require ventilatory assistance. If the patient is continuously producing oral secretions, you should suction his airway for 15 seconds and then ventilate him for 2 minutes. Continue this alternating pattern until his airway is clear of secretions or an advanced airway device (ie, ET tube, multilumen airway, supraglottic airway) has been inserted.

Following a head injury, a young female is semiconscious and is bleeding from the nose and left ear. You should: • A:place a pressure dressing over her ear to prevent blood loss. • B:insert a nasal airway to keep her tongue from blocking the airway. • C:cover her ear and nose with a loose gauze pad to collect the blood. • D:control the bleeding from her nose by pinching her nostrils closed

You selected D; The correct answer is C; Reason: Blood draining from the ears or nose following a head injury may contain cerebrospinal fluid (CSF) and indicates a skull fracture. In these cases, do NOT attempt to stop the flow of blood. Applying excessive pressure may force the blood leaking from the ears or nose to collect within the cranium, which could increase the pressure on the brain and cause permanent damage. Loosely cover the ears or nose with a sterile gauze pad to collect the blood and help keep contaminants out (patients with a skull fracture and CSF leakage are at risk for meningitis). The nasopharyngeal (nasal) airway is contraindicated in patients with a possible skull fracture, especially if blood is draining from the nose. The airway adjunct may inadvertently enter the cranial vault through the fracture.

When an error occurs while using the AED, it is MOST often the result of: • A:malfunction of the microchip inside the AED. • B:excess patient movement during the analyze phase. • C:misinterpretation of the patient's cardiac rhythm. • D:battery failure secondary to operator error

You selected D; This is correct! Reason: AEDs have a high specificity for recognizing shockable rhythm (eg, V-Fib, pulseless V-Tach); this means that they are highly reliable. It would be extremely rare for the AED to recommend a shock when one is not indicated or fail to recommend a shock when one is indicated. When an error does occur, it is usually the operator's fault. The most common error is not having a charged battery. To avoid this problem, many AEDs are equipped with an alarm that warns the operator if the battery is not fully charged. Some of the older AEDs, however, are not equipped with this feature. Therefore, it is important to check the AED daily, exercise the battery as often as the manufacturer recommends, and always have a spare, fully-charged, battery on hand.

Prior to applying a nonrebreathing mask on a patient with difficulty breathing, you should: • A:set the flow rate to no more than 10 liters per minute. • B:perform a complete exam to assess the degree of hypoxia. • C:insert a nasopharyngeal airway to maintain airway patency. • D:prefill the reservoir bag to ensure delivery of 100% oxygen.

You selected D; This is correct! Reason: After attaching the nonrebreathing mask to the oxygen source, the flowmeter should be set to between 12 and 15 L/min. The reservoir bag is then prefilled with oxygen, which will allow the delivery of high-flow oxygen. Unless the patient has a decreased level of consciousness, a nasopharyngeal airway is not required before applying a nonrebreathing mask. The need for supplemental oxygen should be determined early in your assessment; do not perform an in-depth exam before deciding to administer oxygen.

The method by which you administer supplemental oxygen to a hypoxic patient depends MOSTLY on the: • A:suspected underlying cause of the hypoxia. • B:presence or absence of cyanosis. • C:patient's level of consciousness and heart rate. • D:severity of hypoxia and adequacy of breathing.

You selected D; This is correct! Reason: All hypoxic patients, whatever the cause of their condition, should be treated with high-flow oxygen. The method of oxygen delivery depends on the severity of the hypoxia and the adequacy of the patient's breathing. For example, a hypoxic patient who is breathing adequately (eg, normal rate, adequate tidal volume) should receive oxygen via nonrebreathing mask. However, if the patient is breathing inadequately (eg, fast or slow rate, shallow breathing [reduced tidal volume]), he or she may require ventilation assistance with a bag-mask device. The absence of cyanosis does not rule out hypoxia; cyanosis is a later sign and indicates significant hemoglobin desaturation. A patient's level of consciousness and heart rate can give you clues as to the severity of his or her hypoxia; a decreased level of consciousness and a rapid, weak pulse rate are signs of significant hypoxia

A 50-year-old man presents with crushing chest pain of sudden onset. He is diaphoretic, apprehensive, and tachypneic. You should: • A:obtain baseline vital signs. • B:ask him if he takes nitroglycerin. • C:perform a complete physical exam. • D:apply supplemental oxygen.

You selected D; This is correct! Reason: All of the interventions and assessments listed in this question should be performed on a patient who presents with chest pain, pressure, or discomfort. However, supplemental oxygen is indicated for any patient with a potential cardiac problem and should be given as soon as possible; this is especially true when the patient has potential respiratory involvement as well (ie, dyspnea, tachypnea). Administer oxygen in a concentration sufficient to maintain an oxygen saturation of 94% or greater. Aspirin (up to 325 mg) should also be administered as soon as possible, unless the patient is allergic to it. After applying oxygen and administering aspirin, you should perform a physical exam and obtain baseline vital signs. You would inquire about any prescription medications the patient is taking (eg, NTG) during the SAMPLE history.

While assessing a patient with chest pain, you note that his pulse is irregular. This indicates: • A:acute myocardial infarction or angina pectoris. • B:high blood pressure that is increasing cardiac workload. • C:a dysfunction in the left side of the patient's heart • D:abnormalities in the heart's electrical conduction system.

You selected D; This is correct! Reason: An irregular pulse indicates abnormalities in the electrical conduction system of the heart. The electrical conduction system, beginning with the sinoatrial node as the primary pacemaker, is responsible for initiating the electrical impulses that stimulate the myocardium to contract. An irregular pulse could indicate potentially lethal arrhythmias that could result in cardiac arrest. You should document an irregular pulse and report this important finding to the emergency department.

Which of the following injuries or conditions should be managed FIRST? • A:Fluid drainage from both ears • B:A large open abdominal wound • C:Bilateral fractures of the femurs • D:Bleeding within the oral cavity

You selected D; This is correct! Reason: Any injury or condition that jeopardizes the airway has priority over all else. If blood or other secretions within the mouth are not suctioned immediately, aspiration may occur; this significantly increases mortality. After securing a patent airway, control any external bleeding. Ideally, you and your partner should treat airway problems and external bleeding at the same time.

A middle-aged male was found unresponsive by his wife. When you arrive at the scene, you assess the patient and determine that he is apneic and pulseless. You should: • A:perform CPR with a compression-to-ventilation ratio of 15:2, apply the AED, and request backup. • B:immediately apply the AED, analyze his cardiac rhythm, deliver a shock if indicated, and begin CPR. • C:immediately begin CPR, reassess for a carotid pulse after 60 seconds, and then apply the AED. • D:begin CPR starting with chest compressions, apply the AED as soon as possible, and request backup.

You selected D; This is correct! Reason: As soon as you determine that a patient is unresponsive, pulseless, and apneic, you should begin CPR starting with chest compressions. The appropriate compression-to-ventilation ratio for adult CPR (one- or two-rescuer) is 30:2. A compression-to-ventilation ratio of 15:2 is used for two-rescuer infant and child CPR. Request a backup ambulance as soon as possible; however, do not interrupt CPR to do so. One EMT should perform CPR while the other radios for assistance. Continue CPR and reanalyze the patient's cardiac rhythm every 2 minutes. If indicated, deliver a single shock and immediately resume CPR, starting with chest compressions. If the AED gives a no shock message, resume CPR, starting with chest compressions. Continue CPR, rhythm analysis every 2 minutes, and defibrillation (if indicated), until backup arrives or the patient starts to move.

After defibrillating a man in cardiac arrest, you resume CPR. As you are about to reanalyze his cardiac rhythm 2 minutes later, your partner tells you she can definitely feel a strong carotid pulse. You should: • A:remove the AED pads from the patient's chest. • B:ask her to obtain a blood pressure reading. • C:continue with the rhythm analysis. • D:assess the patient's breathing effort.

You selected D; This is correct! Reason: If return of spontaneous circulation (ROSC) occurs, your first action should be to reassess the patient's airway status and breathing effort. If the patient is still apneic or is breathing inadequately, continue ventilations and frequently reassess his or her pulse. If the patient is breathing adequately, apply high-flow oxygen via nonrebreathing mask. If the patient is still unresponsive, as is often the case, insert an appropriate airway adjunct (if not already done) to assist in maintaining airway patency. After reassessing airway and breathing, obtain the patient's blood pressure and treat hypotension if needed. Do not remove the AED pads from the patient's chest, even if ROSC has occurred. The risk of cardiac arrest is still high and the patient may need further defibrillation. You should, however, disconnect the pads from the AED or simply turn the AED off.

A 60-year-old man is in cardiac arrest. You begin CPR while your partner applies the AED. What should you do if you receive a no shock message? • A:Ensure that the AED electrodes are properly applied. • B:Reanalyze his cardiac rhythm after 30 seconds of CPR. • C:Assess for a carotid pulse for up to 10 seconds. • D:Resume CPR, starting with chest compressions.

You selected D; This is correct! Reason: If the AED gives a no shock advised message, you should immediately resume CPR, starting with chest compressions. After 2 minutes of CPR, reanalyze the patient's cardiac rhythm and follow the AED voice prompts. You should not assess for a pulse if the AED gives a no shock message; this will only cause an unnecessary delay in performing chest compressions. Rarely, if ever, does CPR alone restore a normal cardiac rhythm and pulse. If the AED electrodes are improperly applied, it will not analyze the patient's cardiac rhythm; instead, you will receive a "check patient" or "check electrodes" message. Continue CPR, rhythm analysis every 2 minutes, and defibrillation (if indicated) until ALS personnel arrive or the patient starts to move.

You are assessing a 70-year-old male who complains of pain in both of his legs. He is conscious and alert, has a blood pressure of 160/90 mm Hg, a pulse rate of 110 beats/min, and respirations of 14 breaths/min and unlabored. Further assessment reveals edema to both of his feet and legs and jugular venous distention. This patient's primary problem is MOST likely: • A:left heart failure. • B:pulmonary edema. • C:chronic hypertension. • D:right heart failure.

You selected D; This is correct! Reason: If the right side of the heart is damaged, fluid collects in the body (edema), often showing in the feet and legs. The collection of fluid in the part of the body that is closest to the ground is called dependent edema. The swelling causes relatively few symptoms other than discomfort. Another feature of right heart failure is jugular venous distention, which is an indication of blood backing up into the systemic circulation. Left heart failure typically presents with shortness of breath due to fluid in the lungs (pulmonary edema), which indicates blood backing up from the left side of the heart into the lungs. In severe pulmonary edema, the patient may cough up pink, frothy sputum. Right heart failure and/or left heart failure are also referred to as congestive heart failure (CHF). Chronic hypertension cannot be established on the basis of a single blood pressure reading.

Priority treatment for a large avulsion includes: • A:cleaning the wound. • B:assessing distal circulation. • C:immobilizing the injured area. • D:controlling any bleeding.

You selected D; This is correct! Reason: Immediate treatment for any soft-tissue injury begins with controlling any external bleeding. Once the bleeding is controlled, distal circulation, motor, and sensory functions should be assessed, the wound dressed and bandaged, and then distal circulation, motor, and sensory functions reassessed. The injured area can be immobilized as well to prevent further injury and to help reduce bleeding. Generally, open wounds are not cleaned in the field unless they are grossly contaminated with large debris.

Snoring respirations in an unresponsive patient are usually the result of: • A:swelling of the upper airway structures. • B:foreign body airway obstruction. • C:collapse of the trachea during breathing. • D:upper airway obstruction by the tongue.

You selected D; This is correct! Reason: In an unresponsive patient, the muscles of the tongue, which attach to the mandible, relax and fall back over the posterior pharynx. This makes obstruction by the tongue the most common cause of airway obstruction in the unresponsive patient. Foreign body upper airway obstructions and upper airway swelling typically produce stridor, a high-pitched sound heard during inhalation. Collapsing of the trachea during breathing would likely present with marked respiratory distress.

Despite direct pressure, a large laceration to the medial aspect of the arm continues to bleed profusely. You should: • A:pack the inside of the laceration with sterile gauze. • B:locate and apply pressure to the brachial artery. • C:continue direct pressure and elevate the extremity. • D:quickly apply a tourniquet proximal to the injury.

You selected D; This is correct! Reason: In most cases, external bleeding can be controlled with direct pressure. However, if a wound continues to bleed profusely despite direct pressure, a proximal tourniquet should be applied without delay. If the external bleeding is that severe, elevating the extremity would be of little help. You should not pack anything inside an open wound. Evidence has shown that locating and applying adequate pressure to an arterial pressure point is difficult and time-consuming. If not promptly controlled, severe external bleeding will result in hemorrhagic shock and death

Shallow respirations are an indication of: • A:increased minute volume. • B:increased carbon dioxide removal. • C:increased oxygen intake. • D:decreased tidal volume.

You selected D; This is correct! Reason: Shallow respirations are an indication of decreased tidal volume. Tidal volume is the amount of air (in milliliters [mL]) breathed into or out of the lungs in a single breath. Adequate tidal volume is needed to bring in adequate amounts of oxygen and eliminate adequate amounts of carbon dioxide. Patients with shallow breathing often need some form of positive-pressure ventilation assistance (eg, bag-mask or pocket face mask device), especially if they have a decreased mental status. Minute volume is the volume of air that is moved through the lungs per minute; it is a product of tidal volume multiplied by the respiratory rate. If tidal volume is reduced, minute volume will be reduced as well unless there is a compensatory increase in the respiratory rate.

White blood cell

blood cell that functions in defending the body against infections and cancer cells; also called a leukocyte

A 56-year-old man was the unrestrained driver of a small passenger car that rolled over twice after he rounded a corner too fast. He is unresponsive; has rapid, shallow respirations; and has a rapid, weak pulse. His left arm is completely amputated just below the elbow. As you and your partner are treating the patient, other responders are trying to find the amputated arm. Which of the following statements regarding this scenario is correct? • A:Your priority should be to recover the man's arm because a vascular surgeon may be able to successfully reattach it. • B:Quickly move the patient to the ambulance, continue treatment, and wait for the other responders to recover his arm. • C:You should transport the patient immediately, even if the other responders recover his arm before you depart the scene. • D:If the patient's arm has not been recovered by the time you are ready to transport, you should transport without delay.

You selected D; This is correct! Reason: Life takes priority over limb. The patient is in shock, which may be complicated by a head injury; therefore, he requires rapid transport to a trauma center. Although efforts should be made to recover an amputated body part, this must not delay transport of a critically injured patient. If the arm has not been recovered by the time you are ready to transport, you must transport without delay. If his arm is located after you depart the scene, it can be transported separately. If his arm is recovered before you depart the scene, however, you should take it with you; surgeons may be able to successfully reattach it. Care for the amputated part in accordance with your local protocols.

Medications such as albuterol (Ventolin) relieve respiratory distress by: • A:contracting the smaller airways in the lungs. • B:dilating the large mainstem bronchi of the airway. • C:constricting the bronchioles in the lungs. • D:relaxing the smooth muscle of the bronchioles.

You selected D; This is correct! Reason: Medications such as albuterol (Ventolin) and metaproterenol (Alupent) are in a class of drugs called bronchodilators. They relax the smooth muscle found within the bronchioles in the lungs, which causes them to dilate. This effect opens the air passages and improves the patient's ability to breathe.

Which of the following patients would MOST likely present with vague or unusual symptoms of an acute myocardial infarction? • A:75-year-old male with hypertension • B:55-year-old obese female • C:66-year-old male with angina • D:72-year-old female with diabetes

You selected D; This is correct! Reason: Not all patients experiencing acute myocardial infarction (AMI) present with the classic signs and symptoms one would expect. Middle-aged men often minimize their symptoms and attribute their chest pain or discomfort to indigestion. Some patients, however, do not experience any pain. In particular, elderly women with diabetes may present with vague, unusual, or atypical symptoms of AMI; their only presenting complaint may be fatigue or syncope. Do not rule out a cardiac problem just because a patient is not experiencing chest pain, pressure, or discomfort; this is especially true in elderly females with diabetes.

Occasional, irregular breaths that may be observed in a cardiac arrest patient are called: • A:Biot respirations. • B:Cheyne-Stokes respirations. • C:ataxic respirations. • D:agonal gasps.

You selected D; This is correct! Reason: Occasional, irregular breaths, called agonal gasps, may be observed in some patients shortly after their heart stops beating. They occur when the respiratory center in the brain sends stray signals to the respiratory muscles. Agonal gasps are not adequate because they are infrequent and result in negligible tidal volume. Biot respirations are characterized by an irregular pattern, rate, and depth of breathing with intermittent periods of apnea; they are commonly associated with severe brain trauma. Ataxic respirations are ineffective, irregular breaths that may or may not have an identifiable pattern; they are also commonly associated with severe brain trauma. Cheyne-Stokes respirations are characterized by a crescendo-decrescendo pattern of breathing with a period of apnea between each cycle (fast, slow, apnea). Cheyne-Stokes respirations may occur in healthy people during certain phases of the sleep cycle; however, if they are grossly exaggerated or occur in a patient with a head injury, they are an ominous sign.

Patients with a hypoxic drive: • A:are accustomed to low levels of carbon dioxide in the blood. • B:may hypoventilate if given low concentrations of oxygen. • C:rarely become cyanotic because of high blood oxygen levels. • D:are stimulated to breathe by low oxygen levels in the blood.

You selected D; This is correct! Reason: Patients with chronic respiratory diseases (eg, emphysema) maintain decreased levels of oxygen and increased levels of carbon dioxide in the blood. The sensors in the brain become accustomed to this. Unlike a healthy person, whose primary respiratory drive is influenced by increasing carbon dioxide levels in the blood, the primary respiratory drive of a patient with a chronic respiratory disease is influenced by low levels of oxygen in the blood (hypoxic drive). Cyanosis is common due to chronic hypoxemia. Some patients with the hypoxic drive may hypoventilate if given high concentrations of supplemental oxygen, although this is highly uncommon. High-flow supplemental oxygen may fool the brain into thinking the body has sufficient oxygen, causing it to send fewer signals to the diaphragm and intercostal muscles.

You should suspect that your patient has pulmonary edema if he or she: • A:has swollen feet and ankles. • B:has a dry, nonproductive cough. • C:is hypertensive and tachycardic. • D:cannot breathe while lying down.

You selected D; This is correct! Reason: Pulmonary edema is often caused by failure of the left side of the heart. When the patient is lying down, he or she experiences worsened difficulty breathing (orthopnea) because more blood backs up in the lungs. Patients with severe pulmonary edema often produce pink, frothy sputum when they cough; this is another sign of blood backing up in the lungs. A dry, nonproductive cough is not common. Hypertension and tachycardia are common in patients with pulmonary edema; however, many other conditions can cause these findings. Swelling of the feet and ankles is commonly seen in patients with right heart failure, and occurs when blood backs up beyond the right atrium; it is not a common sign of left heart failure and pulmonary edema.

A 40-year-old man has burns to the entire head, anterior chest, and both anterior upper extremities. Using the adult Rule of Nines, what percentage of his total body surface area has been burned? • A:18% • B:45% • C:36% • D:27%

You selected D; This is correct! Reason: Using the adult Rule of Nines, the head accounts for 9% of the total body surface area (TBSA), the anterior chest for 9% (the entire anterior trunk [chest and abdomen] accounts for 18%), and the anterior upper extremities for 4.5% each (each entire upper extremity is 9% of the TBSA). On the basis of this, the patient has sustained 27% TBSA burns

Shock following major trauma is MOST often the result of: • A:head injury. • B:long bone fractures. • C:spinal injury. • D:hemorrhage.

You selected D; This is correct! Reason: Shock following major trauma is usually caused by hemorrhage (bleeding), which can be external and obvious (gross), internal and hidden (occult), or both. Trauma to the chest and/or abdomen and multiple long bone fractures are common causes of hemorrhage that result in shock. An isolated head injury usually does not cause shock; it causes increased intracranial pressure. If the patient with a seemingly isolated head injury has signs of shock, look for other injuries. Major trauma may also be associated with spinal injury. If the spinal cord is injured, the patient may develop shock because the nerves that control the diameter of the blood vessels are damaged, resulting in widespread vasodilation (neurogenic shock).

Which of the following mechanisms of injury would necessitate performing a rapid head-to-toe assessment? • A:A 5 foot, 9 inch tall adult who fell 12 feet from a roof and landed on his side • B:An impaled object in the patient's lower extremity with minimal venous bleeding • C:Amputation of three toes from the patient's left foot with controlled bleeding • D:A stable patient involved in a motor-vehicle crash, whose passenger was killed

You selected D; This is correct! Reason: Significant mechanisms of injury include, among others, falls of greater than 15 feet (or three times the patient's height), penetrating injuries to the trunk and head, high-speed motor vehicle crashes, rollover motor vehicle crashes, ejection from a motor vehicle, and motor vehicle crashes in which another person in the same passenger compartment was killed. In cases such as this, you must assume that the same violent forces that killed the passenger were sustained by the patient, regardless of whether the patient is stable or not.

Signs of inadequate breathing in an unresponsive patient include: • A:an irregular pulse. • B:warm, moist skin. • C:symmetrical chest rise. • D:cyanotic oral mucosa.

You selected D; This is correct! Reason: Signs of inadequate breathing in both responsive and unresponsive patients include a respiratory rate that is too slow (less than 12 breaths/min) or too fast (greater than 20 breaths/min); shallow (reduced tidal volume), irregular, or gasping respirations; asymmetrical (unequal) chest rise; abnormal respiratory sounds, such as wheezing, stridor, or gurgling; and abnormal skin color and condition (ie, cool or cold skin, pallor, diaphoresis, cyanosis). An irregular pulse indicates a cardiac dysrhythmia.

If the level of carbon dioxide in the arterial blood increases: • A:a reduction in tidal volume will occur. • B:the respiratory rate and depth decrease. • C:the respiratory rate slows significantly. • D:the respiratory rate and depth increase.

You selected D; This is correct! Reason: Special receptors, called chemoreceptors, sense the levels of oxygen and carbon dioxide in the arterial blood. The central chemoreceptors are located in the brain; the peripheral chemoreceptors are located in the aorta and carotid arteries. The level of carbon dioxide in the arterial blood stimulates the healthy patient to breathe (primary respiratory drive). If the carbon dioxide level rises above normal, the chemoreceptors send messages to respiratory centers in the brain, resulting in an increase in respiratory rate and depth (tidal volume). Conversely, if the level of carbon dioxide is too low, respiratory rate and depth decrease accordingly.

Which of the following sets of vital signs is MOST suggestive of increased intracranial pressure in a patient who has experienced a traumatic brain injury? • A:BP, 92/60 mm Hg; pulse, 120 beats/min; respirations, 24 breaths/min • B:BP, 160/72 mm Hg; pulse, 100 beats/min; respirations, 12 breaths/min • C:BP, 84/42 mm Hg; pulse, 60 beats/min; respirations, 32 breaths/min • D:BP, 176/98 mm Hg; pulse, 50 beats/min; respirations, 10 breaths/min

You selected D; This is correct! Reason: The body responds to a significant traumatic brain injury by shunting more oxygenated blood to the injured brain; it does this by increasing systemic blood pressure. In response to an increase in blood pressure, the pulse rate decreases. Pressure on the brain stem often causes an irregular breathing pattern that is either slow or fast. Therefore, patients with increased intracranial pressure present with hypertension, bradycardia, and irregular respirations that are fast or slow (Cushing's triad). Vital signs representative of shock (eg, hypotension, tachycardia) are not common in patients with an isolated head injury and increased intracranial pressure. If the patient with a seemingly isolated head injury is hypotensive and tachycardic, look for other injuries; internal or external bleeding is likely occurring elsewhere.

A 23-year-old unrestrained female struck the steering wheel with her chest when her passenger car collided with a tree at a high rate of speed. Your assessment reveals that she is conscious, but has signs of shock and an irregular pulse. The MOST appropriate treatment for this patient includes: • A:high-flow oxygen, summoning a paramedic unit to the scene to assess her cardiac rhythm, a cervical collar, and transport as soon as possible. • B:insertion of an oral airway, assisted ventilations with a bag-mask device, full spinal precautions, and rapid transport. • C:applying an AED in case she develops cardiac arrest, high-flow oxygen, full spinal precautions, and rapid transport. • D:high-flow oxygen or assisted ventilations as needed, full spinal precautions, blankets to keep her warm, and rapid transport.

You selected D; This is correct! Reason: The cause of this patient's shock may be a myocardial contusion, or bruising of the heart muscle. Blunt trauma to the chest can injure the heart, making it unable to maintain adequate blood pressure. In a myocardial contusion, the pulse is often irregular, but dangerous rhythms (eg, V-Fib, V-Tach) are relatively uncommon. There is no special diagnostic test at this time, and there is no prehospital treatment for the condition. Therefore, waiting at the scene for a paramedic unit would only waste time. Apply high-flow oxygen, assist ventilations if the patient is breathing inadequately (eg, slow or fast respirations, shallow breathing [reduced tidal volume]), treat the patient for shock (eg, cover her with blankets, elevate her lower extremities [if local protocol permits]), and transport rapidly. Because of the mechanism of injury, full spinal precautions should be taken. The patient is conscious and likely has an intact gag reflex; therefore, an oral airway is contraindicated. The AED is only applied to patients who are in cardiac arrest

During your rapid assessment of a critically-injured patient, you should assess the chest for: • A:crepitus and distention. • B:distention and guarding. • C:rigidity and guarding. • D:symmetry and pain.

You selected D; This is correct! Reason: When assessing the chest during the assessment, you should check for symmetry (equal rise of the chest), assess for pain upon palpation, and the presence of equal breath sounds bilaterally. Crepitus also should be noted if present, but not purposely elicited. Rigidity, guarding, and distention should be assessed for when evaluating the abdomen. Because of the ribcage, the chest is rigid by nature.

Which of the following BEST describes the mechanism of injury? • A:The product of mass, force of gravity, and height • B:The energy of an object in motion • C:Your concern for potentially serious injuries • D:The way in which traumatic injuries occur

You selected D; This is correct! Reason: The mechanism of injury (MOI) is the way in which traumatic injuries occur; it describes the forces (or energy transmission) acting on the body that cause injury. Index of suspicion is your concern for potentially serious underlying and unseen (occult) injuries. A significant MOI (eg, fall from a significant height, ejection from a motor vehicle) should increase your index of suspicion for serious injuries. The energy of an object in motion is called kinetic injury. Potential injury is the product of mass (weight), force of gravity, and height; it is mostly associated with the energy of falling objects.

Sudden cardiac arrest in the adult population MOST often is the result of: • A:accidental electrocution. • B:myocardial infarction. • C:respiratory failure. • D:a cardiac arrhythmia.

You selected D; This is correct! Reason: The most common cause of sudden cardiac arrest (SCA) in the adult population is a cardiac arrhythmia—usually ventricular fibrillation—in up to 70% of cases. This fact underscores the importance of early defibrillation to shock the heart back into a perfusing rhythm. Evidence has shown that cardiac arrest—again, most often the result of an arrhythmia—occurs in up to 40% of patients experiencing an acute myocardial infarction (AMI). The risk of cardiac arrest is highest within the first few hours following the onset of an AMI. Respiratory failure is the most common cause of cardiac arrest in children, not adults. Children generally have healthy hearts and rarely experience cardiac arrest due to a primary cardiac event.

A 22-year-old female woman was shot by her husband. Law enforcement is at the scene and has the husband in custody. The patient is conscious, but extremely restless, and is pale and diaphoretic. As your partner administers high-flow oxygen, you should: • A:keep her warm by applying blankets. • B:take her BP to detect hypotension. • C:compare her carotid and radial pulses. • D:look for and control any bleeding.

You selected D; This is correct! Reason: The primary assessment of any patient includes ensuring a patent airway, assessing breathing adequacy, administering high-flow oxygen or assisting ventilations, assessing circulation, and controlling all active bleeding. You and your partner must work as a team; as your partner administers high-flow oxygen, you should be looking for her gunshot wound(s) and ensuring that all bleeding is controlled. After the primary assessment and management, begin treating her for shock (eg, applying a blanket, elevating her lower extremities [if local protocol permits]) and perform a rapid head-to-toe assessment to search for other injuries that may not have been obvious during the primary assessment. Assess the patient's vital signs after all life-threatening injuries or conditions have been identified and corrected.

When caring for a critically injured patient, it is MOST appropriate to perform your secondary assessment: • A:immediately after taking baseline vital signs. • B:immediately following the primary assessment. • C:after all life threats have been ruled out. • D:while you are en route to the hospital.

You selected D; This is correct! Reason: The secondary assessment is a detailed exam of the patient; it focuses on detecting and correcting injuries or conditions that were not grossly obvious during the primary assessment. Because the secondary assessment can be time-consuming, it should be performed en route to the hospital if your patient is critically ill or injured. The only actions that you should perform at the scene of a critically ill or injured patient are the primary assessment, correction of immediate life-threats (eg, problems with the ABCs), and spinal precautions if necessary. The quicker you begin transport of a critically ill or injured patient, the quicker the patient will receive definitive care at the hospital. In some patients, you may not have time to perform a secondary assessment; this is especially true with critically ill or injured patients who have ongoing problems with airway, breathing, or circulation.

A patient who is breathing with reduced tidal volume would MOST likely have: • A:a prolonged inhalation phase. • B:warm, moist skin. • C:a respiratory rate of 14 breaths/min. • D:shallow respirations.

You selected D; This is correct! Reason: Tidal volume, a measure of the depth of breathing, is the amount of air (in milliliters [mL]) that is moved into or out of the lungs during a single breath; in the average adult male, this is about 500 mL. Tidal volume cannot be quantified (that is, it cannot be assigned a numeric value) by the EMT; however, it can be estimated by observing the adequacy of chest rise during inhalation. A patient who is breathing with reduced tidal volume will have a shallow depth of breathing; his or her chest rises minimally during inhalation. If a patient is not breathing with adequate tidal volume, he or she will eventually become hypoxemic, which will cause the skin to become cool and clammy and pale or cyanotic. Conversely, a patient with a prolonged inhalation phase (eg, taking a deep breath) would experience an increase in tidal volume. Minute volume is the amount of air moved through the lungs each minute; it is calculated by multiplying tidal volume and respiratory rate. A respiratory rate of 14 breaths/min with adequate tidal volume would result in adequate minute volume. Minute volume is affected by tidal volume, respiratory rate, or both.

Treatment for a patient with congestive heart failure and shortness of breath may include: • A:prophylactic suctioning of the airway. • B:supine positioning and elevation of the legs. • C:hyperventilation with a bag-mask device. • D:up to three doses of sublingual nitroglycerin.

You selected D; This is correct! Reason: Treatment for patients with congestive heart failure (CHF) includes supplemental oxygen as needed to maintain an oxygen saturation equal to or greater than 94%, continuous positive airway pressure (CPAP), ventilatory assistance with a bag-mask device if needed (do NOT hyperventilate the patient), placing the patient in an upright or sitting position to facilitate breathing (a supine position will clearly make it more difficult to breathe), monitoring the patient's vital signs, and transporting to the hospital without delay. Suction the airway only if there are secretions in the mouth; prophylactic suctioning is not indicated. Nitroglycerin (NTG) may be of value if the patient is not hypotensive and he or she has the medication prescribed to him or her. As a vasodilator, NTG causes systemic venous pooling of blood, which reduces the amount of blood returned to the heart (preload), and thus, the amount of blood available to backup in the lungs. Follow your local protocols or contact medical control as needed regarding the use of NTG for patients with CHF.

paradoxus pulsus

blood pressure declines as one inhales and increases as one exhales

A nonrebreathing mask is MOST appropriate to use on patients who: • A:are semiconscious and breathing shallowly. • B:are cyanotic and have a low oxygen saturation. • C:are breathing less than 12 times per minute. • D:have an adequate rate and depth of breathing.

You selected D; This is correct! Reason: With the oxygen flow rate set at 15 L/min, the nonrebreathing mask can deliver an oxygen concentration of 90% or greater. Unlike the bag-mask or pocket mask devices, which deliver oxygen via positive pressure, the nonrebreathing mask delivers oxygen passively; therefore, the patient must have an adequate rate and depth (tidal volume) of breathing in order to open the one-way valve in the nonrebreathing mask and inhale oxygen from the reservoir bag. Shallow (reduced tidal volume) breathing, bradypnea (slow breathing), cyanosis, a low oxygen saturation, and a decreased level of consciousness are signs of inadequate breathing, and should be treated with some form of positive-pressure ventilation assistance.

You receive a call for a 54-year-old female who is having a seizure. When you arrive at the scene, the patient is actively seizing. Her husband tells you that she has a history of seizures, and that she has been seizing continuously for 20 minutes. It is MOST important for you to recognize that: • A:her husband's description is consistent with status epilepticus. • B:the patient may have stopped taking her seizure medication. • C:her seizure could be the result of a low blood glucose level. • D:the patient is hypoxic and needs oxygen as soon as possible.

You selected D; This is correct! Reason: There are many causes of seizures, including noncompliance with prescribed anticonvulsant medications, hypoglycemia, stroke, head trauma, and poisoning, among others. Regardless of the underlying cause, it is most important to recognize that patients who are actively seizing, especially if the seizure is prolonged, are hypoxic and need high-flow oxygen as soon as possible; in many cases, it may be necessary to assist the patient's ventilations. The husband's description of the seizure (eg, continuous for 20 minutes) is consistent with status epilepticus, which is all the more reason to recognize that she is severely hypoxic. Seizure-related deaths are the result of cerebral hypoxia.

While caring for a trauma patient, blood splashes into an EMT's eyes. This is an example of: • A:infection. • B:indirect contact. • C:transmission. • D:exposure.

You selected D; This is correct! Reason: Exposure occurs when an individual comes in direct contact with blood or other bodily fluids. Examples of direct exposure include blood splashing into the eyes or mouth and an accidental stick with a contaminated needle or other sharp object. Exposure does not always result in disease transmission and subsequent infection; that depends on whether the patient has an infectious disease. Routine use of standard precautions will afford the EMT the best protection from exposure to an infectious disease.

A 16-year-old, 125-pound male ingested a bottle of aspirin approximately 20 minutes ago. Medical control orders you to administer activated charcoal in a dose of 1 g/kg. How much activated charcoal should you administer? • A:54 g • B:60 g • C:51 g • D:57 g

You selected D; This is correct! Reason: First, you must determine the patient's weight in kilograms (kg). Either of the following formulae can be used to convert pounds to kilograms: Formula 1: weight (in pounds) ÷ 2.2 = weight in kg. Formula 2: weight (in pounds) ÷ 2 - 10% = weight in kg. On the basis of the above formulae, a 125-pound patient weighs 57 kg. Using formula 1, the equation is as follows: 125 (weight in pounds) ÷ 2.2 = 56.81 (57 [rounded to the nearest tenth]). Using formula 2, the equation is as follows: 125 (weight in pounds) ÷ 2 = 62.5 (63 [rounded to the nearest tenth] - 6.3 (10% of 63) = 56.7 (57 [rounded to the nearest tenth]). Since the drug order is for 1 g/kg, you should administer 57 g of activated charcoal to your 125-pound patient.

You respond to a grocery store where a 39-year-old man reportedly experienced a seizure. When you arrive at the scene, a clerk begins to escort you to the patient. She tells you that the man stopped seizing about 5 minutes ago. If the patient truly experienced a seizure, you will MOST likely find that he: • A:has a slow heart rate. • B:is fully conscious and alert. • C:is not breathing and is cyanotic. • D:is confused and disoriented.

You selected D; This is correct! Reason: Most seizures are followed by a postictal phase, in which the patient is unresponsive at first and then gradually regains consciousness. In most cases, the postictal patient appears dazed, confused, or disoriented; in some cases, he or she is combative. The postictal phase typically lasts from 5 to 30 minutes. During the seizure, the patient is often not breathing or is breathing inadequately; however, when the seizure stops, breathing usually resumes. You will also find that most postictal patients are tachycardic; this is the result of an adrenalin surge that occurs during the tonic-clonic phase of the seizure.

While caring for an injured patient, you remove blood-soaked clothing in order to treat his injuries. You should dispose of the clothing by: • A:leaving it at the hospital. • B:leaving it at the scene. • C:placing it in a regular trash can. • D:placing it in a biohazard bag.

You selected D; This is correct! Reason: The appropriate method for disposing of soiled clothing or any other "nonsharp" contaminated item is to place it in a red biohazard bag. The biohazard insignia as well as the red color alerts others that the items within the bag are contaminated.

You have just delivered a baby girl. Your assessment of the newborn reveals that she has a patent airway, is breathing adequately, and has a heart rate of 130 beats/min. Her face and trunk are pink, but her hands and feet are cyanotic. You have clamped and cut the umbilical cord, but the placenta has not yet delivered. You should: • A:reassess the newborn every 5 minutes and transport after the placenta delivers. • B:massage the lower part of the mother's uterus until the placenta delivers. • C:give the newborn high-flow oxygen via a nonrebreathing mask and transport. • D:keep the newborn warm, give oxygen to the mother if needed, and transport.

You selected D; This is correct! Reason: The newborn is stable and does not require care beyond providing thermal management and monitoring. Oxygen is indicated for the newborn if it has central cyanosis (cyanosis to the face and trunk), and should be delivered via the blow-by technique with the flowmeter set at 5 L/min. Cyanosis to the periphery of the body (eg, hands and feet), which is called acrocyanosis, is a normal finding in the newborn. You should not wait at the scene for the placenta to deliver; it can take up to 45 minutes for this to occur. Begin transport, keep the newborn warm, and give oxygen to the mother if indicated. Massaging the uterine fundus (top part of the uterus) is indicated for women with postpartum vaginal bleeding (ie, BOTH the baby and placenta have delivered).

Abruptio placentae

a disorder of pregnancy in which the placenta prematurely separates from the wall of the uterus

The EMT should avoid focusing all of his or her attention on a single critical patient during the triage process because: • A:the patient will most likely die before he or she can be transported. • B:all of his or her supplies will likely be depleted on that one patient. • C:three EMTs are required to effectively manage a critical patient. • D:other patients may die of problems that may have been corrected.

You selected D; This is correct! Reason: Triage is the process of rapidly assessing patients in order to determine their treatment priority. Focusing your attention on one patient during the triage process not only defeats the purpose of triage (to do the greatest good for the greatest number of people), but it neglects other patients who may die from injuries or conditions that may have been corrected if detected earlier.

Which of the following scene size-up findings is LEAST suggestive of an unsafe environment? • A:The sound of breaking glass as you approach a residence • B:Screaming and yelling coming from inside a residence • C:Liquid draining from a car that struck a telephone pole • D:A large man standing in his yard awaiting your arrival

You selected D; This is correct! Reason: When approaching a residence, findings that would suggest an unsafe scene include, among other things, the sound of breaking glass, screaming and yelling, and an unusual silence. Liquid leaking from a wrecked automobile should be assumed to be gasoline and, therefore, dangerous. Although intimidating in appearance, there is no correlation between a person's physical size and his or her potential for violence.

A 6-year-old boy complains of pain to the right lower quadrant of his abdomen. Assessment of this child's abdomen should include: • A:palpating the right lower quadrant first. • B:auscultating bowel sounds for 2 minutes. • C:avoiding palpation of the abdomen. • D:palpating the left upper quadrant first.

You selected D; This is correct! Reason: When assessing the abdomen of any patient, you should determine the location of the pain and palpate that area last. Begin by palpating the abdomen furthest away from the area of pain; in this case, the left upper quadrant is furthest away from the right lower quadrant. Palpating the painful area first will interfere with the rest of your assessment because the patient will be in significant pain and will likely not remain still during the remainder of the assessment. This is especially true in children. Auscultation of bowel sounds is generally not performed in the prehospital setting; little, if any, information will be gained from doing so.

When calling your radio report to the receiving hospital, you should: • A:include the patient's name. • B:break your report into 60-second increments. • C:only give your report to a physician. • D:be brief, concise, and factual.

You selected D; This is correct! Reason:A radio report should be brief, concise, and factual. It should include the patient's age and sex, his or her chief complaint, associated assessment findings, vital signs, treatment that you provided, and the patient's response to your treatment. Avoid speculative statements regarding the patient's condition; report only what you know to be fact. Longer radio reports should be broken into 30-second increments; after 30 seconds, pause and ensure the listener heard your previous traffic. The patient's name is not vital to your report; thus, there is generally no need to disclose it. Unless you are requesting medical direction, it is acceptable, and routine practice, to give your report to a registered nurse.

Which of the following is the MOST significant finding in a patient experiencing an allergic reaction? • A:Widespread rash. • B:Abdominal cramps. • C:Headache • D:Hoarseness

You selected D; This is correct! Reason:A rash (urticaria), headache, abdominal cramps, and hoarseness are all signs of an allergic reaction. However, the presence of hoarseness is the most significant finding because it indicates upper airway swelling. Other significant findings include swelling of the face, neck, or tongue; wheezing and/or stridor; difficulty breathing; tachycardia; and hypotension. Death due to a severe allergic reaction (anaphylaxis) is caused by vascular collapse (caused by massive vasodilation) and respiratory failure (caused by upper and/or lower airway swelling).

A 9-year-old girl was struck by a car while she was crossing the street. Your assessment reveals a large contusion over the left upper quadrant of her abdomen and signs of shock. Which of the following organs has MOST likely been injured? • A:Kidney • B:Liver • C:Pancreas • D:Spleen

You selected D; This is correct! Reason:Abdominal trauma commonly occurs in children as the result of motor vehicle versus pedestrian accidents. The contusions over the left upper quadrant and the signs of shock suggest significant injury to the spleen. The liver lies in the right upper quadrant, and the pancreas and kidneys lie in the retroperitoneal space. Although the exact injury cannot be determined in the field, you must treat the patient for shock and provide rapid transport.

Activated charcoal is contraindicated for a patient who is: • A:conscious and alert and has ingested a large amount of Motrin. • B:emotionally upset and has ingested two bottles of aspirin. • C:agitated and claims to have ingested a bottle of Tylenol. • D:awake and alert and has swallowed a commercial drain cleaner.

You selected D; This is correct! Reason:Activated charcoal adsorbs (sticks to) many ingested substances, preventing them from being absorbed into the body by the stomach or intestines. In some cases, you may give activated charcoal to patients who have ingested certain substances, if approved by medical control or local protocol. Activated charcoal is contraindicated for patients who have ingested an acid or alkali (ie, drain cleaner) or a petroleum product (ie, gasoline), who have a decreased level of consciousness and cannot protect their own airway, or who are unable to swallow.

Which of the following assessment parameters is a more reliable indicator of perfusion in infants than adults? • A:Level of orientation • B:Pulse quality • C:Blood pressure • D:Capillary refill

You selected D; This is correct! Reason:Capillary refill time (CRT) is a reliable indicator of perfusion in children less than 6 years of age. When the capillary bed (eg, fingernail, forehead) is blanched, blood should return to the area in less than 2 seconds. Because peripheral perfusion decreases with age, CRT is a less reliable indicator of perfusion in older children and adults. Note that cold temperatures can affect CRT. Pulse quality is reliable in patients of any age; weak or absent peripheral pulses indicate poor perfusion in anyone. Blood pressure is the least reliable indicator of perfusion in patients of any age; it usually does not fall until the body's compensatory mechanisms have failed. Assessing an infant's level of orientation is not possible; infants do not know who they are, where they are, what happened, and what day it is. When assessing an infant's mental status, note his or her level of alertness and interactivity (eg, tracking with his or her eyes, crying versus quiet).

You arrive at the scene of a young male who was stabbed when a burglar broke into his house. Law enforcement officers are present. The patient, who is unresponsive with several stab wounds to the chest, is lying in a narrow space between a couch and coffee table. You should: • A:move the coffee table, document what you did, and begin treatment. • B:treat the patient where he is, without moving the coffee table. • C:obtain permission from law enforcement before moving any furniture. • D:move the coffee table, begin patient care, and notify a police officer.

You selected D; This is correct! Reason:After ensuring your own safety, your primary responsibility when functioning at a crime scene is to provide patient care. Optimally, you should help preserve the scene at the same time. In this situation, however, the patient requires immediate treatment; he is unresponsive and has life-threatening injuries. It would be difficult to provide effective care in his present position, and notifying law enforcement before you move any furniture would unnecessarily delay care. Therefore, you should move the coffee table out of the way and begin treatment. When possible, notify a law enforcement officer of what you moved, where you moved it to, and why you moved it. You should also document this on the patient care report, AFTER you have cared for the patient.

Which of the following is the MOST practical method of standard precautions when treating multiple patients during a mass-casualty incident? • A:Asking each patient you treat if he or she has a communicable disease • B:Placing clean gloves over soiled gloves in between patient contacts • C:Thoroughly washing your hands in between patient contacts • D:Changing your gloves in between contact with different patients

You selected D; This is correct! Reason:Although the most effective means of preventing the spread of disease involves thorough hand-washing, this is not practical at the scene of a mass-casualty incident; you usually do not have the time or facilities to do this. In a situation where you are treating multiple patients, you should change your gloves in between patients; this will help prevent cross-contamination. The concept of standard precautions is based on the assumption that ALL bodily fluids are potentially infectious; thus, there is no need to ask a patient if he or she has a communicable disease. Furthermore, to make such an inquiry is unethical.

You are called to the residence of a 39-year-old male with flu-like symptoms. The patient tells you that he has been feeling ill for the last 2 days. He has a fever, headache, and diarrhea, and asks that you take him to the hospital. His blood pressure is 120/60 mm Hg, his pulse is 110 beats/min, and his respirations are 16 breaths/min. You should: • A:request an ALS ambulance to the scene to start an IV line. • B:ask him if he has a history of HIV infection or hepatitis. • C:advise him that he can drive himself to his family physician. • D:offer oxygen and transport him in a position of comfort.

You selected D; This is correct! Reason:Although the patient is likely experiencing the flu, there are other diseases, some of which are communicable, that can cause similar symptoms. The patient is requesting EMS transport; failure to comply constitutes abandonment. Although he is tachycardic, the remainder of his vital signs are stable; therefore, requesting an ALS ambulance to the scene to start an IV is not necessary. Simply transport him in a position of comfort and offer him supplemental oxygen. If the patient is infected with HIV or hepatitis, he may choose to voluntarily disclose that information. However, to inquire about infection with such diseases is unethical.

A 30-year-old man with a history of schizophrenia cut his wrists and is bleeding profusely. He is confused, combative, and has slurred speech. With the assistance of law enforcement personnel, you and your partner physically restrain him in order to provide care and transport. In this situation, a court of law would MOST likely: • A:conclude that you should have had a court order to restrain. • B:determine that the patient had decision-making capacity. • C:agree that you and your partner are guilty of assault and battery. • D:consider your actions in providing care to be appropriate.

You selected D; This is correct! Reason:An adult with decision-making capacity (ie, a mentally competent adult) has the legal right to refuse medical treatment, even if that treatment involves lifesaving care. In psychiatric cases, however, a court of law would likely consider your actions in providing lifesaving care to be appropriate, particularly if you have a reasonable belief that the patient would harm him- or herself or others without your intervention. In addition, a patient who is in any way impaired, whether by mental illness, medical condition, or intoxication, may not be considered competent to refuse treatment and transport. If you are unsure of a patient's decision-making capacity, err on the side of treatment and transport. Few would argue that it would be easier to defend why you treated a patient than to justify or defend why you abandoned a patient.

You arrive at the scene shortly after a 3-year-old female experienced a seizure. The child, who is being held by her mother, is conscious and crying. The mother tells you that her daughter has been ill recently and has a temperature of 102.5°F. What is the MOST appropriate treatment for this child? • A:Oxygen via nonrebreathing mask, place the child in a tub of cold water to lower her body temperature, and transport. • B:Oxygen via the blow-by technique, transport, and request a paramedic intercept so an anticonvulsant drug can be given. • C:Oxygen via nonrebreathing mask, avoid any measures to lower the child's body temperature, and transport at once. • D:Oxygen via the blow-by technique, remove clothing to help reduce her fever, and transport with continuous monitoring.

You selected D; This is correct! Reason:As evidenced by her recent illness and fever (102.5°F), this child has likely experienced a febrile seizure. Appropriate treatment for the child following a febrile seizure involves ensuring a patent airway, administering oxygen (the blow-by technique is generally better tolerated in children than a mask), removing the child's clothing to facilitate heat loss, and transporting to the hospital. Avoid cooling the child with water; doing so may cause the child to shiver—a mechanism that produces body heat—which may cause an abrupt rise in body temperature and another seizure. Since the child is no longer seizing, an anticonvulsant drug is not indicated.

Failure of the EMT to obtain consent from a responsive patient before taking his or her blood pressure may constitute: • A:assault. • B:negligence. • C:abandonment. • D:battery.

You selected D; This is correct! Reason:Battery is defined as unlawfully touching another person without his or her consent. Obtaining consent from every responsive patient prior to rendering care is of paramount importance. Assault is defined as instilling fear into another person, but does not involve actually touching him or her. Negligence occurs when the EMT fails to act as another prudent EMT would have acted in the same or similar situation. Abandonment occurs when the EMT terminates patient care without the patient's consent or transfers care of a patient to a provider of lesser training.

Which of the following statements regarding pediatric anatomy is correct? • A:Relative to the overall size of the airway, a child's epiglottis is smaller. • B:The child's trachea is more rigid and less prone to collapse. • C:Airway obstruction is common in children because of their large uvula. • D:The occiput is proportionately larger when compared to an adult.

You selected D; This is correct! Reason:Compared to adults, infants and small children have a proportionately larger head, specifically the occiput (back of the head). Therefore, when positioning an infant or a child's airway, padding in between the shoulder blades is often needed to maintain neutral alignment of the head. Infants and children are at risk for an airway obstruction because their entire airway is smaller, not because their uvula is large. An infant or a child's trachea is less rigid than an adult's; therefore, it collapses more easily during respiratory distress. Relative to the overall size of an infant or a child's airway, the epiglottis is larger; it is also floppier.

Of the following, the MOST detrimental effect of gastric distention in infants and children is: • A:less effective chest compressions. • B:acute rupture of the diaphragm. • C:tracheal rupture. • D:decreased ventilatory volume.

You selected D; This is correct! Reason:Gastric distention can be lethal if not detected and managed appropriately in any patient, especially infants and children. As air insufflates the stomach, the diaphragm is pushed into the thoracic cavity, which decreases the amount of air that can fill the lungs. This results in decreased ventilatory volumes during artificial ventilation. Gastric distention also increases the risks of regurgitation and aspiration. You must be able to deliver adequate ventilations to the patient. Remember, respiratory failure is the most common cause of cardiopulmonary arrest in children.

A 60-year-old man presents with a severe right-sided headache. He is conscious and alert. As you are assessing him, he tells you that the left side of his body feels weak. His blood pressure is 190/100 mm Hg, pulse is 88 beats/min, and respirations are 14 breaths/min. His past medical history includes diabetes, arthritis, and hypertension. You should be MOST concerned with: • A:his current blood pressure. • B:the location of his headache. • C:his blood glucose level. • D:the presence of hemiparesis.

You selected D; This is correct! Reason:Headaches are a common medical complaint and have numerous causes, most of which are not life-threatening. Common causes of a headache include migraine and cluster headaches, sinusitis, and muscle tension, among others. In some patients, however, a headache can signal a serious problem. In this patient, you should be most concerned with his unilateral weakness (hemiparesis); this is not a common finding in patients with otherwise benign headaches and suggests a serious underlying problem (eg, intracranial bleeding, brain tumor). The fact that his headache is to the right side of his head and his weakness is to the left side of his body suggests that the problem lies in the right side of the brain. It is doubtful that his blood glucose level (BGL) is abnormal; he is conscious and alert. However, it would not be unreasonable to assess his BGL. His blood pressure, which is significantly elevated, could be causing his headache or may be the result of other factors, such as noncompliance with his antihypertensive medication, anxiety, or pain.

When restraining a violent patient, you should make sure that: • A:the patient is restrained using maximal force. • B:at least two EMTs restrain the patient. • C:consent for restraint has been obtained from a family member. • D:someone talks to the patient during the process.

You selected D; This is correct! Reason:If a violent patient needs to be restrained, you must ensure the presence of at least four people (one per extremity). One of the EMTs should continuously talk to the patient to explain what is happening, even if the patient is not listening. Restraint is a last resort used to protect the EMT as well as the patient. Consent is not needed from a family member prior to restraining the patient. Just enough force to effectively restrain the patient is all that is required to prevent causing unnecessary injury.

Upon delivery of the baby's head, you note that the umbilical cord is wrapped around its neck. You should: • A:immediately clamp and cut the cord and continue the delivery. • B:give the mother high-flow oxygen and transport her on her side. • C:keep the cord warm and moist and transport without delay. • D:make one attempt to gently remove the cord from around its neck.

You selected D; This is correct! Reason:If the umbilical cord is wrapped around the baby's neck (nuchal cord), the EMT should make one attempt to gently remove the cord from around its neck. If this is unsuccessful, clamp and cut the cord and continue with the delivery. A nuchal cord can cause fetal asphyxia and must be treated immediately upon discovery.

While assisting a paramedic in starting an IV on a patient, you are inadvertently stuck by the contaminated needle while attempting to place it in the sharps container. You should: • A:notify your supervisor and request an HIV test. • B:hold the paramedic liable for the needle stick. • C:cease patient care immediately. • D:seek medical care as soon as possible.

You selected D; This is correct! Reason:If you receive an exposure while providing patient care, you should notify your supervisor or designated infection control officer and seek medical care as soon as possible. However, this should be done after you have completed patient care. You will be tested for bloodborne pathogens such as HIV and hepatitis B, and based on the degree of exposure, the physician may suggest immediate treatment. The paramedic did not intentionally stick you with the needle; therefore, you cannot hold him or her liable. Careful and appropriate handling of needles and other sharps will reduce your risk of an accidental stick.

As soon as you begin transport of a patient to the hospital, you should: • A:notify the receiving facility. • B:conduct a detailed examination. • C:contact medical control. • D:advise dispatch of your status.

You selected D; This is correct! Reason:Immediately upon departing the scene with a patient, you should first inform the dispatcher that you are en route to the hospital. Never leave the dispatcher in the dark, for it is the dispatcher's job to know what units are available to answer emergency calls. Notifying the receiving facility, contacting medical control, and performing a detailed assessment of your patient all can occur while you are en route to the hospital.

Which of the following injuries is MOST indicative of child abuse? • A:Multiple bruises to the shins • B:Small laceration to the chin • C:Burned hand with splash marks • D:Bruising to the upper back

You selected D; This is correct! Reason:In order to detect child abuse, you must be familiar with injury locations and patterns consistent with an accident versus those that were intentionally inflicted. It is common for children to trip, fall, and strike their chin or forehead on a solid object; therefore, chin lacerations and hematomas to the forehead are common injuries. Small children frequently hit their legs on coffee tables, resulting in bruises to the shins. If a child accidentally sticks his or her hand in hot water, the hand is quickly pulled back by reflex, resulting in a splash pattern of burns. Injuries found in anatomically unlikely areas, such as the torso (back or front), upper arms and legs, or genitalia, should raise your index of suspicion. Burns that are not accompanied by splash marks should also make you suspicious. For example, if a child's hand or foot is intentionally held in hot water, you will see a clear line of demarcation (stocking-glove effect) without evidence of splash burns.

A conscious and alert 92-year-old woman with chest pain is refusing EMS treatment and transport to the hospital. Her family insists that you transport her. This situation is MOST appropriately managed by: • A:transporting the patient as the family wishes. • B:obtaining a signed refusal from a family member. • C:transporting the patient as you explain your actions. • D:advising the patient of the risks of refusing care.

You selected D; This is correct! Reason:Just because the patient is 92 years old does not mean that she does not have decision-making capacity and cannot make an informed decision. In cases where any patient refuses care, after determining that the patient has decision-making capacity (ie, is of legal age, is not impaired by drugs or alcohol), you must inform the patient of the potential risks of refusing care, namely death. If the patient is aware of and willing to accept the potential consequences of his or her refusal, a refusal of treatment and/or transport form should be signed by the patient. A non-biased witness (ie, law enforcement officer, emergency medical responder [EMR]) should also sign the refusal form.

After arriving at a mass-casualty incident where other ambulances are already present, you should notify the dispatcher and then: • A:initiate care for the most critically injured patients. • B:obtain information from the fire service commander. • C:repeat the triage process. • D:report to the staging area.

You selected D; This is correct! Reason:Once you arrive at the scene of a mass-casualty incident where an incident command system has already been established, you should report to the staging area, the area designated for all incoming ambulances and resources. The staging officer will know where help is needed the most and will be able to direct your actions accordingly.

Upon arriving at the scene of a multiple vehicle crash, you can see that at least two patients have been ejected from their vehicles. You should: • A:immediately triage the two patients. • B:gather all of the patients together. • C:treat the most critical patient first. • D:request at least one more ambulance.

You selected D; This is correct! Reason:One ambulance and two EMTs can effectively care for only one critical patient or two non-critical patients. As soon as you determine that the patient count exceeds your capabilities, you should immediately call for additional help. After doing this, you should begin the processes of triage and treatment.

The safest emergency vehicle operator is one who: • A:has a positive attitude. • B:is physically fit. • C:drives with lights and siren. • D:drives with due regard.

You selected D; This is correct! Reason:One of the most important attributes of a safe emergency vehicle operator is the ability to drive with due regard for others. This means that the operator must be aware of others around him or her and to keep their safety in mind. The EMT should never assume that all drivers will see or hear the ambulance. A positive attitude about one's ability to safely operate an emergency vehicle is also an important attribute. Although sometimes indicated for the patient's condition, the use of lights and siren increase the risk of an ambulance crash.

According to the United States Department of Transportation (USDOT), minimum staffing for a basic life support ambulance includes: • A:at least two EMTs in the patient compartment. • B:a minimum of two EMTs in the ambulance. • C:an EMT who functions as the driver. • D:at least one EMT in the patient compartment.

You selected D; This is correct! Reason:The United States Department of Transportation (USDOT) requires at least one EMT in the patient compartment of a basic life support (BLS) ambulance. Although the driver does not have to be an EMT, it is preferable. Regardless, the person operating the ambulance must be able to safely and effectively operate an emergency vehicle. Regulations regarding minimum staffing of an ambulance vary from state to state.

Law enforcement requests your assistance to evaluate a young female who they believe has overdosed. The patient is conscious, is displaying bizarre behavior, and is very restless. Her BP is 170/90 mm Hg and her heart rate is 130 beats/min. You see needle tracks on both of her arms. She is MOST likely under the influence of a: • A:benzodiazepine. • B:sedative-hypnotic. • C:powerful narcotic. • D:methamphetamine.

You selected D; This is correct! Reason:The needle tracks on the patient's arms indicate narcotic abuse, most likely heroin; however, her signs and symptoms are not consistent with those caused by a narcotic or any other central nervous system (CNS) depressant (eg, benzodiazepine [a sedative-hypnotic]). Signs of CNS depression include a decreased level of consciousness, hypoventilation, bradycardia, and hypotension. Her signs and symptoms are consistent with a sympathomimetic (upper) overdose. Sympathomimetic drugs mimic the effects of the sympathetic (fight or flight) nervous system; therefore, restlessness or hyperactivity, hypertension, tachycardia, and pupillary dilation are common findings in patients who overdose. Amphetamine and methamphetamine (eg, ice, speed, crack) are examples of sympathomimetics.

When treating an unresponsive man who was struck by lightning, you should: • A:manually stabilize his head and open his airway. • B:apply full spinal precautions before moving him. • C:begin CPR at once if he is apneic and pulseless. • D:ensure that you and the patient are in a safe place.

You selected D; This is correct! Reason:Contrary to popular belief, lightning can (and does) strike in the same place twice. After lightning strikes, the ground remains electrically charged for a period of time; this increases the chance of a second strike within a short period of time. You must first ensure that you and the patient are safe by moving to a sheltered area, preferably indoors. After you have ensured the safety of yourself and the patient, begin treatment as dictated by the patient's condition. Do not let the life you save, or attempt to save, TAKE your own!

Which of the following components is NOT part of an injury or illness prevention program? • A:Teaching new parents how to properly install a child safety seat • B:Inspecting the home of young parents for child safety locks • C:Educating teenage students on the dangers of drinking and driving • D:Training a group of daycare employees on infant and child CPR

You selected D; This is correct! Reason:The operative word in injury and illness prevention is "prevention." An injury or illness prevention program should focus on actions that prevent illness or injury. These include, but are not limited to, teaching new parents how to properly install a child safety seat, educating teenage students on the dangers of drinking and driving, and conducting a courtesy inspection of the home of young parents for the presence of child safety locks. Training laypeople on CPR, although a valuable service, is not part of a prevention program; CPR is not needed until an injury or illness has already occurred.

While en route to the scene of an injured person, dispatch advises you that law enforcement personnel are at the scene. This indicates that: • A:the scene is safe for you to enter. • B:a crime has been committed. • C:the patient is critically injured. • D:the scene is potentially unsafe.

You selected D; This is correct! Reason:The presence of law enforcement at the scene indicates, at a minimum, that the scene is potentially unsafe, otherwise they would not be there. While you are en route, you should make radio contact with the police officers at the scene to determine if it is safe for you to enter; you should also inquire about the severity of the patient's injuries. The more information you obtain before you arrive, the better. For all you know, you could arrive and find yourself in the middle of a fire fight! The mere presence of law enforcement at the scene does NOT ensure a safe environment.

The primary clinical feature associated with exposure to a vesicant agent is: • A:vomiting blood. • B:muscle twitching. • C:tachycardia. • D:skin blistering.

You selected D; This is correct! Reason:The primary route of exposure of blister agents, or vesicants, is the skin. If vesicants are left on the skin or clothing long enough, they produce vapors that can enter the respiratory tract. Vesicants cause burn-like blisters to form on the victim's skin as well as in the respiratory tract (if inhaled). Vesicant agents include sulfur mustard (H), Lewisite (L), and phosgene oxime (CX). The symbols H, L, and CX are military designations. Vesicants usually cause the most damage to damp or moist areas of the body, such as the armpits, groin, and respiratory tract.

A set of regulations and ethical considerations that define the extent or limits of an EMT's job is called: • A:the Medical Practices Act. • B:confidentiality. • C:a duty to act. • D:scope of practice.

You selected D; This is correct! Reason:The set of legal regulations and ethical considerations that define the job of the EMT is called the scope of practice. The scope of practice provides a clear delineation of the EMT's roles and responsibilities. Duty to act is defined as a legal obligation to respond to every call for help while on duty and in your jurisdiction, whether you are paid for your services or not. Confidentiality entails not releasing any patient information to those not directly involved in the care of the patient. The Medical Practices Act describes the minimum qualifications of those who may engage in emergency medical care and establishes a means of certification.

The bite of a brown recluse spider is characterized by: • A:two small puncture marks, swelling, and delayed onset of pain. • B:immediate pain, swelling at the site, and painful muscle spasms. • C:rapid swelling within 30 minutes and a decline in mental status. • D:delayed onset of pain, swelling, and blister formation at the site.

You selected D; This is correct! Reason:The venom of a brown recluse spider is cytotoxic; that is, it causes tissue and cellular damage. The bite itself is usually painless; however, the patient typically begins to complain of pain within a few hours. The area becomes swollen and tender, developing a pale, mottled, cyanotic center and possibly a small blister. Over the next several days, a scab of dead skin, fat, and debris will form and dig down into the skin, producing a large ulcer that may not heal unless promptly treated. The bite of a brown recluse spider is very small; therefore, puncture marks are not visible. Unlike the brown recluse spider, the black widow spider bite is characterized by immediate pain. Its venom is neurotoxic; that is, it supresses the central nervous system. The patient typically complains of intense muscle spasms, especially of the abdomen, and systemic signs of nervous system involvement (eg, dizziness, chest pain, difficulty breathing).

A 70-year-old man with a history of emphysema and congestive heart failure is in cardiac arrest. His wife tells you that he collapsed about 5 minutes before your arrival. Your partner begins one-rescuer CPR as you prepare the AED. As you are applying the AED pads, the man's wife tells you that she wants you to let him die in peace. You should: • A:comply with her request and cease all resuscitative efforts. • B:cease resuscitation only if the AED does not indicate a shock. • C:perform rescue breathing only and contact medical control. • D:continue performing CPR and ask her if he has a living will.

You selected D; This is correct! Reason:When faced with a situation in which a family member does not wish for you to attempt resuscitation of a loved one, you should inquire about the presence of a living will or out-of-hospital do not attempt resuscitation (OOH-DNAR) order. If a valid living will or OOH-DNAR order is produced, it is generally acceptable to cease resuscitative efforts; consult medical control as needed. If a valid living will or OOH-DNAR order is not available, the most prudent action would be to continue CPR and contact medical control. Even in the absence of such documentation, medical control may advise you to cease resuscitation based on the wishes of the family and the patient's medical history. When in doubt, err on the side of the patient and attempt resuscitation. Few would argue that it is preferable to defend why resuscitation was attempted as opposed to why it was not.

A 28-year-old woman has severe lower quadrant abdominal pain. When assessing her abdomen, you should: • A:encourage the patient to lie supine with her legs fully extended. • B:ask her where the pain is located and palpate that area first. • C:auscultate for bowel sounds for approximately 2 to 5 minutes. • D:ask her where the pain is located and palpate that area last

You selected D; This is correct! Reason:Assessment of a patient's abdomen includes asking where the pain is located and then palpating that area last. Palpating the painful area first may interfere with the rest of your assessment because of the significant pain the patient will be in. Bowel sounds are of little value in the field and generally are not included in the abdominal assessment. Patients with severe abdominal pain typically prefer to lie on their side with their knees drawn up into their chest (fetal position). Moving them from this position will aggravate their pain.

When called to the scene of a structural fire to stand by in case any injuries occur at the scene, you should: • A:depart the scene and return to service after the fire has been completely extinguished. • B:enter the structure with the firefighters so you can provide immediate care to any victims. • C:park the ambulance close to the fire so you can rapidly access any patients. • D:ensure that your ambulance does not block or hinder other arriving fire apparatus.

You selected D; This is correct! Reason:When standing by at the scene of a structural fire, you must ensure that the ambulance is parked at a safe distance; this will not only keep you safe from the fire itself, but also minimize your risk of injury if the structure collapses. You must also ensure that the ambulance does not block or hinder access to the fire by fire apparatus that arrives after you. You should receive instructions from the fire officer in charge regarding the appropriate location to park the ambulance. It is not the EMT's job, nor is it safe, to enter a structure fire; if victims are located by firefighters, they will be brought to you. Do not depart the scene and return to service until the fire officer in charge releases you; many dangers still exist after the fire has been extinguished.

Which of the following conditions would MOST likely cause flushed skin? • A:Low blood pressure • B:Blood loss • C:Hypothermia • D:Exposure to heat

You selected D; This is correct! Reason:Whenever the body temperature rises (ie, heat exposure, fever), the peripheral blood vessels dilate, which draws warm blood to the skin and gives it a flushed (red) appearance. Blood loss, shock, low blood pressure (hypotension), and hypothermia generally cause the skin to become pale; these conditions cause peripheral vasoconstriction, which shunts blood away from the skin.

A 55-year-old woman with a history of diabetes is found unresponsive with rapid, shallow respirations. The patient's husband tells you that he does not know when his wife last took her insulin. Management of this patient should include: • A:subcutaneous injection of insulin and 100% oxygen. • B:oral glucose and oxygen via nonrebreathing mask. • C:assisted ventilations and oral glucose. • D:assisted ventilations and rapid transport.

You selected D; This is correct! Reason:Without knowing if and when the patient last took her insulin, it is difficult to determine if she is experiencing diabetic coma or insulin shock. Nonetheless, her rapid, shallow respirations—which are likely not producing adequate tidal volume—should be treated with ventilation assistance. Because she is unresponsive and obviously unable to swallow, oral glucose is contraindicated. Assess the patient's blood glucose level and pass this information along to the hospital; if possible, arrange for a paramedic intercept so she can receive intravenous dextrose. If the patient is experiencing diabetic coma, insulin is what she truly needs; however, insulin is rarely, if ever, administered in the prehospital setting—even by paramedics. After ensuring adequate oxygenation and ventilation, transport the patient without delay.

Which of the following is considered minimum personal protective equipment (PPE) when suctioning an unresponsive patient's airway? • A:Gloves and a mask • B:Gloves, gown, and eye protection • C:Gloves, head cover, and eye protection • D:Gloves and full facial protection

You selected D; This is correct! Reason:You should wear gloves when assessing or treating any patient. The level of personal protective equipment (PPE) used beyond gloves is dictated by the risk and type of exposure. When managing a patient's airway (eg, suctioning, ventilating with a bag-mask device), you should use, at a minimum, gloves and full facial protection. This will protect you from oral and eye exposure if the patient coughs or vomits. A gown should be worn, in addition to gloves and full facial protection, any time there is a risk of blood splatter. Examples of when this may occur include delivering a baby or caring for a combative patient with severe external bleeding. The level of PPE you use in a given situation should be guided by reasonable judgment and common sense.

While you are inside a crashed vehicle assessing a patient who is entrapped, the rescue team should be: • A:actively extricating the patient using whichever extrication method they deem necessary. • B:preparing for a simple extrication process since you were obviously able to access the patient. • C:awaiting specific instructions from you as to how to proceed with the extrication process. • D:assessing exactly how the patient is trapped and determining the safest way to extricate.

You selected D; This is correct! Reason:Your ability to access the patient, with or without difficulty, does not indicate the extent of entrapment or method of extrication necessary. The EMT's job is to assess and treat the patient; the rescue team's job is to determine the degree of entrapment and decide how to extricate. As you are assessing the patient, providing any care that you can within the confines of the vehicle, the rescue team should be assessing the vehicle and the degree of patient entrapment; this will enable them to determine the safest, most effective extrication approach. It is critical for the EMT in the vehicle and the rescue team outside the vehicle to constantly communicate. Once the patient has been assessed and provided any immediate life-saving care, and the rescue team has determined the best extrication approach, extrication can begin.

You are called to a local park for an ill person. It is a hot day and the humidity is high. When you arrive, a bystander directs you to the patient, a young male who is semiconscious. His skin is flushed, hot, and moist. Your FIRST action in the management of this patient should be to: • A:begin rapid cooling measures. • B:ensure an open airway. • C:administer high-flow oxygen. • D:move him to a cool area.

You selected D; This is correct! Reason:Your first action in a heat-related emergency is to move the patient to a cooler environment. Once you have moved the patient to a cooler place, you should begin your assessment and treat the patient accordingly. Remember, you must FIRST prevent further harm to the patient.

Following a call in which a 6-week-old infant in cardiac arrest did not survive, your partner is exhibiting significant anxiety and irritability. How can you MOST effectively help him? • A:Report his behavior to the medical director. • B:Tell him that he needs psychiatric help. • C:Recommend at least 12 hours of sleep. • D:Allow him to voice his feelings to you.

You selected D; This is correct! Reason:Your partner is clearly having difficulty coming to terms with this call's bad outcome. As his partner, you can be most effective during this time by simply listening and allowing him to voice his feelings. Bad feelings should never be kept bottled up. If your partner is still having difficulties, a formal critical incident stress debriefing (CISD) may be needed. Some EMS providers may require psychiatric or psychologic assistance; however, this is not a determination that you can make. At least 8 hours of sleep is healthy; however, in the face of a crisis, sleep does not make the problem go away.

A 36-year-old male, who is a known diabetic, presents with severe weakness, diaphoresis, and tachycardia. He is conscious, but confused. His blood pressure is 110/58 mm Hg, pulse is 120 beats/min and weak, and respirations are 24 breaths/min. The glucometer reads error after several attempts to assess his blood glucose level. In addition to high-flow oxygen, medical control will MOST likely order you to: • A:transport only and closely monitor him. • B:assist the patient in taking his insulin. • C:give him a salt-containing solution to drink. • D:give at least one tube of oral glucose.

You selected D; This is correct! Reason:The patient's signs and symptoms indicate hypoglycemia. When you are in doubt as to a patient's blood glucose level, you should err on the side of caution and give sugar; this is what medical control will likely order you to do. The patient, although confused, is conscious and will likely be able to swallow. Insulin is not administered to patients in the field, even if hyperglycemia is documented; EMTs and paramedics are usually not familiar with all of the different types of insulin and their respective doses, and profound hypoglycemia, potentially resulting in death, can occur if too much insulin is given. After giving oral glucose, reassess the patient's mental status and vital signs.

You have just assisted in delivering an infant with a pink body, a pulse rate of 106 per minute, and a good muscle tone. The infant is crying lustily. How should you care for this newborn? A. wrap the newborn in clean towels and give her to the mother to hold during transport B. provide positive pressure ventilations at the rate of 60 per minute with a bag valve mask C. monitor the infant for one minute and reassess vital signs to see if the heart rate increases D. administer free-flow oxygen by holding an oxygen mask or tubing over the newborn's face

`

fowler position

a bed sitting position with the head of the bed raised to 45 degrees

Full thickness burn

a burn in which all the layers of the skin are damaged. There are usually areas that are charred black or areas that are dry and white. Also called a third-degree burn.

Partial thickness burn

a burn in which the first layer of skin (epidermis) is burned through and the dermis (second layer) is damaged. Burns of this type cause reddening, blistering, and a mottled appearence. (Also called second-degree burn).

Superficial burn

a burn that involves only the epidermis, the outer layer of the skin. It is characterized by reddening of the skin and perhaps some swelling. An example is a sunburn. Also called a first-degree burn.

Status epilepticus

a condition in which there are continuing attacks of epilepsy without intervals of consciousness

Simple partial seizures

begins in one area can move, patient aware and conscious, expreience change in taste/smell cause nausea

Pulmonary arteries

carry deoxygenated blood out of the right ventricle and into the lungs

COPD

chronic obstructive pulmonary disease; chronic bronchitis - bronchiole lining inflamed, excess mucus formed; emphysema - walls of alveoli break down; HYPOXIC DRIVE (not in asthma)

Cerebrospinal fluid

clear liquid produced in the ventricles of the brain

Occluded

closed off

V-fib

completely erratic rhythm with no identifiable waves; fatal

Aortic aneurysm

congenital or developed weakness in wall of lower lumbar region (aorta), balloons out, anterior abdominal wall pulses, untreatable if ruptures

Online Direction

consists of orders from the on-duty physician given directly to an EMT-B in the field by radio or telephone.

Ruq

contains majority of liver, gallbladder, small portion of pancreas, right kidney, small intestines, and colon.

Llq

contains parts of the small and large intestines, left ovary, left fallopian tube, left ureter

Rlq

contains parts of the small and large intestines, right ovary, right fallopian tube, appendix, right ureter

Febrile seizure

convulsions brought on by a fever in infants or small children. During a febrile seizure, a child often loses consciousness and shakes, moving limbs on both sides of the body.

You receive a call for a 3-year-old girl with respiratory distress. When you enter her residence, you see the mother holding the child, who does not acknowledge your presence. This finding indicates that the child: • A:is reacting normally for her age. • B:has severe hypoxia. • C:probably is sleeping. • D:is afraid of your presence.

correct answer is B; Reason:A 3-year-old child typically is very attentive to his or her surroundings, especially when a stranger enters the environment. The fact that this child does not acknowledge your presence is an abnormal sign and indicates significant hypoxia. This child must therefore be managed aggressively to prevent respiratory arrest and subsequent cardiac arrest.

Hypoxia

deficient amount of oxygen in tissue cells

Pulmonary veins

deliver oxygen rich blood from the lungs to the left atrium

basket stretcher

designed to surround and protect the patient, this stretcher is used to move a patient from one level to another to over rough terrain.

Hypoglycemia symptoms

hunger, fatigue, weakness, sweating, headache, dizziness, low bp, cold or clammy skin

Complex partial seizure

impairment of consciousness for a few min or less; usually has rhythmic movements of face or extremities

cerebrum

large part of the brain that controls the senses and thinking

Hyperglycemia symptoms

lots of eating, peeing, drinking. blurred vision, fatigue, weight loss

flexible stretcher

made of canvas or rubberized or other flexible material, often with wooden slats sewn into pockets and three carrying handles on each side. can be useful in restricted areas or narrow hallways

Obstructive shock

mechanical obstruction or compressing that prevents blood from reaching the heart

Petit mal

milder form with loss of consciousness for a few seconds. Common in children. May disappear by late adolescence

diaphragm

muscular partition that separates the thoracic cavity from the abdominal cavity and aids in respiration by moving up and down

Placenta stage

placenta is expelled-afterbirth

Hypovolemic shock

shock caused by severe blood or fluid loss

Cardiogenic shock

shock that results from failure of the heart in its pumping action.

Acute Stress

short term ; fight or flight response ; effects disappear quickly after it is over

Acute coronary syndrome

sudden symptoms of insufficient blood supply to the heart, sudden symptoms of insufficient blood supply to the heart indicating unstable angina or myocardial infarction

Acute abdomen

sudden, sever abdominal pain that is less than 24 hours in duration

Seizure

sudden, transient disturbances in brain function resulting from abnormal firing of nerve impulses (may or may not be associated with convulsion)

Epidural hematoma

temporoparietal skull fracture; tear middle meningeal artery

cerebellum

the "little brain" attached to the rear of the brainstem; its functions include processing sensory input and coordinating movement output and balance

Right date, right patient, right drug, right dose, right route

the 5 rights

Asystole

the absence of a heartbeat

Acetabulum

the cup-shaped hollow in the hipbone into which the head of the femur fits to form a ball-and-socket joint

Standard Care

the degree of care that a reasonably prudent person should exercise under the same or similar circumstances

Perineum

the general region between the anus and the genital organs

semi fowler position

the head of the bed is raised 30 degrees; or the head of the bed is raised 30 degrees and the knee portion is raised 15 degrees

mid clavicular

the line through the center of the clavical

brain stem

the part of the brain continuous with the spinal cord and comprising the medulla oblongata and pons and midbrain and parts of the hypothalamus

Bloody show

the pink-tinged secretions that follow the loss of the mucous plug b/c of exposed cervical capillaries. this is a sign that labor will begin within 24-48 hours.

Type 1

this type of diabetes mellitus is an autoimmune situation. The pancreas produces and releases NO insulin, so no glucose transport.

Hypodermis

tissue layer beneath the dermis; contains adipose tissue, a connective tissue that includes fat-storing cells and blood vessels

lower airway

trachea, bronchi, bronchioles, and aveoli (gasses travel through the structers to and from the blood)

TIA

transient ischemic attack; temporary interference with the blood supply to the brain

Hypoglycemia and acute ischemic stroke can present similarly because: • A:both oxygen and glucose are needed for brain function. • B:the majority of stroke patients have a history of diabetes. • C:the most common cause of a stroke is hypoglycemia. • D:they are both caused by low levels of glucose in the blood.

• A:both oxygen and glucose are needed for brain function. Reason: Although stroke and hypoglycemia are two distinctly different conditions, their signs and symptoms are often similar. This is because the brain requires both oxygen and glucose to function normally. An acute ischemic stroke is caused by a lack of oxygen to a part of the brain due to a blocked cerebral artery, whereas hypoglycemia (low blood glucose level) deprives the entire brain of glucose. In either case, the patient presents with signs of impaired brain function (ie, slurred speech, weakness, altered mental status). Both conditions may lead to permanent brain damage or death if not treated promptly.


Related study sets

Origin, Insertion, action, innervation

View Set

Chapter 11 - Network Performance and Recovery

View Set

Unit 7: Creation and Termination of Agency

View Set

5. PROBLEM SOLVING MODELS SARA - SCAN, ANALYSIS, RESPONSE, ASSESSMENT

View Set

Chapter 16 Practice Test: Nursing assessment

View Set

Chapter 7 Principles of Information Security.

View Set

Frankenstein Chapters 11 - 14 For Quiz

View Set

MedSurg PrepU Ch 35 Assessment of Musculoskeletal Function

View Set